Old Exam Questions 2 Flashcards

1
Q

After an uncomplicated appendectomy for acute appendicitis, pathologic exam reveals a carcinoid tumor in the specimen. All of the following are indications for repeat operation and R hemicolectomy EXCEPT

A. Tumor size <1 cm 
B. LVI
C. Presence of goblet cell features
D. Invasion of the appendiceal mesentery
E. Tumor location at the base of the appendix
A

A

Neuroendocrine tumors (NET aka carcinoids) of the appendix are the most common malignant neoplasm of the appendix. Histologically identified in <1% of appy specimens, they are typically dx post op as an incidental finding on path review.

Indications for R hemi, regardless of tumor size, include LVI, presence of goblet cells, invasion of the appendiceal mesentery, and tumor location at the base of the appendix. With tumor size as the most important prognostic factor for risk of mets, repeat OR with R hemi should be performed for NET >2 cm. Mgmt of NET between 1 and 2 cm is controversial; R hemi should be considered b/c up to 1/2 of pts may have regional LN mets. Long term follow up incl plasma chromogranin A levels and CT imaging at 6 and 12 months post op and then annually

How well did you know this?
1
Not at all
2
3
4
5
Perfectly
2
Q

36M has an abdo CT scan after MVA. No injuries are found but his bp is 160/100 mmHg. The CT scan shows a 3 cm adrenal mass. Appropriate initial biochemical evaluation should include all of the following except

A. Plasma metanephrines
B. Plasma aldosterone level 
C. Low dose overnight dexamethasone suppression test
D. Plasma renin level 
E. Serum ACTH
A

E

Adrenal mass incidentally discovered for nonadrenal indications is often called an adrenal incidentaloma. Patients with identified adrenal mass should be evaluated for risk of malignancy and hormonal activity. Pts with hyperfunctioning or potentially malignant tumors should undergo adrenalectomy.

Biochemical eval is performed to dx hormonally active tumors, including pheo. aldosteronomas, and cortisol producing adenomas.

Plasma metanephrines and normetanephrines are the most sensitive markers for pheo; levels 2x N or higher dx. If doubt of dx, 24 hr urine metanephrines and catecholamines should be done.

Concern for primary aldeosteronoma, characterized by HTN and hypokalemia, should be evaluated with plasma aldosterone and renin; an aldosterone to renin activity ration > 20 is suggestive of dx. Confirmatory 24 hr urine aldosterone should be performed.

Hypercorticolism from an autonomously secreting tumor is best dx with a low dose overnight dexamethasone suppression test. a single 1 mg dose of dexamethasone is given at 2300 followed by a morning cortisol level; if this does not suppress the morning cortisol level to less than 5, confirmatory testing with 24 hr urine free cortisol should be performed.

Biochemical eval includes plasma metanephrines. plasma aldosterone, low dose overnight dex suppression test and plasma renin. Screening ACTH is not indicated at this time. Midnight salivary cortisol determination can also be used to dx hypercortisolism. If hypercortisolism is confirmed, an ACTH would be required to ensure the adrenal mass and not the pituitary or an ectopic source is the cause of the elevated cortisol

How well did you know this?
1
Not at all
2
3
4
5
Perfectly
3
Q

73M undergoes lap for perf’ed diverticulitis with widespread feculent peritonitis. Resusc incl 7L crystalloid and 2 units PRBCs. At lap, diverticulitis is identified with free perf of the sigmoid and widespread feculent peritonitis. Pt continues to receive fluid resus and requires norepi to maintain SBP >90. Indications to perform a damage control procedure include all of the following except

A. Arterial pH <7.2 
B. Plt count <50 
C. PTT >50% of normal 
D. Temp <35
E. Lactate > 5
A

B

Damage control surgery is used in pts who are in extremis and dying due to the triad of hypothermia, coagulopathy and acidosis. Limit the operation to essential interventions, namely controlling hemorrhage and limiting enteric contamination and to return the pt to the SICU for physiologic restoration.

Common indications include arterial pH <7.2, PTT >50% of normal, Temp <35, lactate >5, and base deficit > 15 mmol/L (or >6 mmol/L in pts >55 yrs). PLt count is not typically used as a variable in the decision process. Once pts are resusc and their lab parameters have normalized, they are returned to the OR for definitive repair and closure of abdo

How well did you know this?
1
Not at all
2
3
4
5
Perfectly
4
Q

All of the following are indications for elective splenectomy in adults except

A. Hairy cell leukemia with splenomegaly
B. Warm autoimmune hemolytic anemia without response after 3 weeks of steroids
C. Persistent thrombocytopenia in ITP after failure of medical mgmt
D. Severe neutropenia with Felty syndrome
E. Persistent anemia requiring transfusion with hereditary spherocytosis

A

A

Splenectomy may be considered for multiple hematologic disorders.

Warm autoimmune hemolytic anemia is most comomnly dx between age 40 and 70. Initial tx is with corticosteroid therapy (up to 2 mg/kg/day); improvement in Hb is typically seen within 1 week and remission occurs in up to 60%. If remission does not occur within 3 weeks or if hemolysis is not controlled with low dose of steroids (15 mg/day), splenectomy is indicated

ITP is characterized by circulating antiplatelet antibodies that bind plts resulting in thrombocytopenia due to macrophage clearance in spleen and liver. Pts with ITP present with N sized spleen. petechiae, and purpura. Secondary causes that should be excluded include HIV< SLE, antiphospholipid antibody, hep C, lymphoproliferative d/o, cocain, gold, heparin, quinidine, and certain abx and anti-HTNs. Initial tx is medical. Prednisone at 1-1.5 mg/kg/day is administered once plt counts are < 20-30; although 50-75% of pts will iniitally respond to steroids, more than 80% will relapse. Pts who fail steroids may be tx with IVIG and rituximab. Splenectomy is indicated for those who fail medical mgmt.

Felty syndrome consists of neutropenia, splenomegaly and RA. Neutropenia is caused by immune complexes coating the WBCs with resultant sequestration and clearance by the spleen. Initial tx is MTX, corticosteroids, and antirheumatic drugs. Splenectomy is indicated in pts with severe neutropenia or failed medical therapy with recurrent infections.

Hereditary spherocytosis, an autosomal dominant d/o, is the most common RBC membrane d/o in NA. Spherical erythrocytes are destroyed by the spleen, resulting in hemolytic anemia. Although some pts may have a mild form of disease with only mild jaundice, those with persistent anemia, particularly pts requiring repeated transfusions, should undergo splenectomy. To reduct the risk of overwhelming postsplenectomy infection, splenectomy may be delayed in pts <5 yrs.

Splenectomy was originally adovocated for tx in hairy cell leukemia with up to 70% improvement. new purine analog tc with pentostatin and cladribine has become tx of choice with 80-90% remission rates. Splenectomy is no longer performed as part of staging laparotomy for pts with Hodgkin lymphoma

How well did you know this?
1
Not at all
2
3
4
5
Perfectly
5
Q

All of the following can be used as first line therapy for ongoing bleeding from esophageal varices except

A. Endoscopic band ligation
B. IV beta blocker
C. TIPS
D. IV somatostatin
E. IV vasopressin
A

C

Pts who present with UGIB from esophageal varices need to be stabilized rapidly and a definitive dx and hopefully therapeutic procedure will need to be performed expeditiously. Airway control with intubation may be necessary for emergency endoscopy. Two large bore IVS should be placed and resusc with NS whould be begun. Pt should be typed and crossmatched and coag profile and CBC sent.

Pts with a hx of esophageal varices may already be on a nonselective beta blocker for ppx again variceal bleeding. This may be continued if they are hemodynamically N after volume resusc.

Vasoactive drugs, such as somatostatin and vasopressin, should be started if variceal bleeding is suspected. When using vasopressin for severe variceal hemorrhage, a nitroglycerin drip may be beneficial to counteract the severe vasoconstrictive properties on the coronary vessels.

Endoscopic band ligation in combo with pharmacotherapy has beomce a mainstay in tx of acute variceal bleeding. It can be effective in >85% of pts. Although sclerotherapy is also possible, the complictions exceed those of band ligation, thereby making ligation the preferred technique. TIPS should be used in hemodynamically N pts who are refractory to medical and endoscopic mgmt.

How well did you know this?
1
Not at all
2
3
4
5
Perfectly
6
Q

All of the following statements about closure of abdominal wounds are true, EXCEPT

A. Continuous closure with rapidly absorbing sutures has a significantly higher incisional hernia rate compared with continuous closure with slowly absorbing suture closure
B. Abdominal wall closure with continuous nonabsorbable sutures has a higher incidence of suture sinuses and prolonged wound pain compared with absorbable suture closure
C. Significant differences in the incidences of incisional hernia are found between continuous and interrupted abdominal wall closure
D. Midline incisions should be closed with suture length to wound length ratio of at least 4 and a stitch length <1 cm from wound edge
E. When a long stitch length is used, bites of >1cm from the edge can lead to higher incidence of wound infections and incisional hernias

A

C

Statistically sig incr in ventral incisional hernia when rapidly absorbing sutures are used as opposed to slowly absorbing sutures in continuous midline closures. Failed to prove differences in rates with continuous slowly reabsorbing sutures adn nonabsorbable. Incr wound pain and suture sinuses when using nonabsorbable sutures. Did not shows any significant difference between continuous and interrupted midline abdominal wall closures. Most studies favored continous b/c of ease and decr OR time.

Suture length to wound length ratios of <4 might incr the risk of ventral incisional hernia. Recent prospective RCT comparing short stitches (5-8 mm from wound edges) with long stities (>1 cm from wound edge) demonstrate shorter stitches at shorter intervals have a significantly lower rate of surgical site infectinos (SSIs) and incisional hernias. Can be assoc with an incr in OR time. Longer stitch is also assoc with a significantly higher incidnece of SSIs and incisional hernias on multivariate analysis. Postulated mechanism of higher SSI and hernia formation with longer stitches suggest that longer stitches cut through or compress tissue, leading to necrosis and surgical site infection, such as slackening and eventual hernia formation.

How well did you know this?
1
Not at all
2
3
4
5
Perfectly
7
Q

Each of the following is true about paraduodenal hernias EXCEPT

A. Paraduodenal hernias may present to the L or R side of the duodenum
B. L sided paraduodenal hernias are encased in a peritoneal sac that lies between the stomach and pancreas
C. Majority of pts with paraduodenal hernias present with SBO
D. Pts with paraduodenal hernias may present with chronic intermittent, non specific GI symptoms
E. Paraduodenal hernia presenting on the L side accounts for 75% of reported cases

A

C

Internal abdominal hernias not related to acquired adhesions are rare. >50% pts with such hernias have congenital paraduodenal hernias.

Approx 75% hernias occur on the L side of the abdomina through Landzert fossa. This congenital defect in the descending mesocolon is located behind the 4th portion of the duodenum. Formed by a peritoneal fold creased by the IMV and L colic artery as they course along the lateral side of the ascending duodenum. Cross sectional imaging demonstrated sac encased small intestinal loops between the pancreas and the stomach to the L of the ligament of Treitz.

Most patients experience non specific symptoms such as N/V, and abdo pain but do not have typical symptoms and signs of SBO.

Rarer R paraduodenal hernias present with small intestinal hernia through Waldeyer fossa located in the first portion of the jejunum mesentery inferiot ot eh 3rd portion of the duodenum and posterior to the SMA. Cross sectional imaging of these hernias demonstrates sac encased small intestinal loops lateral and inferior to the descending duodenum in the R transverse mesocolon or behind the ascending mesocolon.

How well did you know this?
1
Not at all
2
3
4
5
Perfectly
8
Q

53M smoker presents with a 3 cm symptomatic umbilical hernia. Which of the following herniorrhaphies is least likely to lead to recurrence?

A. Figure of 8 suture
B. Simple interrupted suture
C. Simple continuous
D. Mesh
E. Vest over pants
A

D

Umbilical hernias account for 10% of all primary hernias. Reported recurrence rates excessed 25%. Classic repair is Mayo hernioplasty. Vest over pants imbrication of the superior and inferior aponeurotic segments is performed. Smaller umbilica hernias are closed with a simple interuppted, figure of 8, or continuous nonabsorbable sutures. Mesh herniplasty is often used for umbilical defects >2-3 cm and are usually placed in an onlay (abve the anterior rectus fascia) or sublay in the preperitoneal space. In RCT and observational studies, there was a significantly lower recurrence rate (10 fold) when the repair was perforemd with mesh than without. There was no significant difference in rates of wound complications

How well did you know this?
1
Not at all
2
3
4
5
Perfectly
9
Q

65M underwent a total proctocolectomy with end ileostomy for Crohn’s colitis. He now presents with a large reducible parastomal hernia.

What is the least likely reason for which this pt developed a parastomal hernia?

A. Preop siting
B. Aperture size at creation
C. Patient age
D. Ileostomy rather than colostomy
E. Length of follow up
A

D

Parastomal hernia is a freq complication of stoma formation. Overall incidence may be as high as 50%. Multifactorial and relative contribution of several factors varies from 1 pt to another. In general, length of follow up is assoc with an incr in parastomal hernia dx. Stoma siting throug the rectus abdominis may decr herniation rates b/c the strong muscle provides support to the stoma. Preop siting in lying, standing and sitting decr the chances that the stoma will be placed in a less than optimal site than when the surgeon tries intraop to guess the best site. Aperture size at time of somta creation directly affects the rate of hernia formation: in 1 study, each additional 1 mm incr in aperture size awas assoc with a 10% increase in risk of hernia formation. Older pts and obese patients with a waist circumference >100 cm are at higher risk for stoma herniation. Colostomies are 2x higher risk for parastomal hernias than ileostomies

How well did you know this?
1
Not at all
2
3
4
5
Perfectly
10
Q

65M underwent a total proctocolectomy with end ileostomy for Crohn’s colitis. He now presents with a large reducible parastomal hernia.

What would you recommend initially to maange the patient’s symptoms?

A. Weight loss 
B. Hernia belt
C. Local revision with fascial repair
D. Local hernia repair with mesh
E. Stoma relocation
A

B

Most parastomal hernias do not require surgical intervention. Overall, 10-30% of pts with a parastomal hernia eventually undergo surgery. Conservative mgmt with abdominal support devices such as a hernia belt, counselling regarding wt loss, reevaluation and modification of the stoma appliance by et nurse will often provide symptomatic relief. Sx is reserved for complications such as impairment of stoma function (obstructive symptoms), incarceration, strangulation, or inability to maintain skin integrity.

How well did you know this?
1
Not at all
2
3
4
5
Perfectly
11
Q

65M in otherwise excellent health develops jaundice. CT scan demonstrates a mass in the head of the panc and a solitary lesion in the liver. Percutaneous core needle bx of the liver lesion confirms a neuroendocrine tumor. Which of the following is the best tx option?

A. Peptide receptor therapy
B. RFA of the pancreatic lesion followed by octreotide
C. Radiation
D. Enucleation of both lesions
E. Whipple with resection of the liver lesion

A

E

Gastroenterohepatic neuroendocrine tumor (GEP-NET) is a unifying concept of related tumors including carcinoid tumors, functional endocrine tumors (e.g. insulinoma, gastrinoma), and nonfunctioning neuroendocrine tumors (e.g. islet cell tumors). Broad range of clinical presentations and behaviors. Approx 1/2 are malignant, the endoendocrine carcinomas behave in an indolent fashion.

Resection is potentially curative, even in the face of mets. Functional GEP-NETs should be resected, if possible, to palliate symptoms from hormonal production. If surgical tx is being considered, nonfunctional tumors with mets should undergo resection of both theprimary and met lesions and can lead to significant long term survival.

In this pt who is an otherwise excellent surgical cnadidate, a combo of Whipple with liver resection with concurrent or staged resection of the liver is the best tx option.

Peptide receptor therapy can be used to palliate endocrinopathies for pts with met neuroendocrine tumors but would not be appropriate for tx of the primary. Octreotide is a valuable tx in functinoal neuroendocrine tumors that are otherwise not resectable but does not have a role as a sole therapy for resectable lesions. Primary neuroendocrine tumors do no usually respond to radiation therapy. Although enucleation is an acceptable tx for isolated small tumors involving the body and tail, a larger tumor involving the head of the panc is not amenable to enucleation. In addition, enucleation does not address the liver met. Patients with multiple hepatic lesions not amenable to resection can undergo liver direct tx with chemoembolization, RFA, cryotherapy or other regional therapies.

How well did you know this?
1
Not at all
2
3
4
5
Perfectly
12
Q

Which of the following is the best mgmt approach for a symptomatic splenic cyst?

A. Percutaneous aspiration only
B. Percutaneous aspiration with injection of a sclerosing agent
C. Operative unroofing of the cyst
D. Partial splenectomy including the cyst
E. Splenectomy

A

D

Splenic cysts are categorized as parasitic, usually from Echinococcus infection, or non parasitic. Nonparasitic cysts can be further subdivided into primary (congenital) or secondary (pseudocysts). Primary cysts are relatively rare entities. Symptoms are generally vague LUQ discomfort, although many cysts are completely asymp and are discovered when imaging is done for other purposes.

Indications for surgical intervention include symptoms and cysts > 5cm. Perc aspiration leads to poor results with reaccumulation of the cystic fluid being the norm. Variety of sclerosing agents added to perc aspiration are used. Although success rates for eradicting the cyst have improved, recurrence rates remain high. Unroofing of the cyst still leaves behind a portion of the cyst lining on the spleen. Therefore recurrences are still possible. Splenectomy would be an effective tx of the cyst and was prev considered the standard of care. However with splenectomy, all of the functionining splenic parenchyma is removed, with the subsequent short and long term complication assoc with the asplenic state.

Newer technique of partial splenectomy offers best mgmt option of eliminating the entire cyst wall thereby minimizing recurrences, yet maximizing the the remaining functional splenic parenchyma. Done open or laparoscopically.

How well did you know this?
1
Not at all
2
3
4
5
Perfectly
13
Q

Which of the following is TRUE regarding surgical outcome fro pts with cirrhosis and painful umbilical hernia?

A. Surgical repair should be performed only if the hernia becomes incarcerated
B. Use of mesh should be avoided
C. Preop control of ascites is essential
D. Recurrence rates are identical to the noncirrhotic population
E. Presence of a patent umbilical vein should not affect the decision for repair

A

C

Historically, umbilical hernia repair with cirrhosis is assoc with a high periop morbidity and mortality. Led many surgeons to limit repair to pts presenting with life threatening emergencies, such as incarceration or skin ulceration with ascitic leak. Safe elective repair comparable to noncirrhotic populations can be accomplished, however, with appropriate pt selection. If ascites is not clearly present on physical exam, it is essential to use imaging to look for evidence of hepatic decompensation. Preop control of ascites significantly decr hernia recurrence and allows safe use of mesh for closure of the abdominal wall defect. Ascites control may be accomplished through medical diuresis coupled with serial abdo paracentesis or TIPS. Peritoneal drains may be used to aid postop ascites mgmt if a more urgent repair is required. Herniorraphy should be avoided in the presence of a patent umbilical vein b/c ligation during herniorraphy may alter the portal circulation and lead to acute portal vein thrombosis. Ensuing liver failure may necessitate emergent liver transplant.

How well did you know this?
1
Not at all
2
3
4
5
Perfectly
14
Q

23F with brittle diabetes has been referred for cholecystectomy. She states that she has frequent bouts of N/V 3 hrs after a meal. In the last month, she has had 4 episodes of severe ketoacidosis assoc with RUQ pain and prolonged vomiting lasting for up to 6 hrs. An US of the gallbladder is N. All liver function tests, amylase, and bilirubin are N. Which of the following would you recommend?

A. Lap chole
B. Gastric emptying test
C. ERCP
D. Oral cholecystography with rapid cholecystokinin infusion
E. Cholecystokinin cholescintigraphy during an episode

A

B

Although biliary like symptoms in the presence of a N gallbladder by US may represent gallbladder dyskinesia and potentially warrant chole, this dx requires careful evaluation to exclude other etiologies of episodic abdo pain. In this case, severe gastroparesis, a common complication of diabetes could readily account for the pt’s presenting symptoms and can be rapidly evaluated by a radiolabeled gastric emptying test. Serologic testing of liver and pancreatic enzymes and upper endoscopy to r/o peptic and other primary gastric d/o are also essential before proceeding to surgery.

Assessment of gallbladder emptying with oral cholecystography, both with and without CCK stimulation has sufficiently variable results to make it an unreliable diagnostic tool. Currently CCK cholescintigraphy provides the most reliable measure of gallbladder EF. Should be avoided during an acute episode of abdo pain and it should not be used as a provocative test. GBEF <35% in conjunction with episodic RUQ/epigastric pain lasting >30 mins –not relieved by antacids, BM, or position change–and assoc with N serum liver enzymes and amylas is highly supportive of biliary structural abN and aid evaluation of possible sphincter of Oddi dysfunction, this relatively invasive test would not be a 1st step in the WU of this pt. Furthermore, a low GBEF may ocur in healthy asymp individuals; pts with a varied of medical conditions, including diabetes, celiac, and IBS and as a side effect of opiods, CCBs, OCPs, histamine 2 receptor antagonists, and benzos. Cholecystectomy should be undertaken only when there is a high index of suspicion coupled with supportive evidence of primary gb dyskinesia and when other dx have been eliminated

How well did you know this?
1
Not at all
2
3
4
5
Perfectly
15
Q

CT scan was performed on a 56F for epigastric pain. CT revealed a cystic lesion in the tail of the panc. Patient underwent a distal pancreatectomy with splenectomy. Histology was consistent with a 4 cm, well differentiated, nonfunctional neuroendocrine cystic neoplasm, metastatic to 1 of 9 peripancreatic nodes. Which of the following would you recommend?

A. Octreotide injecitons (long acting release( 
B. Sunitinib
C. Hepatic artery embolization
D. Temozolonide chemo 
E. Observation
A

E

NETs are generally divided into 3 types: well differentiated tumors or carcinoids, which are benign; well differentiated carcrinomas or malignant carcinoids, which show a low grade of malignancy and poorly differentiated carcinomas or small cell carcinomas, which have a high grade of malignancy.

Intestinal tumors are more freq than primary panc tumors, with the ileum as the most common site. Panc NETs account for ~3% of panc malignancies. Although the classic clinical syndromes of hormonally active tumors, such as insulinomas and gastrinomas, are well recognized, the majority are non secretory and metastatic at dx.

Mgmt of localized moderately to well diff neuroendocrine pancr tumors is primarily surgical with resection to clear margins. 5 yr survival rates range from 92% for pts with stage 1 disease (tumor up to 4 cm and limited to panc) to 525% for stage IV tumors ( any T, any N, M1). Histologic grade is a strong prognostic indicator although functional tumor status for panc NET does no significantly affect median survival. This pt would be staged as stage II (T2N1M0) and would be expected to have an 85-90% 5 yr survival after resection alone

Adjuvant therapy for NET is generally reserved for mets and may vary depending on the site of origin and presence or absence of a clinical syndrome, suggesting a “functional” tumor with a biologic target. Tx with octreotide improves progression free survivial in pts with advanced mid gut carcinoid. Pts with panc NET mets to liver may also respond to streptozocin and temozolomide based chemo. The tyrosine kinase inhibitor sunitinib improves progression free survival for some intestinal primary NET but has not been applied to the panc ENT subset. Limited hepatic resection or hepatic artery embolization may be beneficial for those pts with hepatic predominant met disease

How well did you know this?
1
Not at all
2
3
4
5
Perfectly
16
Q

54 alcoholic man was adm with S&S of acute panc 6 mos ago. His course was complicated by panc necrosis and the development of a large pseudocyst, which was found to be infected on percutaneous aspiration. The cyst was tx with external catheter drainage and abx for 1 month, at which time the catether was removed. He now returns with early satiety and epigastric discomfort. His abdo CT scan shows a fluid collection posterior to the stomach and perigastric varices. What would be your recommendation?

A. Percutaneous drainage
B. Endoscopic transgastric drainage
C. Operative cyst-gastrostomy
D. Transpapillary endoscopic drainge
E. Continued observation
A

C

Presence of complication (infection, GOO or biliary obstruction or bleeding) or persistent symptoms should prompt consideration of a drainage procedure for patient with chronic panc pseudocyst. Endoscopic drainage, whether transpapillary or transmural (gastric or duodenal) is becoming the preferred approach, b/c it is less invasive, avoids the necessity of an external drain, and has a high long term success rate. Perc external drainage is generally reserved for infected pseudocysts,. However, open surgery may still be reqd, particularly when portal HTN results from compression or obstruction of the splenic vein/portal vein, either by the cyst alone or in conjunction with underlying chronic pancreatitis. Under these circumstances, open cyst gastrostomy, with or wtihout splenectomy, may be the only safe tx modality. This pt’s CT shows perigastric varices making endoscopic drainage less desirable. A persistent pseudocyst due to a panc stricture that is not amenable to stenting or duct occlusion may also require open internal drainage.

How well did you know this?
1
Not at all
2
3
4
5
Perfectly
17
Q

35 y.o. healthy woman presents to ER with complaints of LUQ pain. She gives a hx of having been in a MVC 3 mos ago. Exam reveals that she is febrile, tender in the LUQ, and has a WBC of 19. CT scan of abdo and pelvis heterogenous collection in LUQ with a bubble of air. After 1 week of abx, she remains febrile with continued elevation of her WBC. Which of the following is the most appropriate next step?

A. Internal cyst drainage
B. Aspiration
C. Percutaneous drainage
D. Operative drainge
E. Splenectomy
A

E

Although uncommon, splenic abscesses can be lethal if not tx appropriately. Most common etiology is hematogenous spread to the spleen from another septic focus, such as endocardidits, diverticulitis, or directly from IV drug abuse. Trauma to the spleen can make the organ more susceptible to infection if there is a devascularized segment of splenic parenchyma. Pts present with fever, elevated WBC and LUQ pain. Dx is made by CT.

IV Abx and splenectomy provide the best means of source control. Aspiration or perc drain may occasionally be successful; they are assoc with incr rates of abscess recurrence (50-60%). This is not a cyst and internal drainage of an abscess is usually not performed. Common organisms include staph and strep and gram neg enteric organisms.

OPSI is a highly lethal complication of splenectomy. Seen more commonly in pts who have eihter had seplnectomy for hematologic reasons, in those who are immunocompromised or in children. When elective splenectomy is considered, the pt should receive vaccines for the following encapsulated organism: streptococcal pneumoniae, haemophilus influenzae and neisseria menengitides.

How well did you know this?
1
Not at all
2
3
4
5
Perfectly
18
Q

23F has had 2 days if nausea, emesis and mid epigastric abdo pain. On physical exam, her temp is 36.8, HR 76, bp 124/54, and RR 14. Scleral icterus is present and her abdo is soft with tenderness in the mid epigastrium and RUQ. Lab data are as follows WBC 56, ALP 128, T bili 4.1, D bili 2.9, Lip 2430. An abdo US confirms cholelithiasis without any gallbladder wall thickening or pericholecystic fluid and a CBD of 6 mm. She is adm to the hospital and 24 hrs later remains afeb; her abdo pain has resolved. Which of the following is the most appropriate next step for this pt at this time?

A. CT scan of the abdo
B. Lap chole w/intraop cholangiogram
C. ERCP
D. IV abx
E. Continue monitoring liver function and symptoms
A

B

Transient obstruction of the CBD and panc duct by gallstone migration may trigger acute biliary pancreatitis. Typical presentation includes nausea, emesis and midepigastric abdo pain. Dx is confirmed with elevation in serum lipase in the setting of cholelithiasis on abdo U/S. Initial mgmt includes NPO, IV fluids and control of symptoms. Lap chole is the cornerstone of surgical tx to prevent recurrent attacks, which may occur in up to 60% of pts.

Timing of lap chole depends primarily on severity of pancreatitis. Early lap chole within 48 hrs of adm in pts with mild to mod biliary pancreatitis reduces the length of hospitalization (4 days vs 7 days). Waiting for normalization of preop lab data does not reduce morbidity and mortality in pts with mild to mod biliary pancreatitis undergoing lap chole

Abdo CT in pts with biliary pancreatitis is indicated in the setting of clinical deterioration concerning for sepsis. IN the absence of cholecystitis and acute cholangitis, use of IV abx is not indicated. Preop ERCP in pts with mild to moderate pancreatitis without evidence of cholangitis has not been shown to affect overall complications or mortality. Approx 60% of pts with biliary pancreatitis and ampullary obstruction show spontaneous relief of obstruction within 48 hrs of symptom onset. In RCT of preop ERCP in pts with mild to moderate biliary pancreatitis without cholangitis, only half of the pts were found to have CBD stones

How well did you know this?
1
Not at all
2
3
4
5
Perfectly
19
Q

Which of the following regarding mgmt of choledocholithiasis is TRUE?

A. ERCP is assoc with higher morbidity than lap CBDE
B. Stone impaction, periampullary diverticula and Mirizzi syndrome incr the possibility of failure of endoscopic CBD stone clearance
C. Predictors of successful stone clearance include proximal stones, large stones and numerous stones (>5)
D. LCBDE is less effective in clearing CBD stones than ERCP
E. Surgical CBD exploration requires placement of a T tube

A

B

Lap CBDE is an ideal alternative to pre or post op ERCP in the mgmt of CBD stones. Review of RCT comparing LCBDE and ERCP demonstrated equivalent duct clearance rates (87.6%), morbidity, and mortality with a trend toward shorter hospital stay in pts undergoing LCBDE. Selection depends largely on local expertise, in addition to anatomic and pathologic considerations.

ERCP requires at least 1 additional procedure unless performed intraop, with potential complications inlcuding pancreatitis, duodenal perf, and bleeding. In addition, stone impaction, hx of gastrectomy, Roux en Y anatomy, recurrent bile duct stones after hepaticojej, periampullary diverticular and Mirizzi syndrome have been shown to incr the incidence of failure of endoscopic CBD stone clearance. Failure of post op ERCP, which occurs in 4-10%, mandates surgical CBD exploration and clearance

Various methods to remove stones from CBD include flushing, Fogarty catheters, and use of choledochoscope and basket retrieval device. USed with either of the 2 primary techniques for LCBDE. Transcystic approach is successful in up to 90% of cases for small stons (<6 mm) and for stoens in the CBD distal to the insertion of the cystic duct. After performed an intraop cholangiogram, a wire is placed through the cholangiogram catheter, followed by a dilating balloon and sheath. Used of a flexible choledochoscope aids in visualization of the stones, which can be removed with a basket retrieval device. Proximal stones, strictures and the presence of numerous stones limit the success of transcystic LCBDE. Alternatively, presence of numerous stones limit the success of transcystic LCBDE. Alternatively, the CBD may be explored laparoscopically by making a choledochotomy. Mgmt options for the choledochotomy includ primary closure, external drainage via an externalized cystic duct drain, or closure over a T tube. Advantages of T tube closure include access to the biliary system for postop evaluation and stone removal but may be complicated by bile leak, peritonitis, biliary fistula and later stricture. Studies comparing T tube and primary closure show similar complication rates, morbidity, and mortality.

How well did you know this?
1
Not at all
2
3
4
5
Perfectly
20
Q

39F is referred for splenectomy for ITP. Which of the following preop factors is most likely to be predictive of a positive response to splenectomy?

A. Age < 40 yrs 
B. Response to corticosteroids
C. Time from dx to sx
D. Preop platelet count
E. Sex
A

A

ITP is an autoimmune disease characerized by the production of antibodies against platelet surface antigens. The spleen has a dual role in pathogenesis, b/c it serves as the primary site of antibody production and platelet sequestration and destruction. Acute ITP is most common in children and is self limiting in 70% of cases. Typical manifestations include petechiae, purpura, and bleeding. Medical mgmt including corticosteroids and IVIG, has only a 20-25% remission rate in adult pts. Splenectomy results in long term remission rates in 66-85% of pts with ITP and this is the preferred option for definitive tx in medically refractory pts.

Multiple factors have been evaluated to assist in predicting clinical response to splenectomy. Age < 40 is the most widely acknowledged predictor of positive response to splenectomy. Refractor or recurrent disease is more common in older pts. Use of surface nuclear scanning to aid in the identification of the site of plt sequestration suggests that older ppl have a higher likelihood of developing extrasplenic sites of sequestration (liver), which may explain failure with splenectomy. Response to cortiocsteroids, time from dx to sx, preop plt count, and gender have not been consistently shown to affect response to splenectomy in patients with ITP

How well did you know this?
1
Not at all
2
3
4
5
Perfectly
21
Q

35F currently on OCPs has a 12 cm lesion in the R lobe of the liver. On review of the triphasic CT scan, the arterial phase displays nodular peripheral asymmetrical enhancement and delayed filling in the same area. Which of the following is the most likely dx?

A. Adenoma
B. Hemangioma
C. Met neuroendocrine 
D. HCC
E. FNH
A

B

Adenomas typically have heterogeneous enhancement on arterial phase and are hypointense on the venous phase. They are also without a central scar on imaging. They are typically present in women and are assoc with risk of rupture and malignancy

Hemangiomas are the most common benign lesion seen in the liver. They are not assoc with OCPs and carry no malignant risk. On CT, periphearl asymmetrical enhancement with delayed vascular filling is characteristic. Risk of rupture is exceedingly low and the indication for resection is typically pain

Met neuroendocrine cancer is hypervascular on the arterial phase and hypoattenuating on the venous phase

HCC shows hypervascular enhanacement on the arterial phase and a characteristic portal venous washout on the venous phase

FNH shows enhancement on the arterial phase and the lesion is difficult to see on the venous phase. A central scar may also be present for FNH. With the exception of its characteristic central scar, FNH enhances homogenously during the arterial phase of contrast enhanced imaging studies. They are typically present in women and are not assoc with risk of rupture or malignancy

How well did you know this?
1
Not at all
2
3
4
5
Perfectly
22
Q

In addition to total bilirubin and creatnine, which of the following is included in the Model of End Stage Liver Disease (MELD) score?

A. Presence of ascited
B. Encephalopathy 
C. Plt count
D. Ammonia
E. INR
A

E

MELD score is an accurate and reproducible scoring system for severityy of liver disease.It is a prospectively developed and validated scale that uses the quantitative, objective values of serum bili, serum Cr and INR. It was iniitially developed to predict death within 3 mos of surgery in pts who had undergone a TIPS. MELD score can prognosticate for mortality from major interventions, such as nontransplant s in pts with cirrhosis. Mortality correlates linearly with the MELD score. Mortality at 30 days ranged from 6% for a MELD score < 8 to mroe than 50% for a MELD score >20.

Child classification is a commonly used scoring system that was originally used to predict the likelihood of variceal bleeding in cirrhotic pts. Uses the presence of clinical ascites, encephalopathy, and serum bilirubin, INR and albumin. Total score of 5-6 is considered grade A (well compensated disease), 7-9 is grade B (significant functional compromise), and 10-15 is grade c (decompensated disease).

Studies comparing MELD and Childs show MELD scoring to be more accurate at predicting mortality in cirrhotic pts undergoing surgery due to a greater scale of objectivity.

How well did you know this?
1
Not at all
2
3
4
5
Perfectly
23
Q

Which of the following is a TRUE statement regarding peritoneovenous shunt for intractable malignant ascites?

A. Complication rates are high
B. Quality of life is improved
C. Survival is improved
D. Medical mgmt is inferior to shunting
E. 30% of pts have symptomatic relief of ascites
A

A

Medical mgmt of scites include specific tumor therapy, active diuresis, dietary restriction, and repeat paracentesis.

Diuretics and salt restrictive diets make sense but they are used inconsistenytly and no good clinical trials support these approaches in pts with malignant ascites. Paracentesis is useful for pts with symptomatic intraabdominal pressure and dyspnea, nausea, and pain are often temporarily relieved. Complication rates are low with this approach but repeat paracentesis are often required to achieve approx 95% effective control.

Major complications of peritoneovenous shunting include pulmonary edema, PE, CV events, overt DIC, infection and hemorrhage. Minor complications including shunt failure, subclinical DIC (which occurs in most pts), wound infection, ascites leak, and the like are common. Thus, the overall complication rate of PVS is very high. The potential for improved survival cannot be clearly states. Same is true for quality of life measures. Wide range of results reported or the control of malignant ascites by PVS (40-100% relief of ascites). As such, individual considerations are the key

How well did you know this?
1
Not at all
2
3
4
5
Perfectly
24
Q

After a severe episode of acute alcoholic pancreatitis a 42M presents with painless hematemesis. Initial upper GI scope reveals N duodenum, mild esophagitis and prominent vessels in stomach. Which of the following is the most likely dx?

A. Splenic vein thrombosis 
B. H pylori infection
C. Mallory Weiss tear
D. Budd Chiari syndrome
E. Dieulafoy lesion
A

A

Images show prominent gastric varices and a N duodenum and the patinent has mild esophagitis. With this hx of recent acute pancreatiits, splenic vein thrombosis with resultant R sided or sinistral, portal HTN leads the ddx. Splenic vein occlusion causes the spleen to engorge and the short gastric vessels become the route for venous decompression. B/c gastric mucosal folds can mimic gastric varices, it is important to fully distend the stomach during endoscopy to reveal whether HTN veins can be displayed. Fastric varices are sometimes difficult to appreciate on routine endoscopy, so alerting the endoscopist to this
likelihood is important.

Tx options for portal vein HTN generally do not apply to sinistral HTN b/c the portal vein is not hypertensive. BUdd Chiari is not assoc with gastric variced. Dieulafoy lesions are usually not detectable by endoscopy. H pylori infection is not assoc with gastric varices. A MW tear would usually present after a bout of forecful vomiting and would be obvious in EGD

How well did you know this?
1
Not at all
2
3
4
5
Perfectly
25
Q

A pt, 3 wks after an alcoholic binge, arrives in the ER with intense upper abdo and back pain. Vital signs are N. He is tolerating a regular diet and is not experiencing diarrhea. A CT scan is obtained in ER and shows a large pseudocyst. Which of the following represents the best surgical care at this time?

A. Stapled cyst gastrostomy plus bx 
B. Perc aspiration and drain placement
C. Endoscopic cyst gastrostomy
D. Octreotide and NPO
E. Multivitamins and pain meds
A

E

By hx, pseudocyst is just forming. Most pseudocysts, even large ones, will resolve without invasive procedures. Best approach is to provide pain control and replace B vitamins from pt’s diet. Octreotide does not improve the resolution rates of panc pseudocysts and should be used very selectively, if at all, for high output panc fistulae. The invasive options are competitive and complementary and should be reserved for mature pseudocysts (>6 wks old) that are symp and large. Cysts < 6 cm usually resolve without invasive procedures. Cysts > 6 cm also usually resolve without invasive procedures but these cysts are more likely to remain symptomatic and large.

Complications of unresolved, large panc pseudocyts include chronic pain, compression of surrounding structures, bleeding, pseudocyst infection, and other life threatening problems. When it comes time to intervene, a cyst gastrostomy with bx of the cyst wall to r/o malignancy is the gold standard option, especially if the etiology of the cyst is in question. This procedure can be accomplished through an open or a lap approach. Endoscopic cyst gastrostomy can be accomplished with the placement of drainage stents into the cyst. Percutaneous aspiration or drain placement can be achieved although the results are not as good as surgical drainage.

How well did you know this?
1
Not at all
2
3
4
5
Perfectly
26
Q

Which of the following is TRUE regarding infected pancreatic necrosis?

A. They can be tx with abx alone
B. mgmt w/perc drain and abx is effective
C. Octreotide prevents development of panc fistula
D. Panc necrosectomy is ultimately required in all pts
E. Incidence is incr in acute pancreatitis with enteral feeding

A

B

Panc necrosis occurs in ~10% of pts with pancreatitis and put pt at risk for multiple organ failure, infection of necrotic tissue, or both. Risk of death is estimated 20-30% for pts with these complications. Dx is best made with CT to determine which areas of the pancreas are perfused. Nonperfused areas are considered necrotic. Definitive dx of infected panc necrosis is more difficult and can be made with a FNA and gram stain/cultures of the aspirate or visualization of the presence of gas within the fluid collection on CT. Pts with sterile necrosis can still have multiple organ failure. Decreasing mortality as low as 4% due to better ICU, avoidance of early open intervention to allow for stabilization, and demarcation of peripanc fluid collections and liquefied necrosis. Intervention should be delayed until ~4 wks after onset of pancreatitis to allow this walling off and demarcation, as well as liquefaction.

Classical tx of infected necrosis is laparotomy and panc debridement; however, this approach is being challenged. MIS panc necrosectomy is described as endoscopic debridement performed via transgastric, laparoscopic, or RP routes. Study prospectively compared open necrosectomy with a step up approach in which the first step was perc drain, followed, if necessary by VARDs with end points of major complications or death. Patients tx with step up had a lower incidence of multiple organ failure and systemic complications but not difference in rate of death. 35% required only perc drain and did not need to go on to necrosectomy

Tx with abx alone is not described. Effectiveness of abx used ppx to prevent infection in cases of nec panc is hotly debated and a 2010 Cochrane review stated “although we cannot confirm benefit from the use of ppx abx, consistent trend towards a beneficial effect nonetheless remain”.

Somatostatin analogs such as octreotide are used to tx panc fistula, although they are not thought to be of benefit. Octreotide is not effective as a preventive measure for panc fistula

Enteral nutrition, compared with TPN, reduces mortlaity, multiple organ failure, systemic infections and the need for operative interventions in pts with acute pancreatitis.

How well did you know this?
1
Not at all
2
3
4
5
Perfectly
27
Q

25F is 12 wks pregnant and presents to ER with R sided abdo pain for 24 hrs, fever, emesis, and WBC 12. Her exam reveals a gravid uterus and R sided tenderness without focal peritoneal signs. Her UA is N; US reveals a viable fetus and no cholelithiasis. The appendix is not seen on the US study. Which of the following is the most appropriate next step?

A. Perform laparoscopic exploration
B. Obtain CT Abdo/Pelvis
C. Perform open appy
D. Obtain MRI of abdo and pelvis
E. Admit and observe
A

D

Acute appy occurs in approx 1 in 1500 pregnancies and is 1 of the most common indications for sx in pregnancy. During pregnancy, physiologic incr in WBC and upward migration of appendix makes dx of acute appy more difficult. Delay in dx ca be assoc with serious complications. Uncomplicated appy results in a 2% rate of fetal loss and 4% rate of early delivery. Perf appy can result in fetal loss rates between 6-30% with an 11% rate of early delivery. B/c of the difficulties in dx, failure to use preop imaging resulted in a neg appy rate of 23-33%

US avoids use of ionizing radiation and assists in identification of gyne dx and is the initial imaging for suspected appy in preg pts. Sensitivity of US in the dx of appy is as low as 20%. If the dx cannot be ascertained via US, MRI avoids ionizing radiation and has a high sensitivity and specificity for appy and alternative dx in pregnant women. Although the cost of MRI is greater than CT or US, it is a minor expense compared with that incurred during a neg appy.

How well did you know this?
1
Not at all
2
3
4
5
Perfectly
28
Q

65 y.o. obese male smoker with DM presents to ER with a 3 yr hx of foul smelling purulent drainage from his abdo wall. He has experienced incr pain, fevers, and chills. His hx is notable for a colostomy, colostomy takedown complicated by a surgical site infection (SSI) and ventral incisional hernia (VIH) and 2 prior VIH repairs with prolene mesh. Which of the following statements is TRUE?

A. An occult underlying enterocutaneous fistula can present as a mesh infection
B. Any tx must remove all existing mesh
C. Gram neg bacteria are the most likely cause
D. Prior use of expanded PTGE would lower the risk of mesh explantation
E. Bowel resections at the time of VIH repair do not alter SSI rates

A

A

This pt has a mesh infection based on purulence draining from his abdo wall in the setting of an implanted mesh Definitive dx of a mesh infection relies on postive deep cultures of fluid surrounding the mesh or mesh cultures. Mesh infections occur with a reported incidence of 6-10% in open repairs and 0-3.6% in laparoscopic ventral incisional hernia repair. Mesh infections in the early post op period are more likely to be assoc with an enterocutaneous fistula. They are the 3rd leading cause of mesh explantation. Staph aureus is the organism most implicated in mesh infections, although other organisms, such as staph epi, strep pyogenes and gram negs are implicated. Tx of mesh infections depends on the pt’s clinical status. Pts who may be septic require abx and explantation. Concomitant procedures performed via the same incision as hernia repair, enterotomy, surgical site infection and ECF are assoc with higher rates of of mesh explantation. Expanded PTFE mesh used in open ventral incisional hernia repairs is significantly more likely (4x) to need explantation than prolene or expanded PTFE mesh used in lap repairs. Well incorporated mesh does not need to be removed during an explantation procedure

How well did you know this?
1
Not at all
2
3
4
5
Perfectly
29
Q

Which of the following is TRUE aout bile leaks from subvesical ducts of Luschka?

A. They are the 2nd most common cause of postcholecystectomy bile leaks
B. They are the sole drainage from assoc liver parenchyma
C. They represent disruption of drainage into the gallbladder
D. They are most commonly seen after cholecystectomy for acute inflam
E. Enteric drainage is usually required for tx

A

A

Leaks from the subvesical ducts of Luschka are next in freq to those from the cystic duct. Luschka ducts are accessory, not aberrant, biliary ducts that drain subsegmental areas of liver into the extrahepatic bile ducts. Ducts of Luschka are estimated to occur in up to 50% of pts. Aberrant hepatocystic ducts are the only source of drainage from their assoc liver parenchyma, whereas accessory ducts are not. The hepatocystic duct may drain into the gallbladder or cystic duct. Luschka ducts do not. Injuries to Luschka ducts are not predicted by the presence of acute chole or by the elective or urgent nature of a chole. Tx of a Luschka duct leak not recognized at the time of chole is often successfully accomplished by sphincterotomy and transampullary stenting to decr the pressure in the biliary tree. Enteric drainage is not usually required.

How well did you know this?
1
Not at all
2
3
4
5
Perfectly
30
Q

83 y.o. emacited female resident of a nursing home who has never undergone abdominal operation presents with a distal small intestinal obstruction and R thigh and knee pain that began 3 days earlier. Which of the following is TRUE?

A. Celiotomy is unlikely to cure this patient
B. Abdominal/pelvic CT is useful in establishing the dx
C. Hip osteoarthroplasty with assoc ileus is the most likely cause
D. NG suction should resolve the pt’s symptoms
E. End of life care should be provided

A

B

Mechanical SBO in elderly pt who have not undergone prior sx suggest the dx of malignancy or internal hernia. Obturator hernia are rare but thay are identified typically in such patients, particularly multiparous women. Ipsilateral thigh and knee pain (the Howship Romberhg sign) is caused by obturator nerve compression. Sign occurs in approx half of pts. CT have improved the dx accuracy of obturator hernias. 1/3 of pts have evidence of intestinal ischemia at the time of sx and hernia reduction. B/c many elderly pts have degenerative joint disease, nonspecific nature of pain in patients with obturator hernias may suggest a more common finding of hip and lumbar spine disease. However, mechanical SBO confirmed by CT should raise suspicion for an obturator hernia. Early dx of this disease is assoc with a mortality of only 5% in a group of pts with a mean age of 80.

How well did you know this?
1
Not at all
2
3
4
5
Perfectly
31
Q

Which of the following statements about spigelian hernias is MOST accurate?

A. They should always be repaired using an open anterior abdominal approach
B. They can be dx solely bt Hx and Px
C. They penetrate hte transversus abdominus and internal oblique muscles and lie posterior to the external oblique aponeurosis
D. They develop above the arcuate line
E. The present in or before the 3rd decade of life

A

C

May present without significant abdo wall bulges b/c they are generally interparietal, penetrating the transversus abdominis and internal oblique muscles, but lying posterior to the external oblique aponeurosis. Pts present most freq in 5th and 6th decades of life. Hernias occur anywhere along the Spigelian fascia that runs between the lateral edge of the rectus abdominis and the semilunar line. Most freq develop at or inferior to the arcuate line. Often cannot be dx on hx and px alone. Physical exam false neg in 36% of pts and false positive in half of pts. CT improves dx accuracy but false neg scans in 1/3 of pts. RCT of open vs las repairs found that both techniques were effective inn preventing recurrences. Advantages of laparoscopic; these were greater number of pts having outpt sx and shorter hosp stays

How well did you know this?
1
Not at all
2
3
4
5
Perfectly
32
Q

Compared with open repair, lap ventral hernia has

A. Lower recurrence rate
B. Less post op pain
C. Fewer post op seromas
D. Fewer surgical site infections
E. Higher enterotomy rate
A

D

Meta analysis of 10 RCTs involving 880 patients, there was no statistical difference between recurrence rates for lap or open repair of ventral hernias, although follow up was limited. In the 4 trials in which early post op pain was recorded, there was no difference in pain intensity between the two techniques Seroma formation was relatively common with both techniques. Most consistent finding across all studies was a significant decr in incidence of surgical site infections (3% vs 13%). Enterotomy was uncommon in both groups.

Laparoscopic abdominal wall hernia repair had a significantly lower likelihood of developing morbidity but there was no effect on mortality

How well did you know this?
1
Not at all
2
3
4
5
Perfectly
33
Q

Compared with delayed lap chole (>6 weeks), early lap chole for acute cholecystitis (<7 days) is assoc with

A. Incr open conversion rate
B. Higher bile duct injury rate
C. Higher rate of bile leak
D. Lower mortality rate
E. Lower cumulative length of stay
A

E

Initially there was concern that lap chole at the time of acute inflam would result in higher open conversion rate and a higher incidence of serious bile duct injuries. Despite growing evidence with lap chole, some surgeons still cool the pt off with abx and delay OR for 6 wks.

In a meta analysis (5 trials of 451 pts) of RCTs comparing early (<1 week after onset of symp) vs delayed lap chole (>6 wks after onset of symp) for acute chole, there was no difference in rates of conversion to open, bile duct injury or bile leakage. No mortality in either group. Early lap chole results in a 4 day reduction in total hospital LOS

How well did you know this?
1
Not at all
2
3
4
5
Perfectly
34
Q

26F presents to ER with a chief complaint of RUQ pain for 12 hrs. On physical exam, she is afeb, anicteric, has mild RUQ pain to palp and has no Murphy’S sign. Labs shows a WBC of 7, AST28, ALT 32, ALP 331, and Total bili 3.2. U/S shows a N size CBD and no evidence of acute chole. Which of the following studies is most likely to confirm the presence of choledocholithiasis preop?

A. HIDA scan
B. CT scan
C. MRCP
D. ERCP
E. EUS
A

C

Option for preop ERCP if CBD stone is suspected.

Deciding factor to pursue preop eval of CBD depends on probability a CBD stone would be present based on clinical findings, findings on preop labs and US. Very strong = CBD stone on US, Clinical ascending cholangitis, Bili >40; Strong = dil CBD on US >6 mm with gb in situ and bili 1.8-4; Moderate = AbN liver tests other than bili, Age >55, Clinical gallstone panc

If low probability of choledocholithiasis–> surgery. High probability of choledocholithiasis–> ERCP. Intermediate probability–> Surgery + IOC or preop imaging

HIDA is reasonable if acute chole is being considered but limitations in evaluation of CBD.

MRCP sensitivity is greater than 90% and specificty >95% which is better than CT of US.

ERCP and EUS are invasive. EUS has sensitivity and specificity similar to MRCP. More sensitive for stones <5 mm. ERCP typically reserved for pts with very high probability of CBD stones. B/c incr risk of complications includ pancreatitis, hemorrhage and perf.

Pt has 1 strong risk factor present (elevated bili). Make her intermediate probability . 2 options are MRCP or EUS.

How well did you know this?
1
Not at all
2
3
4
5
Perfectly
35
Q

30M involved in MVC. He is evaluated and his only injury is a grade IV liver injury with an active blush. He is taken for angio, where as arterial bleed is identified and managed with selective embolization. Seven days later he develops a fever of 39. CT scan shows a large collection with air in the liver. Which of the following is the most likely dx?

A. Recurrent hepatic arterial bleed
B. Biloma
C. Hepatic parenchymal necrosis
D. Hepatic abscess
E. Hemobilia
A

D

Liver is one of the most commonly injured organs after blunt trauma. Range from minor to severe, most are managed without surgical intervention. As grade of liver injury incr, likelihood that the pt will require some sort of intervention incr. With intro of angio evaluation of liver injuries, arterial abN that otherwise would have gone unnoticed are now being identified and tx. Most common tx is embolization.

After significant liver injury, complications are assoc with the injury itself as well as the perc intervention. Complications broken down into early and late. Most significant early complciation is ongoing or recurrent bleeding. Typically seen within first 48 hrs after injury. Presentation may include incr abdo pain, distension, peritonitis, hemodynamic instability, abd dec hct.

Late complications incl hepatic necrosis, abscess, and biloma. Differentiated on CT by their appearance. Enhancement is more characteristic of hepatic abscess. Another test to differentiate is the measurement of serum bilrubin. If this is elevated, dx is more consistent with biloma

Hemobilia is a late bleeding complication assoc with liver injury. It presents with a sudden drop in Hct sometimes assoc with hemodynamic instability. Bc the CBD is filled with blood, an elevation of serum bili also occurs.

How well did you know this?
1
Not at all
2
3
4
5
Perfectly
36
Q

In addition to abx, which of the following approaches to infected panc necrosis has the lowest incidence of morbidity?

A. Observation
B. Perc drain
C. Perc drain and min invasive RP drainage
D. Open necrosectomy
E. Open necrosectomy and panc packing
A

C

Panc debridement with either open packing or drainage is assoc with significant complications. Open technique with packing of peripanc space and RP gave reasonable control of infectious and necrotic processes but led to significant complications assoc with fistula formation and prolonged inflamm response. With debridement and drainage there was the ned for multiple procedure to ensure that a complete debridement was undertaken. Injury to the intraabdominal viscera was not uncommon with the many procedures. In addition, b/c the operative procedures, there was a continued generation of inflam response.

Minimal access technique for panc necrosectomy. Placement of drain into the RP space and then serial dilation of the drain tract until a RP endoscopic approach can be undertaken. Area is debrided and irrigated, and drain catheters left at the completion of the procedure. Significant decr in complications assoc with nec panc. Decr rates of postop organ failure, ICU support and complications (e.g. multisystem organ ffailure, bleeding, visceral perf, and death)

2010 study compared open mgmt vs step up approach and found the step up approach has statistically significant decr in number of complications. New onset multisystem failure also occurred less freq. No difference in the number of deaths in each group. Step up with perc drain and advancement to endoscopic drainage appears to provide best outcome for these complicated pts.

How well did you know this?
1
Not at all
2
3
4
5
Perfectly
37
Q

As part of a study protocol, a healthy, asymp 22F undergoes an abdo US. A 3 mm polyp is found in her gb. What would you recommend for this pt?

A. FU US in 6 months
B. EUS
C. CA 19-9 and CA125 
D. ERCP with biliary cytology
E. Lap chole
A

A

Use of screening US often leads to incidentalomas. With regard to gb, most common incidental findings include gallstones and gb wall polyps. Cholelithiasis is an asymp pt is a relatively straightforward dx. If a pt is not having symptoms from gallstones, chole is not indicated. A gb polyp, conversely, has the potential to be neoplastic. For this reason, in the past, it was recommended that gb poylps, even if asymp, necessitated cholecystectomy

Most recent data have providede guidelines to the val and mgmt of gb poylps. Recommendations divided into gb assoc sympts, assoc conditions, polyp size, and polyp morphology. Pt with sympts attributed to gb who has a polyp on US should undergo chole. Chole is indicated in the asymp pt if the polyp is >6 mm or sessile, or if the pt has PSC. If the pt is asymp, polyp is pedunculated and 6 mm or smaller, follow up US is indicated. If on repeat US, polyp incr to more than 6 mm or beings to develop sessile features, chole is indicated.

In this pt with an asymp 3 mm polyp, a follow up US 6 months would be the correct recommendation. No further eval is required if the polyp is stable and the pt remains asymp.

How well did you know this?
1
Not at all
2
3
4
5
Perfectly
38
Q

Regarding parastomal hernias, which of the following is TRUE?

A. Use of mesh at initial stoma formation reduces the incidence of parastomal hernia
B. Mesh placement at initial stoma formation increase the complication rate compared with no mesh placement
C. Primary repair of a parastomal hernia provides durable long term results
D. The complication rate is similar whether synthetic mesh or biologic mesh is used for parastomal hernia repair
E. The results of laparoscopic parastomal hernia repair are superior to open repair.

A

A

Ppx mesh placement at the time of stoma formation has conclusively been shown to reduce the incidence of parastomal hernia. Two systematic reviews of mesh vs no mesh stoma formation did not identify any incr morbidity or mortality with the use of mesh. Analysis included biologic and synthetic mesh

Mesh placement is used in the repair of established parastomal hernias, b/c primary direct suture fascial repair leads to unacceptable recurrence rates (46-100%). Type of mesh used may affect the rate of complication; synthetic mesh repair is assoc with a higher rate of complications than biologic mesh, often leading to the need for mesh explantation. Mesh infection, extrusion or erosion, chronic pain, bowel obstruction, seroma formation, and intestinal fistulization occur more often synthetic mesh. Sufficient robust long term data are not currently available to conclude that lap mesh placement is superior to open mesh placement

How well did you know this?
1
Not at all
2
3
4
5
Perfectly
39
Q

25M presents to ER with a newly identified R inguinal hernia. It is easily reducible and you are now counseling him regarding the mgmt of his hernia. Which of the following statements is TRUE?

A. Risk of strangulation is currently 5%
B. Risk of chronic groin pain at 5 yrs after hernia repair is 10-20%
C. Risk of chronic groin pain at 5 yrs is the same whether an open or lap repair is performed
D. Likelihood of conversion from watchful waiting to surgical repair is 80%
E. Likelihoof of developing chronic testicular pain is higher with an open repair than a lap repair

A

B

In RCT comparing watchful waiting and early surgical repair with open tension free techniques, only 2 of 364 watchful waiting pts experienced acute incarceration and non experiences strangulation during the 4.5 yr follow up. 23% converted from watchful waiting to surgical repair during this time and hernia related pain was the most common reason providede. Surgical outcomes from pts initially assigned to surgical repair and those who crosed over to repair were similar.

Chronic pain after surgery is also an important outcome to consider. In randomized multicenter study analysing outcomes after lap TEP or open Lichtenstein hernia repair, authors folowed 1370 pts and found the total incidence of chronic pain to be 9.4% vs 18.5%. This finding includs pts reproting mildpain, defd as occ discomfort or pain not limiting daily activities. By contrast, moderate or severe pain, defined as occ or daily interference with daily activities was noted in only 1.9% and 3.5% after 5 yrs. Long term follow up of RCT reported that testicular pain was more common finding in pts who underwent a TEP compared with open mesh repair.

How well did you know this?
1
Not at all
2
3
4
5
Perfectly
40
Q

18M presents to trauma room after sustaining a single GSW to the lower abdo. He is initiall resusc and taken to OR for abdo exploration. On exploration, he is found to have multiple small bowel enterotomies as wellas a R ureter injury. Bowel is repaired. Which of the following statements is TRUE regarding injuries to the ureter?

A. Injuries to the lower third of the ureter are managed by ureteroneocystostomy
B. An anterior wall bladder (Boari) is the preferred technique to repair an injury to the upper third of the ureter
C. A psoas hitch is appropriate for injury to the middle third of the ureter
D. Ligation with neph tube drainage and delayed repair decr morbidity
E. Mobilization of the ureter is safe secondary to its collateral blood supply

A

A

Ureter injuries are relatively rare, but missed injuries lead to significant morbidity adn mortality. Injuries to the ureter are classified and managed based on the degree and location of injury. Divided into upper, middle and lwoer thirds. Upper extends from UPJ to the area where it crosses the SI joint. Middle ureter courses the bony pelvis to the iliac vessels. Lower portion extends from the iliac vessels to the bladder. Blood supply to the ureter (utreteral artery) is tenous and runs longitudinally along the ureter without collateral flow in 80%. Reconstruction options for the upper 1/3 include ureteroureterostomy or ureteropyelostomy. Recon options of the middle 1/3 include ureteroureterostomy, transureteroureterostomy or anterior wall bladder flap (Boari). Recon options for the lower 1/3 include ureteruneocystostomy with direct reimplantation and ureteruoneocystostomy with a psoas hitch. Early identification and surgical repair improves overall morbidity; therefore drainage and delayed operative repair is not a preferred mgmt strategy

How well did you know this?
1
Not at all
2
3
4
5
Perfectly
41
Q

57M presents to Er wuth a 4 day hx of vague abdo pain, low grade fever, and mild nausea. Evaluation demonstrates N vitals except temp 38.8. He is tender in RUQ without diffuse peritonitis. CT shows a heterogenous collection in liver . Beside broad spectrum abx, which of the following is the most appropriate therapy?

A. Open sx drainage
B. Lap resection
C. Perc drain
D. Hypertonic saline injection
E. Thiabendazole
A

C

Pt presents with pyogenic liver abscess. As tx of appy, diverticulitis and other intra abdominal infections has improved, incidence has decr. Biliary obstruction, stenting or instrumentation is now a more common primary source than seeding through portal blood flow.

Tx is primarily abx and perc drain. Needle aspiration of small <5 cm, simple abscesses may suffice but large multiloculated abscesses freq require multiple catheters and interventions to resolve the abscess. In the presence of yeast in the abscess or demonstration of communication with the biliary tree may predict failure of perc tx strategy. Perc drain found to produce statistically significant improvements in morbidity, LOS, and hospital costs. Preferred to open or lap drainage or liver resection

How well did you know this?
1
Not at all
2
3
4
5
Perfectly
42
Q

53F with prior hx of Bassini repair of a R inguinal hernia presents with c/o R groin pain, nausea and vomiting. Evaluation revelas N vitals and distended, non tender abdo. Exam reveals a firm mass in the R groin below the level of the inguinal ligament. Mass is non reducible and moderately tender. In the OR a piece of dusky small bowel is found. Which of the following statements is TRUE regarding this condition?

A. The most likely dx is a recurrent inguinal hernia
B. Lap inguinal hernia repair decr the risk of this complication
C. Men are more likely to present with this dx
D. Exp lap is necessary
E. This hernia, if asymp, should not be repaired

A

B

Pt presents with strangulated femoral hernia after prev open inguinal hernia repair. Fem hernias comprise 2-8% of all groin hernias in adults and are extremely rare in children. Most are found in adults age 40-70 and are 4-5x more common in women. In contast to inguinal hernias, 30-50% of femoral present as emergencies due to incarceration or strangulation. B/c of higher risk of incarceration or strangulation, once dx, femoral hernias should be repaired in pts of suitable surgical risk. Some physicians now recommend lap repair of all inguinal hernias in women b/c of higher risk of clinically undetectable femoral hernias (37%). If a strangulated hernia is encountered during open groin exploration, the bowel may be freed by dividing the inguinal ligament and then assessing for viability. If a bowel resection is necessary, this can most often be accomplished through the groin incision without converting to laparotomy. Repair of the hernia will depend on the level of suspected contamination of the surgical field. If the bowel is incarcerated but viable, standard tension free techniques can be used. If there i contamination, mesh should be avoided and either a tissue repair or implantation of a bioprosthetic should be considered

How well did you know this?
1
Not at all
2
3
4
5
Perfectly
43
Q

48F presents with several months of chronic abdo pain, 48 hrs of incr jaundice, and low grade fever. An ERCP shows no ductal stones but a smooth narrowing of the CBD where the cystic duct enters the CBD. Which of the following statements about this condition is TRUE?

A. Lap chole is the operative approach of choice
B. Cystic duct is at R angles to CBD
C. Gallstone ileus is a common complication
D. It is assoc with a 25% risk of gb cancer
E. Prev chole excludes the dx

A

D

Pt has Mirizzi syndrome. Obstructive jaundice due to extrinsic compression of the CHD caused by a stone impacted in the neck of the gb or cystic duct. Depending on degree of impairment and chronicity of condition, may be a cholecystocholedochal fistula. Rare complication of gallstones occurs in approx 0.1-0.7% of pts who have gallstones and this condition is distinct from gallstone ileus. Up to 25% of pts have incr risk of gallbladder cancer

4 factors contribute to development: 1) cystic duct must be anatomically parallel to CHD, 2) stone must become impacted in cystic duct or gallbladder neck, 3) CHD must be obstructed by the stone or by the secondary inflammatory response, 4) longstanding obstruction must cause intermittent or constant jaundice with occasional cholangitis

B/c stone is characteristically in either the neck of the gb or cystic duct, prev chole does not rule it out as a possible dx.

Csendes
I - Pressure on CHD due to an extrinsic stone in cystic duct
II- A cholecystobiliary fistula <1/3 circumference of duct wall
III- A cholecystobiliary fistula involving 2/3 circumference of the ductal wall
IV- A cholecystobiliary fistula involved the entire circumference of the ductal wall

Lap chole can be successful with Type I Mirizzi syndrome or external compression of the CHD by a stone impacted by the cystic duct or Hartmann pouch but the authors cautioned that open chole is the method of choice for type 2 Mirizzi syndrome, a fistula between the gb and common duct from inflam and erosion. Another retrospective study indicated that of 14 pts who underwent laparoscopic sx, 11 required conversion to an open procedure.

How well did you know this?
1
Not at all
2
3
4
5
Perfectly
44
Q

49M with hx of anemia presents with mild abdo pain and bloating. 5 sm (<1cm) gastric polyps in the proximal stomach are identified by upper endoscopy. These polyps are confirmed to be carcinoid tumors. The histopathologic analysis also confirms strophic gastritis. Pt’s serum 5 HIAA (1.0) is normal but his serum gastric is elevated (1500). Which of the following statements regarding this pt is TRUE?

A. These polyps are the result of enterochromaffin like cell proliferation
B. The atrophic gastritis is likely caused by antibodies directed against the goblet cells
C. The incidence of nodal mets from these tumors is 50%
D. Endoscopic surveillance of this pt is not recommended
E. Gastric resection of the lesions to include the Antrum is recommended

A

A

Gastrin is secreted by G cells in the gastric Antrum. Acts on parietal cells to stimulate HCL after ingestion of food. Gastrin production is inhibited by somatostatin secreted by D cells in response to gastric acid. Hypergastrinemia can can occur secondary to uninhibited gastric production or in response to decr acid secretion. Pts who have a gastrinoma as their source of uninhibited gastrin secretion usually present with peptic ulcers secondary to acid hypersecretion. By contrast, unopposed gastrin secretion in response to achlorhydria typically occurs in pts with atrophic gastritis or who take PPI. Hypergastrinemia occurring with gastritis related to H pylori infection is caused by the decr somatostatin release by D cells due to incr pH and circulating cytokines around D cells.

Gastric carcinoid also termed NETs are rare in US, comprising 4.1% of all carcinoid tumors. Subclassified into 3 distinct groups: those assoc with chronic atrophic gastritis/pernicious anemia (Type 1 70-80%), those assoc with MEN type 1 ZES (type 2 2.5%) and sporadic NETs of the stomach (type 3 15-20%). Both type 1 and 2 NETS of the stomach are assoc with hypergastrinemia. Type 3 develop in the absence of hypergastrinemia and tend to pursue an aggressive clinical course

B/c type 1 and 2 pursue an indolent course, tumors <2 cm (up to 6 in number) should be resected endoscopically, with subsequent interval follow up. Pts with tumors >2 cm, those with recurrent tumors or those with >6 tumors generally require more aggressive mgmt and local sx resection is recommended. In Type I NET arising in the setting of chronic atrophic gastritis, antrectomy may be performed to eliminate source of gastric production. Antrectomy results in tumor regression. In Type 2 NET of the stomach secondary to ZES/MEN I syndrome, tx with somatostatin analogs may be initiated and result in tumor regression. The surgical mgmt of type 3 isolated sporadic NETs of the stomach require more aggressive surgery, generally with partial gastrectomy and LN dissection

This pt has anemia, hypergastrinemia, and atrophic gastritis consistent with Type I gastric carcinoid. Tumors in pts with type 1 gastric carcinoid are caused by gastric stimulation of ECL cells proliferation which can progress to ECL cell hyperplasia and type 1 gastric carcinoid. This progression occurs in approx 5% of pts with pernicious anemia, an autoimmune condition in which antibodies are directed against parietal cells. Destruction of parietal cell mass results in atrophic gastritis, loss of acid production, hypergastrinemia, and macrocytic anemia from loss of intrinsic factor and vit b12 absorption. Risk of LN mets in this pt is low. Reduction in gastrin from Antrum through antrectomy alone should result in regression of the tumors without resection of the tumors themselves

How well did you know this?
1
Not at all
2
3
4
5
Perfectly
45
Q

Which of the following statements regarding the nutritional sequelae after RYGB is TRUE?

A. Patients should consume 30 g of protein daily
B. Oral replacement of thiamine is unnecessary
C. Microcytic anemia is common
D. Fat malabsorption is uncommon
E. An incr in serum PTH levels indicates an overdosing of oral vitamin D

A

C

RYGB combines gastric resection with a minor malabsorptive state to achieve wt loss. Affects gut hormone, such as gherkin, glucagon like peptide 1 and peptide YY that influence eating behaviours and body wt. Change in eating habits combined with the malabsorptive state can result in significant nutritional consequences.

Major physiologic consequence of RYGB involves lipid absorption. Under physiologic conditions, fat passes into duodenum and stimulated CCK. CCK stimulates gb and pancreas to release bile and lipolytic enzymes. After RYGB, secretion of bile and lipolytic enzymes is reduced, b/c lipids never pass through the duodenum. Lipids including triglycerides, phospholipids and cholesterol travel through the Roux conduit as intact structures until they reach the jejunojejunostomy. Delayed breakdown of dietary fats and the delayed formation of micelles limits the amt of fat available for absorption. Undigested fat passes into the colon, producing fat malabsorption and steatorrhea.

Intolerance of protein rich foods, such as meat and dairy products, is common. Many pts fail to meet the recommended protein intake, which should avg 60-120 g/day. Meat and dairy products also contain sever critical moicronutrients; therefore routine supplementation and monitoring for deficiencies in iron, vit b12, calcium, vit D, folate (vit B9), and thiamine (vit B1) is recommended. In addition, iron, calcium, and thiamine are absorbed primarily in the duodenum, which is precluded in RYGB.

Iron stores decline after gastric bypass, making iron deficient microcytic anemia very common. Orally administered ferrous sulphate, fumarate or gluconate may be needed to prevent iron deficiency. Vit C should be added to incr iron absorption and ferritin levels. Oral iron supplements can decr absorption of ca, mg, and zn, so these should be taken at different times of day. RYGB alters the absorption of vit B12 by isolating the source if intrinsic factor, the distal stomach, from the alimentary system. However, substantial deficiencies in vit B12 do not occur until at least 1 yet after surgery. Deficiency can result in macrocytic anemia and neuropathy.

Wt loss that occurs after RYGB is commonly accompanied by calcium deficiency and bone loss. Loss of fat absorption is believed to contribute to vit D def b/c it is a fat soluble vitamin. This is believed to incr bone turnover and to decr bone mass. However, calcium deficiency and loss of bone density can occur in the presence of normal vit D and PTH levels. For. This reason, monitoring of bone density and serum levels of calcium, vit D and PTH is recommended. An incr in serum PTH is indicative of neg calcium balance or vit D deficiency. If deficient vit D can be supplemented with ergocalciferol or cholecalciferol.

How well did you know this?
1
Not at all
2
3
4
5
Perfectly
46
Q

You are performing a sphincter preserving, LAR of a rectal ca 2 cm from the anal verge. The proximal site of transection is the jcn of the colon and rectum. To obtain adequate length of proximal colon for reconstruction, you divide the IMA at its origin. After this maneuver, you detect no arterial flow at the level of the bowel transection with intact flow to the remainder of the bowel. Which of the following is the most likely etiology?

A. Inadvertent ligation of the L colic artery and not the IMA
B. SMA occlusion with an intact arc of Riolan
C. Occlusive disease of L iliac artery
D. Incomplete marginal artery of Drummond
E. Aneurysm also dilation of the infrarenal aorta and L common iliac artery

A

D

Division of IMA at its origin from the aorta (high ligation) is often performed to remove the LN basin at risk for mets and provide adequate mobilization of the proximal bowel for a tension free anastomosis. In this case, arterial flow occurs through collateral vessels.

Anatomic variations and postsurgical alterations in arterial anatomy can greatly alter the collateral arterial flow necessary to provide oxygenized blood to the proximal bowel necessary for healing a low colorectal or coloanal anast and prevent leak or stricture. Line of division describes in the questions is at the Sudeck point and the IMA is divided at its origin

Ischemia of the proximal bowel would not occur if the L colic were divided instead of IMA, b/c superior hemorrhoidal branches arising from IMA trunk provide arterial flow. Arc of Riolan is an inconstant artery that connects proximal SMA or 1 of its primary branches to the proximal IMA of 1 of its primary branches. It is classically described as connecting the middle colic branch of the SMA with the L colic branch of the IMA. Forms a short loop that runs close to the root of the mesentery. An important connection between SMA and IMA in the setting of arterial occlusion or significant stenosis such as proximal SMA occlusion. In this event, high ligation of proximal IMA would result in ischemia within the SMA distribution (sm bowel and R colon) as well as the transected distal colon

Marginal artery of Drummond is the anastomoses of the terminal branches of the ileocolic, R colic and middle colic arteries of SMA and of the L colic and sigmoid branches of the IMA. Form a continuous arterial circle or arcade along the inner border of the colon known as the marginal artery of Drummond. Important connection between SMA and IMA and provides collateral flow in the even of occlusion or significant stenosis. Jcn of SMA and IMA territories is at the splenic flexure. Vascular anast here are often weak or absent, hence the marginal artery at this point, known as Griffiths point, is often vocally small or discontinuous. Poor arterial flow across this point is most likely to produce ischemia at the point of transection of the distal colon.

The aorta and its iliac branches are within the systemic arterial circulation; therefore occlusive or aneurysmal disease of these vessels is unlikely to cause ischemia at the point of bowel transection. AAA can occlude the origin of IMA, resulting in collaterals that should be carefully preserved

How well did you know this?
1
Not at all
2
3
4
5
Perfectly
47
Q

Which of the following is the primary tx for nonulcerated, nonnodular Barrett esophagus with high grade dysplasia?

A. Observation
B. Antireflux operation
C. Endoscopic radiofrequency ablation 
D. Photodynamic therapy 
E. Esophagectomy
A

C

Barrett’s is a pathologic change in the squamous epithelium of the esophagus to intestinal metaplasia. Places pts at high risk for esophageal adenoca. Risk of progression of Barrett esophagus to adenoca is dependent on the pathologic grade of its dysplasia. When no dysplasia is present, risk of 0.7% per year. High grade dysplasia carries highest risk, with 15-30% of resected esophagectomy specimens already harbouring carcinoma and a detection rate of approx 14.6% per yrs. This group of pts require intervention to reduce the risk of adenoca

Observation with PPI is accepted mgmt strategy. Requires endoscopy with bx q3-6 months. Strategy does not actually address carcinoma risk, but rather relies on early detection of carcinoma. This strategy is inferior to RFA. Antireflux operations, although good for treating the symptoms of GERD, have not been conclusively demonstrated to reduce the cancer risk. Photodynamic therapy, usually with light sensitizing agen such as porfirmer sodium, has fallen out of favour due to issues related to buried glands (i.e. Islands of Barrett’s under a layer of seemingly normal squamous epithelium) and stricture formation. Esophagectomy was considered an acceptable option b/c of the high rate of occupation carcinoma with high grade dysplasia. However, given the efficacy of RFA, esophagectomy is reserved for pts in whom the high grade dysplasia cannot be completely eradicated or in whom other signs incr the suspicion of carcinoma presence. A randomized, sham controlled trial showed that endoscopic RFA can eradicate Barrett metaplasia and dysplasia and reduce the progression to carcinoma. Now considered the primary tx for Barrett’s esophagus with high grade dysplasia in pts without other signs of carcinoma, such as ulceration or nodularity.

How well did you know this?
1
Not at all
2
3
4
5
Perfectly
48
Q

Which of the following is the most common presentation of jejunal diverticulosis?

A. Perforation
B. Hemorrhage
C. Obstruction
D. Recurrent abdominal pain
E. Wt loss
A

A

Small bowel diverticula are categorized by location and type. Location categories include duodenal and jejunoileal. Type categories include true and false (pulsion) diverticula. Duodenal and Meckel’s diverticula are true diverticula and are the most common, with duodenal diverticula accounting for 15% of all small bowel diverticula. These are generally found in the periampullary region and are asymp, although occasionally can cause biliary obstruction. Diverticula of the jejunum and ileum are pulsion diverticula similar to that of the colon. Although the exact incidence of ileojejunal diverticulosis is unknown, b/c most of these pts are asymp, it is generally considered far less common than colonic diverticulosis. Prevalence of jejunoileal diverticula at approx 2% or less. Jejunum is a more common site of diverticula than ileum. Diverticula occur on the mesenteric side of bowel where the blood vessels enter the bowel from the mesentery. Perforation, bleeding and obstruction are the most common urgent presentations. Perforation accounts for >20% of emergency surgical interventions for jejunoileal diverticula compared with hemorrhage, which accounts for <2%. Recurrent abdo pain is rarely assoc with jejunoileal diverticulosis and wt loss is not likely related to jejunal diverticula.

How well did you know this?
1
Not at all
2
3
4
5
Perfectly
49
Q

In the repair of paraesophageal hernia

A. Primary suture repair has a recurrence rate of 20-30%
B. Biologic meshes have recurrence rates similar to nonabsorbable synthetic meshes
C. Rates of esophageal erosions and strictures by nonabsorbable mesh are less than 1%
D. Symptomatic improvement is superior with mesh compared with primary repair
E. Radiologic hiatal hernia is frequently symptomatic

A

A

Repair of PEH has several areas of controversy. First is whether they should be repaired at all. Repair of symptomatic PEH in pts who are acceptable surgical risk is indicated; however, many surgeons now chooses to observe asymp pts b/c the risk of catastrophic incarceration, and gastric necrosis is rarely the first presenting symptom.

Other areas of controversy are open vs lap repair, need for an esophageal lengthening procedure, and routine use of mesh. Primary suture repair has recurrence rates of 20-30%, although some reports as high as 50% exist. Recurrences are usually identified radiographically, and most are asymp sliding hiatal hernias. Only a minority of radiographically identified recurrences are symptomatic

No difference in rates of symptomatic improvement between primary and mesh repair, as long as complications do not occur. If mesh is used, the next area of controversy is whether a prosthetic mesh or a biologic mesh should be used. Meta analyses comparing biologic with prosthetic meshes have generally shown that prosthetic meshes have a lower recurrence rate. However, there is no standardization as to the type of mesh, its configuration or fixation. In addition, studies have varied as to type and length of follow up. One of the most devastating complications of mesh use, especially prosthetic mesh, is erosion and stricture formation. Although data for the rate of these occurrences are limited, erosions and strictures appear to occur in more than 2% of pts. If this occurs, the only tx is esophagectomy, gastrectomy or some combo of both.

How well did you know this?
1
Not at all
2
3
4
5
Perfectly
50
Q

For colon cancer, which of the following factors is assoc with lower survival rates?

A. Bleeding at presentation
B. Longer duration of symptoms
C. Intestinal obstruction at presentation
D. FamHx of colon cancer
E. Peutz-Jeffers syndrome
A

C

Colon cancer may be dx by routine screening before symptoms develop, but when symptoms are the reason the pt seeks medical attention, those symptoms are usually LGIB, change in bowel habits (incl stool caliber), abdo pain, or obstruction. Bleeding as the presenting symptom is not assoc with worse prognosis, and there is no assoc between duration of symptoms and survival in pts with colon ca.

Patients with intestinal obstruction as their presenting sign of colon cancer form a subgroup that tends to have a short duration of symptoms with a poorer prognosis. Both a famhx of colon cancer and Peutz Jeghers syndrome incr the risk of colon ca with approx 40% of pts with Peutz Jeghers developing colon ca. However, these pts are subjected to incr screening compared with the general population. Thus, their cancer is often detected at an earlier stage, resulting in a good prognosis and improved survival

How well did you know this?
1
Not at all
2
3
4
5
Perfectly
51
Q

Which of the following statements is TRUE about pancreatic adenocarcinoma?

A. Resection of the tumor without curative intent is superior to bypass alone
B. Chemo and radiation improve progression free survival in pts with unresectable cancer
C. In resectable tumors, neoadjuvant chemo improves survival compared with those with primary resection and adjuvant chemo
D. Neoadj chemo and radiation commonly convert unresectable tumors into resectable ones
E. Median survival for an R0 resection is 5 yrs

A

B

Significant more morbidity in pts with panc adenoca undergoing palliative resection compared with those undergoing bypass procedures without a significant incr in the length of survival: 8.2 mos for palliative resection and 6.7 mos for bypass. 2009 meta-analysis of chemo for locally advanced and met panc ca showed improved progression free survival in pts receiving gemcitabine based combo therapy, but that improvement was offset by toxicity in those pts

Survival for pts with unresectable tumors by preop imaging is reported to be essentially the same, whether they were given neoadj chemo or were resected followed by adj chemo. It is uncommon for neoadj chemo and radiation to convert unresectable tumors into resectable ones. In pts with resectable tumors, medial survival is <2 yrs in most reports.

How well did you know this?
1
Not at all
2
3
4
5
Perfectly
52
Q

50M with hep C undergoes a CT scan after screening US suggests a liver mass. A CT shows a 8 cm liver mass. His AFP is 1200. His bili is 10. All other liver function tests are N. Which of the following tx would you recommend?

A. Liver transplant
B. Transarterial Embolization
C. RFA
D. Primary liver resection
E. Interferon alpha
A

D

Pt has an 8 cm solitary isolated focus of HCC in R hepatic lobe and evidence of well compensated cirrhosis. HCC is one of the most freq encountered malignancies worldwide. Degree of underlying cirrhosis directly influences choice of therapy and its efficacy. Surgical resection is reserved for non cirrhotic pts or those with well preserved liver function and relatively limited tumor burden (ideally a single lesion). Bili values >11 and evidence of portal HTN (esophageal varices, ascites, thrombocytopenia, portal vein pressure >10mmHg) are predictors of post op hepatic failure and preclude resection

RFA and alcohol injection are therapies usually reserved for pts who are not primary surgical resection candidates due to # or distn of lesions. Both are most effective for masses <3 cm. TACE involves injection of chemo agent such as doxorubicin or cisplatin, in conjunction with occlusion of hepatic artery supplying the liver lobe containing the tumor. Generally used in pts with sufficiently high tumor burden or more advanced cirrhosis to obviate either resection or ablation. B/c recurrence and progression of malignancy can be very rapid under immunosuppresion, strict pt selection criteria if liver transplant is considered. Milan criteria limit selection to a pt with a single tumor 6.5 cm or smaller or 2-3 lesions 4.5 cm or smaller, totalling 8 cm or less, who shows no evidence of vascular invasion or extrahepatic spread. Ideal transplant candidates may benefit from bridging ablative or TACE while awaiting an organ. Interferon alpha has no role in tx of HCC but is used in mgmt of active hep C.

Despite high risk of multicentric recurrence (70% at 5 yrs), primary resection offers best therapeutic option for this pt, with potential for a 50% survival rate at 5 yrs.

How well did you know this?
1
Not at all
2
3
4
5
Perfectly
53
Q

55F has undergone Whipple for adenoca in the head of the panc. At the time of the OR, the remainder of the panc was noted to be soft, and the primary panc duct measured 3 mm. On POD 6, she resumes oral intake and the output from a peripancreatic drain changes from serous to cloudy. Output over the next 24 hrs is 575 cc and the amylase in the effluent is 28,000 units/mL. Which of the following statements regarding the use of octreotide in the pt is TRUE?

A. Preop octreotide prevents fistula development
B. Octreotide decr fistula output
C. Use of octreotide decr the length of ICU stay
D. Reoperation can be avoided by the use of octreotide
E. Octreotide prevents assoc post op pancreatitis

A

B

Incidence of post op pancreas fistula after Whipple ranges from 10-40%. Several studies have documented small duct size, residual soft panc parenchyma, BMI, poor nutritional status, and malignancy as factors contributing to increased risk of postop fistula development. Although somatostatin analogues such as octreotide do decr fistula output, a cochrane review of 17 RCTs concluded that its use does not affect the development of perioperative pancreatitis, post op mortality, reoperation rates, or the ICU length of stay. Preop use of octreotide has not been shown to prevent fistula development, even in high risk pts.

How well did you know this?
1
Not at all
2
3
4
5
Perfectly
54
Q

Which of the following statements is TRUE regarding the mgmt of a pt with an uncomplicated pilonidal sinus?

A. Excision with primary closure decr the incidence of recurrence
B. Carcinoma in the pilonidal sinus accounts for up to 5% of recurrences
C. Shaving and local hygiene decr the necessity for a surgical procedure
D. Preop IV abx before chronic sinus excision will decr wound complications
E. Excision should be recommended for obese pts, even if asymp

A

C

Pilonidal sinus disease is widely attributed to hair follicle in growth and subsequent fb reaction. Local hair control with shaving or laser epilation, coupled with improved hygiene, has been demonstrated to decr total hospital admission days and the necessity for surgical procedures, even in obese pts.

Pilonidal disease presenting as an abscess should be tx with simple incision and drainage; approx 60% of such pts will heal without further surgical intervention after this initial episode. Formal surgical mgmt should be reserved for chronic or recurrent disease. Preop IV abx before excision of a chronic sinus has not been shown to decr wound complications, improve healing or affect recurrence. Surgical options include complete sinus excision with primary closure or healing by secondary intention, with or without marsupialization of the wound edges. Success of primary closure requires a narrow field of excision and care to avoid sitting in the immediate post op period. Sinus recurrence, however, is more freq after primary closure compared with healing by secondary intention. Carcinoma arising in a pilonidal sinus is rare, accounting for approx 0.1% of pts with chronic untx or recurrent pilonidal disease. These tumors classically present as an ulcer with rapidly progressing, fungating margins, and show an aggressive biology.

How well did you know this?
1
Not at all
2
3
4
5
Perfectly
55
Q

54M present with progressive dysphasia and a 10 lb wt loss. His serum albumin is 2.9 and his serum transferrin is 11. EGD is performed and shows a mass at the GE junction. Bx demonstrates presence of invasive adenoca. Based on staging with esophageal US and CT, pt is scheduled for neoadjuvant chemo and radiation, followed by surgical resection. Which of the following is the preferred form of alimentation for this patient?

A. TPN
B. Enteral nutrition via nasojejunal feeding tube
C. Enteral nutrition via perc endoscopic gastrostomy tube
D. Oral alimentation after placement of an uncovered metal stent
E. Oral alimentation after placement of a covered silicone stent

A

Pts with esophageal ca often present with progressive dysphagia. Along with tumor induced cachexia, this progressive dysphagia can lead to malnutrition, manifested as depressed levels of serum albumin and transferrin on lab testing. Predicts incr risk of morbidity and mortality at the time of esophageal resection.

Neoadjuvant chemorads presents an additional burden on these pts, further compromising their nutritional parameters. Radiation induced esophagitis develops in 15-28% of tx pts, further worsening dysphagia. Side effects of 5 FU and cisplatin, the most common chemo regimen include N/V and diarrhea. Malnutrition reduces the response of the tumor to chemorads and impairs the pt’s ability to tolerate full course of tx. Nutritional deficiencies may also contribute to the trend of incr period morbidity and mortality among esophageal ca patients receiving neoadj therapy compared with pts undergoing esophageal resection alone. For esophageal ca pts with dysphagia receiving neoadj therapy, relief of symptoms, maintenance of nutrition and minimization of periop complications are important tx goals.

Correction of malnutrition before sx can markedly reduce the morbidity and mortality of esophageal resection. Traditional options for nutritional supplementation during weeks to allow for wound healing and resolution of local inflam and contamination at insertion site. Avoid PEG b/c of potential to render stomach unusable as replacement conduit for the esophagus. NJ tubes freq become dislodged, resulting in an interruption in nutrition support or more serious complications, including aspiration pneumonia. Complications of feeding tube placement can be devastating in esophageal ca pts b/c of their malnourished state and ongoing malignancy. In addition, use of enteral tube feeding does nothing to address dysphagia, typically the dominant symptom.

Preop esophageal stenting has emerged as an alternative approach to promote nutritional repletion in pts undergoing neoadj chemorads before esophageal resection. Options include uncovered bare metal stents or covered silicone stents. Uncovered metal stents are best suited for palliation of pts with inoperable esophageal cancer, b/c they become incorporated into the wall of the esophagus and are not removable. Covered silicone stents offer more effective relief of dysphagia and can be removed later, either endoscopically or at the time of surgery.

Compared with enteral feeding, oral alimentation after placement of a covered silicone stent results in better relief of dysphagia, higher performance status, better tolerance of chemorads and better mean improvement in albumin. Reduces incidence of major operative complications by >50% compared with enteral feeds. Incidences of stent related complications are <5%. No difficulties with stent removal or intraop dissection at time of sx are reported. Oral alimentation after placement of a covered silicone stent is a safe and effective means to relieve dysphagia and promote nutritional repletion in pts undergoing neoadj chemorads followed by surgical resection for esophageal ca

How well did you know this?
1
Not at all
2
3
4
5
Perfectly
56
Q

73M with hx of cirrhosis and ascites presents 48 hits after the acute onset of chest and upper abdo pain after an episode of vomiting. CXR and gastrograffin swallow are performed and show a leak of contrast into the L chest. Which of the following is the next best step in the mgmt of this pt?

A. L tube thoracostomy and covered stent placement to distal esophagus
B. L thoracotomy and direct esophageal repair
C. L tube thoracostomy and bare metal stent placement to the distal eosphagus
D. Exclusion of the esophagus in the neck and at the GE jcn and creation of an end esophagocutaneous fistula in the neck
E. Creation of a side esophagocutaneous fistula in the neck and placement of PEG

A

A

Acute onset of chest and upper abdo pain after an episode of vomiting suggests a spontaneous perf of esophagus. Spont perf of esophagus known as Boerhaave syndrome, is a full thickness longitudinal tear in the esophageal wall as a result of vomiting. Perf typically occurs in the intrathoracic esophagus just above the GE jcn. Significant delay in dx and continued PO intake result in mediastinal contamination are common. Of the factors that critically influence prognosis in these pts, including pre existing comorbidities and esophageal disease, the most significant is delay in dx

Successful mgmt of an esophageal leak requires prompt control of sepsis and elimination of ongoing contamination of the mediastinum. Traditional therapy had been urgent operative repair, including primary surgical repair when possible, exclusion and diversion or esophagectomy. Despite advances in critical care, antimicrobial therapy, diagnostic imaging, and surgical technique, spontaneous esophegeal perf continue to be assoc with excessive rates of morbidity and mortality. When tx is performed within the first 24 hrs, mortality rates are between 16 and 24%. When tx is delayed and performed after >24 hrs, mortality rates incr rapidly up to 50%. B/c of these high mortality rates, some surgeons have advocated nonop mgmt, with cessation of PO intake and iv abx until healing is confirmed. Approach is also assoc with long hospital stay in many pts and it often requires enteral or parenteral nutrition support.

With the improvement in endoscopic techniques and ICU mgmt, additional of endoluminal stent placement to nonop mgmt has emerged as a feasible and safe alternative for the tx. Stent placement closes the site of perf, stops ongoing contamination of the mediastinum, preserves esophagogastric continuity, avoids potential morbidity and mortality of open sx and allows for earlier oral intake and a decr in hosp stay. Plastic stents are mainly using for the tx of leaks and strictures of benign disease b/c of their ability to be removed more easilty with less damage to the esophageal wall. To ensure complete occlusion, it is important to allow adequate stent coverage on either side of the leak and the stent length chosen should be at least 3-4 cm longer than the leak. Easily performed under direct endoscopic control with an occlusion rate of 94%. Stent migration occurs in <5% of cases. Stent removal is typically 4-6 wks after implantation

Pt’s delayed presentation and hx of cirrhosis and ascites markedly incr the risk of surgical intervention to repair the site of perf. At 48 hurts after spontaneous perf, both direct esophageal repair and esophageal exclusion are assoc with mortality rates approaching 50%. Creation of a side esophagocutaneous fistula in the neck and placement of a PEG tube does not close the site of perf and allows ongoing contamination of the mediastinum. Under these circumstances, a less invasive approach that seals the site of perf is a safer option. Placement of a chest tube will drain infected material from the involved hemithorax.

How well did you know this?
1
Not at all
2
3
4
5
Perfectly
57
Q

38 y.o. Black F is seen in ER with 4 days of dizziness, SOB, and melena. She has no abdo pain or significant findings on exam. Hb on adm is 72 and she is FOBT positive. After transfusion, she undergoes upper endoscopy, which shows a 5 cm submucosal mass in the body of the stomach along the greater curve. There is an adherent clot of over the mass. CT abdo confirms the presence of a gastric mass without any other abN. Which of the following is the next best step in the mgmt of this pt?

A. EMR
B. Subtotal gastrectomy with lymphadenitis you
C. Imatinib
D. Gastric wedge resection of mass
E. Epirubicin, cisplatin, and 5 FU
A

D

GISTs are the most common mesentery all tumors of the GI tract. Most are found incidentally at the time of endoscopy or radiologic imaging. Symptoms may include abdo pain, GI bleed, or obstruction, depending on the size and location of the tumor. Most commonly found in stomach (60%) and small bowel (35%). GISTs originate from interstitial cells of cajal which express cd117 (c-kit), present in 95% of tumors.

Complete surgical resection to grossly neg margins is tx of choice. Lymphadenectomy is not required for GISTs b/c these tumors spread hematogenously. Lap vs open depending on tumor size, location and extent of invasion. EMR suggested for smaller tumors.

Resection offers potential for cure, disease recurs in 40-90% of surgically tx pts. Prognostic indicators include tumor size, mitotic index, tumor location and tumor rupture during sx. Adjuvant therapy with imatinib, which targets the tyrosine kinase c kit receptor, is recommended for intermediate to high risk GISTs and may have a role in cytoreduction and organ preserving sx but it is not indicated. Epirubicin, cisplatin, and 5FU are the chemo agents used for gastric adenoca.

How well did you know this?
1
Not at all
2
3
4
5
Perfectly
58
Q

Which of the following preop factors are predictive of symptomatic relief in pts with achalasia undergoing lap esophagomyotomy?

A. Prior response to injection of Botox
B. Megaesophagus or sigmoid esophagus
C. Resting lower esophageal sphincter pressure >30 mmHg
D. Prev tx with pneumatic dilation
E. Presence of lengthy, high amplitude esophageal contractions on manometry

A

C

Achalasia or impaired relaxation of LES, is the most common d/o of esophageal dysmotility. Symptoms incl dysphagia and CP. Dx eval includes barium swallow, endoscopy and manometry. Classic radiologic findings incl a dilated esophagus with narrowing and the GE jcn (bird’s beak). Endoscopy is used to exclude malignancy, b/c pt with achalasia are at incr risk of both SCC and adenoca of the esophagus. Manometric findings of achalasia incl elevated resting LES pressure and a peristalsis of the body with secondary and tertiary contractions.

Tx options include medical mgmt, Botox, pneumatic dilation and surgical myotomy. Medical mgmt consists of CCB and nitrates, which are reserved primarily for pts who are not candidates for more invasive therapies. Effects of these meds are short lived with variable symptom relief and significant side effects. Botox inhibits the release of acetylcholine, resulting in reduction in LES pressure. Efficacious in reducing CP and dysphagia in pts with achalasia; however its effects are time limited with relapse of symptoms in >50% after 3 months. Repeated injections in initial responders may confer some long term benefit up to 2 yrs. Endoscopic pneumatic diln is considered a good 1st line option. Compared with Botox, pneumatic diln has much higher long term response rates, with low complication rates. Age and successful disruption of LES fibres, as measured manometrically, are predictors of symptom relief with pneumatic diln.

Surgical myotomy results in complete disruption of the LES fibres from the stomach to the esophagus, thus reducing resting LES pressure. A partial funds may be performed with the myotomy, depending on the presence of reflux symptoms preop. Most consistent preop predictors of successful surgical myotomy is elevated resting LES pressures >30 mmHg. Failure of surgical myotomy is assoc with megaesophagus and lengthy, high amplitude esophageal contractions. Prior pneumatic dilation and tx with Botox have not been shown to significantly affected surgical outcomes in pts with achalasia

How well did you know this?
1
Not at all
2
3
4
5
Perfectly
59
Q

87M presents to ER with N/V for 2 days. CT shows air in gb, air fluid levels in the small intestines and TP in the distal small intestines. Operative mgmt will require which of the following?

A. Cholecystostomy
B. Stricturoplasty
C. Enteroscopy
D. Enterotomy
E. Cholecystectomy
A

D

Biliary enteric fistula (gallstone ileus) is a recognized complication of cholelithiasis. Presentation may be intestinal obstruction when the gallstone lodges in the distal ileum. Classic radiographic findings comprise a triad of gastric or small bowel diln with pneumobiliar and intraintestinal gallstone on CT.

B/c the cholecystenteric fistula is usually large, recurrent symptoms are rare and choelcystostomy and fistula mgmt are not mandatory. Abdo exploration will involve an enterotomy proximal to the site of obstruction to remove the stone. Operative intervention should include manual exam of the entire small intestine, b/c a second stone may be present. Drainage of gab has occurred by means of the fistula, thus cholecystostomy is unnecessary. Enteroscopy is not recommended in this setting.

How well did you know this?
1
Not at all
2
3
4
5
Perfectly
60
Q

An 18M undergoes a CT scan to assess for injury after a MVC. Although no acute injuries are identified, a 3 cm fusiform CBD is noted. Which of the following would you recommend as the next step?

A. Repeat CT in 1 year
B. Ca 19-9 
C. ERCP with brushing and stent
D. MRCP 
E. EUS with FNA
A

D

Choledochal cyst disease may present as diln in a variety of locations within the biliary tree.

Type I fusiform dilation of extrahepatic duct
Type II DIverticulum of the extrahepatic duct
Type III Intraduodenal (choledochocele proper)
Type IVa Intra and extra hep diln of the bile duct
Type IVb Multiple diln of extrahepatic bile duct
Type V Multiple intrahepatic biliary cysts (Caroli disease)

Incidence of carcinoma in bile duct cysts is estimated to be 2.5-15%, compared with an incidence of 0.012-0.48% in pts without bile duct cysts. Classical triad of jaundice, RUQ pain, and abdo mass, occurs in <20% of all pts with biliary cysts. Choledochal cysts should be surgically resected when possible to avoid long term consequences of cholangitis, liver cirrhosis, pancreatitis and malignant transformation

MRCP provides a noninvasive test to image the biliary tree and assess feasibility of surgical resection. Such imaging should supersede ERCP, brushings, tumor markers or FNA

How well did you know this?
1
Not at all
2
3
4
5
Perfectly
61
Q

Compared with standard open hemorrhoidectomy, stapled hemorrhoidectomy is assoc with which of following?

A. Lower rate of infection
B. Lower recurrence rate
C. Improved patient satisfaction
D. Delayed return to normal activities 
E. Less post op pain
A

E

Stapling device for hemorrhoidectomy has less pain at the cost of higher recurrence rates. Comparison trials have shown that patient satisfaction is similar between procedures–possible due to patient self selection, that is, one is often likely to be happy with the chosen option. Less pain results in a quicker return to N activities in controlled studies. Infection rates are similar.

Conventional hemorrhoidectomy may be used more often in pts with prominent external skin tags, and this is the source of the incr pain scores. Randomization of subjects removes this selection bias but pts in the conventional hemorrhoidectomy group may suffer the pain of skin tag removal. With these differences between stapled and open hemorrhoidectomy, it is clear that patient selection and a good preop discussion of options is relevant

How well did you know this?
1
Not at all
2
3
4
5
Perfectly
62
Q

One yr after a lap RYGB, a 42F presents with a bowel obstruction. Her original OR indicates that her gastric bypass was performed in a retrocolic antegastric fashion. Which of the following is the most likely cause of her symptoms?

A. Obstruction secondary to adhesions
B. Port site hernia
C. Internal hernia
D. Stricture of gastrojejunostomy
E. Stricture of jejunojejunostomy
A

C

Adhesive SBO is less common after LRYGB than open, the overall rate of bowel obstruction remains approx equal between 2 approaches and is possibly even higher with lap. Due to incr incidence of bowel obstruction caused by internal hernia, which occurs at a rate of approx 2-5% after LRYGB.

3 potential spaces of internal herniation are created during the procedure. First is Peterson defect between the Roux limb mesentery and the mesocolon. Second is jejunojejunostomy meseteric defect. Third occurs only in retrocolic RYGB.

Overall incidence of SBO in retrocolic cohort is 5.1%. Internal hernia causes half of these bowel obstructions and approx half were adhesive disease, scarring at the mesocolic space, port site hernia and jejunojejunostomy stenosis. 3/4 of internal hernias were at the defect in the transverse mesocolon, a space that is not created in the anetocolic technique. In antecolic LRYGB, overall incidence of SBO was 1.7% and obstructions were caused by stenosis at the jejunojejunostomy, adhesive disease, internal hernia and port site hernia, in desc order of freq.

When evaluation a pt with bowel obstruction after LRYG, important to note the route of Roux limb and timing of the obstruction. Early obstruction (within 30 days) are more likely caused by technical error causing obstruction at the jejunojejunostomy. Late obstruction is more likely due to internal hernia or adhesive disease. Internal hernia is thought to be a late complication b/c the pt’s wt loss causes a decr in intraperitoneal fat and subsequent enlarging of mesenteric defects.

Dx can be difficult. Valuable diagnostic tool is lap reexploration and may SBO after LRYGB can be tx laparoscopically.

Stricture at gastrojej causes a GOO rather than SBO. Pts are intolerant of even small amts of food and generally do not have bloating and abdo distension.

How well did you know this?
1
Not at all
2
3
4
5
Perfectly
63
Q

Which of the following is TRUE regarding the mgmt of sigmoid diverticulitis?

A. Patients with perforated diverticulitis require sigmoidectomy and end colostomy (Hartmann procedure)
B. Likelihood of needing emergency surgical therapy is not affected by the number of prev episodes of uncomplicated diverticulitis
C. Colonoscopy is not recommended in pts <40 yrs after resolution of uncomplicated diverticulitis
D. Elective diverticular disease resection is assoc with lower rates of morbidity and mortality than elective CRC resection
E. In men <40 yrs, an episode of uncomplicated diverticulitis warrants elective sigmoidectomy

A

B

Hinchey I Pericolic or mesenteric abscess
Hinchey II Walled off Pelvic abscess
Hinchey III Generalized purulent peritonitis
Hinchey IV Generalized feculent peritonitis

Tx of sigmoid diverticulitis continues to evolve. Significant morbidity and mortality assoc with Hartmann’s and subsequent colostomy take down. This OR is commonly considered for Hinchey III or IV. However in clinically stable pts with favourable anatomy, it is possible and perhaps better to perform resection with colorectal anastomosis with or without diverting loop ileostomy. Subsequent ileostomy takedown is easier. Other approaches for Hinchey III and IV include damage control sigmoid resection with a delayed anastomosis as well as irrigation and drainage alone. Thus, all pts do not need a Hartmann.

Applying elective resection after a first attack of uncomplicated diverticulitis has little effect on the incidence of pts requiring emergency procedures, b/c the type of presentation tends to be similar to their first attack and not necessarily more sever. The ASCRS now recommends “the number of attacks of uncomplicated diverticulitis is not necessarily an overriding factor in defining the appropriateness of sx”. Studies based on decision analysis models suggest that life expectancy will be optimized if elective sx is performed after the 3rd or 4th attack of uncomplicated diverticulitis. This finding is particularly relevant b/c elective diverticular disease resection has a significant rate of morbidity and mortality–higher than that of elective CRC resection

Colonoscopy is recommended after nonop tx and before operative tx of sigmoid diverticulitis in pts of all ages.

How well did you know this?
1
Not at all
2
3
4
5
Perfectly
64
Q

US of RUQ incidentally reveals a 5 mm polyp in the gallbladder. Mgmt consists of which of the folllowing?

A. Cholecystectomy with excision of gb bed
B. Repeat US in 6 months
C. CT scan and EUS
D. An extended cholecystectomy with LN dissection
E. Lap Chole

A

B

Polyps of go are typically incidental findings detected during radiologic imaging of the abdo. Significance is related to their potential for malignancy.

Polyps <0.5 cm are usually benign and most freq represent cholesterolosis. Asymp pts with cholesterol polyps do not need tx. However, a repeat US exam at 6 and 12 months may be appropriate. FU exam are not necessary if polyp is unchanged.

Polyps at least 1 cm in diameter may represent cholesterol polyps, adenomas, or carcinomas. Multiple polyps, pedunculated polyps, and those that are hyperechoic compared with the liver are usually cholesterol polyps, whereas solitary and sessile polyps are isoechoic with the liver are more likely to be neoplasticism and a lap chole should be performed

Lesions >1.8 cm are usually malignant. B/c these lesions may represent advanced cancer, pts should undergo preop staging with CT scan and EUS. If malignancy is proven, an extended cholecystectomy with LN dissection and partial hepatic resection of the gb bed is required.

How well did you know this?
1
Not at all
2
3
4
5
Perfectly
65
Q

Which of the following statements is TRUE regarding Meckel diverticulum?

A. Most common neoplasm in a Meckel diverticulum is adenocarcinoma
B. It is a true diverticulum
C. A Grynfellt-Lesshaft hernia contains a Meckel diverticulum
D. Stapled diverticulectomy is adequate tx for a bleeding Meckel diverticulum
E. Heterotophic mucosa is best detected by capsule endoscopy

A

B

Meckel diverticulum is a true diverticulum, containing all 3 layers of the intestinal wall. Neoplasms within Meckel are exceedingly rare, but of such neoplasms, carcinoid is most common. In a population based study 77% were carcinoid.

A hernia containing a Meckels is a Littre hernia. These hernias can be inguinal (50% in 1 series), umbilical or femoral. A grynfeltt-lesshaft hernia refers to a hernia of abdo contents through the superior lumbar triangle (formed by quadratus lumborum, 12th rib and internal oblique).

Acid producing heterotrophic gastric tissue within a Meckel’s can result in ulceration of adjacent N small bowel mucosa, resulting in GI bleed. An adequate resection needs to include both the diverticulum and the ulcerated intestinal mucosa. A diverticulectomy alone is insufficienct mgmt of a bleeding Meckel diverticulum if it does not resect the ulcerated intestinal mucosa.

In the elective setting, a technetium 99 pertechnetate scan is the study of choice to detect heterotrophic tissue within a Meckel’s. Capsule endscopy is often used in the WU of small intestinal bleeds but it is not the preferred study if a Meckels is suspected

How well did you know this?
1
Not at all
2
3
4
5
Perfectly
66
Q

Which of the following statements is TRUE regarding surgical tx of diverticular disease?

A. A single episode of uncomplicated sigmoid diverticulitis mandates resection
B. Two separate episodes of uncomplicated sigmoid diverticulitis mandate resection
C. Surgical resection should include all areas of diverticulosis
D. The distal resection margin should be at the peritoneal reflection
E. The proximal resection margin should be located in an area without hypertrophy of the muscularis propria

A

E

Controversy remains regarding the need for elective colectomy in pts who are successfully managed nonop during an acute diverticulitis episode. After a single episode of uncomplicated sigmoid diverticulitis, risk for recurrent diverticulitis is low; however, with the second episode, the risk for a third episode becomes substantial. Data indicate that although the risk of subsequent episodes of acute diverticulitis is high, risk of complicated diverticulitis remains low. IT is currently recommended that the decision to proceed should be made on a case by case basis and that the number of attacks of uncomplicated diverticulitis is not necessarily an overriding factor in determining the appropriateness of sx.

For pts who experience diverticulitis complicated by abscess formation requiring perc drain, sigmoid resection is recommended even if otherwise full recovery from the episode of diverticulitis with conservative mgmt is achieved.

When performing elective sigmoid resection for diverticular diseases, not all diverticula bearing colon must be removed. Distal margin of resection should extend to where the taenia coli diverge into the upper rectum, b/c diverticulum formation below this level is very uncommon. Proximal margin should be an area of pliable colon without hypertrophy or inflammation.

How well did you know this?
1
Not at all
2
3
4
5
Perfectly
67
Q

Pancreatic anastomotic lead after Whipple

A. Occurs in <5% of cases
B. Typically gives rise to chronic pancreatic fistula
C. Is best tx with early operative intervention
D. Does not resolve quicker with octreotide
E. Occurs less often with pancreaticogastrostomy

A

D

Anast leaks after Whipple are a significant source of morbidity and mortality from this procedure. Anast leaks occur in 15-25% of cases. Fortunately, most leaks heal with conservative mgmt and do not give risk to chronic panc fistulas. When a chronic fistula does occur, early operative intervention should be avoided, b/c most of these fistulas will close spontaneously with observation. Although the use of octreotide can decr the output from a panc fistula, octreotide therapy dose not aid in the healing of a fistula. Meta analysis of RCTs has not shown a significant difference between pancreaticojejunostomy and pancreaticogastrostomy with regard to panc anast leak or fistula formation

How well did you know this?
1
Not at all
2
3
4
5
Perfectly
68
Q

57F recently found to have an invasive adenoca of the cecum. Her preop work up should include a PET CT under which of the following circumstances?

A. Routinely
B. When initial CT abdo fails to show met disease
C. When initial CT abdo shows a single met to the R lobe of the liver
D. When initial CT abdo shows >5 met lesions to the liver
E. When the patient has UC

A

C

Preop work up should include full inspection of colonic mucosa, preferably by colonoscopy, CBC, chem, CEA and CT scan of Chest/Abdo/Pelvis.

PET CT is not routinely indicated as part of preop work up. If CT scan demonstrates potentially curable met disease, further evaluation with PET-CT is warranted. Under such circumstances, purpose is to evaluate unrecognized mets that would prevent possibility of surgical cure. Pts with clearly unresectable mets (>5 met lesions in liver) should not have a baseline PET CT b/c the results will not affect the clinical mgmt. Should not be used to assess the response to chemo, b.c the scans can provide a transient falsely neg result after the use of chemo. False positive PET can occur in presence of tissue inflam such as in active UC

How well did you know this?
1
Not at all
2
3
4
5
Perfectly
69
Q

Rectoanal inhibitory reflex

A. Is elicited by distension of the distal rectum
B. Results in relaxation of the external anal sphincter
C. Is accentuated in Hirschsprung disease
D. Is best measured with perineal electromyography
E. Is absent in pts with pelvic floor dysmotility

A

A

Rectal inhibitory reflex (RAIR) is normal reflex that occurs in response to distension of the distal rectum. With the RAIR, there is a relaxation of the internal anal sphincter. RAIR allows rectal contents to be “sampled” by the sensory area of the proximal anal canal, thus providing a means to determine stool from gas.

RAIR is absent in Hirschsprungs, b/c the aganglionic segment prevents the relaxation of the internal sphincter. Patients with other forms of chronic constipation, including colonic inertia and pelvic floor dysfunction, will typically have a detectable RAIR. The RAIR is best measured with anal manometry. RAIR does not result in relaxation of the external anal sphincter. In fact, transient contraction of the external sphincter is often seen with the reflex

How well did you know this?
1
Not at all
2
3
4
5
Perfectly
70
Q

Hirschsprung disease in adults

A. Typically occurs in pts >40 yrs old
B. Usually involves > 10 cm of distal rectum
C. Results from an aperistaltic proximal colonic segment
D. Is best tx with mechanical anal dilation
E. Is assoc with the absence of the rectoanal inhibitory reflex

A

E

Hirschsprung is a functional obstruction that results from aperistalsis due to congenital loss of ganglion cells within the distal colon and rectum. Occurs in 1 in 5000 births, with the majority being dx and tx in neonatal period. Rare cases, remain undx in adulthood. Adult Hirschsprung is same entity as in peds, differing only in its degree of severity and timing of dx. Clinical course of adults is characterized by chronic debilitating constipation since birth. Most pts are dx before age 40. Oldest person dx was 69 yrs. Almost always involves a short segment of distal rectum and thus gives rise to less severe symptoms. Dx is suggested by abscence of rectoanal inhibitory reflex as measured by anal manometry. RAIR is a normal reflexive response where distension of the distal rectum results in decr in internal anal sphincter tone. Appropriate tx consists of proctectomy and removal of aganglionic segment and anal mucosectomy with coloanal anastomosis. Anorectal myomectomy provides varying success. Simple anal dilation is not likely to affect the clinical course of Hirschsprung disease

How well did you know this?
1
Not at all
2
3
4
5
Perfectly
71
Q

45M has a 5 yr hx of idiopathic UC. Although the last few yrs he has been asymptomatic from his colitis, he was recently dx with early PSC. Which of the following mgmts should this pt undergo?

A. Total proctocolectomy with ileal pouch anastomosis to prevent progression of PSC
B. Annual screening colonoscopy to assess for presence of dysplasia beginning at 10 yrs from dx of UC
C. Annual screening colonoscopy to assess for presence of dysplasia beginning at 5 yrs after the dx of PSC
D. Annual screening colonoscopy to assess for the presence of dysplasia begnning now
E. Screening colonoscopy beginning at the age of 55

A

D

PSC is characterized by inflam and fibrosis of the intrahepatic and extrahepatic bile ducts. PSC is immune mediated progressive disorder than can progress to the development of cirrhosis, portal HTN, and hepatic decompensation. Occurs in ~5% of pts with UC. IN a majority of such cases, dx of UC precedes dx of PSC by several yrs. Colectomy for UC does not appear to improve or prevent PSC. In fact, PSC may develop several yrs after colectomy. Patients with idiopathic UC are at an incr risk for development of CRC. Risk of cancer is related to the duration of colitis. B/c of this risk, all pts suffering from UC should undergo annual surveillance colonoscopy, with multiple bx beginning 8-10 yrs after dx of colitis. Pts who suffer from both UC and PSC are known to be at incr risk for development of dysplasia and CRC compared with other pts with UC. Given this incr risk, UC pts with PSC should undergo annual surveillance colonoscopies beginning at the time of dx of PSC.

How well did you know this?
1
Not at all
2
3
4
5
Perfectly
72
Q

Which of the following statements about cholecystectomy after endoscopic clearance of choledocholithiasis is TRUE?

A. Ppx cholecystectomy reduces mortality
B. Deferring chole does not incr the risk of cholangitis
C. Deferring chole does not incr the risk of biliary tract symptoms
D. Conversion rate to an open operation is unchanged if chole is deferred
E. Pancreatitis is common in pts who defer chole

A

A

Cochrane analysis of RCTs found that early chole after endoscopic choledocholithotomy decr the risk of mortality by 78% compared with wait and see approach. In ASA IV or V groups, mortality was decr from 13% in wait and see to 7% in early chole. Significant decr in incidence of biliary pain and cholangitis. In pts managed by wait and see, 5.4% developed recurrent jaundice or cholangitis, but only 0.9% developed pancreatitis. In a prospective RCT, 47% managed nonop developed greater than 1 biliary related event during 2 yrs of follow up compared with 2% of pts who underwent lap chole after initial endoscopic choledocholithotomies. Significant incr in conversion to open in wait and see pts who underwent choles after symptomatic recurrence (55%) compared with early chole pts (20%). Early removal of gb after clinical presentation with choledocholithiasis decr risk of death and complications and this improvement is seen even in pts at high operative risk

How well did you know this?
1
Not at all
2
3
4
5
Perfectly
73
Q

Compared with urgent operation for acute, left sided colonic obstruction due to a potentially curable colon cancer, use of self expanding metal stents as a bridge to elective operation has

A. Higher medical complications
B. Lower hospital length of stay
C. Lower risk for stoma formation
D. Worse long term oncologic outcome
E. Higher mortality rate
A

C

~15% of pts with CRC present with obstruction. Mgmt must include decompression to avoid subsequent perf. Accomplished by either a palliative stoma or Hartmann procedure in 25% of pts. However in up to 40% of Hartmann’s are never reversed. Morbidity of operation for acute obstructing CRC is high and mortality rates are 9-27%. Self expandable metal stents are advocated as an alternative to operation. For pts with mets, self expandable metal stents may allow palliation without operation or stoma formation. For pts with potentially curable CRC, these stents may serve as bridge to sx by allowing for decompression and subsequent 1 stage surgery without stoma formation.

Approx 50-60% of pts with acute obstructing CRC are candidates for self expandable metal stents. Technical success rate, defined as the ability to deploy stent and relieve obstruction is high (88-100%). Results in a shortened hosp stay by up to 5-8 days, as well as decr need for ICU admission. With relief of obstruction, need for emergency OR and stoma formation at any point in tx is reduced. Use of stents as a bridge to subsequent resection in colon ca is not assoc with worse oncologic outcome or higher mortality, although better long term data are needed.

Patients with short segment obstruction and distal obstructions are the best candidates for self expandable metal stents. Overall complication rate is ~20% with migration betting the most common problem (10%). Perf from stent placement occurs in up to 4%. Most important risk factor for perf is the use of balloon dilation

How well did you know this?
1
Not at all
2
3
4
5
Perfectly
74
Q

2 weeks after a ventral hernia repair, a 30F returns to ER with a 24hr hx of nausea and intractable bilious vomiting. Her hx is remarkable only for remote antrectomy and RYGB for GIST. A CT is obtained and shows intussusception of small bowel in L side of abdo. Which of the following is the next step in her management?

A. NG decompression and medical mgmt
B. Barium upper GI study
C. EGD
D. Exp Lap and small bowel reduction
E. Exp Lap and small bowel resection
A

E

Intussusception is an uncommon cause of adult bowel obstruction (<5%). Ped intussusception is benign and idiopathic in 80% of pts anc can be tx with reduction (air or barium). However, adult intussusception has a pathologic lead in up to 90% of pts. Approx 65% of colonic intusussception has a malignant lead point. In the small bowel, up to 30% of these lead points are malignant. Benign causes include polyps, Meckels, strictures or benign neoplasms. Thus, up to 90% of adult intussusception cases require definitive tx with surgical resection. In select cases, when a benign etiology is firmly established, the intusussception may be milked out to limit the extent of bowel resection.

CT Abdo is considered to be the best diagnostic imaging modality, with an accuracy of 60-100%. Classic finding is the “target sign” which is caused by bowel within bowel. Mesenteric vessels within a bowel lumen may be seen as well.

In this pt, CT has established the dx, and given her past hx of GIST, she is very likely to have a pathologic lead point. Thus, ongoing medical mgmt is not appropriate.

How well did you know this?
1
Not at all
2
3
4
5
Perfectly
75
Q

74F presents to ER for evaluation of a 2 day hx of epigastric abdo pain and bloating. During the course of this eval, her pain has become more severe. She is tender and tympanic in the upper abdo but has no guarding. Her WBC and amylase are normal. A CT Abdo shows dilated colon pointing to LUQ. Which of the following is the most appropriate next step in her mgmt?

A. Hydrostatic barium enema
B. Colonoscopy
C. Operative detorsion
D. R hemi
E. Cecopexy
A

D

Patient has cecal volvulus. Volvulus occurs when a large, mobile loop of intestin and its mesentery twist on a fixed point. This torsion leads to a closed loop obstruction with bowel distension. Depending on degree and duration of torsion, ischemia, gangrene and perf can occur. Volvulus occurs most commonly in sigmoid colon, followed by cecum

Cecal volvulus occurs in 2 types: axial ileocolic volvulus (90%) and cecal bascule (10%). In axial ileocolic volvulus, cecum and TI rotate up and over to the LUQ. Cecal bascule occurs when cecum flips upward and anterior in a horizontal plane. Both types require a highly mobile cecum, which is though to occur from failure of mesentery to fuse to the posterior parietal peritoneum in the R parabolic gutter.

Preferred tx of cecal volvulus is operative, which is generally a R hemi. Non viable bowel should be resected without detorsion, b/c this may lead to septic shock. Majority of pts can be reanastomosed after resection even with gangrene and obstruction. however, ileostomy with or without a mucus fistula, remains an option if there is peritonitis or severe bowel distension.

Radiologic guided hydrostatic enema with a water soluble contrast can be used to reduce sigmoid volvulus but is less successful in cecal volvulus. In addition, barium would not be used in this pt b/c of risk of perf. Similarly, colonoscopy can decompress a sigmoid volvulus but has a high failure rate for cecal volvulus (put to 70%). Detorsion of volvulus accompanied with fixation, either cecopexy or cecostomy, is assoc with high complication and failure rates

How well did you know this?
1
Not at all
2
3
4
5
Perfectly
76
Q

Which of the following is TRUE regarding the possible risk factors for post laparotomy adhesive SBO

A. Rates of SBO after open appy are higher than after lap appy
B. Separate closure of peritoneum decr adhesion formation
C. Open adnexal operations have the highest rate of adhesion related admission
D. Age and gender are strong predictors of SBO
E. Presence of cancer incr postop SBO

A

C

Postop adhesions are freq after abdo and pelvic sx and occur in 50-95% of pts who undergo subsequent laparotomy. Adhesion related SBO occurs in nearly 5% of pts who have undergone prior abdo or, of which 3-8% required operative intervention. Although risk factors are difficult to identify, type of sx and method of operation play important roles in development of adhesive SBO. Open chole and hyst are assoc with higher rates of SBO compared with laparoscopy; however, with appy, the rates are similar. Open adnexal OR have the highest rate of adhesion related readmission (23%), mostly due to SBO, followed by ileal pouch anal anast (19%), TAH (15%) and colectomy (9%). Age, gender and presence of cancer do not appear to affect post op adhesion formation, readmission, or SBO. Closure of peritoneum as a separate layer appears to incr adhesion related readmission and SBO

How well did you know this?
1
Not at all
2
3
4
5
Perfectly
77
Q

Which of the following choices regarding the mgmt of pts with nonvariceal UGIB is TRUE?

A. Pts with high risk stigmata of recent hemorrhage can be safely managed as an outpt after endoscopic hemostasis
B. Percutaneous or transcatheter embolization should not be used as an alternative to surgery
C. Upper endoscopy should be delayed until INR is normalized
D. A high dose IV PPI reduces rebleeding and the need for surgery
E. Second look endoscopy is required before d/c

A

D

UGIB has an incidence of 1/1000 pts and has a substantial mortality of approx 10%. Initial mgmt is focused on ABCs, correction of coagulopathy and risk assessment, followed by early endoscopy (within 24 hrs of presentation) focused on hemostasis. Upper endoscopy should not be delayed until correction of coagulopathy.

At upper scope, hemostatic therapy (clips, thermocoagulation, or sclerosant injection) is indicated for all lesions with high stigmata of recent bleeding (active bleeding or visible vessel in an ulcer bed). Epi injection alone is not sufficient. Second look endoscopy not recommended unless bleeding recurs. If upper endoscopy reveals findings of low risk stigmata (clean based ulcer or clot in an ulcer bed), hemostatic therapy is not indicated and pts can be fed within 24 hrs and d/c’ed early with oral PPI. Pts deemed as high risk should be hospitalized for >72 hrs and should received IV PPI. Percutaneous embolization or surgery can be considered when endoscopic therapy has failed.

How well did you know this?
1
Not at all
2
3
4
5
Perfectly
78
Q

Which of the following statements about colon cancer in pts with UC is true?

A. Cancer risk incidence exceeds 50% at 30 yrs after dx
B. Use of 5 ASA decr incidence of cancer
C. Complete mucosectomy significantly reduces a the incidence of cancer
D. Colon cancer is more common in adult onset UC than in childhood onset UC
E. Risk of cancer is unaffected by extent of disease

A

B

Incidence of cancer in pts with UC corresponds to cumulative probabilities of 2% by 10 yrs, 8% by 20 yrs and 18% by 30 yrs. In patients with use of 5 ASA, lower risk of CRC. Mucosectomy does not confer benefit in terms of disease control and there is no significant improvement in cancer risk with mucosectomy

Consensus that CRC is highest in this pts with long duration of disease (adult compared with child) and extent of disease

How well did you know this?
1
Not at all
2
3
4
5
Perfectly
79
Q

Transanal endoscopic microsurgery has limited application in which of the following?

A. Lesions < 10 cm from anal verge
B. Lesions occupying <30% of the circumference
C. Lesions <5 cm in diameter
D. Mucosal lesions
E. Lesions located above the anal canal
A

E

TEM provides a minimally invasive technique for excising a wide variety of beings and malignant rectal lesion. Large diameter operating proctoscope is inserted into the anal canal. With an airtight seal, rectum can be distended with CO2, providing clear visualization of rectum was a small calibre videoscope. Precise dissection for both full thickness and partial thickness excision. Better imaging and access facilitate control of hemostasis. Accurate dissection within the proper planes and with appropriate margins can be achieved. Polyps above the peritoneal reflection of the rectum (>15 cm), polyps > than 8 cm in diameter, and polyps occupying >50% of rectal circumference are amenable to TEMS resection. TEMS is applied to a variety of anorectal diseases. Other benign rectal and extrarectal masses such as carcinoids, and some retrorectal cysts, can also be excised with TEMS. TEMS does not allow for visualization of the anal canal. It used is limited to lesion located above the anal canal.

How well did you know this?
1
Not at all
2
3
4
5
Perfectly
80
Q

For each numbered statement, select the correct lettered statement

  1. Restrictive and malabsorptive
  2. Superior resolution of metabolic comorbidities
  3. Operative mortality <0.5%
  4. Mean excess wt loss > 70%

A. Duodenal switch with sleeve gastrectomy (biliopancreatic diversion)
B. RYGB
C. Both
D. Neither

A
  1. C
  2. A
  3. B
  4. A

RYGB is most commonly performed bariatric procedure. 60 mL proximal gastric pouch is created and anastomosed to Roux limb. Creating a Roux limb 150 cm long enhances wt loss by means of malabsorption of ingested foods. Small gastric pouch restricts volume of food intake, and pts must dramatically reduce meal size to avoid complications. Mean excess wt loss approaches 60% at 1 yr and is somewhat less at 3 yrs. 30 day operative mortality ranges from 0.1-0.5% in large series

BPD includes a sleeve gastrectomy by stapling along a 60 Fr Bougie placed along the lesser curve of the stomach. Duodenum is divided 2 cm distal to pylorus, preserving blood supply and vagal innervation of antrum. Roux limb is created by dividing small intestine 250 cm proximal to IC valve and anastomosing this to the postpyloric duodenal cuff. Bypassed biliopancreatic lumb is sewn to Roux limb 100 cm proximal to IC valve. Technically more challenging than RYGB. Clearly a restrictive procedure, pt acceptance is higher b/c gastric capacity is larger and there is less dumping than RYGB. More effective in reversing premorbid metabolic conditions such as DM, dyslipidemia, and HTN in super obese. 30 day operative mortality 0.5-1.1%. For super obese pt, BPD results in significantly superior sustained wt loss than RYGB at 3 yrs postop

How well did you know this?
1
Not at all
2
3
4
5
Perfectly
81
Q

For each numbered statement, select the correct lettered statement

A. Lap Nissen fundoplication
B. Lap Toupet fundoplication
C. Both
D. Neither

  1. 270 degree wrap
  2. Requires extensive division of short gastrics
  3. Requires posterior esophageal dissection
  4. Less early post op dysphagia
A
  1. B
  2. D
  3. C
  4. B

Lap Nissen involves a 360 degree wrap of esophagus with or without division of short gastric vessels. Division of short gastric vessels is rarely necessary to acheive adequate mobilization of the fundus for either Nissen or Toupet. Results are the same without dividing the short gastric vessels and division of short gastric vessels may actually incr postop bloating. Lap Toupet requires a similar posterior esophageal dissection. However, the fundus is sutured to the R diaphragmatic crura, creating a 270 degree wrap, as opposed to a 360 degree wrap.

Toupet is assoc with less postop dysphagia and markedly less need for esophageal dilation in the early post op period. Nissen and Toupet are equivalent in terms of symptom resolution at 5 and 10 yrs follow up. IN prospective RCT, 85% have clinical success at 5 yrs

How well did you know this?
1
Not at all
2
3
4
5
Perfectly
82
Q

For each numbered statement, select the correct lettered statement

A. UC
B. Crohn’s
C. Both
D. Neither

  1. Infliximab therapy is indicated for active disease resistant to 1st line therapy
  2. Mesalamine is effecfive as first line maintenance
  3. 6MP is used as adjuvant therapy for steroid dependency
  4. Infliximab therapy increases the risk of wound complications after stomal closure
A
  1. C
  2. A
  3. C
  4. D

Combining oral meslamine with mesalamine enemas is better than oral monotherapy for pts with mild to mod active, extensive UC. Combo induces 64% remission within 8 weeks, compared with 43% with oral monotherapy. Combo is appropriate for initial therapy. For pts who need escalation in therapy, moving straight from mesalamine to infliximab is an effective option that can avoid steroids; 33% acheived remission after 8 weeks of therapy with infliximab

Mesalamine cannot be recommended for Crohn’s disease b/c results are inconsistent. Cochrane review found no benefit In meta analysis, infliximab maintained remission in more pts than placebo and incr response and spared pts from corticosteroir therapy.

No significantly incr postop complications after ileostomy closure in pts who received infliximab or other immunosuppresive meds compared with pts who did not.

Both 6 MP and azathioprine are successful in managing disease for pts who are steroid responsive ileal disease or UC.

How well did you know this?
1
Not at all
2
3
4
5
Perfectly
83
Q

For each numbered statement, select the correct lettered statement

A. Lap adjustable gastric band
B. Lap sleeve gastrectomy
C. Both
D. Neither

  1. 30 day mortality >1%
  2. Excess wt loss at 1 yr after procedure >50 %
  3. Plasma ghreline levels decr
A
  1. D
  2. B
  3. B

Obesity exceeds 30%. 3 major procedures are LRYGB, lap gastric band, and lap sleeve gastrectomy

All 3 reverse insulin resistance in msot pts. Ghrelin levels are dec with lap sleeve gastrectomy. This finding has theoretical importance in that lowered levels should decr appetite. 30 day mortality is low in all 3 procedures, wich the highest being in LRYGB (0.4%). Incidence of early reoperation with LRYGB is approx 2% with the other 2 procedures, early reoperation is necessary in approx 1%

30 day complication rate is highest in LRYGB group (4%) but the 1 year complication rate is the same for all 3 (8%). At 1 year, the % of excess wt loss is 60% for LRYGB and lap sleeve gastrectomy and 40% for the lap gastric band.

How well did you know this?
1
Not at all
2
3
4
5
Perfectly
84
Q

For each numbered statement, select the correct lettered statement

A. Adult intussusception
B. Adult malrotation
C. Both
D. Neither

  1. Chronic vague symptoms
  2. Target sign
  3. Whirlpool sign
  4. Assoc with cecal volvulus
  5. PSBO
  6. Currant jelly stool
  7. En bloc resection
A
  1. C
  2. A
  3. B
  4. B
  5. C
  6. D
  7. A

Intusussception and anomalies of intestinal rotation are usually assoc with the peds population, commonly in the neonatal period, particularly with respect to malrotation. Both conditions are also identified in the adult population and some have suggested that these conditions occur with equal incidence to the peds population. These 2 entities should be kept in ddx for adult pts who present with vague or nonspecific abdo complaints. Acute symptoms do occur in adults but not as commonly as they do in children. Classic findings of intusussception in children–palpable sausage shaped abdo mass, currant jelly stools or an acute abdo catastrophe–are not seen in adults. When identified in the adult population, assoc congenital abN are not common.

Intusussception in adults may present without a lead point, in contrast to children, and may occur without symptoms as an incidental finding on CT performed for other reasons. Finding of bowel within bowel or target sign is pathognomonic. Presentation in adults with a lead point may manifest with atypical clinical findings, b/c may are related to neoplastic processes, commonly disseminated carcinomatosis. Some intususscpetions may present with acute bowel obstruction. En bloc resection is recommended and attempts at hydrostatic reduction should not be undertaken

Intestinal rotational anomalies in adults can be complete or incomplete. Often, like in intussusception, they are found during work up for vague abdo complaints or incidentally at the time of sx for other reasons. Contrast intestinal imaging may identify R sided small bowel, a L sided cecum, and inverse relationship between the SMA and SMV or aplasia of the uncinate process of the pancreas. A whirlpool sign, wrapping of the SMV around the SMA with dilation of SMV, is a common CT finding. Cecal volvulus occurs with malrotation, and partial SBO is common with both malrotation and intusussception in adults

How well did you know this?
1
Not at all
2
3
4
5
Perfectly
85
Q

For each numbered statement, select the correct lettered statement

A. Adenoca of stomach intestinal type
B. Adenoca of stomach diffuse type
C. Both
D. Neither

  1. Diets high in preserved foods
  2. Antral location
  3. Routine splenectomy
A
  1. A
  2. A
  3. D

Classic histopathologic classification used for gastric cancer describes 2 distinct adenocas: intestinal and diffuse.

Diffuse gastric adenoca has an approx equal male:female ratio, with an infiltrative submucosal growth pattern, resulting in classic thickened, non distensible stomach known as linitis plastica. It tends to being in the corpus or proximal fundus of the stomach and generall mets directly into peritonum

The intestinal types tends to occur in the older population with a higher male: female ratio. It is somewhat assoc with atrophic gastritis and mets often by hematologic spread to the liver. Assoc with environmental factors such as a diet high in salted, smoked and preserved foods. Surgical resection is the only realiztic modality for cure of the pt. IN large clinical studies, there has been no benefit to ppx splenectomy for curative resection in gastric ca, unless there is macrocystic disease involving the spleen or parasplenic LNs

How well did you know this?
1
Not at all
2
3
4
5
Perfectly
86
Q

For each numbered statement, select the correct lettered statement

A. Rubber band ligation
B. Stapled hemorrhoidectomy
C. Both
D. Neither

  1. Mixed hemorrhoids
  2. Sepsis
  3. External hemorrhoids
  4. Above the dentate line
  5. Sphincter injury
A
  1. D
  2. C
  3. D
  4. C
  5. B

Rubber band ligation and stapled hemorrhoidectomy are used in the mgmt of internal hemorrhoids. Neither can be used in the tx of external hemorrhoids. Internal hemorrhoids originate above the dentate line and have visceral innervation but lack somatic innervation. As a result, internal hemorrhoids may be managed with relatively minimal discomfort to the patient. External and mixed internal-external hemorrhoids are covered by anoderm, a modified squamous epithelium that contains pain fibers. Rubber band ligation of external hemorrhoids would be pain ful and stapled hemorrhoidectomy does not remove external hemorroids

Rubber band ligation is office based procedure, performed withou anesthesia or bowel prep. Redundant hemorrhoid tissues is grasped above dentate line through an anoscope and a double rubber band is applied at the base of the sensation free hemorrhoid effectively cinching the redundant tissue. Depending on pt tolerance, multiple site may be tx in a single visit.

Stapled hemorrhoidectomy is an operative procedure. Circumferential purse string suture is placed above the dentate line through a specialized anoscope and the redundant hemorrhoid tissue excised with a hemorrhoid stapler. 1 to 3 cm ring of mucosa and submucosa is excised. Care taken not to include the sphincter muscle or vagina in the staple line. Fecal incontinence and rectovaginal fistula may result from inaccurate stapler placement

Pelvic sepsis may occur after both rubber band ligation and stapled hemorrhoidectomy. This complication is rare yet may be lethal if not recognized. Pelvic sepsis is usually manifested by pelvic pain, fever, and urinary retention. Initial mgmt consists of EUA, debridement of any compromised tissue and broad spectrum Abx.

How well did you know this?
1
Not at all
2
3
4
5
Perfectly
87
Q

For each numbered statement, select the correct lettered statement

A. Fourth degree strangulated hemorrhoids with necrosis
B. Fourth degree stranfulated hemorrhoids with edema, no necrosis
C. Both
D. Neither

  1. Requires emergency OR
  2. Urinary retention
  3. Staples hemorrhoidectomy
A
  1. A
  2. C
  3. D

Both presentations are representative of hemorrhoid crisis. Patients present with acute pain and nonreducible prolapse, and they may experience urinary retention with either presentation

Gangrene, necrosis and ulceration are absolute indications for a emergency hemorrhoidectomy. There is no role for office based mgmt. All devitalized tissue must be debrided. Wounds should be left open to prevent post op sepsis. Best done under GA.

Visible strangulated prolapsing hemorrhoids may be tx with formal surgical hemorrhoidectomy or may be tx more conservatively in the office with a perianal block, gentle reduction and multiple rubber band ligation. Rubber band ligation can be done either at the time of initial reduction or after edema is resolved. Perianal block is obrained with 0.25% bupivicaine with 1:100,000 epinephrine and hyaluronidase. This approaches is particularly useful in late pregnancy.

Stapled hemorrhoidectomy is used in the mgmt of uncomplicated second and thrid degree internal hemorrhoids. It does not address the external hemorrhoid component and is contraindicated in the face of tissue necrosis.

How well did you know this?
1
Not at all
2
3
4
5
Perfectly
88
Q

All of the following are true of Meckel’s diverticulum except

A. Meckels is most common congenital GI malformation
B. 80% of pts present with symptoms before 2 yrs of age
C. GI bleeding is most common symptom in children
D. The cause of bleeding is ulcerated gastric mucosa
E. More than 10% are symptomatic

A

B in answer key
D is also wrong

Most common congenital Gi malformation. Anomaly results from an incomplete obliteration of the omphalomesenteric duct during gestation. Only 4-6% of pts develop symptoms. When discovered incidentally at exp lap, routine resection is not recommended, regardless of age

GI bleeding is the major clinical finding in children and occurs in up to 50% of cases. Cause of GI bleed is assoc with ectopic gastric tissue in the diverticulum. Symp of intestinal obstruction are most commonly seen in adults and this presentation is second most common clinical finding in children. <50% of pts present with symptoms before 2 yrs of age.

How well did you know this?
1
Not at all
2
3
4
5
Perfectly
89
Q

Patients who are TPN dependent should be considered for small bowel transplant for any of the following except

A. Impending liver failure
B. Thrombosis of at least 2 central veins
C. At least 2 episodes of systemic line sepsis in 1 yr
D. Residual small bowel length <100 cm
E. Freq hospitalizations for pseudo obstruction

A

D

Intestinal transplan is a viable surgical option for pts with irreversible chronic intestinal failure who cannot toelrate or be maintained on TPN. Hepatic injury is the most common reason for pts with intestinal failure symptoms (short gut syndrome) not to be able to toelrate PTN. Most common indication for transplant. Complciations related to venous access such as catheter thrombosis and line sepsis can make TPN impractical. Accepted indications include impending or over liver failure, central venous catheter related thrombosis in at least 2 central veins, or at least 2 episodes per year of systemic sepsis secondary to line infections.. Indicated when freq hosp are required for the mgmt of complications related to intestinal failure such as episodes of volume depletion or repeated episodes of pseudo obstruction. In general, the absolute length of residual small bowel alone is no an indication for transplant, except in those situations where ultrashort bowel exits (<20 cm in adults) and where unmanageable complications of volume depletion and electrolyte imbalances are certain to occur.

How well did you know this?
1
Not at all
2
3
4
5
Perfectly
90
Q

65 M underwent a LAR with primary anastomosis without proximal diversion after neoadjuvant chemoradiationfor a 6 cm rectal cancer located 4 cm from the anal verge. His preop albumin was 3.0. What would be the least likely risk factor for developing anastomotic leak?

A. Baseline albumin < 3.5 
B. Tumor size >5 cm
C. Neoadj chemoradiation
D. Low anastomosis
E. Absence of protective stoma
A

C

Anastomotic leak can occur after any colorectal sx, especailly after sphincter sparing surgeries with low anast for rectal ca. Leak is assoc with longer hospital stay, incr morbidity, and incr mortality and is reported to occur in 2-15% of cases. RF for leak after LAR are large tumors size (esp >5 cm), low serum albumin (<3.5), low anast, higher ASA score and intraop soilage. Temp DLI remains somewhat controversial given the need for an additional operation with potential morbidity, yet when they are used, they are assoc with lower anast leak rates, rates of pelvic sepsis and reoperation. Most experts believe than diversion is appropriate for pts who are at high risk fo anast leak, such as the pts with a large and low tumor. Neoadj chemorads does not appear to influence rate of anast leak.

How well did you know this?
1
Not at all
2
3
4
5
Perfectly
91
Q
Best management of a 2cm anal margin SCC
A. Nigro protocol
B. Wide Local Excision
C. APR
D. Topical Imiquimod
A

B

Anal margin is WLE
Anal canal is Nigro

How well did you know this?
1
Not at all
2
3
4
5
Perfectly
92
Q
Patient with rectal cancer, EUS shows invading internal sphincter.  Best mgmt?
A.  Chemo/rad
B. Chemo
C. APR
D. Low ant resection
E. TAE
F. Intersphincteric dissection
A

C

T2; doesn’t need NACRT, shouldn’t change operation based on NACRT.
Internal = T2
Intersphincteric = T3
External = T4

How well did you know this?
1
Not at all
2
3
4
5
Perfectly
93
Q
Patient with rectal cancer 1cm from the dentate line with normal sphincter function and no evidence of sphincter invasion.  What is the best management?
A.  APR
B. Chemo/rad
C. Chemo
D. Low ant resection
A

A

Dentate line is within anal canal. 1cm proximal would put distal margin at the anorectal ring. Possible could do a coloanal, but APR is safer.

How well did you know this?
1
Not at all
2
3
4
5
Perfectly
94
Q
Patient has a 1 (or 2)cm lesion within the anal canal. Biopsy shows poorly differentiated “carcinoma”.  What is the best next step?
A. Path review
B. Chemoradiation
C. APR
D. Local excision
A

A

Adeno? SCC? Adenosquamous? Path needs clarification.

How well did you know this?
1
Not at all
2
3
4
5
Perfectly
95
Q
Anal margin lesion 1.5cm. biopsy showed SCC. palpable lesion
A. Local excision
B. Imiquimod
C. APR
D. Mitomycin C, 5FU, radiation
A

A

Perianal skin cancer behaves as skin cancer, not anal canal SCC. Treatment of choice is WLE. ASCRS text indicates WLE with 1 cm margins, accepting that APR may be necessary if large or involves sphincter. Up to Date is incorrect when it says that perianal SCC should be treated as anal canal SCC.

ASCRS: Local excision is an appropriate consideration only for small superficial lesions outside the anal canal at the anal margin in most instances

How well did you know this?
1
Not at all
2
3
4
5
Perfectly
96
Q

A 45 yo male patient is found to have SCC in a Pilonidal Sinus. What is the best management?
A. Resect with SLNB
B. Neoajduvant Chemo/Rad then Reesect
C. Resect with Flap
D. Resect with Negative Pressure and delayed closure

A

D

NPWT is Canada consensus; SCC in scar is aggressive with high risk of local recurrence, so covering with a flap is controversial as may hide a recurrence. Ideally need to know margin status before closing; VAC as temporizing measure while waiting for path then followed by flap closure once margins confirmed clear.

How well did you know this?
1
Not at all
2
3
4
5
Perfectly
97
Q
Patient refuses an APR for T3 rectal tumor invading the sphincter complex.  What is the best management?
A. Refer to colleague for second opinion
B. LAR and coloanal
C. Transanal excision
D. Chemorads but do not resect
A

A

How well did you know this?
1
Not at all
2
3
4
5
Perfectly
98
Q
Anal canal SCC with positive inguinal node
A. Chemorads and groin dissection
B. APR and groin dissection
C. APR
D. Chemorads
A

D

Chemoradiation is the treatment of choice for inguinal lymph node disease. Similar to management of the primary anal lesion, the mainstay of treatment for concomitant disease of the perirectal or inguinal nodes is chemoradiation. A complete response has been reported in 79% to 92%. With the identification of any positive inguinal lymph node, bilateral inguinal basins should be incorporated into the radiation fields with the addition of a boost technique. Metachronous lymph nodes are seen in 10% to 20% of patients, normally within 6 months of completing treatment of the primary lesion. These metachronous nodes should also be treated with CRT, and typically respond well. Elective prophylactic lymphadenectomy is generally not warranted and is associated with high wound complication rates as well as lower-extremity complications. Selective inguinal node dissection may be considered for persistent disease following CRT. In small case series, long-term survival has been reported after successful removal of disease.

How well did you know this?
1
Not at all
2
3
4
5
Perfectly
99
Q
Patient is found to have a 2cm polyp 7cm above the dentate line.  This is removed endoscopically and pathology returns as a well-differentiated adenoCa with invasion to muscularis mucosa with 1mm margin EUS is performed and shows no lymphadenopathy.  CT shows no evidence of distant mets.  What is the best treatment?
A. Observation
B. Neoadjuvent therapy
C. LAR
D. APR
E. Transanal excision
F. TEMS
A

F

T1N0
Need margins >1mm

How well did you know this?
1
Not at all
2
3
4
5
Perfectly
100
Q
Rectal cancer 2cm in size at 7 cm. Invasion into muscularis propria. Node negative on imaging. Mgmt?
A. Neoadjuvant then TME LAR
B. TME LAR
C. TEMS
D. APR
A

B

How well did you know this?
1
Not at all
2
3
4
5
Perfectly
101
Q
POD 5 LAR. Now presents with tachycardia, spreading LLQ peritonitis and imaging showing a 1cm leak into left pericolic gutter (exact wording). What to do?
A. Operative drain and diverting loop
B. Conversion to Hartmanns
C. Observe/ABx
D. Perc drain
A

A

ASCRS manual: small leaks can be managed with anastomitic drainage +/- repair and diversion. If managed with Hartmann’s, probably will never be able to reconnect as anast is so low already.

  • Patients with a free leak should be taken to the operating room after fluid resuscitation and intravenous antibiotics are administered.
  • After a thorough washout, the treatment is dictated by the findings.
  • Most colorectal anastomosis will require anastomotic takedown and an end colostomy.
  • To minimize the effects of a friable rectal stump (that cannot be closed with staples or sutures nor brought to the skin surface as a mucous fi stula), placement of transabdominal and transanal drains is indicated.
  • Selective small bowel or ileocolic anastomotic defects can be repaired. However, resection of the anastomosis with creation of a new anastomosis or stoma is the most conservative option. Placement of the repaired anastomosis under the surgical incision will result in an enterocutaneous fistula instead of a second bout of peritonitis should a second leak occur.
  • Any concern regarding viability of the bowel ends necessitates takedown of the anastomosis and creation of a stoma.
  • Small defects in a colorectal anastomosis, in select circumstances, may be repaired and a proximal ileostomy created. This should be avoided when there is a large fecal load between the ileostomy and the repaired anastomosis.
  • A contained anastomotic leak is walled off and typically located in the pelvis presenting as an abscess.
  • If the abscess is small and contrast flows freely into the bowel, the patient can be treated with intravenous antibiotics, bowel rest, and observation.
  • Larger abscesses or those removed from the site of the anastomosis may require radiologically guided drainage.
  • A contained leak rarely requires immediate operative intervention, but surgery may eventually be required if the patient develops a cutaneous fistula, anastomotic stricture, or chronic presacral cavity.
How well did you know this?
1
Not at all
2
3
4
5
Perfectly
102
Q

50ish female, 3 years post LAR for rectal Ca with isolated greater omental met seen on imaging. Management
A. Palliative chemo
B. Isolated resection
C. Chemoreductive surgery with HIPEC

A

C

Sounds like the best candidate for CRS; long interval, young, limited disease. Despite HIPEC/CRS not yet being standard of care, it is supported by RCTs.

How well did you know this?
1
Not at all
2
3
4
5
Perfectly
103
Q
T2N0 pre-op staging rectal Ca, undergoes OR, final path T2N1. What next
A. Chemo and RT
B. Chemo
C. Radiation
D. Observe
A

A (this is an old answer)

New ESMO guidelines July 2017
The European Society for Medical Oncology (ESMO) has updated their guidelines for treatment of rectal cancer [1]. Among the many changes from the 2013 guidelines, they suggest a selective approach to postoperative chemoradiotherapy in patients with resected stage II and III disease (table 1), recommending it only for patients with certain high-risk features identified at the time of surgery (table 2).

Sufficient and necessary
CRM ≤1 mm
pT4b
pN2 extracapsular spread close to MRF
Extranodal deposits (N1c)
pN2 if poor mesorectal quality/defects

Sufficient
pN2 low tumors within 4 cm of anal verge (risk of involved LPLN)
Extensive extramural vascular invasion/perineural invasion close to MRF

Borderline sufficient
pN2 in mid/upper rectum if good mesorectal quality
CRM 1 to 2 mm
Circumferential obstructing tumours

Insufficient and unnecessary
pT1/pT2
pT3
CRM >2 mm
pT4a above peritoneal reflection
pN1
If good-quality smooth intact mesorectum
How well did you know this?
1
Not at all
2
3
4
5
Perfectly
104
Q
Lady in her 50s with resection of a rectal cancer. T2N1. 5 years later has an elevated CEA (17 then 50 a month later). full work up negative (CT, PET, colonoscopy)
A. MRI
B. Repeat the CT in 3 months
C. Diagnostic laparoscopy
D. Chemo
A

B

How well did you know this?
1
Not at all
2
3
4
5
Perfectly
105
Q
Rectal neuroendocrine 2 cm in size invading muscularis propria which is 1 cm above anal verge
A. TEMS
B. APR
C. LAR
D. Follow with colonoscopy
A

B

T2, >2cm means radical resection required. As 1cm above anal verge, APR.

Rectal NET: <1cm, confined to mucosa or submucosa, endoscopic resection adequate. >2cm need radical resection (APR/LAR) Intermediate tumours (1-1.9cm) controversial. Generally, if <1.5cm and no high risk features (LVI, mitotic rate, etc), local excision adequate. If high risk, formal resection.

How well did you know this?
1
Not at all
2
3
4
5
Perfectly
106
Q
Patient had colonoscopy and found to have mass/polyp in rectum, polypectomy was performed.  Turned out to be neuroendocrine tumour with positive margin.  Lesion was 1cm big.  
A. APR
B. LAR
C. Transanal excision
D. Observe
A

C

Small NET 1cm, positive margins, requires local excision NOT radical resection. If >2cm or high-risk features, then formal resection.

How well did you know this?
1
Not at all
2
3
4
5
Perfectly
107
Q

71 year old male with who received neoadjuvant chemoradiotherapy for a T3 adenocarcinoma 1cm from the dentate line not involving the sphincter muscles. On repeat endoscopy, the lesion is no longer visible at the previously tattooed site. What is the BEST management?
A. Repeat endoscopy in 6 months
B. TEMS
C. LAR with handsewn coloanal anastomosis
D. APR

A

D

Observation of rectal cancer with complete clinical response is investigational and should not be done outside of a clinical trial. For exam purposes, need to obtain a 2cm distal margin; in this case 2cm would be below dentate so not feasible.

How well did you know this?
1
Not at all
2
3
4
5
Perfectly
108
Q
2.5cm pedunculated rectal polyp, well differentiated ca, only in mucosa, no invasion to vessels or lymphatics or stalk. Follow up:
A. Anterior resection
B. Colotomy and resection of stalk
C. Colonoscopy and fulguration of stalk
D. Colonoscopy in one year
E. Colonoscopy in 4-6 months
A

E

How well did you know this?
1
Not at all
2
3
4
5
Perfectly
109
Q
67 yo male otherwise healthy, has been found to have a villous adenoma of the rectum. It is 6cm above the anal verge, has firm areas but is not fixed. Biopsies reveal dysplasia, as well as ca in situ. What should be done:
A. Low anterior resection
B. APR
C. Piecemeal excision
D. Transanal resection
E. Trans sacral resection
A

D

How well did you know this?
1
Not at all
2
3
4
5
Perfectly
110
Q

3cm mobile sessile polyp at 4cm above dentate line. Biopsy proven foci of ca. What is the best management:
A. APR
B. Low anterior resection
C. Trans-anal resection

A

C

How well did you know this?
1
Not at all
2
3
4
5
Perfectly
111
Q
What is not an indication for transanal excision of rectal ca
A. Tumor <4cm
B. <40% or circumference
C. T2/N0
D. 12cms from anal verge
E. Moderately differentiated
A

C (most wrong answer)

Uptodate:
Local excision should be limited to the following groups
-Superficial T1 cancer, limited to the submucosa
-No radiographic evidence of metastatic disease to the regional nodes
-Tumor <3 cm in diameter
-Well-differentiated histology, no lymphovascular or perineural invasion
-Mobile, non-fixed
-Margin clear (>3 mm)
-Involving <30 percent of the bowel lumen circumference
-Patient is able to comply with frequent postoperative surveillance

How well did you know this?
1
Not at all
2
3
4
5
Perfectly
112
Q
What is the most important prognostic feature of rectal ca:
A. Depth
B. Nodal involvement
C. Tumor aneuploidy
D. Type of surgical resection
E. POP radiotherapy
A

B

How well did you know this?
1
Not at all
2
3
4
5
Perfectly
113
Q

Which is true regarding the use of NdYAG laser for rectal ca:
A. It may eliminate the need for staged surgical procedure
B. It can not be used above the peritoneal reflection
C. It is useful for tenesmus
D. Use improves cure rates
E. Response is a useful staging tool

A

B in answer key

Neodymium-YAG laser therapy. Used mainly for palliation or in patients who are not surgical candidates

Endoscopic Nd:YAG laser treatment of inoperable lower gastrointestinal cancer.
“It requires no anaesthesia and is the only non-surgical procedure that can be safely carried out above the peritoneal reflection. “

Local control of rectal cancer with the Nd-YAG laser. - NCBI
Three patients with locally recurrent rectal cancer were treated using the Neodymium YAG laser to palliate the symptoms of tenesmus, discharge and bleeding.

How well did you know this?
1
Not at all
2
3
4
5
Perfectly
114
Q
What is the commonest complication occurring in a male after APR:
A. Urinary retention
B. Urinary stress incontinence
C. Urethral-perineal fistulae
D. Bladder neck obstruction
E. Distal ureter injury
A

A

How well did you know this?
1
Not at all
2
3
4
5
Perfectly
115
Q
Most common pathology on ca of the anal canal:
A. Adenoca
B. Melanoma
C. Bowen’s disease
D. Paget’s
E. Epidermoid
A

E

How well did you know this?
1
Not at all
2
3
4
5
Perfectly
116
Q
Which is not a risk factor for development of anal ca:
A. Smoking
B. Alcohol use
C. Lymphogranuloma venereum
D. Chlamydia infection
E. Anal intercourse
F. Condyloma acuminata
G. HSV
A

B

How well did you know this?
1
Not at all
2
3
4
5
Perfectly
117
Q
Anal canal 3cm epidermoid ca. What is the best treatment:
A. APR
B. Local excision
C. RX and chemo
D. RX
E. Chemo
A

C

Basaloid, epidermoid, mucoepidermoid are all subtypes of SCC, but do not influence treatment choice.

How well did you know this?
1
Not at all
2
3
4
5
Perfectly
118
Q

Reason for post-op radiation for rectal cancer as opposed to preop. Which is correct?
A. Decreased incidence of neorectal radiation injury
B. Decreased incidence of bladder radiation injury
C. Decreased sb radiation enteritis
D. Avoids over treatment of lesions of stage lower than t3no
E. Increased survival

A

D

How well did you know this?
1
Not at all
2
3
4
5
Perfectly
119
Q

A 20 yo male with multiple rectal polyps. All of the following are part of treatment except:
A. Ileo anal pouch
B. Ileo rectal anastamosis after colectomy
C. Sulindac
D. Endoscopic surveillance until dysplasia
E. Screen for extra colonic cancer

A

B

How well did you know this?
1
Not at all
2
3
4
5
Perfectly
120
Q
Most common cause of anal canal cancer:
A. Squamous cell carcinoma
B. Basal cell ca
C. Melanoma
D. Adenocarcinoma
A

A

How well did you know this?
1
Not at all
2
3
4
5
Perfectly
121
Q
Previous rectal cancer resection with recurrence.  Which finding would make him unresectable
A. Bilat hydronephrosis
B. Pelvic pain
C. Invading base of bladder
D. Incontinence
A

A

How well did you know this?
1
Not at all
2
3
4
5
Perfectly
122
Q
DVT prophylaxis for rectal cancer procedure in patient with renal failure, previous history of DVT post hernia repair
A. Heparin 5000u sc bid
B. Heparin 5000u sc TID for one month
C. LMWH
D. SCDs until ambulating
A

B

How well did you know this?
1
Not at all
2
3
4
5
Perfectly
123
Q
Patient post-LAR for rectal cancer currently undergoing chemoradiation.  Develops peripheral neuropathy and foot drop. What is the most likely etiology?
A. Radiation
B. Nerve injury during surgery
C. Chemotherapy
D. Disease recurrence
A

D

Most common cause is pelvic recurrence, followed by radiation.

FOLFOX –neurotoxicity is common, but is sensory only.

How well did you know this?
1
Not at all
2
3
4
5
Perfectly
124
Q
Obstructing rectosigmoid cancer. Intraop invading bladder, side wall etc. Mgmt
A. Major en bloc resection
B. Loop ileostomy
C. Loop colostomy
D. Close abdo
A

C

How well did you know this?
1
Not at all
2
3
4
5
Perfectly
125
Q
Best indication for TEM.
A. T1 at 1 cm above dentate
B. T1 proximal rectum
C. T2 midrectum
D. T3 mid rectum
A

B

How well did you know this?
1
Not at all
2
3
4
5
Perfectly
126
Q

Hemorrhoidectomy, path comes back as invasive SCC with negative margins. What should you do?
A. Close observation
B. Modified Nigro
C. Re-excise

A

A

Need to discuss at multidisciplinary case conference

Nigro is treatment for all anal SCC

How well did you know this?
1
Not at all
2
3
4
5
Perfectly
127
Q

Scaly perianal lesion, 6cm, circumferential, bx intraepithelial (intradermal) SCC
A. WLE
B. Imiquimod
C. APR

A

B

Bowen’s or HSIL. <10% will progress to invasive in immunocompetent patients, but can’t predict so treatment favoured. Standard treatment is WLE, although topical therapy (imiquimod) may used in unfit patients OR when surgery would leave a difficult wound

Neoplasms of Anal Canal and Perianal Skin. Clinics in Colon and Rectal Surgery, 2010.

The standard treatment is wide surgical excision.6 To ensure clear resection margins, a systematic four-quadrant biopsy technique, with intraoperative frozen sections has been advocated. The frozen sections should include intra-anal biopsies. Despite use of this technique, recurrence rates up to 30% have been reported. The major disadvantage of wide local excision is the difficulty to primarily close the wound and skin flaps may be necessary. The rotational v-y skin flap has been most frequently described in this setting.When surgery is not feasible or refused, other options are available such as topical chemotherapy (5-FU), immunomodulation (imiquimod), and phototherapy, although the latest guidelines favor radiotherapy.

How well did you know this?
1
Not at all
2
3
4
5
Perfectly
128
Q

Female patient went for short course radiation for rectal cancer.

A. Resection 1 week after radiation
B. Resection 6 weeks after radiation
C. Resection after 12 weeks
D. Observe

A

A

How well did you know this?
1
Not at all
2
3
4
5
Perfectly
129
Q

Patient with grade III hemorrhoids, banding in office immediate pain. Stable.
A. Remove band
B. OR debride
C. Observe

A

A

Band applied below dentate line.

How well did you know this?
1
Not at all
2
3
4
5
Perfectly
130
Q

Patient with anal fistula low lying with Crohns on routine exam. Asymptomatic.
A. Remicade
B. fistulotomy
C. Observe

A

C

Treat underlying CD

How well did you know this?
1
Not at all
2
3
4
5
Perfectly
131
Q

48hr pain to perineum with 1.5cm perianal nodule below the dentate line.
A. Excise
B. Observation
C. Incision/drainage

A

A

I&D of thrombosed external hemorrhoid is inadequate; excision is required. Excise up to 72 hours.

How well did you know this?
1
Not at all
2
3
4
5
Perfectly
132
Q

What decreases urinary retention post hemorrhoidectomy
A. Closed hemorrhoidectomy
B. Prone Jack knife
C. Limit IV fluids operatively

A

C

Schwartz: urinary retention is most common complication, limit perioperative fluids to reduce.

How well did you know this?
1
Not at all
2
3
4
5
Perfectly
133
Q
Anal fissure not responsive to medical treatments, fissure on lateral anal verge. Management?
A. Botox
B. Lateral sphincterotomy
C. Topical CCB
D. Biopsy
A

D

Atypical fissure should be biopsed or excised. Muscle relaxing treatments not effective in atypical lateral fissures.

How well did you know this?
1
Not at all
2
3
4
5
Perfectly
134
Q
25y M two weeks ago had a knee surgery. comes in with 1 week history of a posterior midline fissure best treatment
A. Sitz bath with a stool bulking agent
B. Topical CCB
C. Lateral internal sphincterotomy
D. Botox
A

A

Conservative measures first line for acute fissures.

How well did you know this?
1
Not at all
2
3
4
5
Perfectly
135
Q

Brachytherapy for cervical cancer present with stool per vagina with a 2cm fistula 4cm from the verge
A. Endorectal advancement flap
B. Omental interposition
C. Diverting ostomy

A

C

Complex fistula as likely in high location (near/at cervix) and radiation-related; should be diverted first as poor tissue quality.

Surgical Clinics (radiation-related RVF)
In the absence of recurrent cancer, radiation-induced fistulas can be approached abdominally,  locally, or with diversion. There are several variants of each, with little evidence to support 1 method over the next, but the location and extent of radiation injury usually determine the most prudent approach. Before surgery, the extent of radiation injury, including the compliance of the rectum, needs to be addressed. Compliance can be addressed with manometry and subjectively with attempted insulation during endoscopy. 
Considering the poor quality of the rectum and concomitant inflammation and edema of tissue planes, these repairs are typically performed in conjunction with a diverting stoma. The diverting stoma can be performed simultaneously or several months before the repair, depending on the amount of contamination and tissue integrity. If the vaginal mucosa is uninvolved with radiation and if the fistula is not higher than the apex of the vaginal vault, a vaginal flap can be raised.

Cameron, John L.; Cameron, Andrew M (2013-11-20). Current Surgical Therapy: Expert Consult - Online (Current Therapy)
With the increased use of both brachytherapy and external-beam radiation in the treatment of pelvic malignancies, radiation-induced complications are likely to increase. The first step in management of radiation-induced rectovaginal fistulas is to rule out the presence of residual or recurrent malignancy. This requires detailed imaging and an examination with the patient under anesthesia with multiple biopsies of areas of irregularity or random biopsies if no irregularity exists. Once the presence of malignancy has been ruled out, the condition of the rectum, vagina, and surrounding perineal tissues needs to be evaluated. It is mandatory to wait at least 6 months after the completion of radiation treatment before any repair is attempted. This allows for the full effect of radiation to be realized and for the surrounding tissue to recover. If the local tissues are healthy, a rectal or vaginal advancement flap can be attempted. However, it should be appreciated that because the repair is being performed with radiated tissue, it is less likely to succeed. If one attempt at local repair fails, subsequent attempts will most likely be futile. Interposition flaps with nonradiated tissue (e.g., gracilis flap) or a resection of the involved rectum with a coloanal anastomosis and omental interposition then remains the best available option and is preferable to the classic Bricker procedure.

How well did you know this?
1
Not at all
2
3
4
5
Perfectly
136
Q

Colitis cystica profunda with difficulty with passing stool. Defacography shows internal intussusception, on scope she has an ulcer at 6cm anteriorly.
A. Resect with rectopexy
B. Altmeier
C. Biofeedback

A

C

The diagnosis is made on the basis of histologic finding of fibromuscular obliteration of the lamina propria. We suggest observation alone or treatment with bulk laxatives and biofeedback in patients who are asymptomatic or minimally symptomatic (Grade 2B). We suggest surgery (typically abdominal rectopexy) in symptomatic patients with rectal prolapse rather than conservative therapy (Grade 2C). Surgery may also help relieve symptoms in patients with severe symptoms that are unresponsive to conservative management.

How well did you know this?
1
Not at all
2
3
4
5
Perfectly
137
Q
Grade 4 tear with partial involvement of internal and external anal sphincters during delivery (assuming this means just delivered)
A. Debride
B. Biofeedback
C. Primary repair now
D. Diverting ostomy
A

C

Primary repair up to 48 hours; if longer delay, wait for 6 months

How well did you know this?
1
Not at all
2
3
4
5
Perfectly
138
Q
Urinary retention after hemorrhoidectomy, most likely associated with
A. Spinal
B. Fluid
C. Operative time
D. Use of foley catheter
A

B

Uptodate:
Urinary retention following hemorrhoidectomy is observed in as many as 30 percent of patients [56]. Spinal anesthesia tends to be associated with higher rates of urinary retention [5]. Limiting postoperative fluids may reduce the need for catheterization (from 15 to less than 4 percent in one study) [57]. Warm sitz baths and pain medication also may lessen the incidence of urinary retention and reduce the need for catheterization. Some patients will require urinary catheterization, although some remain relatively asymptomatic.

How well did you know this?
1
Not at all
2
3
4
5
Perfectly
139
Q
Guy starts Aldara (imiquimod) for condyloma. Gets pain/redness/swelling in area. Why?
A. Cellulitis
B. Normal reaction to drug 
C. Allergic reaction
D. Fungal infection
A

B

Occurs in 58-100%

How well did you know this?
1
Not at all
2
3
4
5
Perfectly
140
Q

23 year old male, hx of constipation, dilated sigmoid, narrowed rectum, poor rectoanal inhibitory reflex
A. Proctectomy with coloanal anastomosis
B. Lateral internal sphincterotomy
C. Total abdominal colectomy and coloanal anastomosis
D. Laxative and bulking agents

A

A

Adult diagnosis of Hirschsprungs disease. The rectoanal inhibitory reflex (RAIR) is an involuntary IAS relaxation in response to rectal distension, allowing some rectal contents to descend into the anal canal where it is brought into contact with specialized sensory mucosa to detect consistency

Cameron:
Presence of this reflex effectively rules out Hirschsprung’s disease.

How well did you know this?
1
Not at all
2
3
4
5
Perfectly
141
Q

Posterior anal fissure, needs sphincterotomy, why not do it posteriorly
A. Because its near the fissure
B. Because you don’t was to get into external sphincter
C. Fecal seepage

A

C

From Sabiston – Cochrane review showed that posterior sphincterotomy less effective and more incontinence

How well did you know this?
1
Not at all
2
3
4
5
Perfectly
142
Q

Male going for completion proctectomy, what to avoid to not affect both urinary and sexual dysfunction
A. Presacral nerves
B. Nervi ergenti
C. Prostatic plexus

A

C

Presacral nerves (sympathetic: shoot) (hypogastric plexus): ejaculatory difficulties/ retrograde ejaculation. 
Nervi erigenti (parasympathetic: point) at the lateral stalks, injured if dissection too wide: erectile dysfunction
Prostatic plexus – mixed urinary and sexual dysfcn

The nervi erigentes are located in the posterolateral aspect of the pelvis and at the point of fusion with the sympathetic nerves are closely related to the middle hemorrhoidal artery. Injury to these nerves will completely abolish erectile function.

Finally, dissection near the seminal vesicles and prostate may damage the periprostatic plexus, leading to a mixed parasympathetic and sympathetic injury. This can result in erectile impotence as well as a fl accid, neurogenic bladder.

How well did you know this?
1
Not at all
2
3
4
5
Perfectly
143
Q
Crohns pt, has mildly symptomatic hemmorhoids with mild bleeding. Grade II hemorrhoids.  What to do?
A. Nothing/conservative 
B. Band
C. Hemmorhoidectomy
D. Stapled hemmorhoidectomy
A

A

How well did you know this?
1
Not at all
2
3
4
5
Perfectly
144
Q

Lady with PBC and BRBPR – prior previous episodes. Stable . On exam has rectal varices and grade I hemorrhoids. What to do?
A. TIPS and angioembolize
B. Argon beam coagulation
C. Hemorrhoidectomy

A

A

Incidence of rectal varices in Childs C is up to 70%; endoscopic therapy (banding/sclerotherapy) is 1st line. 2nd line is retrograde balloon occlusion or TIPS. Surgical therapy is last-resort, can include surgical shunts, IMV ligation, or transanal oversewing of varix.

Bleeding ectopic varices – TIPS is useful for acute hemorrhage (Up to Date). This patient is stable and TIPS/embolization seems like too much, although might be getting at prophylaxis. APC is not described anywhere, sclerotherapy is. Hemorrhoidectomy is not necessary for grade I disease. TIPS/embolization is described, so of these options is probably the best.

How well did you know this?
1
Not at all
2
3
4
5
Perfectly
145
Q
Old lady with strangulated rectal prolapse
A. Delorme
B. Perineal proctosigmoidectomy
C. Laparotomy and sigmoid resection
D. Laparotomy and pexy
A

B

Delorme and pexy are not options as does not resect the strangulates bowel. Even gangrenous prolapse can be managed with perineal procedures.

Surgical Management of Rectal Prolapse. JAMA Surgery, Jan1 2005. Wexner, S.
Perineal rectosigmoidectomy is well suited for male patients; patients with incarcerated, strangulated, or even gangrenous prolapsed rectal segment; and patients who have had recurrence after another transperineal repair.

How well did you know this?
1
Not at all
2
3
4
5
Perfectly
146
Q
Young guy with chlamydia proctitis. How to treat?
A. Doxycycline
B. Flagyl
C. Steroids
D. Cortifoam enema
A

A

Up to Date:
Empiric therapy for both chlamydia and gonorrhea is indicated for the treatment of patients with acute proctitis. An empiric regimen of doxycycline (100 mg twice daily) plus a single intramuscular dose of ceftriaxone (250 mg) is active against both [3]. The duration of doxycycline therapy will depend on the severity of symptoms. For patients with mild proctitis, seven days of therapy is adequate. Patients with severe proctitis may have lymphogranuloma venereum infection, which requires a full three-week course of doxycycline.

Chlamydial proctitis, defined as inflammation of the distal rectal mucosa that can cause anorectal pain, rectal discharge, or tenesmus [19], is relatively uncommon and occurs almost exclusively in men who have sex with men (MSM) who have had receptive rectal intercourse [20,21]. However, anal intercourse is not uncommon among heterosexuals [22], and symptomatic proctitis has been reported in women [23]. Chlamydial proctitis may be caused by either the common genital strains of C. trachomatis (serovars D-K) that typically cause uncomplicated genital infection in men and women or the lymphogranuloma venereum (LGV) strains (serovars L1, L2, L3), which can cause severe disease (eg, abundant bloody or mucopurulent discharge, severe pain with defecation, fever)
Untreated can progress to rectal ulceration and stricturing.

How well did you know this?
1
Not at all
2
3
4
5
Perfectly
147
Q
Pt has amyloidosis and the fat pad biopsy is not helpful. What is the next best place to biopsy?
A. Rectum
B. Penis
C. Skin
D. Lung
A

A

1st line is FNA of fat pad; 2nd line rectal biopsy since 1964. And in Up to Date. Kidney or liver has highest overall sensitivity if those organs are involved.

How well did you know this?
1
Not at all
2
3
4
5
Perfectly
148
Q
Profuse bleeding PR after LAR. Anastomosis at 12 cm. POD#1. Patient unstable despite resuscitation.
A. Endoscopy with Clips and Epinephrine 
B. Revise anastomosis 
C. Angioembolization
D. Trans-anal suture ligation
A

A

Real life would attempt endoscopic control 1st; too high for transanal and wouldn’t embolize an anastomosis. This patient is unstable and anast is high enough to revise. Plan: take to OR, attempt endoscopic, revise if fails.

SCNA – Complications of Colorectal Anastomoses
In most cases, the patient remains hemodynamically stable, and no intervention is required. The rate of transfusion requirement is routinely less than 5%. In the review by Martinez-Serrano and colleagues, bleeding in 6 of the 7 patients resolved with conservative treatment including endoscopy. Only 1 patient required surgical treatment, and there was no mortality and no anastomotic leaks in these 7 patients. Cirocco and Golub reported nonoperative therapy to be successful in 14 of 17 patients (82%), using endoscopic electrocoagulation in 6 patients (43%) and blood transfusion alone in another 6 patients (43%). The investigators concluded that endoscopic electrocoagulation can be safely and effectively used on a newly created anastomosis to control unremitting anastomotic hemorrhage. Alternative endoscopic techniques include the use of submucosal injection of 10 mL adrenaline (1:200 000) in saline at the bleeding point, with good results. The use of the endoscopic hemoclip has been well described in upper gastrointestinal procedures and in colonic diverticular bleeding; however, its application in postoperative anastomotic bleeding for colon or rectal anastomosis is lacking. Anecdotally, the author (DR) has successfully used the endoscopic hemoclip to control bleeding at a colorectal anastomosis in the postoperative period. Finally, although rarely required, surgical exploration with oversewing of the anastomosis or resection may be needed for select recalcitrant cases.

Bleeding is a rare event after intestinal anastomosis, and endoscopic techniques have largely replaced the need for laparotomy or other surgical interventions. It is advisable that the operative surgeon performs or be present when endoscopic manipulation of a newly created anastomosis is required.

How well did you know this?
1
Not at all
2
3
4
5
Perfectly
149
Q

During sigmoidoscopy for volvulus, perforation of the anterior rectum occured. You should:
A. Hartmann’s
B. Laparotomy primary repair, presacral drainage
C. Laparotomy, resection, primary repair , defuntioning colostomy
D. Laparotomy, exteriorize colon

A

A

Bowel will be dilated, with fecal contamination, so no anastomosis. If can devolved and decompress to achieve better calibre match, primary anastomosis ok.

How well did you know this?
1
Not at all
2
3
4
5
Perfectly
150
Q

Which best describes the etiology of rectal prolapse:
A. Diastesis of levator ani muscles
B. Laxity of endopelvic fascia
C. Stretching of pudendal nerve
D. Loss of horizontal rectal position
E. Full thickness intussusception of the rectum beginning at 5 to 7 cms from the anal verge

A

E

How well did you know this?
1
Not at all
2
3
4
5
Perfectly
151
Q
24 male undergoes hemorrhoidal banding. Later that night he presented with rectal pain and fever of 38.9. He has difficulty in passing urine and his WCC of 18,000. You should:
A. Insert a foley catheter
B. Abx and EUA
C. Incise and drain
D. Remove rubber band
E. Abx and bed rest
A

B

How well did you know this?
1
Not at all
2
3
4
5
Perfectly
152
Q
What is the muscle expose in an hemorrhoidectomy:
A. Internal anal sphincter
B. Transverse perinei
C. Gluteus maximus
D. Subcutaneous external sphinter
E. Deep external sphinter
A

A

How well did you know this?
1
Not at all
2
3
4
5
Perfectly
153
Q
Anal stricture may occur from:
A. Crohn’s
B. Radiation
C. Lymphagranuloma venereum
D. Hemorrhoidectomy
A

D in answer key

World J Gastroenterol. 2009 Apr 28; 15(16): 1921–1928.
Surgical treatment of anal stenosis

The causes of anal stenosis include surgery of the anal canal, trauma, inflammatory bowel disease, radiation therapy, venereal disease, tubercolosis, and chronic laxative abuse.

LGV causes rectal stricture

How well did you know this?
1
Not at all
2
3
4
5
Perfectly
154
Q

Which of the following statements regarding ischiorectal abscess is most correct:
A. The best treatment is Ab
B. Drainage is indicated as soon as the diagnosis is made
C. Drain when fluctuant
D. Drain into rectum

A

B

Sabiston – drain when the diagnosis is made

How well did you know this?
1
Not at all
2
3
4
5
Perfectly
155
Q

Regarding the treatment of condyloma acuminata with the CO2 laser, one of the following is true:
A. The laser is less painful than coagulation
B. The smoke plume has live virus particles
C. Use of laser avoids recurrence
D. The laser is faster than electrocautery

A

B

How well did you know this?
1
Not at all
2
3
4
5
Perfectly
156
Q

Pruritus ani may be secondary to which of the following:
A. Hemorrhoids
B. Key whole deformity
C. Perianal creams

A

A

How well did you know this?
1
Not at all
2
3
4
5
Perfectly
157
Q

Fistula in ano, most commonly arise from:
A. Intersphinteric abscess secondary to perianal abscess
B. Subcutaneous abscess following anal fissure
C. Horseshoe abscess
D. Ischioresctal abscess

A

A

How well did you know this?
1
Not at all
2
3
4
5
Perfectly
158
Q
Factors predisposing to fistulae in ano:
A. Crohn’s
B. Cryptitis
C. Perianal intersphinteric abscess
D. Diverticulitis
A

C

Most fistulas are thought to arise as a result of cryptoglandular infection with resultant perirectal abscess.

How well did you know this?
1
Not at all
2
3
4
5
Perfectly
159
Q
Treatment of a recurrent fistulae in ano includes all of the following except:
A. Full fistulotomy
B. Staged opening fistulotomy
C. Seton suture
D. Open half and clear
A

D

How well did you know this?
1
Not at all
2
3
4
5
Perfectly
160
Q
What forms the superior border of the ischiorectal fossae:
A. Obturador internus
B. Gluteus major
C. Endopelvic fascia
D. Levator muscle complex
E. Internal anal sphinter
A

D

How well did you know this?
1
Not at all
2
3
4
5
Perfectly
161
Q
Injury to the rectum is possible with all except:
A. Penetrating injury to the buttock
B. Penetrating injury to the abdomen
C. Central fracture to the acetabulum 
D. High thigh injuries
E. Pelvic fractures
A

C

Central fracture to the acetabulum – this is actually possible but less likely

How well did you know this?
1
Not at all
2
3
4
5
Perfectly
162
Q
Long term complications of ileoanal anastomosis:
A. Stricture
B. Inflammation of mucosa 
C. Diarrhea
D. Bleeding
A

B

Inflammation of mucosa (cuff remnant mucositis)

How well did you know this?
1
Not at all
2
3
4
5
Perfectly
163
Q
The reason to resect a sacrococcygeal teratoma early:
A. Cosmetic
B. Propensity for malignancy
C. To improve walking
D. To prevent paralysis
A

B

ASCRS Textbook:

Presacral tumours: 2/3 congenital, 2/3 benign. Can be categorized by congenital, neurogenic, osseous, or miscellaneous categories. Usually present late. More common in females (cystic), solid tumours (chordoma) more common in males.

Dermoid or epidermoid cysts, enteric duplication cysts.
Teratomas are neoplastic, derived from all 3 germ layers. Malignant degeneration in 40-50%. (and in Schwartz)
Neurogenic: meningoceles, chordoma, neurogenic tumours.
Osseous tumours: chondroma, osteochondroma, Ewings, myeloma

How well did you know this?
1
Not at all
2
3
4
5
Perfectly
164
Q
What is most likely cause of urinary retention after APR:
A. Damage of bladder muscle nerves 
B. Damage to bladder neck angle
C. Damage to bladder sphincter nerves
D. Benign prostatic hypertrophy
A

A

Damage to pelvic autonomic/hypogastric plexus, nervi erigentes results in flaccid retention

How well did you know this?
1
Not at all
2
3
4
5
Perfectly
165
Q
Most common part of bowel that is injured by radiotherapy:
A. Proximal jejunum
B. Terminal ileum
C. Rectum
D. Sigmoid colon
E. Duodenum
A

C

Rectum is overall most common, TI most common portion of SB
Ileum most commonly involved – 73%
Many authors argue for always doing an ileocecal resection when operating for CRE as it has lower anastomotic complications and less reoperation in future.
Operative and long term results after surgery for chronic radiation enteritis. Am J Surg, Sept 2001.
Small bowel was involved in 93 (85%) patients: ileum in 68, jejunum and ileum in 23, and jejunum in 2 patients.

How well did you know this?
1
Not at all
2
3
4
5
Perfectly
166
Q

42 year old man, hemorrhoids x 2 banded. Presents 36 hours later with rectal pain, perianal tenderness, fever and leukocytosis. Most appropriate management?
A. Antibiotics and EUA
B. I & D
C. Foley Catheter
D. Remove bands
E. Pain killers, sitz baths, stool softeners

A

A

How well did you know this?
1
Not at all
2
3
4
5
Perfectly
167
Q

58 year old man presents 1 month after APR for cancer with persistent, small volume purulent drainage from perineal wound. 1 cm wound extending to sacrum, 8 cm in length. Best treatment
A. Conservative management for at least 2 further months
B. Open, excise granulation tissue, pack open
C. Open, excise granulation tissue, suture closed over a drain
D. Open, excise coccyx, lower part of sacrum, pack open
E. Open, excise coccyx, suture closed over a drain

A

A

How well did you know this?
1
Not at all
2
3
4
5
Perfectly
168
Q

Causes of faecal incontinence after fistula-in-ano surgery
A. Post op infection
B. Division of puborectalis
C. Division of inferior rectal nerves
D. Division of part of internal sphincter
E, Failure to drain horseshoe abscess

A

D

How well did you know this?
1
Not at all
2
3
4
5
Perfectly
169
Q
Most frequent finding in peri-anal Crohn’s disease
A. Fistula-in-ano
B. Perianal abscess
C. Skin tag
D. Anal fissure
A

C

Up to Date:
Abscess: 50%
Fistula: 20-30%
Fissure: 20%
Skin tag:  not mentioned

Schwartz 1038: most common is anal skin tag
AGA technical review on perianal Crohn’s disease. Nov 2003.
Skin tag: 37%
Fistula: 26%
Abscess: 16%
Fissure: 19%
Very high overall incidence of skin tags; the sources that compare directly all have anal skin tag incidence the highest.

How well did you know this?
1
Not at all
2
3
4
5
Perfectly
170
Q
Most cephalad border of ischiorectal fossa?
A. Puborectalis
B. Pubic tubercle
C. Levator ani
D. External sphincter
E. Pubic ramus
A

C

Boundaries of ischiorectal fossa
Anterior : Perineal membrane (and superficial transverse perineal muscle)
Lateral : Obturator internus muscle (& fascia) on ischial tuberosity
Supero-medial : Levator ani muscle
Posterior :Gluteus maximus (& sacrotuberous ligament)

How well did you know this?
1
Not at all
2
3
4
5
Perfectly
171
Q
26 year old male, acute onset of pain at anus after straining for stool. On exam, small tender blue swelling. Diagnosis?
A. Sentinel pile (Anal fissure)
B. Perianal abscess
C. Prolapsed external haemorrhoid
D. Strangulated internal haemorrhoid
E. Thrombosed external haemorrhoid
A

E

How well did you know this?
1
Not at all
2
3
4
5
Perfectly
172
Q

Which of the following is an indication for 1 stage fistulotomy:
A. Anterior transphincteric fistula in a female
B. Extrasphincteric fistula in a crohn’s patient
C. Lateral suprasphincteric fistula in a male
D. Posterior transphincteric fistula in anyone
E. Fistula secondary to radiation

A

D

How well did you know this?
1
Not at all
2
3
4
5
Perfectly
173
Q
What is the most frequent problem with abdominal repair of  rectal prolapse
A. Urinary retention
B. Constipation
C. Rectal incontinence
D. Rectal bleeding
A

B

Can worsen constipation, incontinence improves only 50% of time

How well did you know this?
1
Not at all
2
3
4
5
Perfectly
174
Q
With respect to chronic anal fissure, which is true?
A. Sublingual nitro is proven
B. Lat external sphincterotomy is proven
C. Botox and topical nitro are effective
D. Fissurectomy is treatment of choice
A

C

Botox and topical nitro are effective (50-60%)
Sx tx is lateral internal sphincterotomy

How well did you know this?
1
Not at all
2
3
4
5
Perfectly
175
Q
Woman with incontinence to gas and liquid since birth of last child 10 yrs ago.  Decreased squeeze and palp anterior defect.  Best treatment:
A. Artificial sphincter
B. Gracillis repair
C. Overlapping sphincteroplasty
D. Parks post repair
E. Colostomy
A

C

How well did you know this?
1
Not at all
2
3
4
5
Perfectly
176
Q

Most common longterm complication of ileo-anal pouch:
A. Stenosis
B. Leak
C. Mucosal inflammation

A

C

Pouchitis 23-59%

How well did you know this?
1
Not at all
2
3
4
5
Perfectly
177
Q
Best treatment for grade II and III hemorroids
A. Hemorroidectomy
B. Banding
C. Sclerosis
D. Coagulation
A

B

How well did you know this?
1
Not at all
2
3
4
5
Perfectly
178
Q
Superficial inguinal lymph nodes drain:
A. Spongy urethra
B. Clitoris
C. Vulva
D. Testicle
E. Ovaries
A

C

How well did you know this?
1
Not at all
2
3
4
5
Perfectly
179
Q
POD10 transvaginal hysterectomy, stool coming from vagina.  Best initial management
A. Loop ileo
B Transvaginal repair
C. Anorectal flap
D. LAR
A

A

How well did you know this?
1
Not at all
2
3
4
5
Perfectly
180
Q
Lap LAR. Sick 5 days later.  Mild tenderness.  CT shows 1 cm contained leak.
A. ABx and observe
B. Laparotomy with Hartman’s
C. Loop ileo
D. Laparoscopy with drainage
A

A

How well did you know this?
1
Not at all
2
3
4
5
Perfectly
181
Q

LAR, sick, CT shows 2 cm leak
A. Hartman’s
B. Repair anastomosis
C. Repair anastomosis with diversion

A

A

How well did you know this?
1
Not at all
2
3
4
5
Perfectly
182
Q
Pouchitis
A. Broad spectrum abx
B. 5 asa enemas
C. Probiotics
D. Steroid enemas
A

A

How well did you know this?
1
Not at all
2
3
4
5
Perfectly
183
Q

30 y male, diarrhea, weight loss and crampy pain, comes to see you because of bothersome anal skin tags. On exam, bluish hemorrhoids, anal skin tags. Mgmt
A. Excise tags
B. Biopsy tags
C. Colonoscopy

A

C

How well did you know this?
1
Not at all
2
3
4
5
Perfectly
184
Q

Young guy with 48 hours of pain with small blue tender nodule distal to dentate.
A. Sitz baths
B. Incision and drainage
C. Excision

A

C

How well did you know this?
1
Not at all
2
3
4
5
Perfectly
185
Q

Woman with Crohn’s bleeding PR, feeling of something going in and out, colonoscopy shows combined internal and external hemorrhoids
A. Conservative management (then band ligation)
B. Hemorrhoidectomy
C. Banding

A

A

Desperately try to avoid hemorrhoidectomy in CD: high rate of wound failure.

AGA technical review on perianal Crohn’s disease. Nov 2003.

Simple hemorrhoidectomy, the newer procedures for prolapsing hemorrhoids, and banding of hemorrhoids in patients with Crohn’s disease are usually contraindicated due to the frequent occurrence of postoperative complications, including poor wound healing, anorectal stenosis, and a high rate of proctectomy,12, 151and152 notwithstanding one recent report suggesting that simple hemorrhoidectomy can be safely performed in selected patients.153 When symptomatic prolapsing or bleeding hemorrhoids fail to respond to conservative measures, in the absence of active anorectal Crohn’s disease, elastic band ligation may be used with great effect (Victor Fazio, unpublished experience, May 2003). It should also be noted that in patients without a preceding history/diagnosis of Crohn’s disease who have a nonhealed hemorrhoidectomy wound 2–3 months postoperatively, investigation including colonoscopy is warranted to rule out occult Crohn’s disease.

How well did you know this?
1
Not at all
2
3
4
5
Perfectly
186
Q
Crohn’s disease post proctectomy and end colostomy for stricture comes in with dehiscence of perineal wound. Elevated WBC and perineal wound draining pus from an 8cm cavity.
A. Antibiotic and daily packing
B. VAC
C. Debridement and primary closure 
D. Gracillus flap
E. Abx and secondary closure
A

A

How well did you know this?
1
Not at all
2
3
4
5
Perfectly
187
Q

Male had radiation for prostate cancer, now with radiation proctitis and requiring multiple transfusion for bleeding. Has tried topical and oral steroids. Still bleeding. Mx?
A. Angio
B. Laser
C. Plasma beam coagulator

A

C

How well did you know this?
1
Not at all
2
3
4
5
Perfectly
188
Q
39yo man presents with five days of perianal pain, improving in the last 24 hours. Exam shows prolapsed thrombosed internal hemorrhoids. Best management?
A. Sitz baths &amp; laxatives
B. Banding
C. Hemorrhoidectomy
D. Incision &amp; drainage
A

A

How well did you know this?
1
Not at all
2
3
4
5
Perfectly
189
Q
Alcoholic male presents with retrosternal pain after vomiting. Imaging shows small contained leak in mid esophagus. Stable. Improves on conservative mgmt.
A. Observe
B. Stent
C. Thoracotomy and primary repair
D. Laparotomy and drain mediastinum
A

A

Surgery is not indicated for every patient with a perforation of the esophagus and management is dependent on several variables—stability of the patient, extent of contamination, degree of inflammation, underlying esophageal disease, and location of perforation.

How well did you know this?
1
Not at all
2
3
4
5
Perfectly
190
Q
Scope screening for Barrett’s, distal esophageal perforation seen in distal mediastinum. Worsens with conservative management
A. Continue antibiotics
B. Stent
C. L thoracotomy
D. Laparotomy
A

C

How well did you know this?
1
Not at all
2
3
4
5
Perfectly
191
Q

Patient with paraesoph. Hiatus hernia with evidence of pain. Taken to the OR with contents reduced, no necrotic stomach. Mgmt?
A. Crural repair
B. Crural repair with prosthetic mesh and fundo

A

A

How well did you know this?
1
Not at all
2
3
4
5
Perfectly
192
Q

Roux en Y esophageal-jejunostomy, CXR post-operatively reveals large pleural Lt effusion. Best initial management.
A. Upper GI series, water soluble
B. Barium swallow
C. Endoscopy

A

A

Gastrograffin followed by thin barium if no leak seen.

How well did you know this?
1
Not at all
2
3
4
5
Perfectly
193
Q

Patient with submucosal lesion of esophagus, best management?
A. Endoscopic biopsy
B. Thoracic enucleation
C. Esophagectomy

A

B

Likley benign esophageal leiomyoma

Biopsy not required. Diagnosis best on barium swallow.
Resect if symptoms, >2cm, or cannot r/o GIST. EUS is recommended by some, biopsy is useful but makes eventual surgery more difficult.

Sabiston
Leiomyomas are slow-growing tumors with rare malignant potential that continue to grow and become progressively symptomatic over time. Although observation is acceptable in patients with small (<2 cm) asymptomatic tumors or other significant comorbid conditions, surgical resection is advocated for most leiomyomas; however, imatinib (a tyrosine kinase inhibitor), as targeted therapy used on other GIST tumors, may have some benefit for esophageal leiomyomas. Surgical enucleation of the tumor remains the standard of care and is performed through a thoracotomy or with video or robotic assistance. Lesions of the proximal and midesophagus are removed through the right chest; those of distal origin are removed through the left chest.

Pearson’s Thoracic and Esophageal Surgery:
However, biopsy of intramural tumors (e.g., leiomyoma) is contraindicated because adequate pathologic material to exclude malignancy is impossible to obtain and violation of the mucosal layer may complicate subsequent surgical resection. Esophageal endoscopic ultrasonography (EUS) may further help in the diagnosis, planning of surgery, and follow-up of these tumors.

How well did you know this?
1
Not at all
2
3
4
5
Perfectly
194
Q
Endoscopy with biopsy for barretts. Develops retrosternal CP in recovery. On imaging see a small contained leak in the distal mediastinum. Start abx and resuscitate and continues to have temp of 38.2 and chest pain. Plan? 
A. Continue resuscitation
B. Transabdominal repair with fundo
C. Left thoractomy and repair
D. Endoscopic stent
A

C

Requires source control and control of contamination (stent and drain is a potential option)

How well did you know this?
1
Not at all
2
3
4
5
Perfectly
195
Q

35y M going for hellers for achalasia with a toupet. 1mm perforation 6cm proximal to GEJxn.
A. Open repair with Thal
B. Stent
C. Lap repair and continue planned surgery
D. Lap repair and Dor

A

A in answer key but that’s crazy
D according to Botkin but C would provide some coverage.

Dor 180-200° anterior fundoplication.
Thal 270° anterior fundoplication.

How well did you know this?
1
Not at all
2
3
4
5
Perfectly
196
Q
80 yr male POD #5 Total gastrectomy for ca with esophagojejunostomy. WBC elevated, dyspneic, febrile. Imaging shows large left pleural effusion. Drains 1600mL foul-smelling clear, Gram +ve organisms. Likely etiology of this presentation?
A. Empyema
B. Pneumonia
C. Anastomotic leak 
D. Subphrenic abscess
A

C

How well did you know this?
1
Not at all
2
3
4
5
Perfectly
197
Q
Open fundo years ago with complete SBO. No radiologic improvement after 24 hours, symptoms improved with NG
A. Laparotomy and LOA
B. Continue non-operative management
C. Long intestinal tube
D. Lap LOA
A

B

How well did you know this?
1
Not at all
2
3
4
5
Perfectly
198
Q
When doing Fundo what do you avoid when going through lesser omentum
A. Vagus branch to the liver
B. Left vagus
C. Right vagus
D. Left gastric artery
A

A

Effect of preserving the hepatic vagal nerve during laparoscopic Nissen fundoplication on postoperative biliary functions.
Ozdogan M et al

The patients (n = 40) were prospectively randomized into two groups. The HB-AVn was preserved during the dissection of the lesser omentum in the first group. The nerve was cut in the second group. Postoperative fasting gallbladder volumes were calculated by ultrasonography. Postoperative gallbladder ejection fraction (GEF) and gallbladder emptying time (GET) were determined by calculating intestinal transit time scintigraphically.

Sacrificing the hepatic branch causes prolongation in the GET. This change in the motor functions of the gallbladder does not cause any symptomatic effect during the early postoperative period. However, the delay in the GET may increase the risk of gallbladder stone formation in the long term.

How well did you know this?
1
Not at all
2
3
4
5
Perfectly
199
Q

EGD for dysphagia. Biopies of distal esophagus show eosinophilic esophagitis
A. Fluticasone proprionate
B. Oral prednisone
C. Repeat EGD in 3-6 months

A

A

Commonly used treatments include:

  • Elimination and elemental diets to decrease allergen exposure.
  • Acid suppression to treat gastroesophageal reflux disease, which may mimic or contribute to eosinophilic esophagitis. In addition, a subset of patients respond clinically and histologically to proton pump inhibitors. Such patients have been referred to as having proton pump inhibitor responsive esophageal eosinophilia.
  • Topical glucocorticoids to decrease esophageal inflammation. (Fluticasone, budesonide, ciciesonide)
  • Esophageal dilation to treat strictures.
How well did you know this?
1
Not at all
2
3
4
5
Perfectly
200
Q
Stricture in distal esophagus in patient with reflux and esophagitis, already on PPI. Rest of work up normal 
A. Collis
B. Esophageal stricturoplasty
C. Dilate and perform Fundo
D. Dilate
A

C

Sabiston
All strictures should
be biopsied to rule out malignant processes and can frequently be
managed with dilation if they are benign. Metaplastic changes (Barrett esophagus) should be biopsied in four quadrants every centimeter to evaluate for dysplasia and cancer. Fundoplication
procedures can still be performed in this setting, but surveillance must continue at regular intervals because regression is rare

How well did you know this?
1
Not at all
2
3
4
5
Perfectly
201
Q
Submucosal lesion ?GIST in mid-esophagus
A. Segmental resection
B. Endoscopic resection
C. EUS biopsy
D. VATS resection
A

D

Enucleate. Don’t biopsy, makes eventual surgery much more difficult.

How well did you know this?
1
Not at all
2
3
4
5
Perfectly
202
Q
Esophageal perf/Boerhaave's with small contained perforation in mediastinum. Mild temp of 37.9. No other vitals given. Given Abx, NPO, etc. 
A. Observe
B. Stent
C. Repair though left thoracotomy
D. Repair though right thoractomy
A

A

How well did you know this?
1
Not at all
2
3
4
5
Perfectly
203
Q

Another post-vomitting retrosternal chest pain and pneumomediastinum. This time has a temp of 38.4 or so.
A. Left thoracotomy
B. CT with contrast
C. EGD

A

B

Should do some sort of imaging to localize perforation, assess for collections, and plan management. Contrast swallow or CT with oral contrast should be done.

How well did you know this?
1
Not at all
2
3
4
5
Perfectly
204
Q

Patient with dysphagia, motility study shows high amplitude nonsynchronous contractions
A. Botox
B. Myotomy
C. Calcium Channel Blockers

A

C

Patient has Diffuse esophageal spasm

Long esophagomyotomy with Dor if fails medical management.

How well did you know this?
1
Not at all
2
3
4
5
Perfectly
205
Q
Patient has dysphagia. Upper GI swallow shows “corkscrew esophagus.” Manometry shows uncoordinated contractions. What is the best management?
A. Calcium channel blockers
B. Botox
C. Dilation
D. Myotomy
A

A

How well did you know this?
1
Not at all
2
3
4
5
Perfectly
206
Q

Guy with Ivor Lewis esophagectomy, presents with a small leak, contained in mediastinum, not septic
A. Observe
B. Stent
C. Thoracotomy for T tube through anastomosis
D. Thoracotomy for external drain

A

B in answer key

In the Ivor Lewis esphagectomy, the esophageal tumor is removed through an abdominal incision and a right thoracotomy (a surgical incision of the chest wall). The esophagogastric anastomosis (reconnection between the stomach and remaining esophagus) is located in the upper chest. For this reason, the Ivor Lewis esophagectomy is suitable for patients with resectable tumor of the middle to lower third of the esophagus and gastroesophageal junction.

Uptodate:
Thoracic anastomotic leaks are more likely to require re-exploration for appropriate control [61], although endoscopic stenting or transluminal vacuum therapy may provide acceptable outcomes in selected circumstances [62]. As an example, 49 patients were treated endoscopically for esophageal anastomotic leaks after cancer surgery [63]. Thirty-one patients had a covered stent placed across the leaks; three patients had the leaks closed with clips. After a median follow-up of 83 days, 88 percent of patients achieved healing of their leaks. Of the 23 patients who received more than one endoscopic intervention, 96 percent healed their leaks. However, stent migration remains a challenge, especially in the thoracic anastomotic position. Furthermore, there remains concern that the radial forces exerted by expandable stents could worsen regional ischemia and thus cause more significant tissue loss. Finally, stent erosion into surrounding structures such as the aorta and airway remains a risk with prolonged use of this technology [64].

How well did you know this?
1
Not at all
2
3
4
5
Perfectly
207
Q

Old guy, previous lung resection for cancer, has dysphagia. Barretts esophagus with high grade dysplasia and a 0.7 cm nodule
A. Esophagectomy
B. EMR
C. Photodynamic therapy
D. Aggressive medical management with repeat scope in 3 months

A

B

EMR for tissue; high likelihood nodule has invasive component, so surveillance is not appropriate. Esophagectomy contraindicated, not fit for surgery. PDT acceptable but contraindicated by nodule.

ACG Guidelines BE 2015
In patients with nonnodular BE, the utility of ablative therapy is becoming clearer. In patients with BE and HGD, ablative therapy should be preferred over either esophagectomy or intensive endoscopic surveillance because of its proven efficacy ( 63 ) and a side-effect profile superior to surgery ( 147 ). Recent data demonstrate that in patients with BE and LGD confirmed by a second pathologist, ablative therapy results in a statistically and clinically significant reduction in progression to the combined end point of HGD or EAC, or to EAC alone ( 142 ).

How well did you know this?
1
Not at all
2
3
4
5
Perfectly
208
Q
Best initial management of patient presenting with esophageal varices
A. Octreotide
B. Vasopressin
C. Banding
D. Beta blocker
A

A

For acute variceal hemorrhage, pharmacotherapy is indicated at the time of diagnosis while awaiting endoscopy. Octreotide is preferred over vasopressin (higher bleeding control, no side effects). Terlipressin used in Europe as first line. Antibiotic prophylaxis indicated as high rate of septic complications with AVH.

Endoscopic BL or sclerotherapy is succesful in 90%., EVL is 1st line

Portal decompressive procedures if fails endoscopic/pharmacologic. TIPS 1st, surgical shunt as last resort, very effective but 50% mortality.

Hemospray, SEMS, esophageal transection also described

How well did you know this?
1
Not at all
2
3
4
5
Perfectly
209
Q
Esophageal ca, what would make this resectable?
A. Involvement of crura
B. Virchow’s node
C. Celiac node
D. Mediastinal node
A

A

How well did you know this?
1
Not at all
2
3
4
5
Perfectly
210
Q

Previous lap fundo, recurrence of symptoms not responding to medical manamgnet. EGD normal.
A. ph and manometry
B. Redo fundo

A

A

Must do motility studies before redo.

How well did you know this?
1
Not at all
2
3
4
5
Perfectly
211
Q
Esophageal cancer, lower third. Which is resectable?
A. Positive virchow’s node
B. Positive paraaortic node
C. Diaphragm invasion
D. Positive cervical node
A

C

How well did you know this?
1
Not at all
2
3
4
5
Perfectly
212
Q
Male, barrett’s 10 cm. Nodule on EGD and high grade dysplasia. Mgmt:
A. Repeat endoscopy in 3 month
B. High dose PPI
C. Photoablation
D. EMR
A

D

Nodule requires resection

How well did you know this?
1
Not at all
2
3
4
5
Perfectly
213
Q

Epiphrenic diverticulum with chest pain. Mgmt.
A. Diverticulectomy, myotomy
B. Diverticulectomy
C. Diverticulectomy, myotomy, fundoplication

A

C

Pearson’s Thoracic and Esophageal Surgery:

Epiphrenic diverticula comprise approximately 20% of all esophageal body diverticula. They are frequently alluded to as separate pathologic entities from other pulsion diverticula. There is, however, sufficient evidence to show that the same underlying mechanisms as in midthoracic pulsion diverticula are present. Hypertensive lower esophageal sphincter and increased tone within the esophagus create a high-pressure area above the sphincter, resulting in outpouching of the mucosa through what is thought to be a weak area of muscle.
Epiphrenic diverticula are often seen in association with achalasiaas well as with diffuse esophageal spasm. The natural history, complications, and principles of treatment are, therefore, the same as for midesophageal diverticula, and a concomitant nonobstructing partial fundoplication is therefore advised.

Surgical Treatment of Epiphrenic Diverticula: A 30-Year Experience, The Annals of Thoracic Surgery, Volume 84, Issue 6, December 2007

Traditional transthoracic resection, long esophagomyotomy, and an antireflux procedure provide excellent long-term functional results with relatively low postoperative morbidity in patients with epiphrenic diverticula.

How well did you know this?
1
Not at all
2
3
4
5
Perfectly
214
Q

Post fundo, recurrent symptoms. No evidence of recurrent hiatal hernia. Mgmt
A. Manometry and pH study
B. PPI
C. Redo surgery

A

A

How well did you know this?
1
Not at all
2
3
4
5
Perfectly
215
Q

Treatment for eosinophilic esophagitis?
A. Endoscopic dilation
B. Oral prednisone
C. Fluticasone proprionate

A

C

How well did you know this?
1
Not at all
2
3
4
5
Perfectly
216
Q
Male smoker with enlarged jugulodigastric node.  Biopsy showed adenocarcinoma.
A. Lung cancer 
B. Salivary cancer 
C. Laryngeal cancer
D. Nasopharyngeal cancer
A

B

Lung cancer – Level 2B node too high for lung primary.

Salivary cancer – only H&N malignancy that is adenocarcinoma.

How well did you know this?
1
Not at all
2
3
4
5
Perfectly
217
Q
Distal esophageal adenoCa, involving crura and enlarged nodes around L gastric.  No distant mets.  Management
A. Total gastrectomy
B. Esophagectomy with resection of crura
C. Chemotherapy
D. Radiation
A

B in answer key

Neoadjuvant seems more appropriate given enlarged LNs

Sabiston
For esophageal adenocarcinoma, mostly located in the distal esophagus or GEJ, we consider nodal disease located in the area from the celiac axis up to the paratracheal region to represent regional disease; nodal disease located outside of these boundaries
is regarded as distant disease.

Surgical resection of the esophagus was the mainstay of esophageal cancer treatment in the past. However, we have learned that even the most radical resections with extensive lymph node dissections
are not adequate to cure locoregionally advanced disease in the majority of cases.

The most notable and frequently quoted trial
that compared chemoradiation followed by surgery with surgery
alone for esophageal and EGJ cancer was the Chemoradiotherapy
for Oesophageal Cancer Followed by Surgery Study (CROSS).57 This trial enrolled an impressive 368 patients during a 4-year period, and 366 patients were included in the final analysis. The
surgery-alone group consisted of 188 patients, whereas 178 underwent
chemoradiation followed by surgery. The majority (75%) of
the patients had adenocarcinoma, and 22% had SCC. The chemoradiation
regimen consisted of a 5-week course of carboplatin and paclitaxel administered concurrently with radiation therapy at a dose of 41.4 Gy given in 23 fractions 5 days a week. Esophagectomy
was performed within 4 to 6 weeks in the treatment group
and immediately after randomization in the control group. The completeness (R0) of resection was higher in the trimodality group
than in the surgery-alone group (92% versus 69%; P < .001). Patients with SCC experienced complete pathologic response
(ypT0N0M0) significantly more than patients with adenocarcinoma
(49% versus 29%; P < .001). Expectedly, nodal positivity
was higher in patients with surgery alone compared with the trimodality group (75% versus 31%; P < .001). At a median follow-up duration of 45 months, patients receiving the trimodality therapy had significantly longer median overall survival. duration (49.4 months) than did patients undergoing surgery
alone (24 months; hazard ratio [HR], 0.65; 95% confidence interval [CI], 0.49-0.87; P = .003). The estimated 5-year survival rate in the trimodality therapy group was 47% compared with
34% (HR, 0.65; 95% CI, 0.49-0.87; P = .003) in the surgery
group. Interestingly, trimodality therapy did not significantly benefit patients with adenocarcinoma histology (HR, 0.74; 95%
CI, 0.53-1.02; P = .07), and inexplicably it benefited patients with clinically node-negative disease (HR, 0.42; 95% CI, 0.23- 0.74; P = .003) but not those with node-positive disease (HR,
0.80; 95% CI, 0.57-1.13; P = .21).

How well did you know this?
1
Not at all
2
3
4
5
Perfectly
218
Q
Patient post-Lap fundo with dysphagia with solids, lost 8kg.  Repeated 24pH and manometry without any abN findings.  Barium swallow shows lumen that’s 4mm at GEJ or close to the wrap.  
A. Redo fundo
B. Nasogastric feeding tube 
C. Prokinetic and liquid diet 
D. Endoscopic dilation
A

B then D after wt improved?

SAGES:

Early postoperative dysphagia rates are up to 50% and the general recommendation is for slow advancement of diet from liquids to solids. Attention should be paid to adequate caloric and nutritional intake in the postoperative period. Expert opinion suggests that most patients will lose 10-15 pounds (4.5 – 7 kg) with laparoscopic fundoplication and hernia repair followed by a graduated diet from liquids to soft solids. If dysphagia persists or weight loss occurs of 20 or more pounds (9 kg) evaluation and intervention for the dysphagia should be considered.

Depends on time. If early (<12 weeks), prokinetic and liquid diet, if >12 weeks, dilation.

Uptodate:
Patients with dysphagia in whom the 13 mm barium tablet passes slowly through the esophagus and who had normal motility preoperatively should be considered candidates for dilation after 12 weeks. Approximately 6 to 12 percent of patients with fundoplication required dilation in various reports.
Patients who have a 360 degree fundoplication may be candidates for revision to a partial fundoplication if dysphagia persists and effective barium tablet passage cannot be established.

How well did you know this?
1
Not at all
2
3
4
5
Perfectly
219
Q
Old guy, smoker, COPD, restricted functional activity.  Doesn’t sound like a good OR candidate.  Has dysphagia and found to have 3cm segment of Barrett’s on scope with LGD on biopsy of this raised area confirmed by 2 pathologists.  Management
A. Photodynamic therapy
B. EMR 
C. Esophagectomy
D. Maximize medical therapy
A

B

EMR – histology, short segment makes it appropriate – also describes nodule

SCNA:

PDT is falling out of favour; can have buried glands, and has high (40%) stricture rate. RFA is actually treatment of choice. EMR is most useful when there is a visible nodule or short segment BE.

RFA is treatment of choice for HGD and should be considered for LGD.

How well did you know this?
1
Not at all
2
3
4
5
Perfectly
220
Q

Old guy w/ poor performance status was found to have 3cm Barrett’s esophagus. Biopsy w/ confirmation path showing HGD. What to do

A. Intense medical Tx
B. Photo dynamic therapy
C. Esophagectomy
D. EMR

A

D

Sabiston:

Endoscopic mucosal resection (EMR) has gained favor for the treatment of Barrett’s esophagus with low-grade dysplasia. Also, it has been used as a diagnostic tool to rule out cancer in a focus of Barrett’s esophagus with high-grade dysplasia. Because of an increase in stricture rate with larger resections, it is not advocated for long-segment Barrett’s esophagus. It is acceptable for patients with high-grade dysplasia who are not acceptable candidates for esophageal resection and useful for patients who have an isolated focus of Barrett’s esophagus with dysplasia.

How well did you know this?
1
Not at all
2
3
4
5
Perfectly
221
Q

Young guy post blunt trauma w/ initial image shows pneumomediastinum. Found on PAD#4 with retrosternal chest pain, fever, CT contrast study show extravasation from distal esophagus into posterior mediastinum w/ severe inflammation. Tx?

A. Cervical esophagostomy, gastrostomy, jejunostomy
B. Esophageal stent
C. Primary esophageal repair w/ gastric fundoplication

A

A

How well did you know this?
1
Not at all
2
3
4
5
Perfectly
222
Q

Type 2 gastric varices, had profuse bleeding
A. Cyanoacrylate
B. Splenectomy
C. TIPS

A
Gastroesophageal varix (GOV) Type 1: Extension of esophageal varices along lesser
Gastroesophageal varix type 2: Extension of esophageal varices along greater curve

Isolated gastric varix (IGV) type 1 near fundus of stomach and
Isolated gastric varix type 2: Varices in stomach or duodenum.

Gastric varices: Classification, endoscopic and ultrasonographic management
J Res Med Sci. 2015 Dec; 20(12): 1200–1207.
Gastroesophageal varix type 1 is the most common type, accounting for 74% of all GV. However, the incidence of bleeding is highest with IGV type 1, followed by GOV type 2. Overall, the most important predictor of hemorrhage is the size of varices, with the highest risk of first hemorrhage (15%/year) occurring in patients with large varices. Other predictors of hemorrhage are decompensated cirrhosis (Child B or C) and the endoscopic presence of red wale marks

The endoscopic sclerotherapy has been less effective in the treatment of gastric variceal bleeding and eradication of GV as against esophageal varices where endoscopic sclerotherapy is one of the effective modes of treatment.[12,18] Because of the high volume of blood flow through GV compared with EV, resulting in rapid flushing away of the sclerosant in the bloodstream. In acute GV bleeding, GVS has been reported to control bleeding in 60-100% of cases[20,21,22] but with unacceptably high rebleeding rates of up to 90%. Mucosal ulcers are also commonly seen, and cause rebleeding. Approximately, 50% of rebleeding is caused by sclerotherapy induced ulcers and is difficult to control, with a success rate between 9% and 44%.GVS appears to be least successful in controlling acute fundal variceal bleeding.[23,24]

Tissue adhesive such as N-butyl-2-cyanoacrylate, which is a monomer that rapidly undergoes exothermic polymerization on contact with the hydroxyl ions present in water, has been used for Gastric variceal obturation.

How well did you know this?
1
Not at all
2
3
4
5
Perfectly
223
Q

Patient with clean ulcer base, no active bleeding, clot removed, best management?
A. IV pantoloc
B. Epinephrine and heater probe
C. Clip

A

A

Acute hemorrhage
Forrest I a (Spurting hemorrhage)
Forrest I b (Oozing hemorrhage)

Signs of recent hemorrhage
Forrest II a (Non bleeding Visible vessel)
Forrest II b (Adherent clot)
Forrest II c (Flat pigmented haematin on ulcer base)

Lesions without active bleeding
Forrest III (Lesions without signs of recent hemorrhage or fibrin-covered clean ulcer base)

Uptodate
Stigmata of recent hemorrhage are present if anything other than a clean ulcer base is seen. However, only patients with active bleeding (spurting or oozing), a nonbleeding visible vessel, or an adherent clot are generally considered to be at high risk for recurrent bleeding. Most patients with high-risk stigmata require endoscopic therapy to decrease the risk of recurrent bleeding, whereas patients without these high-risk stigmata are considered low-risk and do not require endoscopic therapy.

Medscape
Endoscopic treatment is reserved for ulcers demonstrating signs of active or recent hemorrhage (i.e., spurting or oozing ulcers [Forrest Ia and Ib, respectively] and nonbleeding ulcers with visible blood vessels [Forrest IIa]). For-rest Ia, Ib, and IIa ulcers are high-risk lesions with a high probability of re-bleeding, despite achievement of initial hemostasis, and necessitate aggressive endoscopic and pharmacologic management. Nonbleeding ulcers with a clean base or dark spot at the base (Forrest III and IIc, respectively) do not require endoscopic therapy because the probability of rebleeding is low.[15] Management of ulcers covered by an adherent clot (Forrest IIb) is controversial. Most authorities believe that overlying clots should be removed and the underlying ulcer carefully examined. Endoscopic therapy is reserved for patients with high-risk lesions (Forrest Ia, Ib, and IIa ulcers) and withheld in the presence of low-risk lesions (Forrest IIc and III ulcers).[7]

How well did you know this?
1
Not at all
2
3
4
5
Perfectly
224
Q
Patient with clean ulcer base, no active bleeding, clot removed, best management?
A. IV pantoloc
B. Epinephrine and heater probe
C. Clip
D. Antrectomy
A

A

Forrest IIB since there was a clot

Acute hemorrhage
Forrest I a (Spurting hemorrhage)
Forrest I b (Oozing hemorrhage)

Signs of recent hemorrhage
Forrest II a (Non bleeding Visible vessel)
Forrest II b (Adherent clot)
Forrest II c (Flat pigmented haematin on ulcer base)

Lesions without active bleeding
Forrest III (Lesions without signs of recent hemorrhage or fibrin-covered clean ulcer base)

Uptodate
Stigmata of recent hemorrhage are present if anything other than a clean ulcer base is seen. However, only patients with active bleeding (spurting or oozing), a nonbleeding visible vessel, or an adherent clot are generally considered to be at high risk for recurrent bleeding. Most patients with high-risk stigmata require endoscopic therapy to decrease the risk of recurrent bleeding, whereas patients without these high-risk stigmata are considered low-risk and do not require endoscopic therapy.

Medscape
Endoscopic treatment is reserved for ulcers demonstrating signs of active or recent hemorrhage (i.e., spurting or oozing ulcers [Forrest Ia and Ib, respectively] and nonbleeding ulcers with visible blood vessels [Forrest IIa]). For-rest Ia, Ib, and IIa ulcers are high-risk lesions with a high probability of re-bleeding, despite achievement of initial hemostasis, and necessitate aggressive endoscopic and pharmacologic management. Nonbleeding ulcers with a clean base or dark spot at the base (Forrest III and IIc, respectively) do not require endoscopic therapy because the probability of rebleeding is low.[15] Management of ulcers covered by an adherent clot (Forrest IIb) is controversial. Most authorities believe that overlying clots should be removed and the underlying ulcer carefully examined. Endoscopic therapy is reserved for patients with high-risk lesions (Forrest Ia, Ib, and IIa ulcers) and withheld in the presence of low-risk lesions (Forrest IIc and III ulcers).[7]

Am J Gastroenterology
Those with an adherent clot may receive endoscopic therapy; these patients then receive intravenous PPI with a bolus followed by continuous infusion.

How well did you know this?
1
Not at all
2
3
4
5
Perfectly
225
Q

Patient with Crohn’s stricture to 3rd part of duodenum, dilated stomach on CT scan, best management?
A. Gastrojej
B. Strictureplasty
C. Endoscopy dilation

A

A

Difficult location, strictureplasty not effective, G-J best (per Kanthan).

SCNA: Surgical Management of CD
For gastroduodenal Crohn’s disease, bypass is often the optimal treatment. In patients with obstructing duodenal disease, resection has a fourfold increased risk of major morbidity when compared with gastrojejunal bypass. Some investigators advocate for strictureplasty in the case of duodenal disease. To accomplish this, the duodenum must be mobilized, which can be challenging in the setting of inflammatory changes. Gastroduodenal strictureplasty frequently fails, has a high rate of reoperation, and confers little advantage over gastrojejunal bypass. Strictureplasty is best used for selected, proximal duodenal lesions near the pylorus.

How well did you know this?
1
Not at all
2
3
4
5
Perfectly
226
Q

Patient visible vessel on endoscopy, posterior duodenum, no bleeding, management?
A. Epinephrine and heater probe and clips
B. Pantoloc infusion
C. Operative management

A

A

Forrest IIA

How well did you know this?
1
Not at all
2
3
4
5
Perfectly
227
Q
85y with a perforated duodenal ulcer with 5 days of pain. Comorbidities include chronic renal insufficiency, asthma, HTN. contrast study shows an a leak that is largely but contained but flows out into subhepatic space. stable with a fever. 
A. Perc drain
B. Abx
C. OR
D. Endoscopic stent
A

C

Old, comorbid, uncontained leak, delayed presentation are all indications to operate.
UTD: nonoperative management less successful in >70 yrs

Patients chosen for nonoperative management are those with contained perforation, gastrointestinal fistula formation, or limited contamination as judged by imaging, in those who have no signs of systemic sepsis . Not surprisingly, since patients chosen for conservative management in contemporary series are generally less ill, conservative management is often associated with lower rates of morbidity and mortality compared with surgical management.

A conservative approach including antibiotic therapy combined with drainage (effusion, abscess), provision for nutritional support (eg, gastrostomy, feeding jejunostomy), or stent placement may be an appropriate initial management strategy

How well did you know this?
1
Not at all
2
3
4
5
Perfectly
228
Q
68y M UGIB found antral ulcer on mid lesser curve. heavy smoker on PPI. Initially unstable, but then stabilizes
A. Oversew
B. Wedge
C. Antrectomy to include the ulcer
D. V+D
A

C.

Type I ulcer; ulcer excision is preferred, particularly to rule out malignancy. If unstable, oversew; if stable, excise.

Lesser curve wedge is problematic due to vascular arcades, more likely to deform stomach and cause obstruction.

How well did you know this?
1
Not at all
2
3
4
5
Perfectly
229
Q
70M with dilated mucosal veins emanating from antrum, no active bleeding. Had been scoped for hx of iron deficiency anemia with positive FOBT. Management?
A. Endoscopic coagulation
B. Endoscopic banding
C. Antrectomy
D. Address portal hypertension
A

A

GAVE, or watermelon stomach, is an uncommon cause of UGIB that is often confused with Portal HTN Gastropathy, both of which can occur in patients with cirrhosis

The term “watermelon stomach” is derived from the characteristic endoscopic appearance of longitudinal rows of flat, reddish stripes radiating from the pylorus into the antrum that resemble the stripes on a watermelon. The red stripes represent ectatic and sacculated mucosal vessels. A punctate form (in which the red stripes are not apparent) has also been described and appears to be more common in patients with underlying cirrhosis. While acute bleeding may occur, low-grade GI bleeding is more common, often with iron deficiency anemia. It is uncommon for patients to present with acute and massive bleeding.

GAVE is usually an isolated problem but has been associated with cirrhosis and systemic sclerosis. In one series of 744 consecutive patients with nonvariceal UGIB, bleeding was due to GAVE in 4 percent. Portal hypertension was present in 31 percent of the patients with GAVE in this cohort. The most common clinical profile of a patient with GAVE is an older (>70 years old) woman. In the series described above, for example, the median age was 74 years, and 80 percent of the patients were women.

Patients may also present with acute bleeding. The clinical presentation is similar whether portal hypertension is present or not, except that those with portal hypertension may have diffuse antral angiomas rather than the classic linear pattern.

The diagnosis is based on the classic endoscopic appearance. It may be confirmed with endoscopic biopsy, endoscopic ultrasound, tagged red blood cell scan, or computed tomography (CT) scan [66]. Histopathologically, GAVE is characterized by vascular ectasia, spindle cell proliferation, and fibrohyalinosis (picture 8) [63].

Episodic transfusions are required in some patients. Endoscopic coagulation with a heater probe, bipolar probe, argon plasma coagulator, laser therapy, or radiofrequency ablation obliterates the vascular ectasia and decreases the degree of bleeding. Endoscopic band ligation has also been used successfully [68]. (See “Argon plasma coagulation in the management of gastrointestinal hemorrhage”.)

Portal decompression with TIPS does not reliably reduce bleeding, underscoring the uncertain relationship of GAVE to portal hypertension [69,70]. Antrectomy prevents recurrent bleeding but is usually reserved for patients who fail endoscopic therapies. Combination estrogen/progesterone therapy may decrease bleeding, although the ectatic vessels appear to persist [71].

How well did you know this?
1
Not at all
2
3
4
5
Perfectly
230
Q

70s male with prev BII with nausea, bloating relieved by emesis, Unable to intubate afferent limb on EGD, HIDA shows delayed empyting afferent limb
A. Roux-en-y
B. Balloon dilation of anastomosis
C. Prokinetics

A

A

Afferent limb syndrome is an intermittent partial or complete mechanical obstruction of the afferent limb of a gastrojejunostomy.

The syndrome classically refers to obstruction of the upstream limb of a side-to-side gastrojejunostomy, but has also been used to refer to the biliopancreatic limb of a Roux-en-Y gastrojejunostomy.

Afferent loop syndrome is not an uncommon postoperative complication, and one study has estimated that it occurs in 13% of post-pancreaticoduodenectomy patients. Patients usually present with epigastric pain, abdominal distention, nausea, and potentially bilious vomiting. It has been classified as acute (<7 days postoperative) or chronic (>7 days postoperative). Bilious vomiting is presumed to occur from regurgitation of bilious contents in the afferent limb into the stomach after release from intermittent obstruction.

How well did you know this?
1
Not at all
2
3
4
5
Perfectly
231
Q
Post PEG, patient has abdo pain and some subQ emphysema near entry gastrostomy site. What to do
A. Lap
B. Interventional PEG tube
C. Urgent CT 
D. Observe and Feed
A

D? Although C may be reasonable if lot of pain

Uptodate
Subcutaneous air has also been described after PEG tube placement. It occurs from air being introduced between the
cutaneous and subcutaneous tissues [35]. In the absence of other findings, it is inconsequential and should not preclude
feeding [35].

How well did you know this?
1
Not at all
2
3
4
5
Perfectly
232
Q

45 F one year after lap gastric bypass. Now presents with nausea, vomiting. Amylase and lipase three times normal. AXR shows bowel in RUQ. No stones in GB What is the most likely diagnosis?
A. Internal hernia
B. GS Pancreatitis
C. Obstruction of J-J anastomosis

A

B in answer key but A more likely the answer

But gas in RUQ makes it suspicious for Petersen’s hernia

CT showing whorling of small bowel mesentery, presence of cecum and TI in RUQ and existence of majority of small bowel loops on one side of the abdo cavity as triad of CT findings pathognomonic for internal hernia

In RYGB, there is an assoc between SBO and an elevated amylase or lipase. Acute obstruction of biliopancreatic limb can be difficult to dx and in these pts, sensitivity of elevated amylase or lipase is high. It is important to recognize that an elevation of these enzymes is not likely a result of acute pancreatitis

To distinguish BP limb obstruction from pacnreatitis, the latter condition would have lipase elevated in the 1000s.

How well did you know this?
1
Not at all
2
3
4
5
Perfectly
233
Q

Recurrent bleeds from a posterior duodenal ulcer while on PPI, resolved. Not actively bleeding now.
A. Gastrin level
B. V+D
C. Endoscopic injection

A

A

How well did you know this?
1
Not at all
2
3
4
5
Perfectly
234
Q

Duodenal GIST 3 cm which is 5 mm from ampulla. In a guy who has had a bout of acute pancreatitis that is now resolved.
A. Local resection primary transverse closure
B. Local resection duodeno-jejunostomy
C. Whipple
D. Neoadjuvant Gleevec

A

D

Try to increase margin to ampulla to allow localized resection.

How well did you know this?
1
Not at all
2
3
4
5
Perfectly
235
Q
EGD reveals 2 one cm gastric adenomas, removed by polypectomy
A. Wedge Gastrectomy
B. Biopsy for H.Pylori 
C. Repeat EGD in 2 years
D. PPI
A

B

Sabiston:
Adenomatous polyps carry a distinct risk for the development of malignancy in the polyp. Mucosal atypia is frequent,and progression from dysplasia to carcinoma in situ has been observed. The risk for the development of carcinoma is approximately 10% to 20% and increases with increasing size of the polyp. Endoscopic removal is indicated for pedunculated lesions and is sufficient if the polyp is completely removed and there are no foci of invasive cancer on histologic examination. If the polyp is larger than 2 cm, is sessile, or has a proven focus of invasive carcinoma, operative excision is warranted.

Up to Date:
Gastric adenomas typically occur in a background of chronic atrophic gastritis. Gastric adenomas are much less common than fundic gland polyps in patients with FAP; they are typically isolated and located in the antrum and are associated with a relatively low but real risk of progression to cancer.

Adenomas may be flat or polypoid, and are usually <2 cm in size. Adenomas are usually solitary. Most are found in the antrum, but some occur in the corpus and cardia.

It is estimated that 8 to 59 percent of adenomas are associated with synchronous gastric carcinomas [53]. The presence of invasive carcinoma in an adenoma correlates with increasing size, villous contour, and the degree of dysplasia [23,54]. The risk of malignancy is lower in flat adenomas [55]. High-grade dysplasia has been identified in close proximity to a high proportion (40 to 100 percent) of early gastric cancers.

In addition, as adenomatous polyps are associated with atrophic gastritis, the normal-appearing antral and corpus mucosa should be sampled to assess the stage of gastritis and, thus, cancer risk. All patients should be tested for active H. pylori infection and, if present, the infection should be treated

How well did you know this?
1
Not at all
2
3
4
5
Perfectly
236
Q
Pyloric stenosis, patient dehydrated. Best fluid to maintain a 3 kg child after giving NS bolus at 20cc/kg
A. RL at 15 cc/hr
B. D10W + 0.45 NS at 12 cc/hr
C. D something + 5 KCL at 18 cc/hr
D. D5W + 0.45 NS + 20 KCL at 20 cc/
A

D

Pyloric stenosis results in hypochloremic, hypokalemic metabolic alkalosis

Need 20 KCL or more; rehydrate at 150%.

How well did you know this?
1
Not at all
2
3
4
5
Perfectly
237
Q
Gastric biopsy of a 1cm mass showing Mucosal associated lymphocytes. Pathologist then calls to say H.P. also seen on specimen. (??maybe a MALT not sure) What is the best Management?
A. H. Pylori eradication
B. Chemo
C. ChemoRad
D. Antrectomy
A

A

HP eradication is first line. In the rare HP negative patient, RT is very effective with low recurrence. Complete response to eradication can take 12-18 months, so usually defer RT til after that. MALT with t(11:18) is unlikely to respond to eradication and is treated with RT

DLBC lymphoma (high-grade MALT) is treated with upfront chemo (R-CHOP).

How well did you know this?
1
Not at all
2
3
4
5
Perfectly
238
Q

Duodenal sessile polyp 2.1 cm located 5mm from ampulla. Biopsy shows sporadic (not related to FAP) duodenal adenoma with low grade dysplasia…treatment?

A. Whipple
B. Endoscopic resection
C. Transduodenal resection
D. Observation

A

C

Up to 2cm can consider endoscopic resection, but sessile is more difficult. Given size >2cm, sessile, and close to ampulla, transduodenal polypectomy would be best.

Schwartz
Tumors located in the duodenum, including asymptomatic lesions incidentally found during EGD, can pose the greatest therapeutic challenges. These lesions should be biopsied; symptomatic tumors and adenomas, because of their malignant potential, should be removed. In general, duodenal tumors less than 1 cm in diameter are amenable to endoscopic polypectomy. Lesions greater than 2 cm in diameter are technically difficult to remove endoscopically and may need to be removed surgically. Surgical options include transduodenal polypectomy and segmental duodenal resection. Tumors located in the second portion of the duodenum near the ampulla of Vater may require pancreaticoduodenectomy. EUS may offer utility for duodenal tumors ranging in size between 1 and 2 cm in diameter

Recent studies have shown that endoscopic resection of biopsy-proven benign duodenal periampullary adenomas leads to equivalent efficacy to surgery but with lower morbidity. Adenomas can recur; therefore, surveillance endoscopy is required after these procedures.

How well did you know this?
1
Not at all
2
3
4
5
Perfectly
239
Q
Fe deficiency anemia.  EGD shows radial dilation of venous channels at antrum, no active bleeding
A. TIPS
B. Thermocoagulation
C. Banding
D. Antrectomy
A

B

How well did you know this?
1
Not at all
2
3
4
5
Perfectly
240
Q

Lady with occult GIB, endoscopy, CT, angio, RBC scan, all negative, next step
A. Capsule endoscopy
B. Enteroclysis
C. Surgery, with intraoperative endoscopy
D. Push enteroscopy

A

A

How well did you know this?
1
Not at all
2
3
4
5
Perfectly
241
Q

Old guy with perforated ulcer. Found to have 5 cm splenic artery aneurysm incidentally. How to manage aneurysm
A. Ligate proximal and distal to aneurysm
B. Angioembolize
C. Stent
D. Splenectomy

A

A

Sabiston:
Patients with splenic aneurysms may report a history of left upper quadrant or epigastric pain. The term double rupture has been used to describe these aneurysms, but is relatively rare. There is initial contained bleeding in the lesser sac, followed by free hemorrhage into the peritoneal cavity, causing hypovolemic shock. Treatment should be considered in aneurysms larger than 2 cm in diameter. Because of the high mortality rate, treatment is warranted for pregnant women and those of childbearing age. Simple ligation or excision of the aneurysm is preferred to splenectomy. Endovascular repair is emerging as the treatment of choice.

How well did you know this?
1
Not at all
2
3
4
5
Perfectly
242
Q
Dieulafoy lesion, bleeding not controlled with endoscopy
A. Wedge resection
B. Subtotal gastrectomy
C. Gastrotomy with oversewing
D. Angio
A

D

Sabiston: endo, then angio, then wedge

Dieulafoy’s lesion: current trends in diagnosis and management. Ann R Coll Surg Engl. 2010 Oct; 92(7): 548–554.

There is no consensus on the treatment of Dieulafoy’s lesions. Therapeutic endoscopy can control the bleeding in 90% of patients while angiography is being accepted as a valuable alternative to endoscopy for inaccessible lesions. Currently, surgical intervention is kept for failure of therapeutic endoscopic or angiographic interventions and it should be guided by pre-operative localisation

How well did you know this?
1
Not at all
2
3
4
5
Perfectly
243
Q
Young guy, gastric ulcer, bleeding, intractable to medical therapy
A. Wedge resection
B. Subtotal gastrectomy
C. Subtotal with vagotomy
D. Angio
A

B in answer key though D seems like a reasonable choice.

If type I, no vagotomy required

How well did you know this?
1
Not at all
2
3
4
5
Perfectly
244
Q

Aortoenteric fistula. Huge hole in D2 (involving 60%). How to repair duodenum after extraanatomic bypass
A. Resection with distal jejunum to stomach
B. Primary repair
C. Resection with side to side duodenojejunostomy

A

C

Primary repair if small (usual case), otherwise resect portion and do D-J.

How well did you know this?
1
Not at all
2
3
4
5
Perfectly
245
Q

Patient awaiting surgery for brain tumor, having OR for perforated duodenal ulcer. Post op hypotensive, what must be given to patient in his immediate treatment of shock
A. Levophed
B. Activated protein C
C. IV hydrocortisone

A

A

C if previously had been on steroids

How well did you know this?
1
Not at all
2
3
4
5
Perfectly
246
Q
Visible vessel not actively bleeding on scope for UGIB
A. Injection epi and thermocoag
B. Inject epi
C. Observe
D. Oversew
A

A

Forrest IIa; dual methods of control is superior.

How well did you know this?
1
Not at all
2
3
4
5
Perfectly
247
Q
Healthy 65m, mgmt of organoaxial gastric volvulus.
A. Gastropexy
B. Repair hiatal hernia
C. Gastrectomy
D. PEG
A

B

Etiology is hiatus hernia. Organoaxial associated with HH; mesoaxial is not, so treatment for mesoaxial volvulus is gastropexy.

How well did you know this?
1
Not at all
2
3
4
5
Perfectly
248
Q

Third episode of hematemesis and melena. Non healing posterior wall duodenal ulcer. On EGD, non bleeding, no visible vessel, no clot. On max medical mgmt, quit smoking. H. pylori has always been negative, culture and serology. Stable. Mgmt.
A. Serum gastrin
B. TV and antrectomy
C. Gastrectomy

A

A

How well did you know this?
1
Not at all
2
3
4
5
Perfectly
249
Q
UGIB, hypotensive. Responds to fluids. On EGD 3 mm mucosal defect with surrounding normal mucosa. Minimal oozing from area.  Mgmt.
A. Endoscopic injection or coagulation
B. Suture ligation
C. Angio and embolize
D. Treat H.pylori
A

A

Dieulafoy’s lesion. 1st line is endoscopic management.

How well did you know this?
1
Not at all
2
3
4
5
Perfectly
250
Q

First trimester pregnant woman with vomiting then hematemesis. Mucosal tear on EGD, can’t stop bleeding. Mgmt.
A. Angio and embolize
B. Gastrotomy and ligate
C. Gastrectomy

A

B

Sabiston:

If these maneuvers fail, high gastrotomy and suturing of the mucosal tear is indicated. It is
important to rule out the diagnosis of variceal bleeding in cases of failed endoscopic therapy by a thorough examination of the gastroesophageal junction.

How well did you know this?
1
Not at all
2
3
4
5
Perfectly
251
Q

Postop RYGB with abdo pain and large gastric air bubble. Mgmt.
A. Percutaneous gastrostomy
B. NG
C. Laparotomy

A

A

Acute gastric remnant distension.

  • Usually after RYGB
  • Due to edema or obstruction of enteroenterostomy site.
  • Dx within first few days post op.
  • N/V, LUQ pain, bloating, hiccups
  • Tx– perc decompression or reoperation
  • NG tube controversial and will not decompress the remnant stomach
How well did you know this?
1
Not at all
2
3
4
5
Perfectly
252
Q
Patient 2 years post Billroth 2, antecolic, for T2N0 gastric cancer. Complains of postprandial abdo pain, forceful bilious vomiting, does not vomit food. Mgmt.
A. Revise to retrocolic gastojejunostomy
B. Braun enteroenterostomy
C. Prokinetic
D. PPI
A

B

Afferent limb syndrome; revise to R-Y or Braun

Sabiston:
Upper endoscopy demonstrates friable, beefy red mucosa. Most patients suffering from alkaline reflux gastritis have had gastric resection performed with a Billroth II anastomosis. Although bile reflux appears to be the inciting event, a number of issues remain unanswered with respect to the role of bile in its pathogenesis. For example, many patients have reflux of bile into the stomach following gastrectomy without any symptoms. Moreover, there is no clear correlation between the volume or composition of bile and the subsequent development of alkaline reflux gastritis. Although it is clear the syndrome does exist, caution needs to be exercised to ensure that it is not overdiagnosed. After a diagnosis is made, therapy is directed at relief of symptoms. Unfortunately, most medical therapies that have been tried to treat alkaline reflux gastritis have not shown any consistent benefit. Thus, for patients with intractable symptoms, the surgical procedure of choice is conversion of the Billroth II anastomosis into a Roux-en-Y gastrojejunostomy, in which the Roux limb has been lengthened to more than 40 cm.

SCNA (Postgastrectomy Syndromes)
Another possible solution to afferent loop syndrome is an afferent to efferent loop bypass. This bypass procedure, originally described by Braun, is particularly useful when dissection of the original gastrojejunal anastomosis is not required or technically difficult.

How well did you know this?
1
Not at all
2
3
4
5
Perfectly
253
Q
Post Billroth 2 with bile reflux. EGD mucosa is friable, beefy red, ulcerated. Mgmt.
A. PPI
B. Prokinetic
C. Convert to Billroth 1
D. Convert to RY gastro J
A

D

SCNA (Postgastrectomy Syndromes)

Although bile reflux and gastritis are more common after B-I and B-II reconstruction than after Roux-Y, debilitating symptoms are infrequent. For the few patients with severe bile reflux gastritis after partial or subtotal distal gastrectomy with B-I or B-II reconstruction, the best solution is to reoperate and convert to Roux-Y anatomy.

How well did you know this?
1
Not at all
2
3
4
5
Perfectly
254
Q

Patient post billroth II with abdo pain, EGD shows alkaline reflux (they give you the diagnosis) and biopsies show chronic inflammation with intestinal metaplasia. What should you do?
A. Promotility agent
B. Convert to Billroth I
C. R-en-Y gastrojejunostomy

A

C

Intestinal metaplasia is a risk factor for adenocarcinoma, but we don’t really know how to follow or treat these patients.

Up to Date says surveillance is recommended, eradicate H. pylori, and no data to support chemoprevention.

How well did you know this?
1
Not at all
2
3
4
5
Perfectly
255
Q
D1 ulcer eroding through into CBD seen on endoscopy.  Not bleeding, patient stable.  
A. HJ
B. IV pantoloc
C. Endoscopic stent
D. Choledochoduodenostomy
A

B

Shaw – no operation unless obstructing or cholangitic.

How well did you know this?
1
Not at all
2
3
4
5
Perfectly
256
Q

Older male with multiple comorbidities, needs some sort of gastrectomy for ulcer and also found to have 3.5cm splenic artery aneurysm, what to do?
A. Gastrectomy + splenectomy
B. Gastrectomy + ligate prox/distal aneurysm
C. Gastrectomy + endovascular

A

B

Most sources say treat when>2cm. If you’re already there for the ulcer, just ligate the thing and spare him the contrast.

How well did you know this?
1
Not at all
2
3
4
5
Perfectly
257
Q

Familial polyposis, found to have 2 polyps in duodenum during screening. Both about 1cm, away from papilla, tubular villous, no LVI. Management?

A. Whipples
B. Endoscopic resection
C. Open polypectomy
D. Observe

A

B

Endoscopic resection – 1st line; if recurrence then open polypectomy

Uptodate
Endoscopically visible duodenal adenomas are identified in more than half of FAP patients. Approximately half of duodenal cancers are ampullary or periampullary. Complete polypectomy or sampling of duodenal polyps should be performed at the time of initial discovery and on each subsequent examination. An abnormal-appearing papilla should be biopsied. Adenomas identified at the ampulla of Vater should be removed endoscopically if possible. Management of high-grade dysplasia in the periampullary region (surgery/ablative therapy versus more frequent surveillance) is controversial and should be individualized based on the patient’s age and the number of duodenal adenomas. The resection margins should be free of neoplastic tissue. A study that included 26 FAP patients who underwent endoscopic ampullectomy demonstrated the procedure can be performed safely, but ongoing surveillance is required because recurrences were common.

The severity of duodenal polyposis, as determined by the Spigelman stage (0 to IV), is used to guide subsequent surveillance (table 1). Surgery (duodenectomy) is reserved for patients with stage IV polyposis.

How well did you know this?
1
Not at all
2
3
4
5
Perfectly
258
Q
Crohns patient 2 prev SB resections, now with 12cm long SB stricture with obstructive symptoms.  What is the management?
A. Resection strictured segment
B. Finney stricturoplasty
C. Heineke Mukilicz stricturoplasty
D. Isoperistaltic stricturoplasty
A

B in answer key

<12cm, HM; 12-25cm Finney appropriate. Maybe this patient is at risk for short bowel syndrome. Don’t do if longer than 25cm as creates a blind loop diverticulum.

But A is also a reasonable choice if no concerns about short gut cause there is a risk of malignancy with stricture

How well did you know this?
1
Not at all
2
3
4
5
Perfectly
259
Q
Best way to treat active bleed from Mallory Weiss tear?
A. Endoscopic control
B. OR gastrostomy and oversewing
C. Angiography
D. Balloon tamponade
A

A

How well did you know this?
1
Not at all
2
3
4
5
Perfectly
260
Q
Female epigastric/upper AP.  Hx of medications for osteoarthritis.  Hemodynically stable.  Laparoscopy showed bile, and perf DU.  How to management?
A. Suture reapir
B. Open omental patch
C. MIS omental patch
D. Drain
A

C

Depends on age. If >70, increased M&M with laparoscopic repair, so do open.

How well did you know this?
1
Not at all
2
3
4
5
Perfectly
261
Q
Patient with history of celiac disease.  Fatigue, weakness, weight loss.  Imaging shows small bowel mass w/ SBO. Most likely diagnosis?
A. AdenoCa
B. Crohns
C. Lymphoma
D. Melanoma
A

C

Enteropathy Assoc T cell Lymphoma

How well did you know this?
1
Not at all
2
3
4
5
Perfectly
262
Q

Perforated gastric ulcer 5cm from GEJ on lesser curve. EtOH user. Stable. What is best management

A. Omental patch and vagotomy
B. Omental patch
C. Pauchet gastrectomy

A

C

If stable, resect b/c of risk of malignancy. If unstable, oversew but needs repeat scopes.

How well did you know this?
1
Not at all
2
3
4
5
Perfectly
263
Q

Patient post LRYGB with SBO taken to OR found to have intussusception at jejunoileal anastomosis. Management
A. Reduce intussusception
B. Open anastomosis to look for lead point
C. Resect and reconstruct anastomosis

A

C

How well did you know this?
1
Not at all
2
3
4
5
Perfectly
264
Q

Patient post LRYGB POD1, not feeling great but hemodynically stable no fever as I recall but has shoulder pain and hiccups. XR done showing large gastric bubble. Management?
A. NG
B. OR
C. Gastrostomy

A

C

Gastric remnant distension—Gastric remnant distension is a rare but potentially lethal complication following gastric bypass [12,13]. The gastric remnant is a blind pouch and may become distended if paralytic ileus or distal mechanical obstruction occurs postoperatively. Iatrogenic injury to vagal fibers along the lesser curvature may also contribute, possibly by leading to impaired emptying of the bypassed stomach. Progressive distension can ultimately lead to rupture, spillage of massive gastric contents, and subsequent severe peritonitis [14]. The combination of the large size of inoculum (liters) and the injurious contents (acid, bile, pancreatic enzymes, and bacteria) makes this complication much more serious than leakage occurring at the gastrojejunostomy.
Clinical features include pain, hiccups, left upper quadrant tympany, shoulder pain, abdominal distension, tachycardia, or shortness of breath. Radiographic assessment may demonstrate a large gastric air bubble.
Treatment consists of emergent operative decompression with a gastrostomy tube or percutaneous gastrostomy [15]. Immediate oative exploration and decompression are required if percutaneous drainage is not feasible, or if perforation is suspected. Although gastrostomy is not performed routinely by most surgeons at the initial gastric bypass operation, drainage of the gastric remnant can prevent this rare but sometimes fatal complication. Routine gastrostomy should be considered in the elderly, super-obese patients, patients with diabetic gastropathy and as part of revisional surgeries where gastric emptying may be delayed.

How well did you know this?
1
Not at all
2
3
4
5
Perfectly
265
Q

Patient post sleeve gastrectomy, febrile, tachy, increase respiratory rate. Hemodynamically stable. U/O 200cc over last 4 hours. Ab tender but no peritonitis. Management
A. CT PE
B. Endoscopy
C. UGI contrast swallow

A

C

UGI contrast swallow – leak, also to assess distal stomach for outflow obstruction

Several points (based on ASMBS paper):

1) Unstable patients should not have imaging studies but go to the OR for exploration
2) CT with oral and IV contrast is more sensitive
3) For GB, early exploration and repair of leak with remnant gastrostomy
4) For GS, early leaks (within 48 hrs) can be managed with operative repair, drainage, and feeding access, but beyond 48 hrs don’t try to repair the leak, just drain and feed.

How well did you know this?
1
Not at all
2
3
4
5
Perfectly
266
Q
Infant history consistent with pyloric stenosis, patient is hypoK, hypoCl and in metabolic alkolosis, what is the next most appropriate step
A. Rehydrate
B. Open pyloromyotomy
C. Lap pyloromyotomy
D. TPN
A

A

How well did you know this?
1
Not at all
2
3
4
5
Perfectly
267
Q

2cm gastric AdenoCa limited to mucosa on biopsy and EUS. No nodes seen. No ulceration. Does not invade muscularis propria
A. EMR
B. Lap wedge
C. Distal gastrectomy with D1
D. Neoadj chemo and distal gastrectomy with D2

A

A

<2cm, not ulcerated, clinically node negative is candidate for EMR.

Up to Date:
Standard and expanded criteria for endoscopic resection—The general guidelines for the selection of patients with EGC who are appropriate for endoscopic resection with EMR or ESD are outlined below [1-4]:

●High probability of en bloc resection
●Tumor histology
•Intestinal type adenocarcinoma
•Tumor confined to the mucosa
•Absence of venous or lymphatic invasion
●Tumor size and morphology
•Less than 20 mm in diameter, without ulceration
•Less than 10 mm in diameter if Paris classification IIb or Iic

Studies have shown high survival and cure rates in patients with EGC who undergo EMR:

Sabiston
Endoscopic therapy for gastric cancer is well established in Eastern countries. Endoscopic resection is a safe and effective technique for patients who meet the criteria and will continue to play an increasing role in the treatment of this disease. As a matter of standard practice, patients with tumors larger than 2 cm, with ulceration or with any submucosal invasion, should be referred for gastrectomy with lymph node dissection if not part of a clinical trial.

How well did you know this?
1
Not at all
2
3
4
5
Perfectly
268
Q
Subtotal gastrectomy for adeno, fozen margins neg, final path with positive proximal margin
A. Total gastrectomy 
B. Chemotherapy
C. Radiation
D. Endoscopically watch q3months
A

A

Controversial; NCCN says chemorads, but surgeons say in fit patient, R0 resection is only chance for cure and can get wider margins so re-resect to negative margins. Different if esophageal margin is positive. If there is metastatic or N2 disease, probably little benefit from re-resection and treatment should be chemorads (or chemo), but if N0 or N1 in a young person, there is survival advantage. Some evidence that positive margins re-resected is equivalent to initially negative margins.

How well did you know this?
1
Not at all
2
3
4
5
Perfectly
269
Q
Staging laparoscopy most indicated for which lesion in gastric cancer
A. T1 with no mets
B. T2 with liver met
C. Obstructing T3
D. T3 with normal CT
A

D

Standard textbook answer, although Toronto manual 2015 says staging laparoscopy for T2 and higher.

How well did you know this?
1
Not at all
2
3
4
5
Perfectly
270
Q

New diagnosis of gastric cancer in male 45 yo. Father died of gastric cancer, mother wants to know about screening for her child. What is the best way to do this?
A. Look for pernicious anemia
B. Eradicate H. pylori
C. Gastroscopy q5-10 years surveillance
D. Test for E-cadherin and if positive gastrectomy

A

D

Up to Date:
Many families with HDGC have germline mutations in the E-cadherin (CDH1) gene that are inherited in an autosomal dominant pattern. The lifetime cumulative risk for clinically significant gastric cancer in individuals from these families is >80 percent in both men and women by age 80; the median age at diagnosis is 38. Because these early gastric cancers are located beneath an intact mucosal surface, early detection is extremely difficult, and prophylactic total gastrectomy is usually advised after age 20, and before the age of 40.
Modified criteria for selection of patients for genetic testing for HDGC have been proposed based upon the experience of the British Columbia Cancer Agency Hereditary Diffuse Gastric Cancer Program:
●Families with two or more documented cases of gastric cancer in first or second degree relatives, with at least one diffuse gastric cancer diagnosed before age 50 years
●Families with multiple lobular breast cancers with or without diffuse gastric cancer in first or second degree relatives
●Individual diagnosed with diffuse gastric cancer before 35 years of age from a low incidence population
●Potential additional criteria which need additional validation in clinical studies include:
•Three or more cases of gastric cancer in first or second degree relatives, diagnosed at any age, one or more of which is a documented case of diffuse gastric cancer (3 percent mutation rate; 1 positive/30 tested)
•Family with one or more cases of both diffuse gastric cancer and signet ring colon cancer (33 percent mutation rate; 1 positive/3 tested).
•Personal history but no family history of diffuse gastric cancer or lobular breast cancer (mutation rate unknown)

How well did you know this?
1
Not at all
2
3
4
5
Perfectly
271
Q
Old guy GOO long standing reflux,vomiting, biopsy shows LGD with fibrosis and inflammation
A. Antrectomy 
B. Gastrojejunostomy
C. Endoscopic stent
D. Endoscopic dilation
A

D

Antrectomy – diagnosis, therapy, risk of CA, deals with obstruction
Endoscopic dilation – also reasonable, but doesn’t mention deeper biopsy; need to exclude deeper invasive
Dilate, PPI, rescope and rebiopsy. LGD has low rate of progression to invasive, and given age just dilate.

Gastric Preneoplastic Lesions and Epithelial Dysplasia,Gastroenterology Clinics of NA (2007)
Given the low rate of malignant transformation of low-grade dysplasia, annual endoscopic surveillance with rebiopsy typically is performed, and surgical resection is usually not necessary [99]and[100]. It also must be emphasized that low-grade dysplasia occurring in a background of extensive intestinal metaplasia may be associated with a higher risk of malignancy [101]. Patients who have high-grade dysplasia, large adenomatous polyps, or well-differentiated adenocarcinomas no more than 2 cm should undergo definitive therapy. Complete excision of mucosal-based lesions may be performed by endoscopic mucosal resection, obviating the need for surgical resection in many cases [102]. Mucosal lesions that are not amenable to endoscopic resection, and those with a submucosal component, are managed best with surgical resection.

How well did you know this?
1
Not at all
2
3
4
5
Perfectly
272
Q

22 yo F. CDH-1 discovered after D2 distal gastrectomy for T2N0 Gastric Cancer
A. Chemo then oopherectomy
B. Chemo them hysterectomy
C. Serial endoscopy and Bilateral Mammograms
D. Completion Gastrectomy and Bilateral Mastectomy

A

D

Prophylactic gastrectomy between ages 20-40 (peak incidence age 38).

Mammography is not useful as young and dense breasts, and lobular carcinoma not detected on mammo. MRI would be good option.

How well did you know this?
1
Not at all
2
3
4
5
Perfectly
273
Q
EGD reveals 2 one cm gastric adenomas, removed by polypectomy
A. Wedge Gastrectomy
B. Biopsy for H.Pylori 
C. Repeat EGD in 2 years
D. PPI
A

B

Sabiston:
Adenomatous polyps carry a distinct risk for the development of malignancy in the polyp. Mucosal atypia is frequent,and progression from dysplasia to carcinoma in situ has been observed. The risk for the development of carcinoma is approximately 10% to 20% and increases with increasing size of the polyp. Endoscopic removal is indicated for pedunculated lesions and is sufficient if the polyp is completely removed and there are no foci of invasive cancer on histologic examination. If the polyp is larger than 2 cm, is sessile, or has a proven focus of invasive carcinoma, operative excision is warranted.

How well did you know this?
1
Not at all
2
3
4
5
Perfectly
274
Q
50ish male who has gastric cancer resected with a distal gastrectomy and LN resection. Presents with a N/V and partial SBO. Imaging shows ascities and recurrent gastric cancer at the midline wound and in the pelvis and a distal partial SBO. What is the best management?
A. IV Steroids and octreotide
B. Loop jejunostomy
C. Palliative Chemo
D. Jejuno-Transverse Colostomy
A

A

SBO is contraindication to chemo. This patient has very short life expectanct (weeks) so medical management is optimal, as even with surgery likely wont survive to discharge. Surgery for MBO is contraindicated with ascites. NCCN specifically included steroids and octreotide for medical management of MBO.

Multifocal recurrence, incomplete SBO. May be a role for palliative surgery, but overall is not clear exactly what that is. Midline recurrence would require abdominal wall resection otherwise would not heal. Don’t operate for MSBO if generalized carcinomatosis or ascites.

How well did you know this?
1
Not at all
2
3
4
5
Perfectly
275
Q
Gastric outlet thickening 1 cm over a distance of 5 cm in patient with dyspepsia on PPI. Biopsy shows mucosal inflammation
A. H. Pylori eradication
B. Deeper biopsy 
C. Continue PPI
D. Antrectomy
A

Both A and B in answer key

Deeper biopsy – reasonable -exclude lymphoma

How well did you know this?
1
Not at all
2
3
4
5
Perfectly
276
Q

Proven cancer 3 cm from GE junction on lesser curve
A. Wedge resection
B. Subtotal gastrectomy to include include ulcer
C. Total gastrectomy
D. Esophagogastrectomy

A

C

Not Siewerts III as does not invade EGJ, but even if it were:

Patients with Siewert type II and III tumors should undergo a total gastrectomy with a transhiatal resection of the distal esophagus with lymphadenectomy of the lower mediastinum and an extended lymph node dissection including nodes along the hepatic, left gastric, celiac, and splenic arteries as well as those in the splenic hilum. (UTD)

How well did you know this?
1
Not at all
2
3
4
5
Perfectly
277
Q
Man, 2 cm carcinoid of stomach. No evidence of gastritis. H. pylori negative. Mgmt
A. Wedge
B. Endoscopic resection
C. Observation
D. D1 gastrectomy
A

D

TYPE III , so resection and lymphadenectomy. Regardless, resect all 2cm or greater.

Up to Date:
Type 3 (sporadic) gastric carcinoids are treated by partial or total gastrectomy with local lymph node resection. The risk of nodal metastases is dependent on tumor size and depth, and some have suggested that endoscopic resection alone may represent adequate therapy for intraepithelial tumors <2 cm and perhaps for tumors <1 cm invading the lamina propria or submucosa. However, this is not a standard approach.
For type 1 and 2 gastric carcinoids smaller than 1 to 2 cm, endoscopic resection represents adequate therapy. Subsequent endoscopic surveillance is needed every 6 to 12 months since these patients continue to exhibit mucosal changes and hyperplasia of enterochromaffin-like cells (ECL) due to sustained hypergastrinemia.
How well did you know this?
1
Not at all
2
3
4
5
Perfectly
278
Q

Lesser curve ulcer, non healing, multiple biopsies negative for malignancy. Mgmt.
A. Subtotal gastrectomy to include the ulcer and TV
B. Total gastrectomy
C. 90% subtotal gastrectomy
D. Wedge

A

D in answer key but maybe A based on uptodate

UTD
Type I gastric ulcer — Type I gastric ulcers are the most common type of gastric ulcer. These occur along the lesser curvature near the junction of fundic and antral mucosa and occur in the setting of acid hyposecretion.
For most patients with type I gastric ulcer, distal gastrectomy with Billroth I or Billroth II reconstruction is recommended since this approach removes the ulcer and the diseased antrum. It also treats an occult malignancy.
Although type I gastric ulcer has classically been considered the consequence of inadequate gastric mucosal defense, as opposed to increased acid secretion, many advocate the addition of some form of vagotomy to the gastric resection.
Although most prefer distal gastrectomy, highly selective vagotomy has been used for type I gastric ulcer. The value of highly selective vagotomy in gastric ulcer may derive from its ability to decrease acid secretion while maintaining adequate gastric emptying and minimizing postoperative duodenogastric reflux. The procedure is performed as for duodenal ulcer, with the addition of a gastrotomy to resect or biopsy the ulcer bed.

How well did you know this?
1
Not at all
2
3
4
5
Perfectly
279
Q

Gastric cancer 4 cm from GE junction. Full thickness. Mgmt.
A. Esophagogastrectomy
B. Subtotal gastrectomy
C. Proximal gastrectomy

A

A

Siewert classification — subclassification scheme for EGJ adenocarcinomas

  • Type I tumor (located between 5 and 1 cm proximal to the anatomical squamocolumnar junction or Z-line (figure 2)) [16] – Adenocarcinoma of the distal esophagus that usually arises from an area with specialized intestinal metaplasia of the esophagus (ie, Barrett’s esophagus) and that may infiltrate the EGJ from above.
  • Type II tumor (located between 1 cm proximal and 2 cm distal to the anatomical Z-line) – True carcinoma of the cardia arising from the cardiac epithelium or short segments with intestinal metaplasia at the EGJ; this entity is also often referred to as “junctional carcinoma.”
  • Type III tumor (located between 2 and 5 cm distal to the anatomical Z-line) – Subcardial gastric carcinoma that infiltrates the EGJ and distal esophagus from below.

In general, type I cancers more frequently involve lymph nodes in the upper mediastinum (tracheal bifurcation and above) [28]. Patients with type I tumors are not appropriate candidates for a purely transabdominal approach to resection. The standard surgical approach is a transthoracic en bloc esophagectomy combined with resection of the upper part of the stomach and two-field lymphadenectomy.
On the other hand, for type II and III carcinomas, nodal metastases are more frequently found in the lower mediastinum and around the celiac trunk (table 4) [29]. The standard surgical approach is a transhiatally extended total gastrectomy with distal esophageal resection and systemic lymphadenectomy of nodes that drain the stomach.

How well did you know this?
1
Not at all
2
3
4
5
Perfectly
280
Q

Gastric cancer invading tail of panc, spleen, multiple nodes. Mgmt.
A. Chemorads
B. Perioperative chemo
C. En bloc resection

A

B

Perioperative chemo for T3 and higher, N1 and higher.

UTD
For most patients with potentially resectable, histologically proven noncardia gastric adenocarcinoma with invasion beyond the submucosa (clinical stage T2N0 or higher (table 1)) on preoperative staging evaluation, we recommend combined modality therapy over surgery alone.
We generally prefer neoadjuvant therapy over initial surgery, especially for those with a high likelihood of developing distant metastases (ie, those with bulky T3/T4 tumors, visible perigastric nodes by preoperative imaging studies, a linitis plastica appearance, or positive peritoneal cytology in the absence of visible peritoneal disease). There are no randomized trials demonstrating better outcomes from neoadjuvant therapy versus initial surgery followed by any form of adjuvant therapy. However, given the greater chance of delivering therapy in the preoperative setting and the fact that patients who are at high risk of developing distant metastases may be spared the morbidity of unnecessary gastrectomy if evidence of distant metastases emerges after chemotherapy, we favor this approach. However, upfront surgery remains an accepted approach, especially for patients with clinically staged, nonbulky, T2 or T3 tumors with no visible perigastric nodes.

How well did you know this?
1
Not at all
2
3
4
5
Perfectly
281
Q

Gastric cancer, submucosal. Laparoscopy negative, no nodes. Mgmt.
A. Subtotal gastrectomy to negative margins with perigastric node dissection
B.Subtotal gastrectomy with splenectomy, celiac nodes
C. Subtotal gastrectomy, distal panc, splenectomy with celiac nodes

A

A

D1 vs D2, Dutch and British trials did not show survival benefit to D2. Morbidity is related to pancreatectomy and splenectomy, so don’t do it. Current standard of care in Canada is D2

How well did you know this?
1
Not at all
2
3
4
5
Perfectly
282
Q
Bleeding gastric cancer, not stopped with EGD. Intraop, distal gastric Ca, distal       stomach is mobile, single met to left lobe liver. Mgmt.
A. Distal gastrectomy liver resection
B. Distal gastrectomy wedge of liver
C. Close
D. Distal gastrectomy, no liver surgery
A

D

Palliative resection for bleeding, do not resect met!

How well did you know this?
1
Not at all
2
3
4
5
Perfectly
283
Q

2cm mid body stomach lesion, EUS says involving mucosa only, no lymph nodes, CT shows it’s not invading muscle layer
A. EMR
B. Antrectomy with D1
C. Perioperative chemo, gsatrectomy and D2

A

A

How well did you know this?
1
Not at all
2
3
4
5
Perfectly
284
Q
Old man with GOO, endoscopy shows ulcerating tumour in the antrum, laparoscopy has positive cytology.
A. Gastrojej
B. Distal gastrectomy
C. Total gastrectomy
D. Total gastrectomy w D2
A

A

How well did you know this?
1
Not at all
2
3
4
5
Perfectly
285
Q

Old man with large ulcerated lesion in body of stomach, looks like T3, in the OR you see it’s involving the root of the celiac axis and the pancreas.
A. En bloc resection
B. Palliative chemotherapy
C. Total gastrectomy

A

B

How well did you know this?
1
Not at all
2
3
4
5
Perfectly
286
Q

EGD shows 3cm submucosal mass, biopsies are normal mucosa…
A. Open wedge
B. Laparoscopic wedge
C. Subtotal Gastrectomy

A

B

How well did you know this?
1
Not at all
2
3
4
5
Perfectly
287
Q

Patient presents w/ distal gastric CA, no distant mets, good status, laparoscopy is N. What is the best treatment.
A. Total gastrectomy
B. Subtotal gastrectomy w/ D1
C. Periop chemo + Subtotal Gastrectomy + D2
D. Subtotal Gastrectomy + Postop ChemoRT

A

C

MAGIC: perioperative ECF for stage II-IVa; improved 5-yr survival, DFS, recurrence rates.

Up to Date:

Lymphadenectomy:

The Dutch trial has been updated with 15-year follow-up [64]. The survival curves have continued to separate, although the difference in overall survival is still not statistically significant (22 versus 28 percent in the D1 and D2 arms, respectively, p = 0.34). However, the gastric cancer-related death rate is significantly higher in the D1 arm (48 versus 37 percent) while death rates due to other causes were not different. This supports the concept that if the D2 dissection can be done with low operative mortality, similar to that of a D1 dissection (as occurs in high volume centers), there will be a positive survival impact. This mirrors the conclusion of the latest Dutch trial paper, which is that D2 dissection is recommended in patients with potentially curable gastric cancer.

Given the apparent impact of D2 lymphadenectomy on disease-specific survival, most major cancer centers are performing a D2 as compared to a D1 dissection. Treatment guidelines published by the National Comprehensive Cancer Network (NCCN) recommend that D2 lymph node dissection is preferred over a D1 dissection. However, in view of the higher reported rates of operative mortality when this procedure has been performed in randomized trials, this recommendation this should be tempered by where and by whom the operation is being performed

Perioperative Chemotherapy:

A meta-analysis of these three trials [3,22,23] plus two other trials comparing preoperative oral fluoropyrimidine versus surgery alone [24,25] and seven other smaller trials comparing a variety of preoperative chemotherapy regimens versus surgery alone concluded that neoadjuvant chemotherapy was associated with a statistically significant benefit in terms of both overall survival (odds ratio [OR] 1.32, 95% CI 1.07-1.64) and PFS (OR 1.85, 95% CI 1.39-2.46) [27]. Furthermore, neoadjuvant chemotherapy was associated with a significantly higher complete (R0) tumor resection rate (OR 1.38, 95% CI 1.08-1.78), and did not significantly worsen rates of operative complications, perioperative mortality, or grade 3 or 4 adverse effects.

In terms of patient selection for this approach, it is reasonable to utilize the eligibility criteria for the MAGIC trial (patients of any age with a performance status of 0 or 1, a histologically proven adenocarcinoma of the stomach that was considered to invade through the submucosa [stage T2 or higher, with no evidence of distant metastases or locally advanced inoperable disease, as evaluated by CT, ultrasonography, or laparoscopy)

SCNA:
In summary, based on the trials mentioned earlier, the retrospective studies suggest improved survival in patients undergoing ELND, but this is not confirmed in prospective studies. With an experienced surgeon, a D2 lymphadenectomy can be performed safely and provides more accurate staging information

How well did you know this?
1
Not at all
2
3
4
5
Perfectly
288
Q

BII for gastric cancer. In recovery room, found to have fresh sang from NG tube. This persisted and he continued to bleeding with output of 400cc over a few hours.
A. Take back to OR, gastrotomy and oversew bleeder at anastomosis
B. Take back to OR and revise anastomosis
C. Endoscopic hemostasis

A

C

Endoscopic therapy as 1st line is reasonable, even if unstable. We routinely scope unstable UGIB patients for hemostasis.

Mastery (6th ed, p 993). Postoperative intragastric hemorrhage should always be managed first with endoscopy. If bleeding is ongoing after 4U of blood in 24hrs and it is not possible to control endoscopically, reoperation is needed. At relook, should make transverse gastrostomy 3-5cm above the anastomosis and obtain hemostasis. Closure is in horizontal direction

How well did you know this?
1
Not at all
2
3
4
5
Perfectly
289
Q
Patient with large gastric mass then invades into adrenal and pancreas.  Biopsy shows spindle cells, C-kit +.  Best next step?
A. Gastrectomy
B. Enbloc resection
C. Enbloc resection then Gleevec
D. Neoadjuvant Gleevec then resection
A

D

How well did you know this?
1
Not at all
2
3
4
5
Perfectly
290
Q
  1. What is TRUE about achalasia?
    A. Dilatation has a perforation risk of 20%
    B. Need to dilate for 5 minutes
    C. Surgical treatment has a poor prognosis
    D. Treatment does not change risk of cancer
A

D

How well did you know this?
1
Not at all
2
3
4
5
Perfectly
291
Q

A 50 year old man is diagnosed with Barrett’s with a short 2.0 cm segment. Biopsy also shows low grade dysplasia. What would your follow-up be?
A. Repeat endoscopy in 6 months for 2 times then yearly
B. Repeat endoscopy every 3 months for one year then yearly
C. Aggressive medical management only

A

A

How well did you know this?
1
Not at all
2
3
4
5
Perfectly
292
Q
What is the MOST significant prognostic factor in instrumental esophageal perforation?
A. Location of injury
B. Size of the perforation
C. Underlying disease
D. Delay in diagnosis
A

D

Perforation of the esophagus is a surgical emergency. Early detection and surgical repair within the first 24 hours results in 80% to 90% survival; after 24 hours, survival decreases to less than 50%.

How well did you know this?
1
Not at all
2
3
4
5
Perfectly
293
Q
What is correct with respect to Schatzki ring?
A. Dysphagia if < 13 mm
B. Needs surgery
C. Transmural fibrosis will be present
D. Located anywhere in esophagus
E. Marker for GERD and hiatus hernia
A

E

Common cause of intermittent dysphagia, associated with HH, treatment is dilation or endoscopic forcep disruption of mucosal ring.

How well did you know this?
1
Not at all
2
3
4
5
Perfectly
294
Q

65 yo male with dysphagia to solids+liquids for 10 months. Cine-esophagram shows dilated esophagus with bird’s beak suggestive of achalasia. Most appropriate initial step?
A. Refer for OGD and biopsy of GEJ
B. Refer for radiological balloon dilation
C. Refer for esophageal myotomy
D. Start Ca antagonists
E. Start anticholinergics

A

A

How well did you know this?
1
Not at all
2
3
4
5
Perfectly
295
Q
What is an increased risk for SCC Ca of the esophagus?
A. Barrett’s  
B. Celiac
C. Food with fungus in it
D. Achalasia
A

D

How well did you know this?
1
Not at all
2
3
4
5
Perfectly
296
Q

57 yo male on UGI endoscopy shows Barretts. What is true?
A. Barretts develops in 5-8% of patients with GERD.
B. Patients with Barretts are at risk for developing SCC 0.5% per year
C. Patients with low grade dysplasia should have screening every 3-5 yrs
D. Patients with high grade dysplasia should have screening every 5 yrs

A

A

How well did you know this?
1
Not at all
2
3
4
5
Perfectly
297
Q
Young hockey player cant play because of pain due to 6cm splenic calcified cyst, unilocular. What to do?
A. Splenectomy
B. Partial splenectomy
C. Aspirate and check for O&amp;P
D. Perc drain
A

B

Splenic cysts are categorized as parasitic, usually from Echinococcus infection, or nonparasitic. Nonparasitic cysts, can be further subdivided into primary (congenital) or secondary (pseudocysts). Primary cysts are relatively rare entities. Symptoms are generally vague left upper-quadrant discomfort, although many cysts are completely asymptomatic and are discovered when imaging is done for other purposes.
Indications for surgical intervention include symptoms and cysts larger than 5 cm (figure 1). Percutaneous aspiration of the cyst contents leads to poor results, with reaccumulation of the cystic fluid being the norm. Therefore, a variety of sclerosing agents added to percutaneous aspiration are used. Although success rates for eradicating the cyst have improved, recurrence rates remain high. Unroofing of the cyst still leaves behind a portion of the cyst lining on the spleen. Therefore, recurrences are still possible.

Splenectomy would be an effective treatment of the cyst, and, in fact, was previously considered the standard of care. However, with splenectomy, all of the functioning splenic parenchyma is removed, with the subsequent short- and long-term complication associated with the asplenic state. The newer technique of partial splenectomy offers the best management option of eliminating the entire cyst wall, thereby minimizing recurrences, yet maximizing the remaining functional splenic parenchyma. This can be done either through an open laparotomy incision or laparoscopically. It is also less technically demanding if the cyst is located at a splenic pole.

How well did you know this?
1
Not at all
2
3
4
5
Perfectly
298
Q

Patient with infective endocarditis with solitary splenic abscess, large. Best mgmt?
A. IV abx
B. Perc drain
C. Splenectomy

A

C

Although uncommon, splenic abscesses can be lethal if not treated appropriately. The most common etiology of a splenic abscess is hematogenous spread to the spleen from another septic focus, such as endocarditis, diverticulitis, or directly from intravenous drug abuse. Trauma to the spleen can make the organ more susceptible to infection if there is a devascularized segment of splenic parenchyma. Patients present with fever, elevated white blood cell count, and left upper-quadrant pain. Diagnosis is made by CT scan

Intravenous antibiotics and splenectomy provide the best means of source control. Aspiration or percutaneous drainage may occasionally be successful; they are associated with increased rates of abscess recurrence (50–60. Park AE, Godinez CD Jr. Spleen. In: Brunicardi FC, Andersen DK, Billiar TR, eds. Schwartz’s Principles of Surgery. 9th ed. New York, NY: McGraw-Hill;2010:1245–1265.

UTD:
Splenic abscess is usually managed by a combination of antibiotic therapy and splenectomy [44-46,49]. CT-guided percutaneous aspiration is occasionally successful, but this approach has not replaced splenectomy as the standard of care.

How well did you know this?
1
Not at all
2
3
4
5
Perfectly
299
Q
Splenectomy for ITP, Platelets 35, what do you do pre-op
A. Vaccination
B. IVIG
C. High Dose Steroids
D. Transfuse platelets if count < 10
A

A

UTD:

Optimizing the preoperative platelet count – If the platelet count needs to be increased for splenectomy, intravenous immune globulin (IVIG) is often administered several days to a week prior to the procedure (table 2). We prefer to perform splenectomy with a platelet count of ≥50,000/microL; however, some patients with ITP are able to undergo splenectomy safely in the setting of more severe thrombocytopenia with platelets available for transfusion if urgently needed because of intraoperative bleeding

How well did you know this?
1
Not at all
2
3
4
5
Perfectly
300
Q
4cm splenic cyst, non parasitic, in an asymptomatic in a 35 yr male construction worker.
A. Observe
B. Splenectomy
C. Partial splenectomy
D. Perc drain
A

A

Observe if asymptomatic, <5cm.

How well did you know this?
1
Not at all
2
3
4
5
Perfectly
301
Q

25 year old male involved in a trauma. Stable, but admitted for mechanism. On tertiary survey found a 3 cm solid homogeneous adrenal lesion. What is the best management for the adrenal lesion?

a. CT guided biopsy
b. Plasma aldosterone to plasma renin ratio
c. 24 hour Dexamethasone suppression test
d. Repeat CT in 6 months

A

B and D

First determine functionality. If nonfunctional, observe if <5cm.

Tests
-Pheo: 24-hour urine fractionated metanephrines
and catecholamines
-For patients with symptoms of Cushing’s syndrome:
24-hour urinary free cortisol
-For patients lacking symptoms of Cushing’s syndrome:
1 mg overnight dexamethasone suppression test
-Primary Aldosteronism: Plasma aldosterone concentration, plasma renin activity
-For adrenocortical carcinoma: Serum DHEAS

How well did you know this?
1
Not at all
2
3
4
5
Perfectly
302
Q
Patient referred for splenectomy who is an alcoholic cirrhosis with splenomegaly. Platelets 30. WBC 2.5. Hgb about 100. 
A. Lap spleen
B. Open spleen
C. Medical management
D. Splenic embolization
A

C

Splenectomy may be useful in cirrhotics with thrombocytopenia preventing antiviral therapy. IFN 2-a is effective antiviral but can cause thrombocytopenia precluding use.

UTD: splenectomy almost never necessary for thrombocytopenia in cirrhosis unless <10 which is extremely rare.

Clear evidence of portal hypertension; not an indication for splenectomy.

How well did you know this?
1
Not at all
2
3
4
5
Perfectly
303
Q

Guy returns 3 weeks after highly selective splenic embolization with left shoulder pain and a hematoma 12 cm around spleen with fluid into paracolic gutter. Management?
A. Proximal embolization
B. Splenectomy
C. Observe

A

A and B both highlighted in answer key

How well did you know this?
1
Not at all
2
3
4
5
Perfectly
304
Q
Patient went through splenectomy for ITP, responded well with increase platelets, but now recurrent ITP one year later. Why?
A. Splenosis
B. Ectopic spleen 
C. Wrong diagnosis
D. Bone marrow failure
A

B

Ectopic spleen (accessory spleen)- more common in ITP than splenosis

How well did you know this?
1
Not at all
2
3
4
5
Perfectly
305
Q

Female plt 60, on steroids for ITP, still bleeding from mucosal and gingival. What is the best management?
A. IVIG
B. Splenectomy
C. Observation

A

B

Mucous membrane bleeding is considered significant bleeding., so 2nd line therapy is indicated. Splenectomy is the preferred 2nd line therapy, as IVIG only transiently improves counts. Question is odd, however, as clinical bleeding with plts >50 is exceptionally uncommon.

Sabiston

Splenectomy is also the treatment of choice for patients with incomplete response to glucocorticoid treatment and for pregnant women in the second trimester of pregnancy who have also failed steroid treatment or IV Ig therapy with platelet counts less than 10,000/mm3 without symptoms or less than 30,000/mm3 with bleeding problems. It is not necessary to proceed to splenectomy for patients who have platelet counts higher than 50,000/mm3, have had ITP for longer than 6 months, are not experiencing bleeding symptoms, and who are not engaged in high-risk activities. A recent review of short-term and long-term failure of laparoscopic splenectomy has reported an overall approximate failure rate of 28% at 5 years after splenectomy.9

UTD

The goal of ITP therapy is to provide a safe platelet count to prevent clinically important bleeding, rather than to normalize the platelet count [1-4]. We often initiate second-line therapy at a slightly lower platelet count than used for initiating initial therapy (ie, <20,000/microL rather than 30,000/microL that is used for initial therapy)

We consider intravenous immune globulin (IVIG) to be rescue treatment in ITP rather than routine therapy. It is important for patients with clinically important bleeding and for patients in whom a more rapid response is required [5].

Second-line therapy is generally reserved for patients with thrombocytopenia that is associated with significant bleeding symptoms (eg, mucosal purpura, more serious bleeding) or for severe, persistent or recurrent thrombocytopenia (eg, platelet count <20,000/microL), following glucocorticoid-based treatments. For second-line therapies, we recommend splenectomy or rituximab rather than observation or chronic glucocorticoids

For patients who require additional treatment beyond initial therapy with glucocorticoids, we suggest splenectomy, provided the patient can tolerate surgery. This practice is based on greater response rates and longer sustained responses to splenectomy compared with rituximab

If the platelet count needs to be increased for splenectomy, intravenous immune globulin (IVIG) is often administered several days to a week prior to the procedure (table 2). We prefer to perform splenectomy with a platelet count of ≥50,000/microL; however, some patients with ITP are able to undergo splenectomy safely in the setting of more severe thrombocytopenia with platelets available for transfusion if urgently needed because of intraoperative bleeding

How well did you know this?
1
Not at all
2
3
4
5
Perfectly
306
Q

Perf diverticulitis, peritonitis in female and also 3cm hypoattenuating adrenal mass on imaging.
A. Urine metanephrine
B. Alpha block and sigmoid resection
C. Sigmoid resection and observe adrenal lesion

A

C

Patient needs urgent OR for diverticulitis due to peritonitis.

How well did you know this?
1
Not at all
2
3
4
5
Perfectly
307
Q

Male with history of melanoma. Had CT for diverticulitis, incidental finding of mass in adrenal. Plasma metanephrine normal. What do you do next

a. urine cortisol
b. FNA
c. adrenalectomy

A

B

How well did you know this?
1
Not at all
2
3
4
5
Perfectly
308
Q
Another guy HTN, on beta blocker and ACEI, mild CRF.  Found to have adrenal mass.  Plasma metaneprhine midly elevated, Cr slightl elevated, what investigation do you do next?
A. FNA
B. MRI 
C. CT
D. MIBG
A

B

Mild elevation in metanephrines expected with antihypertensives, TCA’s, antipsychotics. CT if first line, non-contrast to evaluate densite (obviously can’t assess washout). MRI would be indicated d/t CT being contraindicated. MIBG non-contributory as a routine test, used if pt has pheo and concern for multifocality.

●With CT, there is some exposure to radiation but no risk of exacerbation of hypertension if current radiographic contrast agents are given. CT with low-osmolar contrast is safe for patients with pheochromocytoma even without alpha or beta blocker pretreatment, as illustrated in a report of 22 such patients [56]. After intravenous (IV) low-osmolar contrast administration for CT scan, there was a significant increase in diastolic blood pressure but no increase in plasma catecholamine levels or episodes of hypertensive crises.
●With MRI, there is neither radiation nor dye. This more expensive test can distinguish pheochromocytoma from other adrenal masses; on T2-weighted images, pheochromocytomas appear hyperintense and other adrenal tumors isointense, as compared with the liver (image 1) [18]. However, MRI lacks the superior spatial resolution of CT.
• If abdominal and pelvic CT or MRI is negative in the presence of clinical and biochemical evidence of pheochromocytoma, one ought first to reconsider the diagnosis. If it is still considered likely, then iodine-123 (123-I) metaiodobenzylguanidine (MIBG) scintigraphy may be done. MIBG is a compound resembling norepinephrine that is taken up by adrenergic tissue. A MIBG scan can detect tumors not detected by CT or MRI or multiple tumors when CT or MRI is positive [6].
●MIBG scintigraphy is superfluous in patients with sporadic solitary adrenal pheochromocytoma identified on CT/MRI [57].
●MIBG scintigraphy is indicated in patients with large (eg, >10 cm) adrenal pheochromocytomas (increased risk of malignancy) or paraganglioma (increased risk of multiple tumors and malignancy) (image 2) [25]

How well did you know this?
1
Not at all
2
3
4
5
Perfectly
309
Q
Another guy HTN, on beta blocker and ACEI, mild CRF.  Found to have adrenal mass.  Plasma metaneprhine midly elevated, Cr slightl elevated, what investigation do you do next?
A. FNA
B. MRI 
C. CT
D. MIBG
A

B

Mild elevation in metanephrines expected with antihypertensives, TCA’s, antipsychotics. CT if first line, non-contrast to evaluate densite (obviously can’t assess washout). MRI would be indicated d/t CT being contraindicated. MIBG non-contributory as a routine test, used if pt has pheo and concern for multifocality.

CT and MRI are equivalent; MRI better for extraadrenal; IV contrast can also precipitate catecholamine crisis.
Non-contrast CT would be best.

●With CT, there is some exposure to radiation but no risk of exacerbation of hypertension if current radiographic contrast agents are given. CT with low-osmolar contrast is safe for patients with pheochromocytoma even without alpha or beta blocker pretreatment, as illustrated in a report of 22 such patients [56]. After intravenous (IV) low-osmolar contrast administration for CT scan, there was a significant increase in diastolic blood pressure but no increase in plasma catecholamine levels or episodes of hypertensive crises.
●With MRI, there is neither radiation nor dye. This more expensive test can distinguish pheochromocytoma from other adrenal masses; on T2-weighted images, pheochromocytomas appear hyperintense and other adrenal tumors isointense, as compared with the liver (image 1) [18]. However, MRI lacks the superior spatial resolution of CT.
• If abdominal and pelvic CT or MRI is negative in the presence of clinical and biochemical evidence of pheochromocytoma, one ought first to reconsider the diagnosis. If it is still considered likely, then iodine-123 (123-I) metaiodobenzylguanidine (MIBG) scintigraphy may be done. MIBG is a compound resembling norepinephrine that is taken up by adrenergic tissue. A MIBG scan can detect tumors not detected by CT or MRI or multiple tumors when CT or MRI is positive [6].
●MIBG scintigraphy is superfluous in patients with sporadic solitary adrenal pheochromocytoma identified on CT/MRI [57].
●MIBG scintigraphy is indicated in patients with large (eg, >10 cm) adrenal pheochromocytomas (increased risk of malignancy) or paraganglioma (increased risk of multiple tumors and malignancy) (image 2) [25]

How well did you know this?
1
Not at all
2
3
4
5
Perfectly
310
Q

When doing a splenectomy for severe ITP, when do you give the platelets?
A. 24 hrs preop
B. Immediately preop
C. Just after the incision is made
D. After the splenic artery has been ligated
E. After the spleen has been removed at the end of the case

A

D

How well did you know this?
1
Not at all
2
3
4
5
Perfectly
311
Q

Regarding splenic artery aneurisms all are true except.
A. More common in women.
B. Most common visceral aneurysm.
C. Occur at bifurcation and are saccular.
D. Double rupture phenomenon
E. Cause splenic infarcts.

A

E

How well did you know this?
1
Not at all
2
3
4
5
Perfectly
312
Q
Treatment of splenic abscess - antibiotics plus
A. Splenectomy
B. Percutaneous drainage
C. Open drainage
D. Drain via 12th rib
A

A

Treatment of splenic abscesses depends on whether the abscess is unilocular or multilocular. In one third of adult patients, the abscess is multilocular. In one third of children, the abscess is unilocular. Unilocular abscesses are often amenable to percutaneous drainage, along with antibiotics, with success rates reported at 75% to 90% for unilocular lesions. Multilocular lesions, however, are usually treated with splenectomy, drainage of the left upper quadrant, and antibiotics. Laparoscopic splenectomy for abscess has been reported.3

Gold standard is actually splenectomy, but better efficacy of perc drain if unilocular.

How well did you know this?
1
Not at all
2
3
4
5
Perfectly
313
Q
Blood supply  to adrenal
A. Ao, Renal artery, Inf phrenic artery
B. Ao, Renal artery, Sup phrenic
C. Lumbar, renal, aorta
D. lumbar, renal, Inf phrenic
E. lumbar, renal, sup phrenic
A

A

How well did you know this?
1
Not at all
2
3
4
5
Perfectly
314
Q
When must accessory spleens be removed?
A. Severe congenital hemolytic anemia 
B. Trauma
C. Staging for lymphoma
D. Myeloid metaplasia
A

A

Severe congenital hemolytic anemia – antibody mediated; must removal all splenic tissue

Myeloid metaplasia – only for relieving mass effect

How well did you know this?
1
Not at all
2
3
4
5
Perfectly
315
Q
2.5 cm incidental adrenal mass on CT for Pancreatitis.  What next?
A. Adrenalectomy
B. Biochemical workup
C. Iodocholesterol scan
D. Repeat CT in 3-4 months
E. Perc bx
A

B

How well did you know this?
1
Not at all
2
3
4
5
Perfectly
316
Q

Transfusion of platlelets indicated for:
A. After 7 U prbcs in trauma
B. Platelets 23 in TTP
C. Preop in ITP
D. In patient with VonWillebrand’s disease
E. After transfusion of 10U PRBCs during a Whipple with microvascular bleeding

A

Both A and E

How well did you know this?
1
Not at all
2
3
4
5
Perfectly
317
Q
All of the following are features of ITP EXCEPT
A. Considered an autoimmune phenomenon
B. Antibodies to platelets
C. Spleen is enlarged
D. Prednisone is helpful
A

C

How well did you know this?
1
Not at all
2
3
4
5
Perfectly
318
Q

All of the following regarding splenic artery aneurysms are true EXCEPT:
A. “Double rupture” phenomenon
B. Tend to be saccular and located at arterial branchings
C. Cause splenic infarcts
D. More common in women
E. Occur in association with splenomegaly

A

C

Splenic Arterial Interventions: Anatomy, Indications, Technical Considerations, and Potential Complications, 2005.

Splenic artery aneurysms are the most common visceral artery aneurysms, with a reported prevalence of 0.8% at arteriography and 0.04%–0.10% at autopsy. Most aneurysms are small (<2 cm in diameter), saccular, and located at a bifurcation in a middle or distal segment of the splenic artery. Splenic artery aneurysms are multiple in 20% of cases. Splenic artery aneurysms are found most often in multiparous women:

Double rupture phenomenon: aneurysm first ruptures into the lesser sac with mild clinical symptoms then the blood overflows into peritoneal cavity through the Winslow foramen with hemorrhagic shock.

How well did you know this?
1
Not at all
2
3
4
5
Perfectly
319
Q
Child with left flank injury: Stable. CT shows free fluid and splenic hilum injury
A. Splenorrhaphy
B. Splenectomy
C. ICU
D. Embolize
A

D

Grade IV injury, embolization improves succes of NOM. Stable patient.

How well did you know this?
1
Not at all
2
3
4
5
Perfectly
320
Q
Post-splenectomy, all except:
A. Decreased red cell life span 
B. Howell-Jolly bodies
C. Decreased properdin
D. Decreased tuftsin
A

A

Decreased red cell life span – lengthened RBC life span

Properdin and tuftsin are opsonins produced in spleen, Howell-Jolly bodies present post-splenectomy. RBC life span is increased.

How well did you know this?
1
Not at all
2
3
4
5
Perfectly
321
Q
After splenectomy which one is not true:
A. Decreased IgM (IgG stays normal)
B. Decreased WCC 
C. Decreased RBC span 
D. Decreased properdin
E. Decreased opsonization
A

C

Decr IgM but IgG stays same
Decr WCC
Incr RBC lifespan, especially in HS

How well did you know this?
1
Not at all
2
3
4
5
Perfectly
322
Q
Child with ruptured spleen. Best way to diagnose:
A. CT
B. Angio
C. Spleen scan
D. US
E. MRI
A

A

How well did you know this?
1
Not at all
2
3
4
5
Perfectly
323
Q
Which would not be and indication for splenectomy:
A. CLL 
B. Autoimmune hemolytic anemia 
C. SLE with hypersplenism
D. Feltys syndrome 
E. SC anemia
F. Hereditary spherocytosis
G. Cirrhosis with splenomegaly
H. Trauma
I. Hairy cell leukemia 
J. Polycytemia rubra vera
A

G and now H based on recent literature

a. CLL – yes, maybe
b. autoimmune hemolytic anemia – yes, if warm
c. SLE with hypersplenism
d. Feltys syndrome – yes, good response to splenectomy
e. SC anemia
f. hereditary spherocytosis
g. cirrhosis with splenomegaly
h. trauma
i. Hairy cell leukemia – SESAP: used to be an indication, but newer medical treatment obviated splenectomy
j. Polycytemia rubra vera (yes, if painful or splenic infarctions)

Basically, splenectomy useful if symptomatic splenomegaly.

How well did you know this?
1
Not at all
2
3
4
5
Perfectly
324
Q
The following are true regarding partial splenectomy, except:
A. Indicated in trauma
B Indicated for a splenic cyst
C. Indicated in ITP
D. Need full mobilization of the spleen
A

C

How well did you know this?
1
Not at all
2
3
4
5
Perfectly
325
Q
Splenomegaly is associated with all except:
A. Acquired hemolytic anemia
B. CML
C. ereditary spherocytosis
D. ITP
A

D

How well did you know this?
1
Not at all
2
3
4
5
Perfectly
326
Q
What is the most common organism identified in splenic abscess in immunocompromised patients:
A. Candida 
B. Staph
C. Strep
D. Bacteroides
E. Aspergillus
A

A

Staph, Salmonella, E. coli (Clinical Infectious Disease)
Sabiston: Staph, Strep, Salmonella, Enterococcus

Candida possible in neutropenic/immunosuppressed, but seems like still less common than bacterial.
Fungal in 25% of immunosuppressed splenic abscescc patients. (Shackleford’s)
Candida – more common then in immunocompetent, but still < staph.

How well did you know this?
1
Not at all
2
3
4
5
Perfectly
327
Q
What is the most cost effective single test to diagnose the presence of a subphrenic abscess suspected after splenectomy:
A. CT
B. US
C. Nuclear scan
D. 3 views abdo
E. MRI
A

A

How well did you know this?
1
Not at all
2
3
4
5
Perfectly
328
Q

Splenectomy is most likely to be difficult to to adhesions in which condition:
A. Septic infarcts
B. ITP
C. Splenomegaly
D. Trauma
E. Hypersplenism secondary to portal hypertension

A

A

How well did you know this?
1
Not at all
2
3
4
5
Perfectly
329
Q
What poses the greatest risk after splenectomy:
A. Anemia
B. Thrombocytopenia
C. Thrombocytosis
D. Sepsis 
E. Splenic vein thrombosis
A

D

Sepsis 1-5%

How well did you know this?
1
Not at all
2
3
4
5
Perfectly
330
Q
After a bx for lymphoma all of the following are acceptable but:
A. Quick section
B. Culture
C. EM
D. Immunohistochemistry
E. Light microscopy
A

A

How well did you know this?
1
Not at all
2
3
4
5
Perfectly
331
Q

Laparotomy reveals additional splenic tissue. Cause:
A. Previous splenic trauma
B. Congenital anomaly
C. Accessory spleens

A

C

Accessory spleens – actually quite common, probably more than trauma

How well did you know this?
1
Not at all
2
3
4
5
Perfectly
332
Q
Left adrenal vein drains into:
A. Left renal vein
B. L gonadal vein
C. Inferior vena cava
D. L phrenic vein
E. Portal vein
A

A

How well did you know this?
1
Not at all
2
3
4
5
Perfectly
333
Q
CT abdo for trauma. Incidental 2cm adrenal mass:
A. Pheo
B. Adenoma
C. Adrenocortical ca
D. Myelolipoma
A

B

How well did you know this?
1
Not at all
2
3
4
5
Perfectly
334
Q
Which is not a feature of primary hyperaldosteronism:
A. Hypertension
B. Hypokalemia
C. Acidosis
D. Muscle weakness
E. Polydipsia
A

C

Hypertension, +/- hypokalemia, hypernatremia, metabolic alkalosis, muscle weakness, hypomagnesemia, CV risk

How well did you know this?
1
Not at all
2
3
4
5
Perfectly
335
Q
Key features of hyperaldosteronism are:
A. Hypokalemia
B. Urine with low sg and resistant to ADH 
C. Alkalosis 
D. Hypochloremia
A

A

Urine with low sg and resistant to ADH (should be high SG)
Alkalosis – not a key feature but may be present

How well did you know this?
1
Not at all
2
3
4
5
Perfectly
336
Q
40 yo female, obese, sudden onset of diastolic hypertension, polydipsia, polyuria, weakness, Na=150, K=2.5.
A. Cushing
B. Primary hyperaldosteronism
C. Pheo
D. Renal vascular hypetension
A

B

How well did you know this?
1
Not at all
2
3
4
5
Perfectly
337
Q
Which of the following is not associated with hyperaldosteronism:	
A. Hypertension
B. Liver failure with ascites
C. Adrenal adenoma
D. Adrenal hyperplasia
A

B

How well did you know this?
1
Not at all
2
3
4
5
Perfectly
338
Q

Cushing syndrome is most commonly due to:
A. Pituitary Adenoma
B. Ca
C. Bilateral cortical hyperplasia
D. Ectopic ACTH
E. Bilateral cortical and central hyperplasia
F. Adrenal Adenoma

A

A

Sabiston – 75% caused by pituitary adenoma.

UTD:

1) iatrogenic 2) ectopic ACTH (SCLC) 3) Cushings disease 4) adrenal tumours

SESAP says pituitary source in 80-85%, ectopic in 15%. UTD says ectopic, but qualifies that it is “probably” the 2nd most common (after iatrogenic) although often undiagnosed.

Traditional answer is adenoma (pituitary) so Cushing’s disease.

How well did you know this?
1
Not at all
2
3
4
5
Perfectly
339
Q
Which of the following are not considered APUDOMAS:
A. Folicullar adenoma
B. Parathyroid adenoma
C. Gastrinoma
D. Carcinoid
E. Pheo
A

A

How well did you know this?
1
Not at all
2
3
4
5
Perfectly
340
Q

The best test for neuroblastoma is:
A. VMA in urine
B. Epinephrine
C. Nor-epinephrine

A

A

How well did you know this?
1
Not at all
2
3
4
5
Perfectly
341
Q

The most common reason for missing Conn’s sydrome:
A. Not adequately repleting K prior to renin test
B. Not adequately repleting Na prior to renin test
C. Missing the tumor on the CT scan
D. Missing the tumor on iodocholesterol test

A

A

How well did you know this?
1
Not at all
2
3
4
5
Perfectly
342
Q
Man w/ ITP. On 60 mg prednisone x 6 weeks. Not bleeding but plt remain at 50,000. Best treatment. 
A. Splenectomy
B. IVIG
C. Increase prednisone to 120
D. Nothing
A

C or D in answer key

IVIG is 2nd line, provides temp support if pts <5 or bleeding. IVIG lasts for days to weeks.

Indications for splenectomy: Duration 3-6 months, ongoing thrombocytopenia,
Splenectomy only for plts <30, or 30-50 and inability to wean from steroids; steroid dosing is 1-1.5mg/kg/d/ 6 weeks is too early to consider splenectomy; should wait 6 months as possibility of spontaneous regression.

Goal is 50,000, so should start to taper. Splenectomy only after 6 months and if unable to maintain plts 50 or higher.

UTD:

In practice, we often wait at least six months from diagnosis of ITP to splenectomy, to allow for late spontaneous remissions; this practice is consistent with a 2010 consensus report [6]. However, many patients with persistently severe and symptomatic thrombocytopenia in spite of first-line therapy may require splenectomy much sooner (eg, within weeks).

How well did you know this?
1
Not at all
2
3
4
5
Perfectly
343
Q
A patient undergoes splenectomy for ITP.  3 days post op develops thrombocytosis of 750,000.  	What is your medication of choice?
A. ASA 325 mg PO OD
B. Plavix
C. Nothing
D. SC Heparin
A

C

How well did you know this?
1
Not at all
2
3
4
5
Perfectly
344
Q

In well selected patients undergoing cytoreductive surgery plus hyperthermic intraperitoneal chemotherapy, or HIPEC, for colorectal carcinomatosis, which of the following is TRUE?

A. Survival is improved in patients with distant disease
B. Methods are standardized in developed countries
C. R0 resections are required
D. Complication rates are high
E. Disease-specific survival is equivalent to standard systemic therapy options

A

D

HIPEC is not offered to pts with distant mets. Methods for HIPEC vary between centres. Difficult to standardize surgical technique between surgeons. Other variables include timing of intraperitoneal therapy with regard to surgical anast; thoroughness of extirpation; and the doses, timing and choice of chemo. R0 resections may not be possible. Disease specific survival compared with standard systemic therapy options is improved. Improvement is on the order of months or occasionally years. Trade off is mortality rates approaching 20% in worst cases but closer to 6% in high volume centres. Major morbidity ranges from 12-52% in high volume centres and can lead to prolongs hosp stays and high costs

How well did you know this?
1
Not at all
2
3
4
5
Perfectly
345
Q

Regarding the effect of aspirin in the prevention of CRC, which of the following statements is TRUE?

A. There is modest reduction in polyps and cancers
B. The incidence of other cancers is reduced
C. Effect is seen mostly in women
D. High dose therapy is required
E. Concurrent statin therapy decreases the protective effects

A

A

Low or high dose aspirin reduces both polyps and advanced lesions in men and women. At least 4 RCTs provided evidence in at risk pts, with good colonoscopy FU regimens. Adverse events were uncommon with no difference between groups for death, MI, bleeding or invasive cancers. Incidence of invasive cancer of the colon was low with this level of surveillance but high risk lesions (e.g. High grade dysplasia, a known precursor to cancer) were seen less freq with aspirin in some studies. Others showed a decr in invasive cancer risk. Latency period from aspirin prevention to benefit appears to be approx 10 yrs; apparent only after at least 5 yrs of therapy. Concurrent statin therapy does not appear to have an effect on polyp or cancer prevention

How well did you know this?
1
Not at all
2
3
4
5
Perfectly
346
Q

In a patient with a hx of colon resection for CRC now discovered to have liver mets, which of the following is a contraindication to liver resection for cure?

A. At least 4 hepatic mets
B. Extrahepatic disease
C. Node positive primary tumor
D. Prev hepatic resection for CRC mets
E. Inability to completely resect all disease in the liver
A

E

CRC liver mets can be resected for long term survival and even cure. Approx 5-10% of 50,000 cases of CRC liver mets each yrs in US are candidates for curative resection. Long term survival can occur in up to 50% of pts.

5 preop factors most affecting outcomes in pts presenting for liver resection of CRC mets: 1) Size >5 cm, 2) disease free interval <1 year, 3) Node positive primary, 4) Greater than 1 tumor, 5) CEA >200

Traditional contradindications to hepatectomy for met CRC were presence of extrahepatic diseases at least 4 hepatic mets, close margins and inability to resect all disease.

Hepatectomy for at least 4 mets is assoc with a 5 yr survival rate of 33% and recurrence rate of 80%. Pts w/ close or involved margins have long term survival, and resection of extrahepatic mets presenting concomitantly with liver mets results in long term survival of selected pts. Extrahep met sites most favourable for resection are lung mets, local recurrence of original tumor, and portal LNs. After resection for hepatic CRC mets, recurrence rates at least 80%, with half of recurrences isolated to the liver. Pts whose disease is amenable and who undergo second liver resection for removal of all disease have 5 yr survival for 30-40%

How well did you know this?
1
Not at all
2
3
4
5
Perfectly
347
Q

14 year old presents with 1 week hx of SOB and night sweats. CXR and CT obtained showing compression of at least half the diameter of trachea in some views. No peripheral adenopathy. CT guided bx is non diagnostic. Unable to lie flat. Which of the following is the next best step?

A. Chest radiotherapy
B. Chamberlain procedure using local anesthesia
C. Chamberlain procedure under light sedation
D. Chamberlain procedure under GA
E. Standard chemo regimen for lymphoma

A

B

Pt has large mediastinal mass. Radio growth of mediastinal masses, such as lymphomas, may distort airway, causing resp distress, wheezing and orthopnea. Imaging depicts enlargement of mediastinum with involvement of airway. At times, pleural or pericardial fluid can be obtained for cytologic analysis. If neither peripheral LNs nor fluids are available for bx or puncture, CT guided needle bx for cytologic analysis is generally next step. If this is still not diagnostic, bx by thoracoscopy or anterior Chamberlain may be necessary. Chamberlain procedure is an anterior mediastinoscopy used to bx a mass or LNs in the center of the chest, which is done through a small transverse incision in an intercostal space.

Mediastinal disease compressing the airway poses a significant risk. In pts with tracheal compression, sedation and anesthesia should be avoided. These pts may prove impossible to ventilate, even with intubation, b/c of distal tracheal or bronchial obstruction. For pts with severe airway obstruction, dx by alternate methods needs to be considered. In this pt, a Chamberlain procedure using local anesth is the safest approach. Radiotherapy or chemo is inappropriate until a dx is made.

How well did you know this?
1
Not at all
2
3
4
5
Perfectly
348
Q

Which of the following is TRUE regarding bevacizumab?

A. Currently indicated for breast cancer
B. Anaphylactic/anaphylactoid reactions are not reported
C. It is an epidermal growth factor
D. It can cause gastrointestinal perforation
E. It improves wound healing

A

D

Anti vascular growth factors (VEGF) recombinant humanized monoclonal antibody. New blood vessel formation is a fundamental event in the process of tumor growth and met dissemination. VEGF and its receptors play a role in tumor proliferation. Approaches to limit VEGF activity include monoclonal antibodies and small molecules, incl the corresponding receptor tyrosine kinase activity.

FDA removed met breast ca indication from bevacizumab labelling, citing a lack of demonstrated safety and efficacy in breast cancer. Does not prolong OS and does not provide sufficient benefit in slowing disease progression to outweigh significant risks assoc with treatment.

Indications include colon, kidney, brain and lung cancers.

Assoc with incr incidence of hypersensitivity reactions in combo with chemo compared with chemo alone. Incidence of anaphylactic/anaphylactoid reactions was 5% in clinical trials.

Patients have significantly incr risk of GI perf with a positive correlation with higher doses of bevacizumab and CRC. Extended used can incr long term risk of wound healing complications for up to 6 mos after cessation

How well did you know this?
1
Not at all
2
3
4
5
Perfectly
349
Q

22F undergoes an uncomplicated lap appy for perf appendicitis. Final pathology reveals carcinoid tumor of the tip of the appendix measuring 3 cm. What is the most appropriate next step?

A. Observation
B. Chemotherapy
C. Cecectomy
D. R Hemi
E. Radiation
A

D

Most common tumor of appendix is carcinoid, a neuroendocrine tumor that is found in 0.3-0.9% of pts who undergo appy. Appendiceal tumors including carcinoid, are usually found incidentally on pathologic exam of an inflamed appendix. Size of tumor determines tx and ppx with tumors >2 cm having the greatest met potential. Formal R hemi is indicated. Other indications for R hemi include mesoappendiceal invasion, tumors at base of appendix with positive margins, high grade malignant tumors with a high mitotic index, and goblet cell adenocarcinoid tumors. R hemi also may be considered for 1-2 cm tumors at the base of appendix; however this remains controversial. Simple appy reserved for tumors <1.5 cm in diameter, b/c chance of lymphatic or distant mets is minimal. Observation and radiation have no role and chemo may be considered in mgmt of met tumors, although it has limited efficacy.

How well did you know this?
1
Not at all
2
3
4
5
Perfectly
350
Q

53F undergoes an EGD for dyspepsia. During the procedure, she has evidence of chronic gastritis. In addition, a small mass is identified in the body of the stomach. Specimens are taken and return a dx of H pylori infection and an extranodal marginal zone lymphoma of mucosa assoc lymphoid tissue (MALT lymphoma). Which of the following is the next step in tx of this patient?

A. H pylori eradication therapy
B. Chemo
C. Chemo and radiation
D. Radiation
E. Partial gastrectomy
A

A

MALT lymphoma aka extranodal marginal zone lymphomas MALT type. Indolent B cell lymphomas that develop secondary to chronic inflamm from either infection or autoimmune process. Any tissue that has mucosa assoc lymphoid tissue can develop this tumor, including salivary glands, resp tract, bladder and small intestine. Most commonly seen in stomach. Gastric MALT lymphomas are typically identified during endoscopy being performed for non specific gastric symptoms. Discrete masses are rare, with the most common endoscopic finding beating erythema, ulcer or erosions. Most are seen in the Antrum and multifocal disease is common. Dx made by bx of area

All patients with gastric MALT need to be evaluated for H pylori. Regardless of H pylori status, all pts with local disease should be tx with eradication therapy b/c of risk of false neg test. Of pts with local disease positive for H pylori, 50-70% will have a favourable response to tx. Complete response can take a several months, and serial endoscopy is needed to evaluate tx response.

Once gastric MALT lymphoma is identified, a staging evaluation is undertaken to determine whether more than mucosal involvement is present. Gastric staging is done with endoscopic U/S. Further staging includes CT chest/abdo/pelvis, bone marrow evaluation and a PET CT scan. Unique to MALT lymphoma is the evaluation of other potential sites of disease including bronchoscopy, colonoscopy and small bowel evaluation

If the patient fails to response, follow up radiation can be used. Chemo is not used if only local disease is present. Resectional therapy is rarely necessary and quality of life is better with gastric preservation. Disseminated disease is tx as any other lymphoma with chemo or antibody based approaches. However, b/c these are indolent tumors, watchful waiting is also reasonable.

How well did you know this?
1
Not at all
2
3
4
5
Perfectly
351
Q

Regarding pulmonary mets assoc with soft tissue sarcomas

A. Surgical resection follows neoadjuvant therapy
B. Recurrent pulmonary mets are tx by surgery
C. Multiple unilateral mets are a contraindication to resection
D. Bilateral mets are a contraindication to resection
E. Lobectomy and lymphadenectomy are required.

A

B

Lung is most common site of mets for soft tissue sarcoma. Similar to the primary tumor, resection of pulm mets is performed to control local diseases. Resection confers a survival advantage and should be performed, if possible, before considering any other tx modality.

Criteria need to be applied to planned resection: Minimal amt of lung should be removed with each specimen. Stapled metastasectomy is procedure of choice. Multiple unilateral resections can be performed, again with lung preservation being of utmost importance. If multiple lesions are present in a single lobe, precluding separate excision, especially if thoracoscopic approach is undertaken. If a patient has recurrent pulmonary disease that can be resected safely, sx should be undertaken using the these same principles.

Soft tissue sarcomas do not met to LNs. Lymphadenectomy would not be part of sx. Neoadjuvant therapy before surgical resection has no benefit.

How well did you know this?
1
Not at all
2
3
4
5
Perfectly
352
Q

An otherwise healthy 70F with painless rectal bleeding at defecation. DRE reveals a 1 cm firm anal mass. Colonoscopy 1 yr ago was N except for diverticulosis. Anoscopic exam shows a mass in anal canal. What would you recommend to this patient?

A. Reassurance and observation
B. I &amp; D
C. Excisional bx
D. Rubber band ligation
E. Laser ablation
A

C

Firm symptomatic anal lesions require path exam to exclude malignancy. Reassurance and observation are contraindicated. Laser ablation does not provide tissue for dx. B/c of anatomical position of this lesion, seen only through the anoscope, office excision would be technically difficult and likely very uncomfortable for the pt. Transanal excision in the OR allows for complete excision with margins under optimal conditions.

This lesion is an anal melanoma. By contrast, with cutaneous melanoma, anal melanomas are often amelanotic

How well did you know this?
1
Not at all
2
3
4
5
Perfectly
353
Q

Level I data support the avoidance of which of the following to reduce and prevent lymphedema after LN dissection?

A. Needle sticks
B. BP measurements
C. Wt gain
D. Extreme heat and burns
E. Exercise
A

C

Lymphedema is accumulation of protein rich fluid in regions where lympahtic system cannot transport interstitial fluid. Occurs most often as a complication of LN dissection for cancer tx. Most commonly assoc with breast cancer mgmt, overall 16% incidence in pts tx for melanoma; sarcoma; H&N; urologic and gyne cancer.

Tx remains suboptimal and usually palliative, focus is on several measures to prevent it, although many measures are anecdotal and no evidence based. Conflicting lower level data regarding needle stick avoidance, no data to support bp measurement avoidance, and level 1 data refuting the notions of avoiding extremes of temp or vigourous exercise. RCT of wt loss after LN surgery shows less lymphedema in intervention group. Findings support a large amt of evidence linking obesity to the development of cancer related lymphedema.

How well did you know this?
1
Not at all
2
3
4
5
Perfectly
354
Q

67F presents 2 yrs after distal gastrectomy for a c-kit (CD 117) positive GIST. Pt has an abdo wall soft tissue mass, and bx reveals recurrent tumor. CT reveals multiple intra-abdo masses and abdo wall masses. Pt is started on imatinib mesylate 400 mg/day. Which of the following statements regarding this tx is TRUE?

A. The most common side effects are HTN and thrombocytopenia
B. Most response occur soon after starting tx
C. Most pts are rendered resectable
D. Tx can be continues up until time of sx
E. Tx incr the risk of post op complications

A

D

GIST, the most common sarcoma of the GI tract, arises from interstitial cells of cajal and possesses unique kinase mutations that can serve as targets for medical therapy, either in the adjuvant setting or when gross tumor is present. Most common mutations at exon 11 of c kit (CD117), a tyrosine kinase. Imatinib is a tyrosine kinase inhibitor specific for such mutations is commonly used as medical therapy for GIST.

Drug stopped right before planned operation does not increase complications over the expected rate. OS at 2 yrs was 91-93%. Given to pts with advanced but possible operable disease, approx 25% converted to resectable tumors and long term survival was documented.

Whether given in the adjuvant, neoadjuvant or palliative setting, imatinib, an oral agent is generally well tolerated. Major side effects include fever, H/A, fluid retention, GI side effects, anemia and elevated LFTs

One of the hallmarks of imatinib therapy of measurable tumors is delayed response. Avg time of overall response in the multi center trial was 13 weeks. Centres using imatinib for advanced GIST in hope of rendering pts resectable generally follow reponding patients for at least 6 months before considering surgical exploration. Soon after starting therapy, some tumors actually swell, giving the false impression of CT that the tumors are progressing. Dynamic studies such as MRI, contrast enhanced CT or PET/CT may show decr intensity in these swollen tumors, indicating a metabolic response. Accordingly the Choi criteria, which include a dynamic measure of response, may be more accurate for assessing response to imatinib than the standard guidelines that rely on size alone.

How well did you know this?
1
Not at all
2
3
4
5
Perfectly
355
Q

50 y.o. Otherwise healthy man undergoes exp lap for a presumed ruptured appendix. At OR, the abdo is filled with gelatinous material adherent to all the peritoneal surfaces and intestinal surfaces. Frozen section of this material is read as “mucinous neoplasm”. Which of the following is the best therapy for this pt’s condition?

A. R Hemi, followed by observation
B. Debulking of all gross disease, followed by observation
C. Systemic chemo
D. Cytoreductive surgery and intraperitoneal chemo
E. Palliative surgery

A

D

Systemic chemo is palliative and generally provides only limited improvement in survival. Over past few decades, CRS and HIPEC were developed for pts with mucin producing peritoneal surface malignancies. Studies have shown survival advantages for these approaches.

CRS is an extensive procedure and incl greater omentectomy, splenectomy,. R and LUQ peritonectomy, chole, less omentectomy, pelvic peritonectomy, amd resection of the most involved parts of the bowel–often R colon, rectosigmoid and sometimes gastric antrum. Completeness of resections is scored on a standard grading scale, with the score correlating with survival.

Tx plan and prognosis hinge on specific histology, which usually can’t be determined by frozen section. PMP is the more rare type of mucinous intraperitoneal neoplasm, characterized by mucinous ascites, low grade neoplasm (often of appendiceal origin) and a better prognosis. PMP has almost no ability to spread by via lymphatics or hematogenous routes. Generally stays limited to peritoneal cavity where it spreads by fluid current and gravity. Repeated CRS can palliation the disease for yrs but rarely yields cure. However, the addition of appropriately timed (before adhesion formation) HIPEC has yielded 5 and 10 yr survival rates of 75-100% and 68%, respectively, although results are worse with higher grade tumors and incomplete resections.

Mucinous colorectal adenocarcinoma (MCA) is essentially stage 4 colon cancer limited to the abdo. MCA is assoc with the ability to spread to lymphatics and a poorer outcome. In selected patients with MCA limited to the peritoneal cavity, CRS and HIPEC can 5 yr survival rates of 27-31%. Role of systemic therapy and bio therapy for MCA remains under study

Although CRS and HIPEC are therefore considered the next step in the otherwise healthy pt presented; such pts may be best tx at centres of expertise and only after exact histology has been determined on permanent pathology. Even a recent report of CRS and HIPEC done at nonspecialized hospitals involved surgeons with special expertise operating at carefully chosen institutions.

How well did you know this?
1
Not at all
2
3
4
5
Perfectly
356
Q

After complete excisional bx of an anal melanoma with neg margins, what is the next step?

A. Nigro protocol
B. Inguinal LN dissection
C. Radiation therapy
D. APR
E. Observation
A

E

Regardless of disease stage at presentation and extent of sx, prognosis for anal melanoma is poor, with overall 5 yr survival rates of 10-20%. Although 20% of pts have inguinal node met disease at presentation, it is unclear that groin dissection has any effect on mgmt or prognosis. Survival after WLE is similar to APR, which has prompted recommendation of transanal excision over radical sx in most cases. Results of systemic therapy are disappointing. Radiation alone provides the options of sphincter preservation but does not incr survival. Chemorads with Nigro protocol is 1st line tx for scc of the anus not melanoma. For a pt whose anal melanoma has already been completely excised, observation is best recommendation.

How well did you know this?
1
Not at all
2
3
4
5
Perfectly
357
Q

25F undergoes emergent appy. Pathology reveals presence of a 1.5 cm well differentiated carcinoid tumor at the tip of the appendix with clear margins. Which of the following should be the next step in mgmt of this pt?

A. CT Abdo
B. Colonoscopy
C. R hemi
D. Octreotide scan
E. 24 hr urine 5-HIAA
A

A

Carcinoid of the appendix is the most commonly type of primary malignant lesion of the appendix. Often asymp and found incidentally during appy. Incidence 0.3-0.9% undergoing appy and they are commonly located at the tip of the appendix

Major of incidental carcinoid cured by appy. Current recommendations of additional sx include inadequate resection margins, tumor >2 cm, goblet type morphology. Complete R hemi is required. In tumors >2 cm, 30-60% will have met spread. Met rate for tumors <2 cm is no higher than 3%. Controversy regarding tx of lesions in 1-2 cm range. In such cases, evidence of mesoappendiceal invasion, vasc invasion, incr mitosis, or presence of proliferation markers must be evaluated and careful pt risk assessment undertaken. Goblet cell carcinoids have features resembling both carcinoid and adenoca and they all require R hemi.

In this pt, tumor size 1.5 cm at the tip with clear margins, surveillance by CT abdo is indicated. Nether has evaluation by colonoscopy nor R hemi is indicated. Octreotide scan is useful for making a dx b/c it can image carcinoids expressing somatostatin receptors, particularly those with receptor subtypes 2 and 5 for which octreotide has high affinity. Urine 5 HIAA is a useful lab marker to make the dx of carcinoid, b/c it provides a summation of tumor secretory activity. Neither of these studies are indicated in this pt, however, b/c a pathologic dx has been made.

How well did you know this?
1
Not at all
2
3
4
5
Perfectly
358
Q

Match the correct lettered answer with the numbered statement

A. Basal cell carcinoma
B. Melanoma
C. Both
D. Neither

  1. UV radiation constitutes a risk factor
  2. Interferon alpha 2b is used for therapy
  3. Minimal 1 cm excision margin is required
  4. Depth of invasion is a prognostic indicator
  5. Smoking constitutes a risk factor
A
  1. C 2. B 3. B 4. C 5. D

20% of Americans over lifetime will develop some type of skin cancer. Divided into nonmelanoma skin cancers and cutaneous melanoma. Nonmelanoma skin cancers account for ~90% of all cutaneous malignancies. Of these tumors, BCC is the most common (75% of all nonmelanoma lesions), followed by SCC (20%) and rare skin lesions (fibrohistiocytic and adnexal cancer 5%). Cutaneous melanoma is the deadliest skin cancer, accounting for 75% of all skin cancer related deaths.

UV radiation from sun exposure is a major RF for all skin cancers, accounting for 90% of all nonmelanoma skin cancers and 65% of all CM. Both UVA (320-400 nm) and UVB (280-320 nm) radiation lead to skin cancer formation. UVA radiation approx 95% of all UV exposure to the skin, causes indirect cutaneous damage via oxygen radical formation and depletion of Langerhands cells in the skin. UVB, the remaining 5% of UV exposure, directly damages DNA within the skin cell. For both BCC and CM, fair complexion, blond or red hair, and blue eyes constitute risk factors. Whereas cumulative UV exposure incr risk of SCC, intermittent UV exposure is assoc with CM and BCC. Hx of sunburn at an early age incr CM risk. Although smoking is not a RF in either BCC or CM, it does incr risk of developed SCC by 3x.

BCC typically raised, pearl color nodule; SCC tends to be a small, scaly, white or red lesion with friable base; CM is characterized by ABCDEs. Care must be taken b/c pigmented BCCs can exist and nonpigmented CM does occur.

Depth of invasion is an important prognostic factor in both CM and SCC due to strong assoc between mets and tumors >1 mm thick. CM tends to met to regional LNs first. Distant mets can arise throughout the body, incl the liver, brain, bone, lung, skin and GI tract. Although rare (<0.03%), BCC can met as well and its risk is assoc with depth of invasion. Other RF for BCC mets include large/recurrent tumor, prev irradiated tumors and immunosuppression. Common sites of mets for BCC are bone and lung.

For lesions requiring surgical excision, recommended margin is predicted on lesion and in certain cases, its size. For CM, 1 cm margin is required for lesions 1 mm or less in depth, whereas 2 cm margin is needed for lesions >1 mm. In BCC, surgical excision with 0.5-1 cm margins is often adequate. SCC lesions usually require margins of at least 2 cm. Other tx options include topical chemo with imiquimod (BCC or CM) or 5FU (SCC), photodynamic therapy (BCC and SCC), radiotherapy (BCC, SCC, and met CM), immunotherapy (interferon alpha 2b or IL2 in met CM), curettage and dissection (BCC), MOHs (BCC) and systemic chemo (BCC, SCC, CM)

How well did you know this?
1
Not at all
2
3
4
5
Perfectly
359
Q

Match the correct lettered answer with the numbered statement

A. Vemurafenib
B. Ipilimumab
C. Both
D. Neither

  1. Approved for stage IV melanoma
  2. Targets specific mutation
  3. Common side effects include skin cancers
A
  1. C 2. A 3. A

Resection of mets may provide a small but real disease free survival. Further, new agents that tx met melanoma may render more pts resectable. Some of the newer agents target either the pt’s T cell receptors or a specific tumor mutation

Ipilimumab is a fully human monoclonal antibody against cytotoxic T lymphocyte antigen 4 (CTLA 4). Causes activated T cells to survive and incr the pool of anti tumor T cells. Adverse events assoc with the drug are mostly immune related and include colitis, rash and elevated LFTs. RCT phase III study of stage IV melanoma pts tx with ipilimumab vs a peptide vaccine showed 3 yr survival rates of 21% and 12%, respectively, and led to FDA approval

Approx 40-60% of CM carry mutations in tyrosine kinase protein BRAF and 90% of those mutations are a substitution of glutamic acid for valine at codon 600 (V600E). Vemurafenib is an inhibitory molecule selective for the V600E mutation. In a large RCT phase III study of stage IV melanoma pts tx with either vemurafenib or dacarbazine, responses rates were 48% and 5% respectively. Led to FDA approval of vemurafenib for stage IV melanoma in pts with V600E mutation. Adverse effects are mostly cutaneous and include appearance of SCC and kerathoacanthomas. Exact mechanism is undergoing study. May be related to activation of alternative pathways, such as mitogen activated protein kinase pathway, and thus it may be assoc with drug resistance.

How well did you know this?
1
Not at all
2
3
4
5
Perfectly
360
Q

Match the correct lettered answer with the numbered statement

A. Hepatic resection
B. RFA
C. TACE
D. Orthotopic Liver Transplant (OLT)
E. Systemic chemo 
  1. Primary therapy for 60M Hep C positive, Child B cirrhotic with two 1 cm foci of HCC in confined segments I and II of the liver
  2. Primary therapy for 65M with three 3 cm met CRC lesions involves segments IV, V, and VIII of the liver
  3. Primary therapy for 75F with COPD who has a solitary 3 cm CRC lesion bridging segments IV and V of the liver abutting the L portal vein
A
  1. D 2. A 3. B

Several modalities exist and therapy chose depends on type of lesion, number present, anatomical locations, size, patient comorbidities, degree of liver dysfunction (i.e. Cirrhosis, portal HTN), and presence of extrahepatic disease. Accurate preop radiologic and functional assessment of liver is essential. Multidetectoy CT with 3D reconstruction and MRI with MRCP are recommended to determine morphologic characteristics of tumor, vascular or biliary involvement, presence of satellite lesions, and presence of chronic liver disease. Determine liver functional status with Child Pugh or MELD score. Liver volumetry can also be performed.

Complete surgical removal of tumor remains mainstay for acheving cure in pts with hepatic involvement. Type of resection (anatomical vs wedge) and width of margin required is based on lesion type. Major vasc resection when necessary and regional lymphadenectomy should be undertaken. Criteria for resectability vary with type of malignancy. In HCC, pts with early cirrhosis (CP A/early B) meeting Milan criteria (1 lesion =5 cm or 3 or fewer lesions with larger = 3 cm) and this without cirrhosis are candidates. In CRC liver met, criteria include =4 lesions, favourable anatomy for resection (ie. proximity to vasc structures, ability to obtain adequate margin), and lack of extrahepatic disease.

Both RFA and TACE are tx options in the setting of unresectable disease. Successfully used to downstage tumors, allowing their resection. In addition,they can serve as bridging therapies in pts who are transplant candidates. RFA uses direct application of high freq alternating current to cause thermally induced coagulation necrosis of tumor cells. Depending on temp generated, this necrosis occurs over a short time interval (50-52) or is immediate (>60). For large tumors, multiple applications are necessary. An advantage of RFA is that it may be used in near large vascular structures b/c they can serve as a heat sink to prevent endothelial damage. Pts with multiple bilateral lesions, central lesions not amenable to anatomic resection, contralat nodules after undergoing hemihepatectomy, and severe liver disease/comorbidities are candidates for RFA.

TACE involves administration of cytotoxic drugs via hepatic artery with subsequent embolization. Direct intra-arterial infusion of chemo drugs is more effective than IV infusion and maximizes local effects while minimizing systemic complications. Often used in the setting of unresectable HCC. In CRC liver met, direct hepatic artery infusion can be used by means of a surgically placed pump. Contraindications to TACE include Child C cirrhosis, severe leukopenia/thrombocytopenia, cardiac/renal insufficiency, ascites, PV occlusion, and atypical hepatic artery anatomy.

Orthotopic liver transplant is preferred therapy for Child Pugh B and C pts with HCC satisfying Milan criteria. In this setting, OLT results are similar to those found in OLT for other cases. 4 yr survival approaches 75% and recurrence free survival 83%

In CRC liver met, new chemo agents such as oxaliplatin, iron ore an and bevacizumab have improves response rates compared with traditional 5FU/leucovorin therapy, allowing for neoadj downstaging of prev unresectable met lesions

75F pt with central CRC liver met abutting portal vein is candidate for RFA b/c she is a poor surgical candidate for major hepatic resection

How well did you know this?
1
Not at all
2
3
4
5
Perfectly
361
Q

Match the correct lettered answer with the numbered statement

A. Osteosarcoma
B. Giant cell tumor
C. Burkitt lymphoma
D. AML
E. Ewing sarcoma
  1. Onion skin appearance on plain XR
  2. Starburst appearance on plain XR
  3. EBV
A
  1. E 2. A 3. C

Classic appearance of Ewing sarcoma on XR is an onion skin appearance. Osteosarcoma has an appearance of a starburst.

Most common cytogenetic changes in Ewing sarcoma is a translocation (11:22). Burkitt lymphoma has cytogenetic change, translocation <8:14). EBV is implicated in multiple types of cancers, including Burkitt lymphoma. Giant cell tumors are usually lyric lucent lesions that are rarely malignant. Account for approx 20% of benign tumors.

AML is assoc with bone pain caused by buildup of leukaemia cells and is sometimes assoc with pathologic #s. It is assoc with translocations that are different from Ewing sarcoma and Burkitt lymphoma.

How well did you know this?
1
Not at all
2
3
4
5
Perfectly
362
Q

Match the correct lettered answer with the numbered statement

A. 45F with T2N1M0 ductal carcinoma of the breast being considered for therapy with trastuzumab (Herceptin)
B. 40M with T3N0M0 adenoca of the R colon and no polyps
C. Daughter of a 50 y.o. Ashkenazi Jewish descent with invasive carcinoma of breast
D. 50 y.o. With 5 cm stromatolites tumor in the stomach
E. 70F with stage III bronchoalveolar carcinoma of the lung being considered for gefitnib

A
  1. B 2. A 3. D 4. C 5. E

Familial susceptibility to breast ca accounts for <25% of all breast ca cases. BRCA 1 and BRCA 2 are high penetrance breast cancer predisposition genes identified by genetic studies. Germaine mutations found in all cells within the body; peripheral monocytes are used for testing. Clinical criteria for referral for BRCA testing include 1) at least 3 breast or ovarian cancer cases, at least 1 in the same family when the pts is <50 yrs, 2) breast can cases when pt is <40 yrs in same family, 3) male breast ca and family member with either ovarian cancer early or early onset female breast ca, 4) pt <60 yrs with breast ca in a family of Ashkenazi Jewish descent, 5) pt < 40 yrs with bilat breast ca, and 6) breast ca and ovarian ca in same pt.

Historically GISTs were through to be of smooth muscle or neural origin. Mid 1990s KIT was recognized to play a key role in development of interstitial cells of Cajal and cells of origin for GISTs. KIT is commonly expressed in GISTs and is a tyrosine kinase whose activity is normally regulated by binding of endogenous ligand, stem cell factor. Binding of stem cell factor to KIT results in receptor homodimerization and activation of its tyrosine kinase activity. In >80% of GISTs, mutation in KIT gene occurs, leading to constitutive activation. Imatinib is a small molecule that is a specific inhibitor of a number of tyrosine kinase enzymes, including c-kit. It inhibits c-kit by occupying the TK active site, leading to decr in activity.

HNPCC is a common, autosomal dominant syndrome characterized by early onset of cancer (avg age <45 y.o.), development of neoplasticism lesions in a variety of tissues (e.g. Endometrial, gastric, renal, ovarian, and skin) and microsatellite instability (MSI). Tumors should be tested for MSI in the following situations: 1) CRC dx in pt <50 yrs, 2) presence of synchronous, metachronous CRC or other HNPCC assoc tumors regardless of age, 3) aCRC with MSI-H histology dx in of <60 yrs old 4) CRC dx in at least 1 first degree relative with an HNPCC related tumor, with 1 of the cancer being dx at <50 y.o, 5) CRC dx in at least 2 first or second degree relatives with HNPCC related tumors, regardless of age

ERBB encodes a tyrosine kinase inhibitor involved in growth signaling and belongs to the epidermal growth factor receptor family. Overexpression secondary to gene amplification is the most common mechanism or ERBB2 (HER2/neu) protein overexpression and can lead to oncogenic transformation. Overexpression in 18-20% of invasive breast carcinomas and is an independent marker for poor clinical outcome in newly dx pts, who usually have ER neg phenotype. From a therapeutic point of view, pts with breast ca and incr Her2 have a good therapeutic response to traztuzumab, a human monoclonal antibody. Patients also have relative resistance to endocrine therapies, especially SERMs (e.g. Tamoxifen)

EGFR encodes a cell membrane receptors with protein tyrosine kinase activity and downstream mitogenic effects. Nearly 90% of lung ca specific EGFR mutations consist of point mutations that lead to constitutive action of protein tyrosine kinases. Testing for EGFR mutations in lung ca has become widespread b/c pts with non small cell lung cancer carrying mutations respond more often to PTK inhibitors such as gefitinib. Trials show mutation status to be an independent predictor of response, progression free survival and OS in pts with NSCLC tx with gefitinib. Mutations are more common in adenocas of female pts who never smoked and are of Asian descent

How well did you know this?
1
Not at all
2
3
4
5
Perfectly
363
Q

Match the correct lettered answer with the numbered statement

A. Papillary thyroid cancer
B. Huerthle cell thyroid cancer
C. Medullary thyroid cancer
D. Anaplastic thyroid cancer
E. Follicular thyroid cancer
  1. Amyloid
  2. Nuclear grooves and inclusions
  3. Oxyphilic cells
  4. Calcitonin
A
  1. C 2. A. 3. B 4. C

Papillary thyroid cancer can be accurately dx by FNA based on presence of hyper cellular aspirate containing nuclear overcrowding, grooves or inclusions. Total thyroidectomy is tx of choice for papillary thyroid cancers >1 cm in diameter. RET gene trans locations are present in some as somatic genetic changes. Should not be confused with RET gene codon mutations, which are assoc with medullary thyroid cancer

Follicular thyroid cancer cannot be dx by FNA. Follicular cancer can be distinguished from benign follicular Adenoma by capsular or vascular invasion, which can be determines reliably only on permanent histology. Follicular cancers or adenomas have an identical appearance on FNA and are collectively called follicular neoplasms. Definitive tx is total thyroidectomy

Huerthle cell cancers consist of oxyphilic cells that appear pink and are filled with mitochondria. Cannot be dx by FNA. Can be distinguished from benign Huerthle cell Adenoma by capsular or vascular invasion., which can be determined reliably only on permanent histology. Huerthle cell cancers and adenomas have identical appearance on FNA and are collectively called Huertle cell neoplasms. Definitive tx is total thyroidectomy

Medullary thyroid cancer is characterized by presence of stromatolites amyloid and absence of thyroid follicles; can be dx by FNA. Amyloid stains strongly with Congo red staining. FNA can’t always distinguish MTC on bases of appearance of cells alone, immunohistochemistry staining of FNA sample for chromogranin A, CEA or calcitonin can confirm neuroendocrine origin of MTC from parafollicluar C cells. Serum calcitonin of CEA elevations also confirm dx. Total thyroidectomy and bill central neck dissection is OR of choice for pts with clinically evidence disease. Ipsilateral lateral neck dissection is indicated for pts who have large primary tumors or who have enlarged LNs in the lateral compartment

FNA is typically sufficient to dx anaplastic thyroid cancer. Immunohistochem can be used to verify cytologic dx. Mgmt is difficult b/c of rapid progression of disease and uniformly fatal outcome. Although surgical resection with aggressive adjuvant chemorads may be considered for rare small tumors confined to the thyroid, almost all pts with anaplastic thyroid cancer will have unresectable disease and should be offered non surgical tx under investigation

How well did you know this?
1
Not at all
2
3
4
5
Perfectly
364
Q
Morbidly obese patient with blunt trauma.  Unstable.  Negative fast.  Normal cxr. Still unstable post fluids, best manage.
A. Ct abdo
B. Peritoneal aspiration diagnostic
C. Laparotomy
D. Pericardiocentesis
A

C

How well did you know this?
1
Not at all
2
3
4
5
Perfectly
365
Q
Lady w 3.5 cm pash. Mng?
A. Excise
B. Excise 1 cm margin
C. Radiate
D. Observe
A

D

Pseudoangiomatous stromal hyperplasia (PASH) is a benign stromal proliferation that simulates a vascular lesion. PASH may present as a mass or thickening on physical examination. The most common appearance on mammography and ultrasound is a solid, well-defined, noncalcified mass.
The characteristic histologic appearance is a pattern of slit-like spaces in the stroma between glandular units. PASH can be confused with mammary angiosarcoma.
If there are any suspicious features on imaging, the diagnosis of PASH on a core biopsy should not be accepted as a final diagnosis, and excisional biopsy should be performed. However, in the absence of suspicious imaging characteristics, a diagnosis of PASH at core biopsy is considered sufficient, and surgical excision is not always necessary. There is no increased risk of subsequent breast cancer associated with PASH.

How well did you know this?
1
Not at all
2
3
4
5
Perfectly
366
Q
Mvc blunt trauma.  Closed head.  Grade 4 spleen with active bleeding. Stable.  Mgmt?
A. Splenectomy
B. Serial hgb
C. Splenic embolization
D. Observe
A

A

From feliciano trauma, head injury is a relative reason to operate

How well did you know this?
1
Not at all
2
3
4
5
Perfectly
367
Q
Young lady. 3 cm lesion that has doubled in size over 2 years, prev core showed fibroadenoma.  Manage.
A. Excise 2 cm margin
B. Excisonal biopsy
C. Observe, fu us 6 mo
D. Core biopsy
A

B

How well did you know this?
1
Not at all
2
3
4
5
Perfectly
368
Q
Old lady w 4 cm phyllodes.  
A. Lumpectomy 
B. Simple mastectomy
C. Simple mastectomy and slnb
D. Mrm
A

A

How well did you know this?
1
Not at all
2
3
4
5
Perfectly
369
Q
Lady w er positive, her2 neu negative ductal carcinoma.  Type?
A. Luminal type a
B. Luminal type b
C. Basal
D. Her2 enriched
A

A

How well did you know this?
1
Not at all
2
3
4
5
Perfectly
370
Q
Young guy presents 2 weeks after embolization of spleen.  Febrile. Tachycardia.  Ct shows 12cm subcapsular splenic hematoma.  Mng
A. Splenectomy
B. Perc drain
C. Embolize main artery
D. IV ABx
A

A

How well did you know this?
1
Not at all
2
3
4
5
Perfectly
371
Q

Gsw left abdo. Pain distended and has pain. Diminished left femoral pulse. Stable. Ct shows bullet on right pelvis. Mng?
A. Exploratory lap and explore retroperitoneum
B. Lap first then angio later
C. Angio
D. Observe

A

A

How well did you know this?
1
Not at all
2
3
4
5
Perfectly
372
Q
Location of superior parathyroid 
A. Medial to superior thyroid vein
B. Dorsal to RLN
C. Ventral to RLN
D. Medial to RLN
A

B
Posterior and superior to nerve
Dorsal and lateral to the RLN.

How well did you know this?
1
Not at all
2
3
4
5
Perfectly
373
Q

Lady with elevated calcium serum, urine, and elevated PTH. Approach with the highest chance of success?
A. Four gland exlploration with intraop frozen section
B. Local exploration with venous sampling
C. Local exploration with intraop PTH
D. Local exploration with gamma probe

A

A

How well did you know this?
1
Not at all
2
3
4
5
Perfectly
374
Q
3.4 cm papillary thyroid cancer in person with no history radiation or famhx of thyroid cancer. Mgn?
A. Total thyroid and central compartment
B. Hemithyroidectomy
C. Total thyroid
D. RAIU
A

C? (B/c old question?)

NCCN says consider total vs lobectomy if no prior rads, no distant mets, no cervical LN mets, no extrathyroidal extension or tumor <4 cm

How well did you know this?
1
Not at all
2
3
4
5
Perfectly
375
Q

Self inflicted injury to anterior neck. At exploration has 1.5 cm laceration in 3rd tracheal ring. Mgn?
A. Tracheostomy through wound
B. Primary repair, no trach
C. Primary repair and trach thru cricothyroid membrane
D. Leave et tube in 48 hours

A

B

Feliciano recommends primary repair for injuries less than 50% of the airway with minimal devitalization.

How well did you know this?
1
Not at all
2
3
4
5
Perfectly
376
Q
Patient 48 hours after trach.  Bleeding around tracheostomy. Mng?
A. Pack with hemostatic
B.Go back to OR and revise
C. Pull trach out and intubate
D. Ct and bronch
A

D

Rule out tracheoinominate fistula is bleeding >48 hrs.
If bleeding <48 hrs likely skin bleed and can do A.

How well did you know this?
1
Not at all
2
3
4
5
Perfectly
377
Q
Middle age guy, with tumor in stomach. Biopsy shows adeno.  EUS shows submucosal invasion. No nodes on CT. Peritoneal washings negative.  
A. D1 gastrectomy 
B. D2 with spleen 
C. Total gastrectomy
D. Neoadj
A

D?

Old answer key says A.
This is a T1b tumor. NCCN says surgery alone vs neoadjuvant then surgery

How well did you know this?
1
Not at all
2
3
4
5
Perfectly
378
Q

Guy with bleeding antral tumor with liver met on ct.
A. Wedge stomach
B. Antrectomy with wedge liver
C.Antrectomy
D. Antrectomy with anatomic liver resection

A

A?

Terrible question. Patient has metastatic gastric cancer. In real life, would try endoscopy or IR to stop bleeding

How well did you know this?
1
Not at all
2
3
4
5
Perfectly
379
Q
Lady epigastric pain, bile on scope. Had BII for gastric ca 1 year ago.  No evidence of disease on ct. Mng?
A. Revise to retrocolic BII
B. Prokinetic
C. Roux en Y
D. BI
A

C

R en Y is treatment for bile reflux and afferent loop syndrome

How well did you know this?
1
Not at all
2
3
4
5
Perfectly
380
Q
Lady with BII for something.  Forceful bilious emesis post prandial without food in it.  Ct with oral shows patent GJ with distended DU.  Mgn?
A. Revise to retrocolic BII
B. Prokinetic
C. Braun
D. Steroids
A

C

Alkaline Reflux Gastritis: patient dosent feel better after vomiting, usuall fails to respond to medical therapy although Urosdeoxycholic acid might be promising.

Surgical Options are: Braun(enteroenterostomy),distal enteroenterostomy , Roux-en-Y , Henley Loop. (C)

How well did you know this?
1
Not at all
2
3
4
5
Perfectly
381
Q
Gastric carcinoid in antrum 1.5 cm.  No gastritis and no fam hx and negative gastrin.
A. PPI
B. D1 antrectomy
C. Endoscopic polypectomy
D. Surveillance
A

B

Type 1, which represent 70 to 80 percent of all gastric carcinoids, are associated with chronic atrophic gastritis. In this condition, serum gastrin rises in response to gastric achlorhydria. The elevated gastrin, in turn, simulates neuroendocrine cell hyperplasia in the stomach and development of multifocal polypoid carcinoid tumors. The clinical behavior of these tumors is usually indolent.

Type 2, which represent approximately 5 percent of gastric carcinoid tumors, also occur as a result of elevated serum gastrin levels stimulating multifocal gastric carcinoid tumors. The underlying cause of type 2 gastric carcinoids is a pancreatic or duodenal gastrinoma (Zollinger-Ellison syndrome). The clinical behavior is usually indolent.

Type 3 gastric carcinoids (sporadic carcinoids) occur in the absence of atrophic gastritis or the Zollinger-Ellison syndrome. They account for 20 percent of gastric carcinoids and are the most aggressive; local or hepatic metastases are present in up to 65 percent of patients who come to resection.

Type 3 (sporadic) gastric carcinoids are treated by partial or total gastrectomy with local lymph node resection. The risk of nodal metastases is dependent on tumor size and depth, and some have suggested that endoscopic resection alone may represent adequate therapy for intraepithelial tumors <2 cm and perhaps for tumors <1 cm invading the lamina propria or submucosa. However, this is not a standard approach.

For type 1 and 2 gastric carcinoids smaller than 1 to 2 cm, endoscopic resection represents adequate therapy. Subsequent endoscopic surveillance is needed every 6 to 12 months since these patients continue to exhibit mucosal changes and hyperplasia of enterochromaffin-like cells (ECL) due to sustained hypergastrinemia.
Progression to a malignant phenotype or disease-related death is rare with small tumors. Metastases occur in less than 10 percent of tumors ≤2 cm.

Antrectomy is a reasonable option for type 1 gastric carcinoids if there are numerous progressive tumors. Antrectomy reduces hypergastrinemia by reducing the gastrin-producing cell mass in the antrum of the stomach; in most cases, this leads to tumor regression. The success of this approach was shown in a series of 51 patients with type I carcinoids, 10 of whom had antrectomy (eight in conjunction with endoscopic removal of the largest tumor)]. Seven of the eight with residual disease became endoscopically tumor-free, and one progressed and died of metastatic disease. In all, 9 of the 10 patients treated with antrectomy remained tumor-free for an average of 65 months.
More aggressive surgical therapy is rarely needed for type 1 gastric carcinoids, unless there is extensive tumor involvement of the gastric wall (which increases the risk for a coexisting adenocarcinoma, tumor size >2 cm (which increases the risk for metastases, poorly differentiated histology, or for emergent bleeding .

How well did you know this?
1
Not at all
2
3
4
5
Perfectly
382
Q
Guy pod 3 for roux en Y gastric bypass.  Has sudden severe abdominal pain.  Contrast swallow shows dilated gastric pouch, and no leak.
A. ppi
B. Surgical exploration
C. PEG tube
D. CT PE
A

C

How well did you know this?
1
Not at all
2
3
4
5
Perfectly
383
Q
Guy pod 1 for rou en Y.  HR 140.  needs 7 l of fluid to maintain UO. Tachypnea. Mgn?
A. Ct
B. Scope
C. Contrast swallow
D. Surgical exploration
A

D

How well did you know this?
1
Not at all
2
3
4
5
Perfectly
384
Q
Lady comes 6 months after lap band.  Sudden onset epigastric pain, radiating to back.  Unable to tolerate fluids or PO.  Best management?  (really it says best management)
A. Deflate band
B. Remove band
C. Contrast swallow
D. Scope
A

B

How well did you know this?
1
Not at all
2
3
4
5
Perfectly
385
Q
79 yr old guy prev lung resection for cancer.  Has gerd.  Scope shows 10 cm segment of barret’s with polyp.  Biopsy from polyp shows HGD. Mgn?
A. Esophagectomy
B. Ablation
C. EMR
D. Repeat scope 3 months
A

C

How well did you know this?
1
Not at all
2
3
4
5
Perfectly
386
Q

Patient with epiphrenic diverticulum. 3 cm. Mgn?
A. Resect
B. Resect and myotomy
C. Resect and myotomy with anti reflux procedure
D. Myotomy and anti reflux procedure

A

C

Small (<2 cm) diverticula can be suspended from the vertebral fascia and need not be excised. In patients with severe chest pain, dysphagia, or a documented motor abnormality, a long esophagomyotomy is indicated. If a diverticulopexy is performed, the myotomy is begun at the neck of the diverticulum and extended onto the LES. If a diverticulectomy is pursued, a vertical stapling device is placed across the neck, and the diverticulum is excised. The muscle is closed over the excision site, and a long myotomy is performed on the opposite esophageal wall, extending from the level of the diverticulum onto the LES. If a large hiatal hernia is also present, the diverticulum is excised, a myotomy performed, and the hiatal hernia repaired. Failure to repair the hernia results in a high incidence of postoperative reflux.
Schwartz: In patients selected for surgery, preoperative manometry is essential to determine the proximal extent of the esophageal myotomy. Most surgeons extend the myotomy distally across the LES to reduce outflow resistance. Consequently, some form of antireflux protection is needed to avoid gastroesophageal reflux.

How well did you know this?
1
Not at all
2
3
4
5
Perfectly
387
Q
Which of these patients can have curative resection in distal esophageal cancer?
A. Virchow node
B. Celiac node
C. Invading crus
D. Mediastinal node
A

B

cervical goes to cervical nodes; mid esoph to mediastinal; distal to abd. Crus is a T4a lesion. Bad nodes are defined as beyond one nodal station, therefore for a distal cancer, b is ok. (B)

How well did you know this?
1
Not at all
2
3
4
5
Perfectly
388
Q
Guy with fundo in past.  Still having reflux symptoms, not controlled by PPIs.  Ct shows fundo in good position. Scope shows normal fundo, no signs of reflux.  Mng?
A. Redo fundo
B. ph and manometry
C. Gastrograffin
D. Continue PPIs
A

B

How well did you know this?
1
Not at all
2
3
4
5
Perfectly
389
Q
Guy with anterior trach injury...you are repairing primary the 3rd ring.  Where will you place your tracheostomy?
A. Cricothyroid cartilage
B. 2nd ring
C. 1st ring
D. 4th ring
A

D?

You want to trach below but 4th is pretty low…

390
Q
Guy presents with anemia.  Scope shows mass..biopsy shows adenoCA.  CT shows invading spleen and pancreas.  Perigastric nodes. Mgn?
A. Chemorads
B. Enbloc resection
C. Neoadjuvant
D. Rads
A

C

391
Q
Ruptured umbilical hernia in child C cirrhotic.  3 cm.  Leaking a little. Mgn?
A. Tips and diuretic
B. Paracentesis and primary repair
C. Mesh repair then diuretics
D. Diuretics and paracentesis
A

B

392
Q
Which hernia repair most urgently in a cirrhotic? 
A. Umbilical
B. Inguinal
C. Femoral
D. Incisional
A

C

393
Q
69 year old lady.  Excisional of mammo lesion.  6mm focus of DCIS, well differentiated, ER postive. 12 mm margin, and small 5 mm focus of LCIS with 1 mm margin.
A. Radiation
B. Tamoxifen
C. AI
D. observe
A

B

For DCIS, can omit radiation if margin >10 mm. Don’t need to do anything further for the LCIS

394
Q
Metastatic colon cancer.  Which regimen has been proven to improve survival and rapid remission?
A. Mayo regimen
B. FOLFOX4
C. 5FU leucovorin 
D. Cisplatin
A

B

395
Q
Lady with excised phyllodes tumor. 3 cm.  5 mm margin. Intermediate grade or malignancy risk. Mgn
A. Radiation
B. Re-excise with 1 cm margin
C. Mastectomy
D. Mastectomy and slnb
A

B?

Wide excision with intention of obtaining surgical margins >/=1 cm. Narrow margins are assoc with heightened local recurrence but are not an absolute indication for mastectomy
No prospective RCT for use of radiation with phyllodes. In the setting where additional recurrence would create significant morbiditiy (i.e. chest wall), radiation may be considered following the same principle as applied to sarcoma

396
Q
5 cm mass in young man, anterior thigh.  Ct suggests sarcoma.  Core biopsy inconclusive.  Mng
A. Radiation
B.Excisional biopsy
C. Observe
D. WLE
A

B
Excisional biopsy is recommended only for small cutaneous or subcutaneous tumors, usually smaller than 5 cm, in which a wide reexcision (if required) is usually straightforward.

397
Q
GSW to neck.  Stable.  Ct angio shows pseudoaneurysm 1 cm distal to bifurcation with clot.  Flow is going passed clot. Mgn?
A. Surgical repair
B. Stent
C. Embolize with angio
D. Asa and heparin
A

A

According to a lit review in the surgical annals of vascular surgery, accessible lesions should have an open repair. (A)

398
Q
7 year old.  Mvc.  No vitals on scene.  Bag mask, and 10 min of CPR.  In ER, still no pressure, good AE bilateral.  EKG shows wide complex rhythm with pulse 30. mgn?
A. Declare patient
B. Intubate, IV and ED thorocatomy
C. Intubate, IV and chest tubes
D. Intubate, IV and massive transfusion
A

A

399
Q
Chest trauma, multiple rib fractures and occult hemothorax with flail.  In ICU 2 weeks, cant wean.  Mng?
A. VATS
B. Thoracotomy and decortication
C. Rib fixation
D. Chest tube
A

C

Indications for repair are failure to wean from the vent. uncontrollable pain despite maximal therapy, major deformity, non-union # and already there for something else.

400
Q
Guy post op from diverticular resection for acute. In recovery, tachy, febrile.  Sat 87% on 10 l bag mask.  NEXT Step?
A. Ct chest
B. Back to OR
C. IV fluids
D. Intubate
A

D

401
Q

At elective laparotomy for diverticular disease, encounter 2 cm abcess and colovesicalur fistula?
A. Hartmann’s
B. Sigmoid resection and repair fistula
C. Sigmoid resection, repair fistula and diverting ileostomy
D. Drain and diverting ileostomy

A

B

Operative management should include resection of the affected segment of the colon involved with diverticulitis (usually with a primary anastomosis) and simple repair of the secondarily involved organ. Suspicion of carcinoma may mandate a wider, en bloc resection.

402
Q

Guy obstructing rectosigmoid cancer. At laparotomy, invading sidewall, ureter and bladder.
A. Ilestomy
B. Colostomy
C. Peel off and resect
D. Enbloc resection colon, bladder and ureter

A

B

403
Q
Contra-indication to resection of recurrent rectal cancer?
A. Bilateral hydronephrosis
B. Incontinence
C. Pain
D. Invasion into bladder
A

A
Contra-indications to re-resection:
Nerve root involvement above the level of L1-2
Proximal (S1,2) sacral invasion extending to the sacral promontory
Involvement of the paraaortic lymph nodes
Tumor encasement of the iliac vessels
Extension of tumor through the greater sciatic notch
Bilateral ureteral obstruction
Unresectable extrapelvic disease
Circumferential involvement of the pelvic wall

404
Q

Anterior duodenal injury 3 cm from ERCP. 40 percent
A. Primary repair with freshen edges and drainage
B. Pyloric exclusion with duodenostomy
C. Duodenostomy
D. Whipple

A

B?

Old answer key said A

405
Q
Post mastectomy hematoma, 25 cm, drain not draining
A. Evacuation in OR
B. Drain at beside with 2cm incision
C. Irrigate drain
D. Observe
A

A

406
Q
35 year old, has BRCA2, mammo with birads 2 and 75% density, what next
A. Mammogram next year
B. MRI now
C. US now
D. Bilateral mastectomy
A

B

407
Q
40F 2cm spiculated mass seen on mammongram, core biopsy came back as radial scar . What is the best MX? 
A. Excisional biosy
B. Do nothing
C. Mastectomy
D. Recore biospy
A

A

Review course–controversy re excision of radial scars but there is a mass here which requires further work up

408
Q
Breast cancer 5cm with 4 positive nodes on dissection. Triple negative. How to treat adjuvantly?
A. Chemo and rads
B. Rads only
C. Chemo only
D. Observe
A

A

Post-mastectomy radiation improves OS and DFS. Indicated in large primary, 4 or more nodes +.

409
Q
40yo lady with persistent serous drainage from breast, single duct at 9 o’clock . What to do FIRST?
A. Bilateral mammo
B. Galactogram
C. U/S
D. MRI
A

A

410
Q

73F with 7.5cm ER+ her2- breast ca. 1.2cm node with normal hilum on imagine. Bone scan consistent with multiple bone mets.
A. MRM and tamoxifen
B. AI followed by tamoxifen if no tumor response
C. AI followed by chemo if no tumor response
D. AI followed by resection if tumor response

A

B

Adjuvant endorine is preferred. Can trial 2nd line endocrine if no response to AI. Also needs bisphosphonate.

411
Q
40F with recurrent breast infections, 2 previous I and Ds, fistula opening found lateral to areola
A. Antibiotics
B. I and D
C. NAC resection
D. Resection subareolar ducts
A

D

412
Q
Metastatic breast cancer, tachypnic, tachy, sbp 95, distended neck veins, b/l effusions and cardiomegaly on cxr
A. Bilat chest tubes
B. Perc pericardiocentesis
C. Echo 
D. Ct angio
A

C

Cardiac dysfunction due to chemo.
Echo will evaluate cardiac function (cardiotoxic chemo?), pericardial effusion, PE. Evaluate cause prior to treatment.

413
Q
35 year old female with axillary node positive for cancer on FNA, no primary lesion found on full workup
A. Modified radial mastectomy
B. Axillary dissection
C. Axillary dissection with radiation
D. Radiation
A

A vs C

Up to Date:
Either MRM or ALND with WBI is acceptable with equivalent outcomes. MRM is standard therapy, and there are no prospective trials validating WBI as an alternative. Systemic therapy for stage II disease.

414
Q
Rapidly growing 4cm angiosarcoma of breast in middle aged ish woman
A. Mastectomy
B. Partial mastectomy
C. Mastectomy and SNB
D. Radiation
A

A

415
Q
Pregnant 12weeks, small invasive breast cancer, palpable nodes (biopsy of node not done). Plan?
A. Lumpectomy and Cx and Rx
B. Lumpectomy and Cx
C. Lumpectomy and Rx
D. Terminate pregnancy
A

B

Doesn’t specify timing of RT in answers–needs to be post partum.

Since end of 1st trimester (like 1 week away!) and palp nodes would consider neoadjuvant then surgery then radiation post partum
If early in 1st trimester then mastectomy

416
Q
Pt with Lupus and has cribiform 0.9mm DCIS focal in upper outer quadrant. How to treat
A. Total mastectomy and SLNBx
B. Lump and Rx
C. Lump and chemo
D. WLE/lump
A

A

417
Q
Diffuse DCIS , mastectomy and deep margin 2mm – what to do?
A. Nothing
B. Radiation
C. Re-excision of scar and deep margin
D. Chemo
A

A

Margin is negative; goal is minimum 2mm margins. If postive, radiate bed. NO data on post-mastectomy RT for DCIS! However, do need to treat the contralateral breast with surveillance and tamoxifen or AI depending on menopausal status.

418
Q
BRCA1 pt, shes 35yo – How to reduce cancer-related mortality?
A. Bilateral mastectomies
B. Bilateral salpingooophorectomy
C. MRI screening
D. Tamoxifen
A

B

419
Q

73 yo M with +ER/PR breast cancer, management?
A. Tamoxifen x 5 years
B. Aromatase inhibitor x 5 years

A

A

AIs not studied in men.

420
Q
Idiopathic granulomatous disease of the breast, recurrent abscesses, management?
A. Steroids
B. Abx and drainage
C. Smoking cessation
D. Partial mastectomy
A

A

Very controversial. Observe if option, as many resolve with time. Ralph George (Toronto breast surgeon) says steroids. Meiers say steroids are also reasonable, but better current answer is methotrexate; treat for long courses, up to 1 year before maximal response expected.

Up to Date: Management—IGM is a self-limiting inflammatory condition but commonly takes 9 to 12 months to resolve. IGM associated with a localized infection usually resolves with antibiotics and drainage [8,21,27,29]. The approach to antibiotic therapy is as outlined above for periductal mastitis. Surgical excision for IGM is often followed by slow wound healing and is not advocated. (See ‘Periductal mastitis’ above.)
There is no role for steroid use; systemic glucocorticoid therapy and local depot steroid injections have been used for treatment of IGM based on the hypothesis that it is an autoimmune disease, although there are no randomized controlled trials that demonstrate efficacy for steroids in the treatment of IGM [27,39,43,44]. Methotrexate for treatment of IGM has been described [45-47]. Discontinuation of these drugs has been associated with rebound inflammation.

421
Q
Female wants breast conserving surgery, best way to increase her chances of success?
A. Lumpectomy
B. MRI 
C. Lumpectomy with frozen section
D. Lumpectomy with imprint cytology
A

A

MRI – higher mastectomy rate

422
Q

Patient with adenocystic carcinoma of the breast, management?
A. Lumpectomy with SLNB and radiation with chemo
B. Total mastectomy with SLNB and chemo
C. Lumpectomy with radiation and chemo

A

A
Some say with no chemo?

ACC is a favourable histology with low risk of LN mets (<1%). It is a breast carcinoma, so safe answer is to do SLNB. Clear that BCS is adequate, and there is survival advantage to RT. No data on chemo, but NCCN recommends adjuvant endocrine for favourable histology >3cm, and chemo for >1cm. However, adjuvant chemo has not been studied specifically for ACC.

Clinical role of sentinel-lymph-node biopsy in breast cancer. The Lancet Oncology. Volume 3, Issue 2, February 2002, Pages 105–110
Pts with ACC benefit from SLNB d/t low risk of LN mets.

Adjuvant radiation therapy is associated with improved survival for adenoid cystic carcinoma of the breast. J Surg Onc, 2010.

“RT after local surgical therapy for ACC of the breast improved both cause-specific and overall survival. Use of RT in this rare tumor should be considered in patients otherwise eligible for RT.”

<1% chance of LN mets

Adenoid cystic carcinoma of breast: Recent advances. World J Clin Cases. 2014 Dec 16; 2(12): 732–741.

a. have t (9:22) translocation
b. luminal and basal features
c. low grade, usually indolent, excellent prognosis vs other TNBC
d. usually cured with BCS +/- RT

423
Q

Isolated tumor cells to SLNB of breast cancer, management?
A. Chemotherapy
B. Axillary dissection
C. Observe

A

C

NCCN factors micromets into adjuvant decision, but ITC probably has no prognostic significance. For small tumours (<5mm) where chemo would not normally be given, it will be considered if Nmi+

424
Q

Female with Her-2 +ve breast cancer
A. Chemo + Herceptin, then Surgery, Then Herceptin
B. Chemo, then Surgery, then Herceptin
C. Chemo, then surgery, then Chemo + Herceptin
D. Chemo + Herceptin, then surgery, then Herceptin

A

D!
(Although in this question A and D are the same answer!)

Herceptin never given alone. Options really are NACT+Herceptin then ACT + Herceptin, or NACT + Herceptin

425
Q
32 years old, 3.2 cm had mastectomy, Triple Negative, with 1 mm LN +ve, 
A. ALND for local regional
B. ALND for staging
C. Axillary Radiation
D. Chemo
A

D

Definitely no ALND for micromets, but MA.20 may apply: high-risk node – or + patients treated with axillary radiation had less recurrence but same OS. CTX is definitely indicated, so any answer must include chemo.

426
Q

Palpable breast mass with squamous metaplasia on core biopsy.
A. Observe
B. Excisional biopsy

A

B

427
Q
Old man with ulcerated firm breast nodule, adherent to pec. no mention about axilla. 
A. Modified radical then adjuvant chemo
B. Sentinel lymph node then neoadjuvant 
C. Neoadjuvant then MRM
D. Resect the pec major with MRM
A

C

Locally advanced breast cancer meeting criteria for NACT; axilla needs staging; SLNB is relatively contraindicted in locally advanced (T4) as has a high probability of nodal involvement.

Pectoralis adherence does not equal T4a. Ulceration is T4b.

UTD

Although there have been no randomized trials, most men with early-stage disease (T1 to T2, N0 to N1) undergo a simple mastectomy rather than a mastectomy with pectoral muscle removal. The data to support less extensive chest wall resection (originally as a modified radical mastectomy rather than radical mastectomy) come from retrospective studies that consistently show the procedures result in an equivalent local recurrence rate and survival outcomes.
Locally advanced disease—Men who present with locally advanced disease (T3N0 or stage III disease) or inflammatory breast cancer are treated similarly to women who present in this way. We offer these patients neoadjuvant chemotherapy because randomized trials of neoadjuvant chemotherapy followed by surgery versus primary surgery followed by adjuvant chemotherapy, which have been predominantly performed in women, show it is associated with high rates of clinical response and a higher likelihood for allowing cosmetically acceptable surgery without compromising survival outcomes. Following completion of chemotherapy, we proceed with a mastectomy in most patients, provided they have a sufficient response to make surgery feasible. For patients who do not have a sufficient response, an alternative regimen should be administered
Management of the regional nodes—Assessment of the regional nodes is an important aspect of surgical breast cancer management, as it is in women. We agree with an expert panel convened by the American Society of Clinical Oncology (ASCO) and feel that sentinel lymph node (SLN) biopsy for men with clinically node-negative breast cancer is “acceptable” Although the role of SLN biopsy is established in women with clinically node-negative breast cancer, its proven role in men is less robust due to the rarity of male breast cancer.
Most studies have restricted SLNB to T1 or T2 breast cancers <5 cm in size, since larger tumors have a higher likelihood of positive axillary nodes. However, some studies have shown that SLNB is accurate in patients with T3 tumors and clinically negative axillae [87,88]. Thus, many clinicians do not recognize large breast tumors as a contraindication to SLNB, as long as the axilla is clinically negative. However, patients with T4 tumors (locally advanced) or inflammatory breast cancer are not considered candidates for SLNB. The false-negative rate is high in patients with inflammatory breast cancer, presumably because of the presence of partially obstructed, functionally abnormal subdermal lymphatics.

428
Q

Palpable mass. hypo echoic on ultrasound. upper outer quadrant. PASH with ADH.
A. Repeat imaging in 3-6 months
B. Excisional biopsy
C. Lumpectomy with SLNB

A

B

Ralph George: don’t need to excise mass-associated PASH; excise ALL if postmenopausal women (if not on HRT), as PASH is estrogen-related lesion.

2 reasons to excise: mass-associated PASH, and ADH

429
Q

High grade DCIS, 1mm superior margin.
A. Resect 1cm margin + Radiation
B. Resect 1cm margin + SLNB + Radiation
C. Radiation

A

A

430
Q
54 years old, Spontanous, unilateral, serous breast discharge, best INITIAL management
A. Galactogram
B. Lumpectomy
C. Ultrasounds
D. Bilateral Mammogram
A

D

431
Q

Breastfeeding woman with pain, erythema tenderness in breast. Ultrasound does not show abscess. Treatment?
A. Stop breast feeding, po antibiotics
B. Continue breast feeding, po antibiotics
C. Stop breast feeding, IV antibiotics
D. Continue breastfeeding, IV antibiotics

A

B

432
Q
Inflammatory breast ca with bulky nodes and 12cm primary. Neoadjuvant chemo given, partial axillary response but no change in the primary.
A. Radiation
B. 2nd line chemo
C. Resect with flap closure
D. Resect with primary closure
A

B

NCCN: if no response, consider 2nd line chemo and/or RT.
UTD: RT, no 2nd line chemo.

IBC is systemic disease so treat systemically.

433
Q
Lady who had breast cancer 5yrs ago, ER+, 5 years of tamoxifen. Shows up with 6cm lesion in liver, Bx shows ER+ve ductal ca. Investigation and imaging show isolated liver met on day that you see her.
A. Resect
B. Endocrine therapy
C. Chemotherapy 
D. Refer for palliation
A

C
Prev years say A

Want to ensure that lesion is responsive and isolated before considering resection per HPB fellow and surg onc staff

434
Q

60s female with previous history breast ca. Presents with chronic abdominal pain. CT shows ascites with peritoneal seeding.
A. Chemotherapy and hormone replacement therapy
B. HIPEC
C. Paracentesis
D. Resect the peritoneal mets

A

C

UTD
Paracentesis is mainstay of management in 90% of patients

435
Q
ER/PR positive and HER 2 negative. 45y with 2.9cm lesion. 
A. Oncotype Dx
B. Chemo
C. Tamoxifen
D. AI
A

A

NCCN: ER + HER2 – T1-2 without LN mets: Oncotype DX to determine RS: If low (<18,), endocrine alone. Intermediate risk (18-31) consider ACT; High (>31) ACT + endocrine
ER + HER2 tumours always get ACT with herceptin if >1cm, considered for >0.6cm

Not entirely clear whether Oncotype is useful for premenopausal; Although TAILORx study showed no difference in recurrence for younger vs older women with low RS.

436
Q
Breast cancer with asymptomatic bone mets. ER/PR positive
A. Chemo with bisphosphonates
B. Bisphosphonates and tamoxifen
C. Radiation plus bisphosphonates
D. AI
A

B

Chemo has slightly higher response rates than endocrine alone, but actually no difference in OS.

Endocrine therapy versus chemotherapy—It is commonly thought that chemotherapy results in higher response rates and more rapid responses than endocrine therapy, and it is often used as the initial treatment for patients with hormone receptor-positive metastatic breast cancer with a poor prognosis, especially those with visceral metastases. A meta-analysis that included eight small randomized trials, all published prior to 1995, compared the response rates for chemotherapy alone with those of endocrine therapy alone [44]. The pooled estimate of response rates showed an advantage for chemotherapy over endocrine therapy (relative risk 1.25, 95% CI 1.01-1.54), although the two largest trials had opposite findings [45,46]. No significant difference was seen in overall survival (OS) (hazard ratio [HR], 0.94, 95% CI 0.79-1.12), and on subset analysis, there was no obvious trend to suggest an effect of age, menopausal status, or pattern of metastatic disease on the efficacy of either therapy.

ASCO Guidelines:

Bone-modifying agent therapy is only recommended for patients with breast cancer with evidence of bone metastases.

437
Q
Post menopausal woman with UOQ 2cm mass, not seen on mammo. CNB showed mammary ectasia with squamous metaplasia.
A. Excisional biopsy
B. Excisional biopsy with sentinel node
C. Observe
D. Re-image
A

A

Meiers: MDE should not be mass associated, so discordant therefore excise.

MDE: Mammary duct ectasia (MDE) is a benign abnormality characterized by dilatation of the mammary ducts (1). The frequency of MDE may range from 1.1% (2) to 75% (3), according to the diagnostic method used, which might be clinical, histopathological or necropsy-based (2–4). MDE occurs most often in women around menopause (5,6), also being able to occur in younger women (5), children (7), and men (8). Some degree of dilatation of the ducts occurs with aging, and this can be considered as a normal involution of the breast tissue (3,9).

MDE is benign. Squamous metaplasia is non-proliferative breast lesion

438
Q
Angiosarcoma UOQ 4cm in 40y obese woman.
A. Lumpectomy
B. Lumpectomy with SLNBx
C. Mastectomy
D. Mastectomy with SLNBx
A

A

Breast angiosarcoma is often multicentric with wider involvement than appreciated, so mastectomy is standard of care. Other primary sarcomas mayb be treated with WLE.

Up to Date:

Surgery represents the only potentially curative modality for breast sarcomas. The type and extent of the operation is based upon both the size of the tumor and the size of the breast, as well as histology:

An adequate resection margin is the single most important determinant of long-term survival with breast sarcomas For larger tumors (ie, those >5 cm), the overall cosmetic result is often better with a mastectomy and reconstruction than with lumpectomy. Deep seated tumors, which are close to or involve the chest wall, may require en bloc resection of the chest wall.With the exception of angiosarcomas, the majority of primary breast sarcomas are not multicentric and negative surgical margins are more important than the extent of surgical resection. Breast angiosarcomas (primary or therapy-related) often affect a much larger field of the breast or chest wall than anticipated, and mastectomy is the standard treatment

439
Q
Melanoma of breast near/at areola. 1.2 mm thick.
A. 1 cm with SLNB
B. 2 cm with SLNB
C. Central Mastectomy with SLNB
D. 2 cm with ALND
A

B

Groot: 2cm margin; don’t compromise oncology for cosmesis. Meiers agrees. Orient incision in a wedge so as to close NAC with good cosmesis.

440
Q
54 F with non-cyclical mastalgia and mild breast tenderness for a few months. No other risk factors on history and physical exam was normal. Mammogram also normal.
A. Tamoxifen
B. Danazol
C. Ibuprofen
D. Vitamin E
A

C

Probably fibrocystic changes; ibuprofen is first line.
NSAIDs are 1st line.

441
Q

18 M with gynecomastia not interested in surgery.
A. Testosterone
B. Danazol
C. Tamoxifen

A

C

Up to Date:

Medical management of gynecomastia: 1) tamoxifen 2) testosterone replacement (if hypogonadic).

First line is tamoxifen in adolescents and men, with 3-6 month trial. Usually partial response.

442
Q
Woman had previous lumpectomy with SLNB and XRT 2 years ago, and is currently on tamoxifen. Now on follow-up mammogram, has a spiculated lesion in the exact location of the previous lesion. Biopsy is done which shows fat necrosis. BEST management?
A. Mammogram in 1 year
B. Mastectomy
C. Excisional biopsy
D. Anastrazole
A

A

443
Q
60 F with a 5cm breast mass. Biopsy shows adenoid cystic carcinoma of the breast.
A. Partial mastectomy + SNB + XRT
B. Partial mastectomy + XRT
C. Lumpectomy
D. WLE
A

A

See above. RT is probably indicated, reducing local recurrence. There is a very low incidence of LN mets (1-5%), and some argue that this means women will benefit from SLNB. This is a low-grade, favourable histology T3 carcinoma, so the safe answer is RT + SLNB, accepting overtreatment.

444
Q
Recurrent breast Ca in scar and infraclavicular, after MRM and ALND
A. Hormones
B. Chemo
C. Radiate, Excise, Chemo
D. Surgery, Radiate, Chemo
A

D

Infraclavicular is local disease, surgery is first line. Resection indicated when possible. NCCN has surgery first, then RT. If unresectable, NACT.

445
Q
Recurrent breast Ca in breast and supraclavicular nodes after lumpectomy and ALND
A. Radiation
B. Chemotherapy
C. Surgery
D. Endocrine therapy
A

B?

NCCN says tx supraclav with radiation if possible an d consider systemic therapy

Patient had prev lumpectomy and therefore would likely have had radiation before

Surgery has no role in supraclavicular recurrent disease

446
Q

Elderly lady with Breast Cancer ?7.5 cm and palpable nodes with Bone Mets (Asymptomatic)
A. Chemo
B. AI followed by Tamoxifen if progression
C. Aromatase inhibitors followed by surgery when shrinks
D. Surgery First

A

B

UTD
Second-line treatment—For patients with disease progression following first-line endocrine therapy, second-line treatment is a reasonable option, provided they are appropriate candidates for endocrine therapy. (See
Patients who develop rapidly progressive metastatic disease should be treated with chemotherapy.
Premenopausal women—For premenopausal women who progress following first-line treatment, ovarian ablation should be offered, which would allow the administration of endocrine agents only approved for postmenopausal women. However, for women with disease progression on ovarian suppression agents, serum estradiol levels should be checked to ensure menopausal status was achieved. If high estradiol levels are noted despite ovarian suppression, ovarian ablation should be pursued.
Once menopause is induced (or confirmed by laboratory testing), the treatment approach for postmenopausal women is used.
For premenopausal women who refuse ovarian suppression or ablation, we recommend chemotherapy.
Postmenopausal women—There is a lack of clinical trials to address the optimal sequence of therapy in second-line and subsequent-line settings. The available options include the administration of a non-cross-resistant AI, tamoxifen, fulvestrant, fulvestrant plus palbociclib, or endocrine therapy plus the mammalian target of rapamycin (mTOR) inhibitor, everolimus, in the second-line setting.

447
Q

Patient with previous I&D for breast abscess. Biopsy shows Idiopathic Granulomatous Mastitis (non-caseating granulomas and other path findings described)
A. Abx and I&D as necessary
B. Steroids
C. Resection

A

B

448
Q

30ish year old female with BRCA-2. Mammograms shows >75% breast density. What is the best Management?
A. MRI
B. Repeat Mammogram in one year
C. Ultrasound

A

A

449
Q

Microglandular adenosis breast
A. Excise
B. Repeat imaging
C. Reassure

A

A

Risk of associated cancer

450
Q

Diffuse calcification mammogram, biopsy showing DCIS. Intraooperatively, cannot find sentinel node
A. Total mastectomy
B. Mastectomy and axillary sampling
C. Mastectomy and level I dissection
D. Mastectomy and levels I and II dissection

A

A

SLNB not successful; axillary staging then is ALND which can be done later IF path shows a cancer in specimen.

451
Q
Young female with breast mass that’s enlarged compared to 2 years ago.  Had FNA 2 years ago of same mass showing fibroadenoma
A. Excisional biopsy
B. Total mastectomy
C. Core biopsy
D. Excision with 2cm margin
A

A

452
Q

Female with prev MRM, ER+ Her negative, 2 positive lymph nodes, on tamoxifen. Now develops 1 cm mass on scar, and supraclavicular lymphadenopathy
A. Axillary dissection
B. Axillary dissection and supraclavicualr dissection
C. Radiation
D. Chemo?

A

C?

453
Q

Breast cancer 2.5cm, node negative, and is a “luminal A” cancer. Best management?
A. Oncotype Dx
B. Hormone
C. Herceptin + chemo

A

A

T1-2, ER +, Her2 -, LN – is indication for Oncotype to determine benefit of chemo

454
Q
Youngish female with LCIS on biopsy and mom who had breast cancer at 51.  Surveillance?
A. Yearly mammo 
B. Yearly MRI
C. MRI and mammo
D. MRI and mammo and U/S
A

A per NCCN

Insufficient information to declare high-risk (5yr >1.66%, lifetime >20%). No guidelines support routine use of MRI for LCIS surveillance. Gail model not applicable in LCIS so can’t use it to stratify as high-risk.

The NCCN Breast Cancer Screening and Diagnosis Clinical Practice Guidelines for women with LCIS include annual mammography and clinical breast examination (CBE) every 6 to 12months, with consideration of annual MRI.68 However, the American Cancer Society (ACS) guidelines do not support routine use of MRI in this setting, stating that there is not enough evidence to recommend for or against MRI screening in women at increased risk from LCIS.69 The ACS guidelines are based on the increased sensitivity of MRI in women at high risk because of an inherited predisposition or strong family history of breast cancer; however, the biology of the breast cancers that develop in women with LCIS differs from those that develop in women at risk from BRCA mutations, and a recent longitudinal cohort study of women in surveillance for a diagnosisof LCIS suggests that routine MRI screening is not warranted in these patients.

455
Q
What’s the benefit of neoadjuvant for breast cancer?
A. Improves survival
B. Changes need for SLNB
C. Better disease free survival
D. Decrease tumour size
A

D

456
Q
For which breast cancer is oncotype Dx most useful?
A. ER negative, Her2 +
B. ER negative, Her2 negative
C. ER +, Her2+
D. ER +, Her2-
A

D

457
Q
Breast biopsy comes back with flat epithelial atypia.  Management?
A. Observe
B. Excision biopsy
C. 	?
D. 	?
A

B

Upgrade of 8-17%.

458
Q

Breast lesion with biopsy previously shown fibroadenoma but has rapidly grown 2X in size. What to do
A. Excisional biopsy
B. Observe
C. Repeat core needle biopsy

A

A

459
Q

Old lady w/ prev MRM for T2N1 disease, on tamoxifen currently and Tx w/ chemotherapy. Found to have chest wall nodules, axilla and supraclavicular LN palpable and biopsy proven to be invasive ductal CA. What to do next
A. Chemotherapy
B. Radiation therapy
C. Axillary dissection
D. Axillary dissection w/ supraclavicular LN dissection

A

B

UTD

5-year survival rate with local and regional recurrence following MRM is 7%. If only SC nodes, 28%.

Postmastectomy locoregional recurrence:
• complete restaging
• complete resection of local chest wall recurrence when possible
• NACT considered if might downsize resection required
• RT is also given if not previously
• ALND for axillary recurrence
• SC recurrence: systemic therapy followed by local therapy (surgery or RT may be used alone depending on prior nodal irradiation. )

460
Q
Breast mass with core biopsy shows PASH.  What to do
A. Observation
B. WLE
C. Excisional biopsy
D. Mastectomy
A

Prev years said C

If certain of dx/no suspicious features, can observe

461
Q
Lady w/ breast cancer  to bone.  Asymptomatic. Tumor biopsy show 1-5% hormonal receptor +ve.  What is the best Tx?
A. Tamoxifen + bisphosphonate
B. Tamoxifen only
C. Radiation 
D. Chemotherapy
A

A

> 1% of tumour cells expressing ER/PR is considered positive, and response to endocrine therapy can be expected. However, historically a cutoff of 10% was used; in patients with 1-9% who don’t tolerate endocrine therapy, it is reasonable to discontinue.

462
Q
Lumpectomy and SLNB for IDC, can’t locate SLN in axilla but radioactive readings in IM node
A. Lumpectomy and ALND
B. Lumpectomy and IM node resection
C. Lumepctomy  only
D. Lumpectomy + ALND + IM node resection
A

A

Still need to stage the axilla. Possible that tumour replacement is reason can’t localize SLN.

Internal mammary nodes—The surgical management of the internal mammary (IM) nodes remains controversial. There is no consensus on the need for IM nodal dissection in women with detection of an IM SLN. The IM nodes are not routinely dissected in patients undergoing breast-conserving therapy or mastectomy with axillary lymph node dissection (ALND). Thus, in the absence of definitive data, dissection of the IM nodes with sentinel lymph node biopsy (SLNB) should be considered investigational.

463
Q
Lady with longstanding history of DMI.  Thickening in breast, biopsy shows keloid lesions ductal fibrosis and lymphocytes, What to do
A. Excisional biopsy
B. Observe
C. Repeat imaging
D. Repeat core biopsy
A

B

Diabetic mastopathy—Diabetic mastopathy, also known as lymphocytic mastitis or lymphocytic mastopathy, is seen occasionally in premenopausal women who have longstanding type 1 diabetes mellitus. The typical presentation is a suspicious breast mass with a dense mammographic pattern. Core biopsy is recommended for diagnostic confirmation. Pathology shows dense keloid-like fibrosis and periductal, lobular, or perivascular lymphocytic infiltration [72-74]. The pathogenesis is unknown, but it may represent an autoimmune reaction as the histologic features are similar to those seen in other autoimmune diseases [75]. Once the diagnosis is established, excision is not necessary and there is no increased risk of subsequent breast cancer.

464
Q
Older lady, prev MRM w/ T2N0 disease ER +ve. Recurrent breast cancer to chest wall, 8cm, invading pec major but not ribs, also positive infraclavicular nodes x2 and lesion is ER +ve. What to do?
A. Radiation
B. Chemotherapy
C. Tamoxifen
D. Desect nodules and LN dissection
A

C?

465
Q

Female with locally advanced breast cancer (matted nodes). ER-, PR-, Her+, what to do
A. Chemo + surgery + hercpetin for 1 yr
B. Chemo + herceptin then surgery, then herceptin for 1 yr
C. Surgery + chemo
D. Chemo + herceptin + surgery

A

B

Her2 + has high likelihood of PCR, NACT indicated; Herceptin given for 1 year.

466
Q

Patient with moderate grade diffuse DCIS for mastectomy and SLN. What’s best for long term cosmesis?
A. Skin spare mastectomy immediate recon w/ prothesis + SLNB
B. Skin spare mastectomy immediate recon w/ flap + SLNB
C. Total mastectomy + SLNB

A

B

At one year following surgery, women with pedicled TRAM flaps, free TRAM flaps, and expander/implants had similar levels of general satisfaction [56], but autologous tissue reconstructions (TRAM procedures) were associated with greater aesthetic satisfaction than expander/implant techniques. At year two, these procedural differences had diminished, but patients continued to be more aesthetically satisfied with autologous tissue compared with expander/implant reconstructions [106].

467
Q
69 women, 2 mm DCIS, low grade, with 12mm margins, LCIS at the margin. Best management
A. Observe
B. Radiation
C. Tamoxifen 
D. Radiation and tamoxifen
A

C

Toronto Manual
There is some evidence, however, that radiotherapy may be safely omitted in some cases of DCIS:
1. Tumour less than 2 cm
2. Margins greater than 10 mm
3. Low or intermediate grade DCIS (grade 1 or 2)19

With wide margins (>10mm), there is little benefit to RT. Tamoxifen effectively reduces risk for both DCIS and LCIS in both breasts. Perfect patient to treat without RT: wide margin, small foci, low grade, LCIS requiring chemoprevention.

468
Q

30 something female, T1 breast cancer with 1 mm sentinel node positive. ER negative. (Can’t remember if they said lumpectomy vs mastectomy) Best management
A. Axillary dissection for staging
B. Axillary dissection for local control
C. Chemo

A

C

No ALND for micromets; would get chemo based on young age and ER negative.

469
Q

Women over 50 y, mammographic density, stereotactic core showed ductal hyperplasia no atypia. Mgmt.
A. Mammogram 6 months
B. Excisional biopsy

A

A

UDH

Usual ductal hyperplasia—Ductal hyperplasia without atypia is a pathologic diagnosis, usually found as an incidental finding on biopsy of mammographic abnormalities or breast masses, characterized by an increased number of cells within the ductal space. Although the cells vary in size and shape, they retain the cytological features of benign cells [5,6]. No additional treatment is needed for ductal hyperplasia. The risk of subsequent breast cancer in women with usual ductal hyperplasia is small and chemoprevention is not indicated.

470
Q

Male breast mass, 1.8 cm, nipple retraction. Bx ductal Ca with dermal involvement. Clinically node negative. Mgmt
A. Neoadjuvant chemo
B. Mastectomy + SLNB
C. Lumpectomy

A

B

Doesn’t meet criteria for locally advanced. AJCC specifies dermal invasion alone is not T4, must have ulceration.

471
Q

Paget’s, no evidence of disease elsewhere in breasts or nodes. Mgmt.
A. Central lumpectomy sparing NAC
B. Central lumpectomy including NAC
C. Simple mastectomy + SLNB

A

C

B is – reasonable but also needs RT to be complete tx (2B evidence, is last of 3 options in NCCN)

472
Q

Inflammatory breast cancer. Post neoadjuvant. Some response in axilla, minimal response in breast. 12 cm indurated, firm mass. Mgmt
A. Radiation
B. Radiation + second line chemo
C. MRM

A

B

Anthracycline-based chemo; if inadequate reponse, taxane-based chemo

473
Q

Previous breast cancer treated with MRM (T2N1), chemo, on tamoxifen. No mention of previous radiation. Recurrence at scar, in axilla, and supraclav. Mgmt
A. Radiation
B. Axillary dissection
C. Chemo
D. Excision scar, axilla, supraclavicular node

A

Prev years picked D–we don’t excise supraclav LNs.

A if possible then consider B

474
Q

Phyllodes, exised, < 1cm margins. Mgmt
A. Re-excision to 1 cm margins
B. Radiation
C. MRM

A

A

475
Q

Pleomorphic calcs, bx sclerosing adenosis and LCIS. Mgmt
A. Wire localized biopsy
B. Repeat mammo 6 months

A

A

476
Q

Woman with single bloody discharge. Mammo normal. No palpable mass. Next step.
A. Galactogram
B. Duct excision

A

B

Prev years picked A but we don’t do this test

477
Q
Lumpectomy, 4 cm DCIS, high grade, multifocal, multiple margins positive. Mgmt
A. Re-excision
B. Radiation
C. Simple mastectomy with SLNB
D. Tamoxifen
A

C

478
Q
Breast cancer ER/PR +ve, Her2 –ve. Histologic type:
A. Luminal A
B. Basal
C. Luminal B
D. Herceptin rich
A

A

Luminal A 
-Er/Pr +, Her2-
Luminal B 
-Triple pos
Basal
-Triple neg
Her 2 enriched
-Er/Pr -, Her2+
479
Q
54 yo female, BIRADS 3, management
A. Short interval follow up mammo
B. Core bx
C. Mammo in a year
D. U/S
A

A

480
Q

Young girl, asymmetric breasts, no mass, 1 breast is double in sized compared to the other
A. Virginal hypertrophy
B. Giant fibroadenoma
C. Phyllodes

A

A

481
Q
History of breast cancer, left leg weakness, no sensory loss
A. CT head
B. Myelogram
C. Bone scan
D. Spine radiography
A

C vs A?

Neurological symptoms and signs, which suggest the possibility of spinal cord compression, must be investigated as a matter of urgency. This requires a full radiological assessment of potentially affected area as well as adjacent areas of the spine. MRI is the method of choice. An emergency surgical opinion (neurosurgical or orthopaedic) may be required for surgical decompression. If no decompression/stabilization is feasible, emergency radiotherapy is the treatment of choice and vertebroplasty is also an option.

MRI best method to differentiate mets vs fragility fracture.

482
Q
Breast simple cyst in 40 yrs female. Normal mammogram
A. Observe
B. Duct excision
C. Aspirate
D. Aspirate and send fluid for analysis
A

A

483
Q

Lady with unilateral spontaneos green breast discharge , normal Mamogram
A. Duct excision
B. Ductography
C. Observe

A

A

Prev years picked B but we don’t do this

484
Q
Diffuse microcalcifications of breast
A. Mastectomy and SLN
B. Mastectomy
C. Lumpectomy and SLN
D. Radiation alone
A

A

485
Q
79.	Woman in her 50s with green nipple discharge that she noticed the last few months when doing breast self-exam. Mammogram and physical exam normal. Management?
A. Galactogram
B. Antibiotics
C. Subareolar duct excision
D. Reassurance
A

D

This is non pathological

INITIAL APPROACH TO PATHOLOGIC NIPPLE DISCHARGE—Women presenting with pathologic nipple discharge should undergo a focused ultrasound. In addition, a mammogram should be performed for women ≥30 years of age [3-5]. Any concerning imaging abnormality, including an intraductal mass on ultrasound (US) or mammogram, warrants surgical evaluation. In patients with spontaneous nipple discharge that is unilateral and/or bloody, surgical evaluation is required even if initial imaging results are negative

Normal ultrasound and mammogram—In patients with spontaneous nipple discharge that is unilateral and/or bloody, further evaluation is required even if initial imaging results are negative.

Unilateral discharge (bloody or nonbloody)—Unilateral, particularly uniductal, nipple discharge requires additional work-up to rule out malignancy, regardless of whether the discharge is bloody or not. If the patient has undergone ultrasound and mammogram, and both are negative, s/he should be referred for either galactography or ductoscopy evaluation, depending upon local expertise. Patients who are found to have intraductal masses by either galactography or ductoscopy are candidates for surgical excision of the involved duct to rule out malignancy.

486
Q
Patient with history of breast cancer, appropriately treated. Imaging shows 3.2 cm liver lesion that has been stable for 1 year. Best management?
A. Resect
B. Continue chemotherapy
C. ?
D. ?
A

A

487
Q

Woman with mammographic abnormality has excision that comes back multifocal DCIS with multiple positive margins. Management?
A. Reexcise
B. Mastectomy with SLNB
C. Mastectomy with radiation

A

B

Prev years picked A. If only one margin A is reasonable but stem says multiple

488
Q
Woman has screening mammogram and ultrasound suggestive of ductal papilloma. Best management?
A. Reassure
B. Galactogram
C. Needle localization excision
D. Core bx
A

D

Never take to OR without a biopsy.
Intraductal papillomas—Intraductal papillomas consist of a monotonous array of papillary cells that grow from the wall of a cyst into its lumen. Although they are not concerning in and of themselves, they can harbor areas of atypia or ductal carcinoma in situ (DCIS). Papillomas can occur as solitary or multiple lesions.
The standard approach to a papilloma diagnosed by core needle biopsy (CNB) is to perform a surgical excision, particularly if atypical cells are identified [14,16-21]. In a meta-analysis of 34 studies that included 2236 non-malignant breast papillary lesions, 346 (15.7 percent) were upgraded to malignancy following a surgical excision [21]. Because of a risk of malignancy, these require surgical excision.
Once the diagnosis of solitary papilloma is confirmed by excisional biopsy, no additional treatment is needed in most patients, unless there is evidence of associated atypia, in whom discussion about endocrine therapy for breast cancer prevention is indicated
Solitary intraductal papillomas may be identified as a mass on a mammogram, ultrasound, magnetic resonance imaging (MRI), ductogram, or they can be found incidentally [23]. Nipple discharge, particularly bloody nipple discharge, is a frequent clinical presentation.
For a solitary papilloma without biopsy evidence of atypia, the need for surgical excision is not clear. Examples in the literature include the following:

489
Q
Perimenopausal women with a 4-month history of breast pain.  Previously healthy.  On examination, A FIRM AND TENDER 2CM LESION IS IDENTIFIED IN THE UPPER OUTER QUADRANT.  What is the BEST initial management?
A. Mammogram
B. Ultrasound
C. FNA
D. Observation
A

Prev year said B because – probably a cyst which may improve/resolve with aspiration, followed by mammo

A seems also very reasonable

490
Q
79.	48 yo female with nonpalpable simple breast cyst on imaging.  What is the BEST management?
A. FNA
B. Serial imaging with ultrasound
C. Reassurance
D. Excision
A

C

491
Q

Old lady with osteoporosis. Previous breast cancer, T2N0, SLN 0/2 negative, treated with total mastectomy and tamoxifen. Recurrence at scar with 5 cm lesion, biopsy ER/PR positive. Positive supraclavicular and ?internal mammary nodes
A. Radiation
B. ALND, excision of mass and radiation for IM node
C. Tamoxifen
D. Aromatase inhibitors

A

A first and then C? depends if recurred while on tamoxifen

492
Q
Microcalcification excised - dcis margin 2 mm, lcis on margin
A. Radiation
B. Observe
C. Aromatase inhibitor
D. Tamoxifen
A

A

493
Q
ALND, nerve injured giving winged scapula
A. Long thoracic
B. Thoracodorsal
C. Intercostalbrachial nerve
D. Axillary nerve
A

A

494
Q

Phyllodes with 0.5 cm margin
A. Reresect to 1 cm margin
B. Mastectomy
C. Radiation

A

A

495
Q

Woman with small invasive cancer, in the OR you’re doing a SLNB and can’t find a node in the axilla but there’s uptake in the ipsilateral 2nd intercostal space.
A. ALND
B. ALND and internal mammary sentinel node resection
C. ALND and internal mammary sentinel node biopsy
D. Internal mammary sentinel node dissection

A

A

496
Q
Mastectomy and SLNB for DCIS, margins show DCIS 2mm from posterior margin, SLNB negative. 
A. Observe
B. Radiation
C. Mammogram at 6 months
D. Re-resect
A

A

497
Q
Biopsy of calcifications, which one can you observe?
A. Intraductal papilloma
B. Radial scar
C. Sclerosing adenosis
D. ADH
A

C

498
Q
Premenopausal woman with breast redness and induration and matted axillary nodes. Breast mammo and MRI show nothing except thickened skin. Biopsy of the skin shows just inflammatory things (?lymphocytic inflitrate but no blocked ducts or anything like that). Biopsy of the axillary nodes shows invasive ductal cancer. What is the best initial treatment?
A. CTX
B. Radiation
C. Mastectomy
D. ALND
A

A

499
Q

86F osteoporotic lady 6-10 years post MRM for T2N0 invasive ca ER+ comes back the 8cm lesion in chest invading major and minor pec but not invading the ribs, axillary nodes and supraclavicular nodes.
A. Tamoxifen
B. AI
C. Resection of chest wall lesion and ALND
D. Radiate the axilla

A

A

Either RT or antiestrogen therapy, but tamoxifen probably lowest risk. This is unresectable disease. AI more effective, but contraindicated given osteoporosis.

500
Q

36 year old woman 12 weeks pregnant with her first pregnancy with 1.5cm breast mass biopsy shows invasive ductal ca with palpable mobile axillary nodes. What should you do (FNA nodes was not an option)
A. Lumpectomy, ALND, CTX and radiation
B. Lumpectomy, SLNB, radiation
C. Lumpectomy, SLNB, CTX, radiation
D. Abortion, neoadjuvant CTX, lumpectomy, CTX and radiation

A

A

Clinically positive nodes, so no SLNB; defer chemo til 2nd TM, RT until after delivery.

501
Q

Woman with biopsy proven Paget disease and no associated mass clinically, on mammo or MRI. Best surgical option?
A. Central lumpectomy with NAC
B. Central lumpectomy with NAC and SLNB
C. Total mastectomy and immediate reconstruction
D. Central lumpectomy sparing the NAC

A

A

UTD

Taken together, these data suggest that central lumpectomy or complete resection of the nipple-areola complex followed by whole breast RT is a reasonable alternative to mastectomy for women with PDB and no palpable mass or mammographic abnormality as long as a good cosmetic outcome and negative margins can be achieved.
Patients with in situ disease do not require axillary investigation unless the disease is extensive enough to merit mastectomy. If mastectomy is planned, SLN biopsy is often (and appropriately) done preemptively in order to avoid complete axillary lymph node dissection in case an invasive component is identified at final pathology. =
●If invasive disease has been identified, patients with clinically node-negative disease should undergo SLN biopsy at the time of wide excision. An SLN can be successfully identified in 97 percent of patients with PDB [56]. Management of a positive SLN is discussed in detail elsewhere.
●For patients with pure PDB, no underlying palpable mass and clinically negative axillary nodes, there is some controversy as to the need for axillary evaluation. Given the high incidence of DCIS alone in patients with PDB and no evidence of an underlying breast mass, SLN biopsy is recommended only if invasive disease is identified or mastectomy is planned

502
Q
Woman morning after simple mastectomy with SLNB, significant ecchymosis of chest wall, 25cm non tense collection, JP not draining, what should you do?
A. Flush the JP
B. Observe
C. Open 2 cm at bedside and drain
D. Go to OR
A

D

503
Q
79.	Woman with CREST and 2cm invasive cancer. What should you do?
A. Lumpectomy
B. Lumpectomy with radiation
C. Mastectomy with SLNB
D. Mastectomy with ALND
A

C

504
Q

Patient with palpable PASH on breast specimen Mgmt?
A. Excise to clear margins
B. Excise with 1 cm margin

A

A

505
Q
Pregnant patient early 2nd trimester.  Early stage invasive ductal Ca, palpable nodes
A. Terminate pregnancy
B. Lumpectomy and radiation
C. Lumpectomy, ALND, chemo, RT
D. Lumpectomy and SLNB
A

C

506
Q

Female with prev MRM, ER+ Her negative, 2 positive lymph nodes, on tamoxifen. Now develops 1cm mass on scar, and supraclavicular lymphadenopathy
A. Axillary dissection
B. Axillary dissection and supraclavicualr dissection
C. Radiation
D. Chemo

A

C?

507
Q

Breast implant w/ MRI showing 1.5 cm invasive ductal CA, axilla clinically negative. Best management?
A. Lumpectomy, SLNBx
B. Lumpectomy axillary node dissection
C. Lumpectomy, SLNBx take out implant
D. Lumpectomy, axillary dissection take out implant

A

A

May have capsular fibrosis with radiation with an implant but this can be managed later by plastics

508
Q

56F referred bc of eczematous rash on right nipple- failed to resolve after 4 wk of topical steroid. o/e breasts and axillae normal. Mammo poor quality but no definite evidence of cancer. Best mgt?
A. Repeat mammo in 2 mo
B. Arrange for excision of major duct system
C. Bx skin of areola
D. Order US of both breast
E. Arrange core Bx of right Breast

A

C

Paget’s disease

509
Q

An 8 year old girl is referred because of a mass in the right breast. The mother says that this has been present and gradually increasing in size for 2 months. On examination there is a centrally placed, non-tender, flat, 2 cm mobile mass beneath the right nipple. The mass is slightly firm but not hard. There is no evidence of inflammation and no dimpling of the nipple. The opposite breast is normal for a prepubertal child. Both axilla and supraclavicular fossa are clear.
The diagnosis of the cause of the breast mass is best ascertained by:

A. Excisional Biopsy
B. Incisional Biospy
C. Mammogram, CXR, and lung tomogram
D. Observation

A

D

510
Q

Which of the following statements concerning ductal carcinoma in situ of the breast is NOT correct?
A. It is rarely multicentric
B. When invasive cancer occurs, it is usually of duct origin
C. It rarely leads to invasive breast cancer
D. When invasive cancer occurs, it has a worse prognosis than other invasive cancers

A

D

511
Q

A 40 y.o woman comes to your office with a smooth, mobile, non-tender mass in her left breast. She is otherwise healthy and the rest of your exam reveals no abnormality. You decide to aspirate the mass in your office. Which of the following considerations is MOST important?
A. The mass should disappear completely on aspiration.
B. If, on further inquiry you find her aunt underwent a mastectomy and her grandmother died of breast cancer, aspiration is contra-indicated.
C. The mass disappears, therefore no further follow-up is indicated.
D. If the mass proves to be cystic, the fluid must be sent for cytology.

A

A

512
Q
During complete axillary lymph node dissection (level I and II), which of the following structures are not routinely identified?
A. Thoracodorsal nerve
B. Medial pectoral neurovascular bundle 
C. Intercostaobrachial nerve
D. Lateral cord of the brachial plexus
E. Long thoracic nerve
A

D

513
Q

45 y.o. F presents with a suspicious lesion containing microcalcifications on screening mammography. Breast exam is normal. Core needle biopsy is performed and shows ADH without evidence of microcalcifications. Which of the following is the most appropriate management?
A. Segmental mastectomy
B. Repeat core biopsy
C. Excisional biopsy following stereotactic wire localization
D. Repeat mammo in 6 months

A

C

514
Q
32 wk pregnant w/ breast cancer. Concerned about mets to the fetus. Which of the following cancers is most likely to metastasize to fetus:
A. Follicular TC
B. Melanoma
C. Breast
D. RCC
A

B

515
Q
Breast mets to liver after a while. 3.5 cm stable x 1 year no other evidence of disease. Best management:
A. Chemo
B. RFA
C. Resect
D. Palliative supportive
A

C

516
Q
Best prognosis for breast cancer?
A. Medullary
B. Mucinous
C. Lobular
D. Metaplastic
A

C

Tubular>mucinous>medullary

517
Q
Which one of the below invasive breast CA can mimic a fibroadenoma on ultrasound?
A. Mucinous
B. Medullary
C. Tubular
D. Lobular
A

B

518
Q
Intracystic papillary cancer margin <1cm. (note: may be an old question – before larger margin was considered –ve).
A. Lumpectomy, SNBx, RT
B. Lumpectomy, RT
C. Re-excise
D. Mastectomy
A

B

Treat like DCIS

519
Q
Invasive ductal 1cm no nodes. Older lady. No other evidence of mets. Staging workup:
A. Nothing
B.  CXR, abdo U/S, bone scan
C. CXR, abdo U/S
D. Bone scan
A

A

For all: H&P, LFT’s, CBC, mammo, u/s
Bone Scan, CT CAP if stage 3, symptoms ?nodes.

520
Q
Older lady on aromatase inhibitor. In addition to annual mammography, should also be followed with:
A. CEA, Ca 15-3
B. Bone density
C. MRI
D. CXR, bone scan, abdo U/S
A

B

521
Q

Cancer removed LCIS at margin. Which increases the risk of LR?
A. ADH
B. LCIS
C. Margin <5mm

A

A

522
Q
What is the most common cause of spontaneous bloody d/c:
A. Intraductal papilloma
B. Paget’s
C. Duct ectasia
D. Invasive ductal
A

A

523
Q
Who gets AI:
A. Premeno ER+
B. Premeno ER-
C. Postmeno ER+
D.  Postmeno ER-
A

C

524
Q
BRCA 2 risk?
A. Breast 80% ovarian 20%
B. Breast 80% ovarian 50%
C. Breast 50% ovarian 20%
D. Breast 50% ovarian 20%
A

A is closest
Breast 70%, Ovarian 20% in BRCA 2
Breast 70%, Ovairan 40% in BRCA !

525
Q

With regards to BRCA

A. > 90% of breast ca in women under 30 yo related to BRCA
B. BRCA is a tumour suppressor gene
C. BRCA1 only causes breast ca
D. BRCA2 only affects women

A

B

526
Q

With regard to BRCA
A. BRCA 1,2 both autosomal recessive
b. Risk of developing breast CA is 80%
c. Bilateral oopherectomy doesn’t decrease incidence of breast CA
d. In post-menopausal women bilateral oopherecomies should be recommended

A

C?

A is wrong. Autosomal dominant
B it’s ~ 70%
D Bilat oophorectomies recommended in patient whol have completed child bearing

527
Q
23.	Women with previous history of breast cancer, presents vague abdominal pain.  CT shows 3cm homogenous adrenal mass.  All tests show that it is not a functional tumor.  What is the next BEST management?
A. MRI
B. Repeat CT scan in 6 months
C. Image guided biopsy
D. Lap left adrenal
A

C

528
Q
Do lumpectomy for core showing invasive ductal. On path specimen find unexpected multifocal invasive ductal w/ DCIS at the margin.
A.  Mastectomy
B. Re-excise
C. RT
D. MRM
A

B

529
Q
Which breast cancer has a tendency for visceral spread?
A. Medullary.
B. Tubular.
C. Lobular.
D. Mucinous.
A

C

530
Q
Lady w/ breast CA triple -ve. Tx?
A. Chemo
B. Chemo + herceptin
C. Chemo + AI
D. Chemo + tamoxifen
A

A

531
Q
A woman presents with a scaly lesion on the nipple areola complex.  Biopsy confirmed Pagets.  There are no abnormalities on mammography, and the lymph nodes are clinically negative. There is no mass.  What is the BEST management?
A. Central mastectomy
B. Simple mastectomy with sentinel node
C. Modified radical mastectomy
D. Radiation
A

B because option A doesn’t include radiation

Tx options

  • Central lumpectomy with radiation
  • Simplet mastectomy with SLNB
532
Q

A 75-year old male on HCTZ and ramipril. Presents with a well defined tender, firm left subareolar disk. What is the most likely cause:
A. Increase prolactin.
B. Decrease testicular hormones with age.
C. Related to medication.
D. Breast cancer.

A

B

Neither HCTZ or ramipril cause gynecomastia. Captopril and enalopril can. This is classic senescent gynecomastia.

533
Q

Lady 22 wks pregnant w/ 3.5cm L UOQ, Bx = infiltrating ductal breast CA.
A. Abort the pregnancy and treat the breast cancer
B. Wait till she delivers then treat the breast cancer
C. Partial mastectomy, AND
D. Partial mastectomy, SLN
E. MRM

A

E

2nd trimester requires too long a delay before RT can be given. Breast RT should be given within 12 weeks in order to be effective.

UTD opinion is that SLNB has not yet been proven safe in pregnancy, so ALND is standard of care, however most institutions will utilize SLNB without blue dye.

SCNA, Schwarz concur: MRM in 1st & 2nd trimesters

534
Q
Several clinical trials have shown survival benefit with adjuvant chemotherapy in node positive breast cancer.  The best description of these trials:
A. Double-blind, randomized, multicentre
B. Double-blind, randomized, prospective
C. Randomized, prospcetive, multicentre
D. Meta-analysis
A

D

535
Q
Woman with no family history has BIRADS 3 lesion – recommendation?
A. Observe
B. Close follow up
C. Image guided biopsy
D. Excisional biopsy
A

B

536
Q
Woman/Male with tender erythematous cord lateral chest wall.  Management?
A. NSAIDS
B. MRI
C. Biopsy
D. Incision and drainage
E.  mammo
F. US
G. ABX
A

A

537
Q

17F with imaging consistent with fibroadenoma. Management?
A. Excisional bx
B. Observe
C. Mammo

A

B

538
Q
50F with 2 cm mass, imaging consistent with fiboradenoma.  Management?
A. Core biopsy
B. FNA
C. MRI
D. Reassure
A

A – because of age. Most are <35 yrs.

539
Q
Benefit of raloxifene vs. tamoxifen in prevention of high risk breast ca.
A. Decrease incidence cataracts
B. Decrease risk MI
C. Decrease breast ca
D. Decreased uterine ca
A

A and D?

540
Q
30yo F has 3 cm lump at the 9 o'clock position of her left breast. It is completely excised with clear margins and SLN biopsy is negative. Pathology reveals invasive ductal carcinoma with ER+, PR+, Her/2Neu +. What is the best systemic therapy you would recommend for this woman?
A.  Hormone therapy alone
B. Hormone therapy and Herceptin
C.  Chemotherapy and herceptin
D. Herceptin alone
A

C

Herceptin based chemo for all HER + tumors node + or >1cm. For tumors <1cm no trials, UTD suggests Herceptin based regimen.

541
Q
45 yo Lady with repeated breast infections and area of patchy infection at the edge of the areola.  What is the diagnosis?
A. Hidradenitis suppurativa 
B. Mammary fistula
C. Folliculitis
D. Paget’s disease
A

B

542
Q
45yo woman with 2cm upper outer quadrant firm breast mass. Mammo and U/S show multifocal lesions. Most appropriate next step:
A.  MRI
B. Core biopsy
C. Excisional biopsy
D. Mastectomy
E. MRI after Bx
A

B

543
Q

40 yo F underwent needle loc nodule excision for an atypical cyst, in specimen of 15mm. Pathology reveals intracystic papillary neoplasia with close margins (<1mm). What is the best management:
A. Re-excision w neg margins
B. Re-excision + radiation
C.Re-excision + SLNBx + radiation
D. Re-excision + axillary node dissection + radiation

A

B

Treat like DCIS

Noninvasive, but based on young age (<50) RT is probably indicated. Normally adjuvant treatment is based on associated pathology, not IPC (eg. DCIS, LCIS, etc). RT is controversial but may improve survival.

544
Q
Old lady with known Stage 4 breast CA with bone mets. Develops tachypnea, has wide mediastinum and bilateral pleural effusion on CXR, visible external jugular veins when sitting. BP: 90/60, HR: 100/min. Radial pulse disappear with inspiration .What is the most useful test?
A.  Vent/perfusion nuclear scan
B. Coronary angiography
C. 2D Echo
D.  Stress test
E. CT scan
A

C

545
Q

Core bx positive for LCIS. What is the best reason to do an excisional Bx?
A. Provide adequate margins
B. Better sampling and prevent misdiagnosis
C. Better survival
D. Better local control

A

B

546
Q
A 60 y.o female comes in with an indeterminate mass on mammo. You get a core and it shows lobular hyperplasia. Management?
A. Mastectomy
B. Mammogram in 1 year
C. Excision
D. Tamoxifen
A

C

Discordant. LH should not be mass-forming.

547
Q

Stage I breast cancer, ER/PR +. Best indication for adjuvant tamoxifen (there really wasn’t any more detail than this):
A. Pre-menopausal woman with a history of depression
B. Pre-menopausal woman with a history of PE
C. Post-menopausal woman with a history of chronic venous insufficiency
D. Post-menopausal woman with bone density <2.5 SD below

A

D

548
Q

Best indication for mastectomy as initial treatment:
A. Inflammatory breast cancer
B. Invasive breast cancer with extensive intraductal component
C. Two separate invasive tumours in 2 quadrants
D. Paget’s disease of the nipple

A

C

549
Q
Female with calcifications, had core biopsy, came back with duct ectasia with squamous metaplasia.  What is the BEST management?
A. Observe
B. Follow-up mammogram
C. Lumpectomy
D. Lumpectomy with SLN
A

B

550
Q
70y M with 2 cm mass behind the R areola.  No other Hx or findings in breast or axilla.  Best management?
A. Mammogram
B. Excisional biopsy
C. Tamoxifen
D. US
E. reassure
A

A in prev years. I would probably pick this as iniitial mgmt.

Question asks best mgmt which would probably be B

551
Q

49 y F underwent breast biopsy showing epithelial hyperplasia. Best management?
A. Excisional Biopsy
B. Tamoxifen
C. F/U only

A

C

No atypia so f/u adequate.

552
Q
63M presented with 1.8 cm mass in the L retroareolar region which is confirmed on core biopsy to be invasive ductal carcinoma. The lesionis solid to palpation and associated with ulceration and erythema of the overlying skin. What is the T stage of this cancer?
A. T1c	
B. T2
C. T3
D.  T4
A

D

553
Q
15 year-old girl with 1.5 cm lesion at 9 o'clock position in left breast.  Has been there for 2 months.  U/S consistent with fibroadenoma.  Management?
A. Low dose OCP
B. Repeat U/S in 6 months 
C. Excise
D. Mastectomy
A

B

554
Q
Patient underwent needle guided excisional biopsy. Pathology shows 6mm LCIS with positive margins. What to do?
A. Re-excision
B. Mastectomy
C. Mammogram in 6m
D. Radiotherapy
A

C

555
Q

Female with locally advanced breast cancer and palpable axillary LN. Underwent chemo with slight response in the axilla and no change in primary. Best management?
A. Radiation with 2nd line chemo
B. MRM
C. Herceptin

A

B

Depends on operability; if operable then yes.
If inflammatory, 2nd line + RT, this doesn’t sound like inflammatory.

556
Q

40 y F with breast mass upper outer quadrant, core biopsy ADH, Best management?
A. Excision
B. Tamoxifen
C. F/U only

A

A

557
Q
14 yo male in a growth spurt is distressed by bilateral subareolar swelling.  What will you do?
A. Reassure
B. Mammo
C. US
D. Bilateral subcutaneous mastectomy
A

A

558
Q
66.	A woman presented with suspicious calcifications on mammography.  Needle-localized excisional biopsy shows LCIS on final pathology.  The calcifications were within the specimen, but the margins are 5 mm.  What is the BEST management for this patient?
A. Tamoxifen
B. Radiation
C. Chemotherapy
D. Re-excision
A

A

559
Q
66.	A patient undergoes needle-localization for a mammographic abnormality, of which pathology shows a radial scar.  What is the BEST management?
A. Mammogram and physical exam yearly
B. Mastectomy
C. Tamoxifen
D. Radiation
A

A

560
Q
Which of these breast lesions has the greatest risk of subsequent malignancy?
A. ADH
B. Adenosing sclerosis
C. Radial scar
D. Duct ectasia
A

A

561
Q
66.	A middle-aged woman is diagnosed with a 2.5 cm [4cm] invasive ductal breast carcinoma with a clinically palpable 2.5 cm axillary lymph node and a biopsy proven involved supraclavicular lymph node. What is the best management?
A. Palliative chemotherapy
B. Neoadjuvant chemotherapy
C. Radiation therapy and chemotherapy
D. Mastectomy
E. Hormonal
A

B

Supraclav nodes = N3c
Locally advanced

562
Q
An elderly women with terminal breast cancer and severe dyspnea.  She is already on maximum oxygen, and maximum ventolin treatments. What is the BEST medication to give?
A. Opioids
B. Lasix
C. Anticholinergic
D. Steroids
A

A

563
Q
66.	A 50 yo woman with unknown primary, presents with an enlarged axillary node.  On biopsy, it shows that it is an adenocarcinoma, ER/PR positive.  What is the BEST management for this patient?
A. Axillary dissection
B. Radiation alone
C. Axillary dissection with radiation
D. Partial mastectomy
A

C

564
Q
66.	A woman presented with suspicious calcifications of the breast on mammography.  She underwent a needle-localized excision biopsy, of which showed LCIS on final biopsy.  The calcifications were within the specimen, but the margins are positive for LCIS.  What is the BEST management for this patient?
A. Re-excise to clear margins
B. Repeat mammogram in a year
C. Completion mastectomy
D. Radiation
A

B

565
Q
Which of these breast cancers has a pleomorphic variety?
A. Lobular
B. Metaplastic
C. Tubular
D. Medullary
A

A

566
Q
A women presents with inflammatory breast cancer, and is treated with neoadjuvant chemotherapy. She has a partial response of the axilla, but no response of the primary.  The mass in her breast still measures 12cm, with diffuse skin involvement.  What is the BEST management?
A.MRM [+/- graft]
B. MRM with skin graft closure
C. Radiation
D. 2nd line chemotherapy [+/- RTx]
A

D

Anthracyclines then taxane as 2nd line.

567
Q
64 year old female has needle local excision biopsy which showed invasive lobular 8mm, ER +, PR+, negative LN. All margin clear, the closest 5mm. What is the BEST management:
A. Tamoxifen
B. Chemo
C. Breast radiation
D. Re-excision
E. MRM
A

C

Lumpectomy + radiation = package deal

Also tamoxifen

568
Q
27 year old woman with malignant phyllodes tumor: breast mass has doubled within 6 months. It measures 3 cms.  After excision, pathology shows it is 3.2cm, and margins are positive.  What is the MOST appropriate management?
A. Re-excise to clear margins 
B. Simple Mastectomy
C. MRM
D. Radiation
E. Close FU
A

A

569
Q
60 year old F with bilateral spontaneous milky discharge. What is your investigation: (As above except should have mammo based on age for screening but NOT necessary for galactorrhea)
A. Bilateral mammo
B. Ductogram
C. Prolactin level and TRH level
D. Send fluid for cytology
E. Duct excision
A

C

570
Q
70 y/o male with a tender breast, with a firm disc behind the nipple areola complex. He is taking ranitidine for GERD. What is the MOST likely cause?
A. Elevated prolactin
B. Medication related
C. Breast CA
D. Low testosterone
A

B

Ranitidine, cimetidine, omeprazole, captopril. Not related to ramipril.

571
Q
Male breast cancer 1.8cm, nipple retraction, involvement to the skin and nipple. Final punch biopsy shows invasive ductal carcinoma with invasion into the skin What is the T stage?
A. T1c
B. T2
C. T3
D. T4
A

D

T4 – Tumor of any size with direct extension to the chest wall and/or the skin (ulceration or skin nodules)

572
Q

60 y/o female with breast cancer (0.8cm) which is well differentiated, ER+ve, PR+ve, HER2Neu +ve, and node negative. What is BEST adjuvant management?
A. Tamoxifen therapy alone
B. Chemotherapy & herceptin
C. Tamoxifen with Herceptin

A

Last year said A.

This is a small cancer but it is Her2 positive. Would at least have a discussion re: chemo and herceptin

573
Q

Which of the following is the BEST indication for a total mastectomy?
A. Paget’s
B. Inflammatory
C. 2 cancers in 2 separate quadrants
D. Invasive ductal with extensive intraductal component

A

B vs C

574
Q
Patient has pleomorphic calcifications over 1 cm, and core biopsy shows sclerosing adenosis, and 2 foci of LCIS. What is the BEST management?
A. Radiate
B. Tamoxifen
C. Needle localization excision
D. Repeat mammogram in 6 months
A

C

575
Q
68 year old lady with palpable upper outer quadrant mass, U/S guided core biopsy shows duct ectasia with squamous metaplasia. What is the BEST management?
A. Observe
B. Lumpectomy 
C. Tamoxifen
D. Lumpectomy with sentinel node biopsy
A

B

Discordant path

576
Q
40F with duct ectasia and squamous metaplasia. Management?
A. Excisional biopsy
B. Lumpectomy
C. Lumpectomy with radiation
D. Observe
A

D

577
Q
In which of the following is DCIS MOST likely?
A. Palpable mass
B. Coarse calcification
C. Paget’s
D. Normal mammogram
A

C

578
Q
What is the BEST procedure to decrease recurrent invasive disease after low grade DCIS?
A. Mastectomy
B. Lumpectomy with 1 cm margin
C. Lumpectomy with radiation
D. Tamoxifen
A

A

579
Q
41 yo female, non-breast feeding patient comes in with a tender right breast in the upper quadrant.  She has been completely healthy.  Temperature is 37.9.  Examination reveals a tender right upper breast with erythema.  No mass.  What is the BEST management?
A. Incision and drainage
B. Antibiotics
C. Biopsy
D. Mammogram
E. FNA
F. MRM
A

B

580
Q
Perimenopausal women with a 4 month history of breast pain.  Previously healthy.  On examination, a firm and tender 2cm lesion is identified in the upper outer quadrant.  What is the BEST initial management?
A. Mammogram
B. Ultrasound 
C. FNA
D. Observation
E. MRI
A

B according to prev years – probably fibrocystic changes; US best modality for that, also needs mammo

581
Q
66.	42 yo female with a recurrent abscess to the left of the areolar complex, draining pus.  She is otherwise non-healthy, no adenopathy.  What is the BEST management?
A. Long term antibiotics
B. Incision and drainage
C. Excision of the affected duct
D. Biopsy
A

C

582
Q
66.	A woman is diagnosed with high grade ductal carcinoma in situ. Which of the following treatments will most decrease her chance of developing invasive cancer? [18mm, Er/Pr+]
A. Lumpectomy and tamoxifen
B. Lumpectomy and radiation
C. Mastectomy 
D. Radiation
E. Lump +AxLN
A

C

In a 2015 observational study that included over 100,000 patients who received a diagnosis of DCIS in the Surveillance, Epidemiology, and End Results (SEER) database, when compared with lumpectomy (with or without radiotherapy), mastectomy resulted in [2]:
●A similar 10-year breast cancer-specific mortality (multivariate hazard ratio [HR] for mastectomy versus lumpectomy 1.2, 95% CI 0.96-1.50)
●Lower rates of ipsilateral invasive recurrence (1.3 versus 3.3 percent)

583
Q
Patient with recurrent breast cancer with nausea and vomiting, thirst and polyuria.  Serum calcium is 2.96.  In addition to fluid rehydration, what is the BEST management?
A. Steroids
B. Bisphosphonates
C. Loop diuretic
D. Mithromycin
A

B

584
Q
Which invasive breast cancer has cells in single file rows?
A. Medullary
B. Lobular
C. Ductal
D. Tubular
A

B

585
Q
9 yo girl with new mobile, non-tender nodule underneath nipple. You should:
A. Reassure parents
B. US
C. FNA
D. Open biopsy
A

A

586
Q
50 yo woman has mammographically confirmed breast mass that the US shows to be complex cyst. Core biopsy shows normal breast tissue. You should
A. Image guided aspiration
B. F/U imaging in 6 months
C. Excise
D. Mastectomy
A

C

587
Q
45 yo woman with palpable mass in upper outer quadrant 2cm. Imaging suggests fibroadenoma. You should
A. Image in 6 months 
B. FNA
C. Core biopsy
D. Reassure
A

C

588
Q
45 yo lady with T1N0 breast cancer comes back for follow-up at one year. What should you do
A. Mammogram
B. Mammogram and bone scan
C. Mammogram, bone scan, CT chest abdo
D. Mammogram and C-135 level
A

A

589
Q
55 yo female with 1.5 cm breast cancer with 1cm margin except the posterior one which is 0.5 cm after lumpectomy. What is the best course for local control
A. Radiation
B. Tamoxifen
C. Re-excision
D. Mastectomy
A

A

590
Q
Erythematous, swollen breast with core biopsy showing adenoca. Next best step is
A. Mastectomy
B. Radiation
C. Chemotherapy
D. Lumpectomy and SLNBx
A

C

591
Q
46 yo woman smoker presents with swollen, tender breast and a temp of 37.9. You should 
A. Biopsy
B. Incise and drain
C. US
D. Antibiotics
A

C

592
Q
What is most suggestive of LCIS?
A. Mass on exam
B. Microcalcifications
C. ?
D. Linear arrangement of cells
A

D

593
Q
On doing SNLBx, patient develops hypotension, tachycardia and skin rash. Most likely diagnosis?
A. Latex allergy
B. Lymphazurin allergy
C. Malignant hyperthermia
D. ?
A

B

594
Q
35 yo lady with history of lupus has a core of left breast showing a 4mm focus of cribriform DCIS. Next?
A. WLE
B. WLE and radiation
C. Mastectomy
D. ?
A

C

595
Q
Diffuse DCIS comes back as 1mm margin on posterior aspect after simple mastectomy. Next?
A. Observation
B. Tamoxifen 
C. Chest wall irradiation
D. Re-excise
A

A vs B

Tamoxifen – for contralateral RR
There are no studies evaluating post-mastectomy RT for DCIS.

596
Q
70 year old woman with remote breast cancer [BCS  + rads], previous node positive, previous ER+ve, previous tamox x 5yrs, with FNA of a supraclavicular node NOW with ER+ve. Best management?
A. 2nd line hormonal therapy
B. Chemotherapy
C. Neck dissection
D. Radiation
E. Completion mastectomy w rads
A

A

Not tx surgically
Prev had radiation

597
Q
Who is most likely to benefit from aromatase inhibitors in breast cancer?
A. 36 yo lady 
B. 60 yo with T more than 2.5 
C. 45 yo post TAH BSO with DVT 
D. 70 yo otherwise healthy
A

C

AI only in post menopausal
AI worsen osteoporosis
45 yo post TAH BSO with DVT -possibly, as is now postmenopausal and AI have less DVT risk than tam

598
Q
45 yo woman with no risk factors comes in with palpable, tender breast mass. Best management?
A. US
B. Aspiration
C. Mammography
D. MRI
A

A

599
Q
Rapidly growing retroareollar well defined mass in 40 yo woman. Most likely?
A. Fibroadenoma
B. Phyllodes
C. DCIS
D. ?
A

B

600
Q
Microcalcifications on mammogram and on core you get hyperplasia with atypia. Best management? 
A. Excise under sterotactic guidance
B. Re-core
C. Vacuum assisted removal
D. F/U
A

A

601
Q
What determines that a phyllodes tumour is malignant?
A. Size
B. Stromal overgrowth
C. Desmoplastic reaction
D. Lymphocytic infiltrate
A

B

602
Q
50 yo woman with eczematous areola and negative mammo and exam. Next?
A. Punch
B. Excise
C. Core
D. MRI
A

A

603
Q
70 yo male with metastatic breast cancer, no mention of ER/PR status.  Treatment
A. Orchidectomy
B. Tamoxifen 
C. Chemotherapy  
D. Androgen therapy
A

C

B also possible since >80% ER +

604
Q
50 yo female presents with 2 cm firm upper outer quadrant mass, mammogram and US consistent with fibroadenoma
A. FNA
B. Excisional biopsy
C. Follow with yearly mammogram
D. MRI
A

B in old answers.

Core bx is probably better

605
Q
Partial mastectomy with invasive ductal carcinoma.  All margins negative, closest was 0.5 cm posterior, tumor <2cm.  What is best for local control:
A. Reexcise posterior margin
B. Tamoxifen
C. Radiation
D. Chemotherapy
A

C

606
Q
Older gentleman with bilateral breast masses.  Drug most likely to cause this:
A. Spironolactone
B. Allopurinol
C. Diazepam
D. Aldactone
A

A

607
Q
Invasive lobular carcinoma, typical features
A. Microcalcifications
B. Worse prognosis
C. Typical mammographic features
D. Single file, layered cells
A

D

608
Q

Which of the following would represent an indication for sentinel lymph node biopsy in breast cancer
A. 1.5 cm lesion fixed to chest wall
B. 1.5 cm lesion with palpable axillary node
C. 1.5 cm lesion with clinically negative axilla
D. 1.5 cm lesion in patient with previous lumpectomy

A

C

609
Q
Intraoperative positive margin doing WLE for DCIS comes back as malignant margin.
A. Close discuss with patient later 
B. Re-resecting margin
C. Mastectomy now 
D. Mastectomy after discussion with kin
E. Nipple sparing subQ mastectomy
A

B

610
Q
12 yo girl with asymmetric breasts.  The right breast is twice the size of the left breast.  Her developmental history is normal.  Her nipple areolar complex is normal in appearance.  What is the MOST likely diagnosis?
A. Virginal hypertrophy
B. Poland’s syndrome
C. Fibrocystic disease
D. Phyllodes
A

A

611
Q
A 15 year old girl presents with a 1.5cm solid breast lump superior and lateral to her left areola.  What is the most likely cause?
A. Hypertrophic breast tissue
B. Fibroadenoma
C. Breast cyst
D. Normal development
A

B

612
Q
52 yo female with unilateral bloody breast discharge.  US, MRI, mammogram normal.  What is most common cause
A. DCIS  
B. Duct ectasia 
C. Paget’s disease  
D. LCIS
A

B

613
Q
60 yo woman with greenish discharge that is non-spontaneous. Most likely diagnosis:
A. DCIS
B. Intraductal papilloma
C. Duct ectasia
D. Mammary fistula
A

C

614
Q
A 45 yo female has unilateral spontaneous green nipple discharge.  The mammogram is normal, and the ultrasound shows a dilated subareolar duct.  What is the diagnosis?
A. Ductal ectasia
B. Papilloma
C. DCIS
D. FCC
A

A

615
Q

Which of the following is most consistent with DCIS
A. Paget’s disease
B. Normal mammogram
C. Erythema of the skin

A

A

616
Q
Pt with peu d’orange.  What is the from
A. Invasion of lymphatics 
B. Venous invasion
C. Perineural invasion
D. Dermal invasion
A

A

617
Q

A 65 yo female with a remote history of breast cancer presents with back pain. There are no neurologic symptoms/signs. X-ray shows collapse of T7. What is the next BEST diagnostic test?

a. Bone scan
b. MRI
c. CT
d. Lumbarpuncture or lumboscintigraphy

A

A in prev years

B seems like a better answer since you need to rule out cord compression or edema

618
Q
45 yo female with breast cancer.  Pathology shows a 2.3 cm grade II, ER/PR negative, Her 2 Neu positive cancer with 4/14 lymph nodes positive for metastatic disease.  Which of the following should be part of her staging workup?
A. Bone Scan
B. CT chest
C. PET
D. CEA and CA 15-3 levels
A

A

In reality, B is also included in staging work up

619
Q
Which of the following is the BEST prognostic indicator in Breast Cancer?
A. Size of primary
B. Lymph node status
C. Age of the patient
D. Grade of the primary tumor
A

B

620
Q
Regarding DCIS, all of the following predict recurrence EXCEPT:
A. Status of margins
B. Age > 65
C. Grade of tumor
D. Presence of comedonecrosis
A

B

621
Q
58 yo female has a non-palpable simple cystic structure in her breast.  What is the best 	management:
A. FNA
B. Excision
C. Follow up in 6 months 
D. Reassure
A

C

Follow up in 6 months – shouldn’t have a new cyst in postmenopausal

622
Q
28 yo female notices a rapidly enlarging mass in her left breast over the last 3 months.  Excisional biopsy shows a Malignant Phylloides tumor which is 3.2 cm and has positive margins on path.  	Management?
A. Mastectomy
B. Re-Excision
C. Lymph node Dissection
D. XRT
A

B

623
Q

Sentinel lymph node biopsy, which is true?
A. It is an accurate indicator of axillary lymph node status
B. It has been shown to be equivalent to ALND on prospective randomized controlled trials
C. Its morbidity is equivalent to ALND

A

A

624
Q
45 yo female has thickened area of breast and no other findings.  Excisional biopsy shows "ductal ectasia with no evidence of hyperplasia".  What is her relative risk of developing cancer based on these findings?
A. None 
B. Three times
C. Six times
D. Nine times
A

A

This is non proliferative

625
Q
40 yo woman has intermediately suspicious microcalcifications on a mammogram and undergoes a core biopsy that shows lobular neoplasia.  What is your management of this patient?
A. Observation
B. Excisional biopsy
C. Tamoxifen
D. Mastectomy
A

B

626
Q

60 yo female with a hard 4 cm central breast lump and skin rash which has not improved with 2 weeks of antibiotics. An incisional biopsy was done which showed a grade III, ER/PR positive, Her 2/Neu negative, invasive ductal cancer. The metastatic workup is negative. What is the next step in management?
A. Total mastectomy with axillary dissection
B. XRT
C. Chemotherapy
D. Lumpectomy with XRT

A

Prev year said A

If this is inflammatory breast cancer, needs chemo before surgery.

627
Q
61 yo female with a stereotactic core biopsy of microcalcifications of the right breast.  Pathology 	shows low grade DCIS.  Which procedure provides the BEST risk reduction for developing future invasive cancer?
A. Lumpectomy with > 10 mm margins
B. Lumpectomy with XRT
C. Lumpectomy with XRT and Tamoxifen
D. Mastectomy
A

Prev year said C but D is best

628
Q

You see a 45 yo woman with a 6.5 cm area of DCIS in her right breast. She tells you that she would like immediate reconstruction. What is true?
A. Reconstruction may mask a breast cancer recurrence
B. She is a candidate for a skin sparing mastectomy
C. You would not recommend immediate reconstruction in her situation
D. She is not a candidate for autologous reconstruction

A

B

629
Q
A 37 yo female has a 1.8 cm breast cancer which is completely excised with negative margins.  On pathology it is shown to be Grade III, ER/PR negative, Her 2 / Neu negative and all of the 14 lymph nodes harvested are negative for metastatic cancer.  What treatment would be the BEST?
A. XRT
B. XRT and Chemotherapy
C. XRT and tamoxifen
D. Mastectomy
A

B

630
Q
Tamoxifen side effects all EXCEPT:
A. Cataracts
B. PE
C. DVT
D. Stroke
A

D

631
Q

Patient present with DCIS with microinvasion. What is true?
A. Should be treated the same as an invasive malignancy
B. It has been shown that tamoxifen plus XRT provide a significant benefit compared to XRT alone

A

A

632
Q
37 year old, presents with 1.8 cm cancer of the breast, high grade, ER/PR –ve, her2neu +/ve, axilla negative.  What is your mgt?
A. No chemotherapy or radiotherapy 
B. Radiation alone
C. Radiation and chemo
D. Chemo alone
A

C

633
Q

A women with DCIS has 5 mm margins after a total mastectomy , grade 3 , what you should do next
A. Follow up
B. Chest wall radiation
C. Tamoxifen chemotherapy

A

A in prev years

We do give C for risk reduction for the contralateral breast

634
Q
67 year-old female has lumpectomy for a breast lump.  Histopathology confirms an invasive lobular carcinoma, with 0.8cm margins.  It is ER/PR positive with none of 10 lymph nodes positive on ALND (LNS 0/10). The further treatment would be:
A. Tamoxifen
B. Radiotherapy 
C. Mastectomy
D. Observation only
A

B

635
Q
Nipple discharge is considered pathologic in all of the following EXCEPT:
A. It presents after 60 years of age
B. It is spontaneous
C. It is expressed
D. It is unilateral
E. It is bloody
A

C

636
Q

LCIS:
A. Premalignant lesion
B. Presence confers RR 5-12 of developing invasive ca
C. Lobular carcinoma develops in these patients
D. Must re-excise if positive margin found

A

B

637
Q

Regarding sentinel lymph node procedure in breast cancer: (MUST BE ALL EXCEPT)
A. 98% chance of finding the SLN in properly selected patient
B. 5-10% false negative rate
C. Lymphazurin blue (but not radio-labelled colloid) is safe in pregnancy and lactation –
D. Standard of care for Stage I and II
E. Not contraindicated for tumour > 5 cm

A

C

638
Q

With regard to DCIS
A. Invasion of basement membrane
B. Invasive form of ca
C. Recurrence related to grade, presence of necrosis, size
D. Mastectomy equivalent to lumpectomy + XRT

A

C

639
Q
All of the following are indications for mastectomy as initial treatment except:
A. Inflammatory breast cancer
B. DCIS with diffuse microcalcifications
C. 3 cm tumour in a small breast
D. 5 cm tumour
E. LCIS in an anxious woman
A

A

640
Q

Regarding the treatment of breast cancer, which is true?
A. Radiation increases long-term survival
B. Tamoxifen will not decrease contralateral breast cancer incidence after five years
C. Chemotherapy decreases the chance of local recurrence
D. Total mastectomy and lumpectomy with adjuvant radiation are equivalent treatments for DCIS

A

D

641
Q
Which of the following has the highest risk of breast cancer?
A. Adenosis
B. Apocrine metaplasia
C. Fibrocystic change
D. Papillomas
E. Mild hyperplasia
A

D

642
Q

Which of the following has the highest risk of breast cancer?
A. Sclerosing adenosis
B. Duct ectasia
C. Squamous metaplasia

A

A

643
Q
Which of the following has the highest risk of breast cancer?
A.  Sclerosing adenosis
B. Intraductal papilloma
C. Radial scar
D. Fat necrosis
A

B

644
Q
What is the best test to diagnose a leaking silicone breast implant?
A. CT scan
B. Mammography
C. MRI
D. Xray
E. Clinically
A

C

645
Q
Long standing hx of UC, now with elevated alk phos of 300, bili normal, distal CBD stricture, ca19-9 was 50
A. Transplant 
B. Ercp, dilate
C. HJ
D. Transhepatic dilation
E. Whipple
A

B

Frequent variations on this question; if CA 19-9 above 130, then oncologic resection.
On oral exam, need to get tissue diagnosis before doing surgery. Some HPB surgeons would do the Whipple based on this. However, in this scenario, CA19-9 is below cutoff for malignancy. UTD & AASLD specify endoscopic therapy first with good success. Spyglass with biopsy and FISH for polysomy are best to evaluate.

Sabiston - Cholangiocarcinoma

Tissue diagnosis prior to resection in operative patients is unnecessary. With obstructive jaundice, bile cytology and brushings are unreliable, and thus a negative cytology report does not exclude malignancy. Therefore, invasive attempts to establish a diagnosis prior to resection carry risk but do not alter subsequent management. Establishing a tissue diagnosis is only important when the patient is not a surgical candidate.

Up to Date

The optimal cutoff value that best discriminates between benign or malignant biliary tract disease is influenced by the presence of cholangitis (suggested by the presence of fever, leukocytosis, and right upper quadrant pain) and/or cholestasis (defined as a serum bilirubin >3 mg/dL [51.3 micromol/L]). In one report, a CA 19-9 cutoff value of ≥37 U/mL was 78 percent sensitive and 83 percent specific for malignant disease in patients who did not have cholangitis or cholestasis. By contrast, if the patient had cholangitis or cholestasis, the sensitivity dropped to 74 percent and the specificity to 42 percent when a cutoff of ≥37 U/mL was used. In patients with cholestasis or cholangitis, increasing the cutoff value to ≥300 U/mL was optimal for increasing specificity (87 percent), but at the expense of reduced sensitivity (approximately 40 percent).

CA 19-9 is an established serum marker for the diagnosis of cholangiocarcinoma, although it is reported to have a wide variation in sensitivity (50 to 90 percent) and specificity (54 to 98 percent) Serum levels of CA 19-9 are widely used for detecting cholangiocarcinoma in patients with PSC. CA 19-9 concentrations >1000 U/mL are consistent with advanced disease, often involving the peritoneum. If initially elevated, serum CA 19-9 levels may be useful for following the effect of treatment and to detect disease recurrence.

We typically use a CA 19-9 value ≥129 U/mL to increase suspicion for a cholangiocarcinoma in patients with PSC, especially in the presence of a dominant hilar stricture, though the current United Network for Organ Sharing policy for transplantation for cholangiocarcinoma uses a cutoff of ≥100 U/mL

646
Q
Hx of 10years ago with HJ for CBD injury, now 60’s, cardiac dz, diabetes, HTN, now with HJ stricture
A. PTC and dilate
B. Ercp and dilate/stent
C. Revise anastomosis at HJ
D. Lifelong abx
A

A

647
Q
GSW to liver, through and through, superficial parenchyma intact, actively bleeding from both sides of tract, in deep portion of liver, best management
A. Balloon tamponade
B. Tractotomy and control veins directly
C. Primarily close both ends
D. Pack
A

A

648
Q
50year old hx of renal cell carcinoma s/p resection, clear cell, now with two lesions in pancreas, head and body, 1.2cm, 2.2cm 
A. Total pancreatectomy 
B. Enucleation 
C. Chemo 
D. Radiation
A

A

There is some recent discussion of pancreatic enucleation for malignancy, but should not be considered standard. First, it may be technically very difficult to enucleate an invasive lesion. Second, there is very high morbidity with leaks, etc. Formal resection should currently be considered standard of care for malignant lesions. Total pancreatectomy for RCC mets to pancreas is well described. Lastly, you won’t be able to do a LAPAROSCOPIC enucleation; would be more feasible if open.

Annals of Surgical Oncology January 2006, Volume 13,Issue1, pp 75-85

Standardized pancreatic resection adapted to the location of the tumor in terms of partial pancreaticoduodenectomy,distal pancreatectomy, and total pancreatectomy was generally recommended for the management of isPMs (Fig.1). Because of the high recurrence rate, atypical local resection was confined to some exceptional cases. Omission of radical resection was reserved for a few exceptional cases. Ten of the 13 pertinent reports of nonresected isPMs specified the survival times. Two of the patients documented underwent what is now known to be an ineffective chemotherapeutic regimen, and another eight went without treatment. The survival data of these 10 patients with nonresected disease were compared with the data of 139 patients who underwent radical resection. The actuarial 3- and 5-year survival rates of 21% and 0% for nonresected metastases were significantly (P = .0383) poorer than those for resected lesions (78% and 72%).

Surgical treatment of isPMs from neoplasms other than renal cell cancer carry a poor prognosis, because they often signal the onset of disseminated metastatic disease. However, a recent retrospective study demonstrated that in a highly selected group of patients with distant pancreatic melanoma metastases, complete resection of the metastases may improve survival. By contrast, the outcome of surgery for isPMs from clear cell renal carcinoma is clearly superior, with a mean survival time of 4 years and actuarial 5-year survival rates of 43% to 75%, and is even better than the outcome of surgery for primary adenocarcinoma of the pancreas.

Because multiple metastases do not have a significantly poorer outcome, omitting or decidedly rejecting radical surgery, if possible, is unfounded. Radical resection for multiple metastases is just as beneficial as it is for solitary metastases. The only difference found was in the type of operation: as would be expected, total pancreatectomy was more often performed in patients with multiple metastases (Table4).

649
Q
35 year old male, mass in head of pancreas, ct read at compression of portal vein and borderline resectable, best way to assess resectability
A. Laparotomy 
B. Laparoscopy
C. Neoadj chemo
D. Neoadj chemorads
A

A vs B vs C

Toronto Manual: neoadjuvant is not standard of care, only true way to assess resectability

NCCN: Sequence of events for borderline resectable: neoadjuvant therapy, restage, laparoscopy, definitive surgery. NACT is usually FOLFIRINOX, may include radiation but that is +/-

Sabiston

For patients who appear resectable on imaging studies alone, laparoscopy identifies additional unresectable disease in up to 30% of cases. Others have argued that with current imaging used properly, the benefit of additional laparoscopy only rarely alters surgical planning. Recently, there has been some consensus on a more selective use of laparoscopy for those at particularly high risk for occult disease, including those with large tumors (>3 cm), significantly elevated CA19-9 level (>100 U/mL), uncertain findings on CT, or body or tail tumors. It may be clinically prudent also to consider laparoscopy for patients with clinical indicators of widespread disease, including significant weight loss, malnutrition, or pain. There are no level 1 data available.

In select patients, the role of neoadjuvant therapy is clearer, particularly those with significant venous or limited arterial involvement who are classified as borderline resectable. In these patients, where up-front surgical exploration has a significant risk of exposing patients to nontherapeutic laparotomy, the argument for neoadjuvant therapy is strengthened. For individuals with significant SMV–portal vein involvement(>180 degrees or short-segment encasement), or hepatic arterial or SMA abutment (<180 degrees) who have been traditionally considered unresectable, neoadjuvant therapy may play an important role in identifying the subset of patients most likely to derive benefit from aggressive multimodality therapy, including surgical resection with vascular reconstruction.51 This type of aggressive treatment should be undertaken only by an experienced multidisciplinary team in the setting of a clinical trial

650
Q
51 male three HCCs all on right lobe. Childs A. What to do
A. Right lobectomy
B. Chemo then right lobectomy
C. Transplant
D. Chemo then transplant
A

A

Childs A not contraindication to PH; however concen about 3 lesions indicating multifocality, in which case PH has a very high recurrence rate. The accumulation of outcomes data has clearly established LT as the gold standard for early-stage HCC in the setting of cirrhosis, BUT: PH acceptable for limited disease.

Multiple studies have demonstrated that a normal serum bilirubin level and the absence of clinically significant portal hypertension (ie, hepatic vein pressure gradient <10 mm Hg) are the best available indicators of acceptably low risk of postoperative liver failure after PH. In the absence of an elevated serum bilirubin and portal hypertension, survival after PH can exceed 70% at 5 years.

Sabiston

Patients with advanced cirrhosis (Child class B and C) and early-stage HCC should be considered for transplantation whereas those with Child class A cirrhosis have similar results with transplantation and resection and should probably be resected.

651
Q

Resection being done for IPMN, doing whipple, margin with PanIN-3 on frozen
A. Resect more distally with repeat frozen
B. Total pancreacteomy
C. Margins enough, you are done

A

A

PanIN 1&2 do
PanIN 3 reresect.

652
Q
Bodybuilder going to the OR for history consistent with appendicitis. Doing the lap appy and the appendix is normal. However, you see 500cc blood in the right gutter and a subcapsular liver hematoma? Anyways, question is what to do about liver bleed. Totally hemodynamically stable.
A. Open and pack
B. CT angiography
C. Serial labs
D. Right hepatectomy
A

B

Likely hepatic adenoma.

Cameron:
Formal resection at the time of acute hemorrhage is generally not recommended, although if the patient has life-threatening hemorrhage or if the lesion is refractory to transarterial interventions, then laparotomy may be necessary to achieve hemostasis. In these instances, the operation is analogous to a trauma laparotomy in which use of inflow occlusion (Pringle’s maneuver), packing and pressure, topical hemostatic agents, and selective hepatic arterial ligation are potentially useful maneuvers.

653
Q
Chronic pancreatitis, pain and jaundice with 3cm stricture in intrapancreatic portion of CBD no mass in pancreatic head – what to do?
A. HJ 
B. Frey 
C. ERCP/stent 
D. Whipple
A

C as initial. A if no mass. D if mass

a. HJ – if no mass or pain
b. Frey – decompresses PD not BD
c. ERCP/stent – only if cholangitic or malnourished as temporizing measure, high failure rate if pancreatic head calcs present
d. Whipple – if mass present or cannot R/O malignancy; only option to deal with pain and obstruction

654
Q
PSC patient with UC.  Has dominant stricture to CBD.  Hx of pruritus and jaundice, fatigue.  Now bili normal but high ALP.  Ca.19.9 raised Bile pathology from ERCP not distinguishable Mgmt?
A. Hepatojej
B. Transplant
C. ERCP dilation
D. Whipple
A

Transplant is the preferred surgical therapy for MOST surgical disease in PSC; the only exception is focal extrahepatic stricture OR complications in early histologic stage disease. Transplant can be considered for peripheral and hilar cholangiocarcinoma as part of a trial – not for extrahepatic/distal CCA.

CCA arising in PSC setting raises possibility of transplant. Overall, transplant is the treatment of choice, particularly as most are intrahepatic or proximal. Frozen section on bile duct should be done and pancreatiocoduodenectomy performed AT THE TIME OF TRANSPLANT if margins are positive. This patient has advanced liver disease (pruritus, jaundice), which is indication for transplant.

D.J. Rea, C.B. Rosen, D.M. Nagorney, et al. Transplantation for cholangiocarcinoma: when and for whom? Surg Oncol Clin N Am, 18 (2009)

UTD
SURGICAL THERAPY—Surgical options for primary sclerosing cholangitis (PSC) include biliary reconstructive procedures, proctocolectomy (in patients with ulcerative colitis), and liver transplantation [74].
Biliary reconstruction—Studies using various methods of biliary-enteric drainage, with or without intraoperative stent insertion, have reported excellent outcomes (free of jaundice and cholangitis) for several years after the procedure [75-77]. However, there may be significant morbidity and mortality, particularly in patients with cirrhosis. In addition, surgery carries a risk of postoperative infection and increases scarring in the porta hepatis, potentially complicating future liver transplantation [75].
The enthusiasm for biliary surgery declined further after a retrospective study found liver transplantation to be superior to biliary surgical procedures [78]. The actuarial symptom-free survival rate in 23 patients treated by nontransplantation biliary surgery was 35 percent at 10 years; the actuarial survival rate from the onset of PSC (56 percent at 10 years) was identical to that expected from the prognostic model. In comparison, the actuarial patient survival rate five years after transplantation in 28 patients was greater than that expected from prognostic models (89 versus 31 percent).
Thus, surgical therapies other than transplantation should generally be avoided in patients with PSC. The only exception may be in patients with isolated focal extrahepatic strictures and early histologic stage disease [79].

655
Q

20 yr female with epigastric pain post pancreatitis 3wks. No fever or chills, or jaundice WBC 14 best mgmt?
A. Serial imaging
B. Cystgastrostomy
C. Perc drain

A

A

656
Q
Elderly women with dyspepsia.  Tumor to pancreas 1.1cm octreotide scan positive. Best mgmt?
A. Central pancreatectomy
B . Whipple
C. Subtotal pancreatectomy
D. Observe
A

D

In this patient would observe. Enucleation of PNET in elderly patients is reasonable. Some centers observe PNETS <2cm.

657
Q

Chronic Hep C patient with Pugh class A. 3 Lesions 4, 4, 5 cms. Mgmt?
A. Transplant
B. R hepatic resection

A

C

Exceeds Milan criteria so no tranplant

658
Q

Thermal damage to biliary tract with 7mm defect. Mgmt?
A. Hepaticojej
B. T-tube through defect

A

A

If the injury occurs to a larger duct, but is not caused by electrocautery and involves less than 50% of the circumference of the wall, a T tube placed through the injury, which is effectively a choledochotomy, usually will allow healing without the need for subsequent biliary enteric anastomosis. Any cautery based injury, in which the extent of thermal damage may not manifest immediately, or an injury involving more than 50% of the duct circumference requires resection of the injured segment with anastomosis to reestablish biliary enteric continuity. When the defect is smaller than 1 cm and not near the hepatic duct bifurcation, mobilization with end-to-end anastomosis of the bile duct can provide acceptable reconstruction. This approach should be accompanied with transanastomotic T tube placement. The tube should be inserted through a separate choledochotomy, and not exit the bile duct though the anastomosis. To ensure a tension-free anastomosis, a generous Kocher maneuver, mobilizing the duodenum and the head of the pancreas out of the retroperitoneum, is necessary.

When the defect is smaller than 1 cm and not near the hepatic duct bifurcation, mobilization with end-to-end anastomosis of the bile duct can provide acceptable reconstruction. This approach should be accompanied with transanastomotic T tube placement.The tube should be inserted through a separate choledochotomy, and not exit the bile duct though the anastomosis. To ensure a tension-free anastomosis, a generous Kocher maneuver, mobilizing the duodenum and the head of the pancreas out of the retroperitoneum, is necessary. Injuries adjacent to the bifurcation or involving more than a 1-cm defect between the ends of the bile duct require reanastomosis
to the gastrointestinal tract

659
Q
Patient with borderline resectable pancreatic cancer with invasion into the portal vein, next step:
A. Laparoscopy
B. Neoadjuvant chemotherapy
C. Neoadjuvant chemorad
D. Laparotomy with tumor exploration
A

B

NCCN: Sequence of events for borderline resectable: neoadjuvant therapy, restage, laparoscopy, definitive surgery. NACT is usually FOLFIRINOX, may include radiation but that is +/-

There is good evidence for benefit of staging laparoscopy; up to 23% are converted to unresectable. Currently NACT or NACRT are being used (NACT more commonly), but there is insufficient evidence to recommend one over the other (ie with or without radiation).

660
Q

Patient with dilated CBD with 3 cm stone, management?
A. CBD exploration and choledochoduodenostomy
B. HJ
C. lithotripsy
D. transduodenal sphincterotomy

A

A on prev years but they were no sure

661
Q

Male patient with fevers, abdominal pain, early satiety, alcoholic, 10 cm collection with solid debris on CT, management?
A. Endocystgastrotomy
B. Percutaneous drainage
C. Operative cystojejunostomy

A

C

Due to debris, A may not work well. B for infected pseudocysts with sepsis or <6 weeks

662
Q

Patient with multiple symptomatic cysts in liver and kidney, management?
A. Liver transplant
B. Observe

A

B

Liver transplant – only used after liver failure, percutaneous treatments first

663
Q

Young female with OCP use, currently has 8 cm hepatic adenoma, best management
A. Stop OCP
B. Hepatic resection

A

B

Resect >5cm, as increased risk of HCC and rupture.

664
Q

Patient 4 years ago had resection for GIST tumor, now has recurrent GIST 12 cm in size in Rt lob of liver. Best management?
A. Gleevac with Rt hepatic resection
B. Rt hepatic resection
C. Resection then Gleevac

A

A

Up to Date:

In general, resection appears to benefit responding patients (ie, those who have a partial response, stable disease, or focal progression, and possibly those with isolated sites of progression) but has little to offer those who experience generalized disease progression while receiving a TKI. Furthermore, resection, even if complete, does not eliminate the need for continued treatment with TKI therapy. Progression-free survival is significantly shorter in patients who discontinue treatment as compared to those who continue the drug after resection
Hepatic resection for liver metastases—The liver is the site of recurrence in as many as 67 percent of patients with relapsed GIST. Prior to the imatinib era, these patients were treated like other soft tissue sarcomas metastatic to the liver, with resection when technically feasible. The available data on GIST liver metastases suggest five-year survival rates from 27 to 34 percent in patients undergoing resection alone.
For patients with isolated liver metastases, hepatic resection combined with imatinib provides the greatest opportunity for long-term disease control. A course of preoperative therapy (ie, three to nine months) is preferred in most cases, as it not only has the potential to reduce the extent of needed surgery, but it also permits the “biologic selection” of the best candidates for surgery, particularly if extensive procedures are planned.

665
Q

Patient with history of choledocholithiasis, previous jaundice, now in your office for referral for laparoscopic cholecystectomy. Currently asymptomatic, labs show N bili, ALP elevated, transaminases elevated, CBD from U/S dilated 8 mm, best management?
A. Lap chole with intraop cholangiogram
B. ERCP
C. MRCP

A

A

666
Q

Patient with history of choledocholithiasis, previous jaundice, now in your office for referral for laparoscopic cholecystectomy. Currently asymptomatic, labs show N bili, ALP elevated, transaminases elevated, CBD from U/S dilated 8 mm, best management?
A. Lap chole with intraop cholangiogram
B. ERCP
C. MRCP

A

A

Gavin says do preop MRCP. Really depends on how choledocholithiasis was managed. Shaw says MRCP is appropriate if trying to avoid cholecystectomy. Patient needs a cholecystectomy; no harm to postoperative ERCP if stones present on IOC.

667
Q

Patient with cholangitis, no ERCP capabilities, multiple stones in GB on U/S, CBD > 12 mm, best management
A. Percutaneous cholecystostomy
B. T-tube with CBD exploration for stones & laparotomy
C. T-tube placement, laparotomy

A

C

668
Q

Patient with 12 cm cystadenoma of Rt lobe of liver, management?
A. Enucleation
B. Rt hepatic liver resection

A

B

Per Karanicolas @ review course & Shaw & Canada consensus

669
Q
HJ after bile duct injury, how do you follow. she doesn’t want to follow up with her local surgeon.
A. 6 mo Blood Work
B  MRCP every 6 mo
C. ERCP at 6 mo
D. Observe
A

A

670
Q
Middle aged woman with intermittent postprandial pain in the RUQ x 6 months. Imaging shows 4mm polyp at cholecystocystic junction. What is the best management?
A. Lap chole
B. Repeat US in 6 months
C. Repeat US in 1 year
D. Hida scan
A

A

671
Q
Post-op double bypass for non-resectable pancreatic cancer. POD 2 has 100mL bile out of the drain, managed conservatively. Now POD 7 with persistent bile leak from drain. What is the best management?
A. Re-do anastomosis
B. PTC Stent
C. ERCP Stent
D. Contrast study through drain
A

B

672
Q

Old Guy Open Chole, 300 cc/day bilious drainage from drain, had ERCP sphincterotomy. Now with 100mL out of drain. What is the best management?
A. Take out JP
B. Take JP off suction
C. ERCP Stent

A

C

673
Q

Old guy. Failed cholecystostomy after imaging shows stone impacted in cystic duct. In OR converted to open and find gangrenous, necrotic fundus of GB. What is the best management?
A. C-Tube
B. Subtotal with C-Tube
C. Cholecystectomy

A

C

674
Q
Fibrolamellar HCC resected 5 years ago. now in her mid 30s. 3cm portal lymph node. best mgmt?
A. Chemo
B. Referral to palliative
C. Resect 
D. Pet CT
A

C in this answer key
D in other questions

Cameron: Fibrolamellar HCC represents nodal metastases in 15% to 30% of cases. Thus the lymph nodes in the porta hepatis, celiac region, and peripancreatic areas should be examined and, if suspect, either sampled or dissected. In cases of nodal recurrence after previous liver resection, exploration and dissection of the recurrent lymph nodes represent rational therapy.

675
Q

Extended R hepatectomy. POD #3 blood in drain, resuscitated for instability. CT shows ruptured subcapsular hematoma left liver. 4 units PRBC. platelet 6 x 10^9. INR 1.4. Hgb 75. Drain still draining blood. Next step
A. Angioembolization of left hepatic artery
B. OR for hemostasis
C. Balanced transfusion and resuscitation
D. CT Angio

A

B

676
Q
Difficult open chole in a 78y male, bile from JP, ERCP shows no stump leak. HIDA shows GB fossa leak. 3 weeks later 100cc per day from the JP. Best mgmt. 
A. ERCP and stent
B. Remove JP
C. Discontinue suction of JP
D. OR
A

A

677
Q
5 days post lap chole and CBDE. 3 small stones. 0.3-0.5cm in size. 
A. Basket removal
B. Lithotripsy
C. Repeat cholangiogram in 2 weeks
D. ERCP
A

C

678
Q

You are in a regional hospital doing a lap chole for a hot GB. thick walled. can’t delineate the triangle. open. cystic node frozen shows adenoca.
A. Chole with frozen section on the cystic duct
B. Close and stage
C. Close and neoadjuvant
D. Radical chole with lymph node dissection and resection of 4B/5

A

B

679
Q

Pancreatic pleural fistula. after pancreatitis. what is the best INITIAL therapy
A. Nutritional support and chest tube
B. Endoscopic stent
C. Distal panc

A

A

Management — Initial management of pancreatic fistulas and resultant complications of pancreatic ascites and pleural effusion include reduction of pancreatic stimulation and octreotide (a long-acting somatostatin analogue) to decrease pancreatic secretion. However, the long-term success of these approaches is limited, and only 50 to 65 percent of fistulas close over four to six weeks [4]. Patients with persistent symptoms require endoscopic stents, preferably bridging the ductal disruption. Surgery for a persistent pancreatic fistula is indicated when endoscopic management fails or is technically unfeasible. The management of pancreatic fistulas is discussed in detail separately.

680
Q

Reformed alcoholic with medically managed pancreatitis. back in clinic 7 weeks later with wt loss, fevers, and CT shows a 12cm pseudocyst with air bubbles abutting the stomach. splenic vein thrombosis and varices. EUS shows fluid and necrotic debris in the cyst.
A. Endoscopic cystgastrostomy
B. Surgical debridement and cystgastrostomy
C. Perc drain
D. Laparotomy with external drainage

A

B

Up to Date
Complex fluid collections and pancreatic necrosis—It is important to identify complex (ie, thick, debris-filled) pancreatic fluid collections since these may require intensive management with several endoscopic procedures. Complex collections are more likely in patients with pancreatic necrosis, which is best identified using MRI.
The decision to proceed with an endoscopic transmural drainage approach depends on how well-organized the necrosis appears. Although post-drainage infectious complications can be managed by expert endoscopists, failure to recognize underlying necrosis may result in inadequate drainage/irrigation and subsequent infection. Endoscopic drainage is less likely to be successful in patients with pancreatic necrosis compared with patients with uncomplicated pancreatic fluid collections. However, endoscopic treatment is a reasonable alternative to surgery in carefully selected patients if an endoscopist skilled in endoscopic necrosectomy is available.

681
Q
Unresectable cholangioca with involvement of right hepatic duct.
A. ERCP stent
B. PTC stent
C. Palliative resection
D. Hepaticojejunostomy of Left Duct
A

B

Need to drain duct; if attempt ERCP and unable to drain, you’ve introduced infection into a static system; PTC more reliable and can later be internalized.

682
Q
Pregnant lady, 28 weeks, cholecystitis, WBC high, bili N. What to do:
A. Abx
B. C tube
C. Lap chole
D. ERCP
A

C

683
Q
Lady bleeding from a ERCP sphincterotomy, failure to control endoscopically. Over next 24 hours requires 4 units PRBC and Hgb 104. stable, NG decreased draining blood. What to do next?
A. OR for oversew
B. Angioembolize
C. Repeat Endoscopy
D. Observe
A

D

If rebleeds could rescope or angio

684
Q
During lap chole, found liver to be cirrhotic and signs of portal HTN including multiple collaterals in porta hepatis, what to do now
A. Subtotal cholecystectomy
B. Total cholecystectomy
C. Cholecystostomy tube
D. Liver core biopsy
A

A

685
Q
Unstable cholangitis. Community hospital, no ERCP available
A. Chole + Duct Exploration
B. Lap T-Tube
C. Open T-Tube
D. Cholecystostomy Tube
A

C

686
Q

Whipple for pancreatic cancer. POD1 there is 50 cc of bright red blood from the JP drain, stopped spontaneously. Patient is stable.
A. Laparotomy
B. Angiography
C. Observe

A

B

Sentinel bleed, suspect GDA stump leak, can be managed with endovascular or surgery. Surgery is gold standard, however increasing role for endovascular therapies so not sure.

687
Q
Patient previously had a really bad gallbladder treated with subtotal cholecystectomy. Now comes back a year later with RUQ pain and a 1 cm gallstone impacted in the cystic duct of the gallbladder remnant and is closely associated with the duodenum. Improves with antibiotics. Management?
A. Resect gallbladder remnant
B. Antibiotics
C. ERCP/Spyglass retrieval 
D. Lithotripsy
A

A

688
Q

Old guy with severe cholecystitis pain improves but still tender. imaging shows air fluid level in GB but no air in CBD (cholecysto-duodenal fistual), no jaundice but acute cholecystitis
A. Antibiotics
B. C-Tube
C. Cholecystectomy/Primary repair of fistula

A

A

689
Q

Doing a CBD exploration when there is some bleeding lateral to the CBD. Most likely cause?
A. Accessory R hepatic artery to the SMA
B. Replaced R hepatic artery to the SMA
C. Accessory L hepatic artery to the L gastric artery
D. Replaced L hepatic artery to the L gastric artery

A

Prev years said A

Replaced R hepatic from SMA 12.5%
Accessory R hepatic from SMA 4%

690
Q
PSC in another patient with multiple intra and extrahepatic strictures and Bili of 80.
A. Transplant
B. HJ
C. PTC
D. ERCP and stent
A

A

691
Q
“Large” impacted stone that failed ERCP. Cant take it out in OR despite “maneuvers”
A. Lithotripsy
B. Hepaticojejunostomy
C. Transduodenal sphincterotomy 
D. Choledochoduodenostomy
A

Prev year said D but review course said C

Keith says sphincterotomy is the best answer – can extend by 2cm, and should be able to get the stone out.

Shaw says transduodenal sphincterotomy is part of the open CBDE, to push it retrograde to deliver also that choledochoduodenostomy is reasonable.

Stones impacted in the ampulla may be difficult for both endoscopic ductal clearance as well as common bile duct exploration (open or laparoscopic). In these cases the common bile duct is usually quite dilated (about 2 cm in diameter). A choledochoduodenostomy or a Roux-en-Y choledochojejunostomy may be the best option under this circumstance.

692
Q

50 F worked up for “dyspepsia”. Find a 9mm lesion found in isthmus of pancreas. Lights up on octreotide scan. Management?
A. Central pancreatectomy without anastomosis
B. Central pancreatectomy with distal Roux en Y pancreaticojejuntomy
C. Subtotal pancreatectomy
D. Repeat CT in 6 months

A

D

SCNA – PNETS

Central pancreatectomy is another parenchyma-sparing procedure that can be considered for small benign lesions.

For even smaller tumors less than 2 cm, surgical resection may not necessarily be mandated. The ENETS’ guidelines state “no data exist with respect to a positive effect of surgery on overall survival in small (<2 cm), possibly benign or intermediate-risk pancreatic endocrine tumors” and advocate careful balancing of surgical risk before proceeding to resection over observation. Lee and colleagues described a cohort of 67 patients with small (median size 1 cm), incidentally found nonfunctional PanNETs who were observed for a median of 45 months. There were no cases of disease progression over this observation period. Therefore, for small tumors less than 2 cm with a low proliferative index (Ki-67), observation may be an appropriate option.

693
Q
50 M with 3 cm cystic lesion at the head of the pancreas. MRCP non-diagnositic. Management?
A. CT guided biopsy
B. EUS
C. ERCP
D. Reimage in 6 months
A

B

694
Q
During a difficult laparoscopic splenectomy requiring multiple staple firings you notice you’ve made a 1cm laceration in the tail of the pancreas
A. Distal panc
B. Leave a drain
C. Close and Monitor Amylase
D. Close and get an MRCP
A

B

695
Q

Patient with previous EtOH pancreatitis. Returns with early satiety and 4kg weight loss. 15 cm pseudocyst against stomach. Splenic vein thrombosis with varices.
A. Cystojejunostomy and Splenectomy
B. Endoscopic trans-gastric drainage
C. Endoscopic trans-papillary drainage

A

B in prev years

But I would be reluctant to do this with varcies. I would probably go with A

696
Q
GB Cancer through lamina propria but not muscularis on post-op final pathology (it does not explicity tell you stage)
A. Liver Resection + LN
B. Chemo
C. Reimage in 6 mo
D. Liver Resection + LN + port sites
A

C (prev year said A)

This is T1a = invades lamina propria. Can tx with lap chole.

T1b = Invades muscle layer which requires further resection

697
Q

Male with symptomatic liver lesion 12 cm that has peripheral arterial enhancement and isointense on “later phase”
A. Angioembolize
B. Resect
C. Reimage

A

B

698
Q
Lady with travel to Europe and South America with pain in RUQ. Imaging shows 3 large simple appearing cysts in the right dome of the liver with no calcifications.
A. Aspirate 
B. Aspirate and Sclerosis
C. Laparoscopic unroofing 
D. Laparoscopic fenestration
A

This is a tricky question. The reference to travel makes you think hydatid but her cysts are simple

B? C?

699
Q

5 days post extended Right Hepatectomy with acities and lower limb edema. Imaging shows signs of torsion of liver causing hepatic vein obstruction . Low BP and low platelets. Lots of clear yellow serous fluid from JP drain.
A. Resucitate and place patient in left lateral decubitus
B. OR
C. Angio/Stent

A

B

700
Q

Arterial bleeding while doing chole from near the traingle, most likely cause
A Accessory right hepatic off SMA
B. Replaced right hepatic off SMA
C. Proper Hepatic

A

B

Replaces is more common than accessory

701
Q
Young lady with 10 cm multiloculated liver cyst with mural thickening
A. Liver resection
B. Deroofing
C. Marsupialization
D. Observe
A

A

? cystadenoma
Multiloculated and mural thickening are all things that make you want to resect

702
Q

2-3 cm HCC, Childs B
A. Transplant
B. Liver resection
C. TACE

A

A

703
Q

HCC, cirrhosis, gave several labs that made it sound like he was Childs A (gave albumin, TB, INR, no ascities, no encephalopathy). Best management
A. U/S to assess portal veins, for consideration for liver resection
B. Transplant multidisciplinary board
C. TACE
D. RFA

A

A

In Barcelona algorithm, presence of portal HTN determines if resect vs transplant for solitary HCC

704
Q

Which imaging is most suggestive of hepatic adenoma
A. Hot on red cell scan
B. Fatty on MRI
C. Central enhancement with washout peripherally
D. Central scar

A

B

705
Q

Presents with abdominal distention. Large retroperitoneal mass on CT with ascites, which is white/creamy on tap. What is the diagnosis.
A. Parasite occlusion of lymphatic channels
B. Lymphoma
C. Carcinoid

A

B

706
Q
2 weeks post CBDE with T-tube, has 7mm stone in common hepatic duct.  Stable.  Management.
A. ERCP
B. Radiologic extraction of stone
C. Repeat cholangiogram in 2 weeks
D. CBDE
A

Last year picked B but this seems early b/c your tract is not mature yet and patient is stable/not septic. I would probably go with C.

707
Q
Chronic pancreatitis, presents with painless jaundice.  Found to have intrapancreatic stricture of distal CBD, smooth, on ercp.
A. Choledochoduodenostomy
B. Stent and dilate ercp
C. Hepaticojej
D. Whipple
A

B
If mass–> whipple
If no mass–> HJ

708
Q
29 yo female with large, well circumscribed mass in head of pancreas.  What is most likely diagnosis
A. Pancreatic adenocarcinoma
B. Pseduocyst
C. Pseudopapillary 
D. IPMN
A

C

Solid pseudopapillary neoplasms—The malignant potential of solid pseudopapillary neoplasms (SPNs) has not been well studied. In a series of 62 patients, nine (15 percent) had malignant SPNs [46]. No factors were identified that predicted malignancy. In a second series with 106 patients who underwent surgery for SPN, 17 patients had high-grade malignant SPN [47]. Tumor size ≥5 cm was associated with an increased risk of high-grade malignancy.
Malignant solid pseudopapillary neoplasms (SPNs) can be cured when completely excised [48], and prolonged survival can be seen even in the presence of metastatic disease with surgical debulking [49,50]. Given the lesion’s malignant potential, combined with its good prognosis if resected, the finding of a pancreatic mixed solid and cystic lesion in a young woman on CT or MRI should lead to resection in most cases.

709
Q

Pseudopapillary tumor removed and confirmed on path. Margins negative. Next treatment
A. Nothing
B. Chemotherapy
C. Radiation

A

A

Patient does need imaging follow up

710
Q

Man presents with jaundice, bili 42 mild elevation of ALT and AST. ALP N, GGT N. Conjugated bili 6. US shows normal bile duct caliber and cholelithiasis. Managment
A. Lap chole
B. Lap chole with intraoperative cholaniogram
C. Ercp

A

A

Patient has Gilbert’s (indirect hyperbilirubinemia)

711
Q

Man with pancreatitis, previous imaging shows 4 cm cyst in pancreas, comes in 6 weeks later with same sized cyst.
A. Perc drain
B. Observe
C. Cystgastrostomy

A

B. Seems asymp

712
Q

History of pancreatitis, presents with episodes of gastric obstuction. On CT has large lesser sac collection with splenic vein thrombosis and left sided hypertension. Best management:
A. Endoscopic cystgastrostomy
B. Open cystgastrostomy
C. Open cystgastrostomy with splenectomy

A

C

713
Q
Pt had an ERCP, presents with pain. CT shows air in retroperitoneum, but no contrast extravasation. Pt is clinically stable. Management?
A. Antibiotics and observe
B. Perc drain
C. Laparoscopy and drain
D. Laparotomy and repair
A

A

Type IV ERCP perforation

Type I Lateral duodenal wall
Type II Periampullary
Type III CBD perforation
Type IV RP air alone

714
Q
Patient with a hilar cholangiocarcinoma on imaging. Which scenario is still resectable?
A. Portal vein encasement
B. Bilateral hepatic artery encasement
C. Right hepatic artery encasement
D. Positive celiac nodes
A

C

715
Q

Cirrhotic patient with umbilical hernia and ascites. Hernia now ulcerated and minimally leaking. Best management?
A. Medical management of ascites then fix hernia
B. TIPS then repair hernia
C. Repair hernia with permanent mesh
D. Paracentesis and diuretics

A

A

Shaw – urgent operation only for incarceration. Medically manage. Leak not a big deal.

Urgent surgery for rupture or incarceration. If ulcerated, eschar, or weeping fluid, medically manage ascites then elective repair. ,

Schwartz

Patients with advanced liver disease, ascites, and umbilical hernia require special consideration. Enlargement of the umbilical ring usually occurs in this clinical situation as a result of increased intra-abdominal pressure from uncontrolled ascites. First line of therapy is aggressive medical correction of the ascites and paracentesis for tense ascites with respiratory compromise. These hernias are usually filled with ascitic fluid, but omentum or bowel may enter the defect after large-volume paracentesis. Uncontrolled ascites may lead to skin breakdown on the protuberant hernia and eventual ascitic leak, which can predispose the patient to bacterial peritonitis. Patients with refractory ascites may be candidates for transjugular intrahepatic portocaval shunt (TIPS) or eventual liver transplantation. Umbilical hernia repair should be deferred until after the ascites is controlled.

716
Q

Recurrent episodes of pancreatitis, now narcotic dependent. Found to have pancreas divisum on MRCP. Best mangagment
A. Endoscopic sphincterotomy of minor duct
B. Whipple
C. Endoscopic sphincterotomy of major duct

A

A

717
Q
Patient with complex cystic lesion of pancreas. FNA done. Which is most consistent with mucinous cystadenoma?
A. Elevated CA 19-9
B. K-ras
C. Elevated CEA
D. Elevated CA-125
A

C

718
Q

Patient 4 months post lap chole presents with jaundice and imaging suggestive of common hepatic duct stricture. Best management?
A. Hepaticojejunostomy
B. ERCP and stent
C. ERCP and dilate

A

A is best. C is initial

719
Q

Patient with necrotizing pancreatitis. What is the best form of nutrition
A. TPN
B. Enteral feeds via NJ
C. Oral feeds

A

B

720
Q
Patient admitted with pancreatitis, transferred to ICU PAD5.  Oliguric, hypotensive requiring levophed, increasing ventilatory suppost required.  CT scan shows 70% non-enhancement of pancreas with no gas or fluid. Liver enzymes improving.  Management
A. Ercp
B. Supportive management
C. Necrostectomy
D. Peritoneal dialysis
A

B

721
Q
Pt with with symptomatic pleural effusion and pancreatitis.  Thoracocentesis postive for amylase.  Best managment. 
A. TPN &amp; Repeat thoracentesis
B. Oral feed and chest tube
C. Octreotide
D. ERCP
A

A

Sabiston

1st line is TPN, octreotide, drainage; 60% respond with this. If continued drainage, ERCP stent across ductal disruption.

722
Q
Lady with vague abdo discomfort.  2.5 cm cystic neoplasm of the pancreas. FNA shows glycogen-rich fluid, normal amylase, CEA. CA 19-9 is slightly elevated. Management?
A. Observe, repeat imaging in 6 months
B. Resect
C. Drain
D. EUS and biopsy
A

A

Serous cystadenoma. Vague discomfort is not convincing for symptoms from this small cyst. Resect if >4cm, symptomatic, growth >0.6cm/year.

723
Q

Patient with acute pancreatitis. CT shows pancreatic inflammation and superior mesenteric vein thrombosis. Management?
A. IV anticoagulation
B. Thrombolysis
C. Surgical thrombectomy
D. Angiographically directed thrombolysis

A

A

Up to Date

TREATMENT—The treatment of established mesenteric venous thrombosis (acute and subacute) is predominantly conservative, consisting of systemic anticoagulation to minimize extension of thrombus, bowel rest, and careful, serial observation for any signs of clinical deterioration. Must also asses for hypercoagulability.

724
Q

35ish woman who has had 4 episodes of pancreatitis recently. Nondrinker, no evidence of gallstones. CT shows 3cm mass in the body and tail of the pancreas (it did say a single mass in the body and tail) “sausage-like” dilation/inflammation of distal pancreas with ductal narrowing (I thought dilation). IgG4 (also Ca19-9?) is normal. Management?
A. Observe
B. Distal pancreatectomy
C. 6 weeks of steroids

A

C

Autoimmune pancreatitis can develop a mass which is confused for carcinoma, but often responds
to steroids. Sausage-like dilation is pathognomonic for autoimmune. Steroids then rituximab. Type 1 AIP has elevated IgG4; Type 2 AIP does not have elevated levels, more commonly associated with IBD. Can have mass, ductal dilation, ductal stenosis, jaundice. Also some overlap with IgG4-associated cholangitis (IAC). Always need to have a strong suspicion for pancreatic CA and rule it out, rather than rule in AIP. This patient, based on age and intermittent symptoms, is very unlikely to have PDAC, so trial of steroids is worthwhile. Sausage-like dilation is characteristic of AIP.

725
Q
HIV cholangiopathy with multiple intra and extrahepatic strictures. Management?
A. Transplant
B. Antibiotics
C. ERCP, balloon dilation, and stent
D. Transhepatic stent
A

C

Shaw: usually 2/2 cryptosporidum; treat AIDS and etiology. Multifocal less amenable to endoscopic therapy. Antibiotics alone ineffective; often have papillary stenosis which is responsive to papillotomy.

726
Q
Women with hypoglycemia. 1.6 cm hypervasc tumor in the pancreas and single lesion in liver, likely met. Mgmt
A. Whipple plus chemoembolize
B. Enucleate plus chemoembolize
C. Whipple plus liver resection 
D. Enucleate plus liver resection
A

C

Surgical resection is the mainstay of therapy for patients who present with locoregional disease. Complete tumor extirpation offers the only opportunity for cure and provides symptomatic relief for patients with functional PNETs. With the exception of most insulinomas, most PNETs are malignant7,8,27 and resection for these neoplasms should
follow sound oncologic principles, including formal pancreatectomy and regional lymphadenectomy. 28,29 Depending on tumor location, pancreaticoduodenectomy, distal pancreatectomy, or total pancreatectomy is typically performed. For patients with benign insulinomas, laparoscopic or open enucleation is the procedure of choice.

727
Q

Women with symptomatic gallstones for elective chole. Best indication for prophylactic abx.
A. Over 40 y
B. Diabetes
C. Thickened GB wall

A

B

Elderly, diabetes, cholangitis, instrumentation, recent biliary colic.

728
Q
Complex cyst in head of pancreas. Amylase elev. CEA elev. What is it?
A. MCN 
B. IPMN 
C. Pseudocyst 
D. Serous cystadenoma
A

B

MCN – lipase N, CEA high
IPMN – lipase and CEA high
Pseudocyst – lipase high
Serous cystadenoma

729
Q

6 y old. Recurrent cholangitis. Ultrasound shows RUQ cystic mass (or something like that). Mgmt.
A. Choledochoduodenostomy
B. Excision and hepaticojejunostomy

A

B

Choledochal cyst

730
Q
Chronic alcoholic pancreatitis with obstructive jaundice. CT shows calcification in the head of the panc with no masses. MRCP shows long, smooth, tapering of intrapancreatic portion of CBD. Mgmt.
A. ERCP and stent or dilate 
B. Whipple
C. Frey 
D. Hepaticojej
A

D

CBD stricture in CP is usually fibrotic. Endoscopic therapy has reasonable short-term efficacy, but not great in long term. Most recurrences after endoscopic therapy occur within 1 year. Presence of pancreatic head calcification predicts failure of endoscopic therapy:

Risk factors for failure of endoscopic stenting of biliary strictures in chronic pancreatitis: a prospective follow-up study.” The American journal of gastroenterology 98.11 (2003): 2448-2453.

CONCLUSIONS:

Endoscopic drainage of biliary obstruction provides excellent short term but only moderate long term results. Patients without calcifications of the pancreatic head benefit from biliary stenting. Patients with calcifications were identified to have a 17-fold (95% CI = 4–74) increased risk of failure of a 12 month course of endoscopic stenting.

TableIV. Indications for biliary drainage (Frey et al. [28])

  1. Symptomatic cholangitis or infected bile.
  2. Biliary cirrhosis (biopsy proven).
  3. CBD stones in association with common bile duct stricture.
  4. Inability to rule out cancer of the pancreas.
  5. Progression of the common bile duct stricture based on radiological assessment of increased dilatation of the common and intrahepatic bile ducts.
  6. Persistent jaundice for over a month.
  7. Persistently elevated alkaline phosphatase (> 3 × normal levels > 1 month).
731
Q

Cirrhotic, 2 cm hypervasc liver mass, bx HCC. Child B. Mgmt.
A. Resect
B. RFA
C. Transplant

A

C

732
Q
Cirrhotic, for elective hernia repair. Which hernia do you need to repair?
A. Ventral
B. Umbilical
C. Femoral
D. Inguinal
A

C

Femoral due to incarceration risk

733
Q

Cirrhotic, mild leakage of ascites from chronic 3 cm umbilical hernia. Mgmt.
A. TIPS, repair
B. Paracentesis, primary repair
C. Repair with mesh

A

A

I think there have been variations of this question where B is the answere but A was not an option

734
Q
Cirrhotic, strangulated inguinal hernia, bowel resection, closure?
A. Peoperitoneal mesh
B. Bassini
C. Lichtenstein
D. Biomesh
A

B

Tissue based repair

735
Q
Palliative cholangio Ca with internal-external drain. 3 weeks later has fever and melena. Cause?
A. Cholangitis
B. Arteriobiliary fistula
C. Vit K deficiency causing coagulopathy
D. Erosion of drain into duodenum
A

B

736
Q

Pregnant woman, 34 weeks, HELLP, tachycardiac, BP not horrible. Perihepatic fluid, large subcapsular hepatic hematoma. Following fluid resuscitation, mgmt.
A. Urgent lapartomy
B. Urgent C section
C. ICU for monitoring, fetal monitoring and serial HB
D. Angio and embolize

A

Prev year picked B. I think D is the right answer

Prev year based their answer that UTD says that delivery is curative and the only effective tx for HELLP and indicated after maternal stabilization in pregnancies >= 34 weeks gestation.

But how they describe the patient makes it seem that she is still tachy

UTD:
The management of a contained hematoma is to support the patient with volume replacement and blood transfusion, as needed. If the size of the hematoma remains stable and her laboratory abnormalities are resolving, the patient may be discharged home with outpatient follow-up. It may take months for the hematoma to resolve completely [42,47].

Percutaneous embolization of the hepatic arteries is a reasonable first-line therapy of hepatic rupture in women who are hemodynamically stable [48,49]. Surgical intervention is indicated if there is hemodynamic instability, persistent bleeding, increasing pain, or continued expansion of the hematoma [50]. A team experienced in liver trauma surgery should be consulted [51]. Operative management includes packing, drainage, hepatic artery ligation, and/or resection of affected areas of the liver. For patients with intractable hemorrhage despite these interventions, administration of recombinant factor VIIa [52] and liver transplantation [53-56] have been successful in case reports.

Patients who survive have no hepatic sequelae.

737
Q

30 y women, acute abdo pain. Tachy, hypotensive. Perihepatic fluid and large subcapsular hematoma. Mgmt
A. Laparotomy and packing
B. Angio and embolize
C. Hepatic resection

A

A vs B

Typical management of ruptured hepatic adenoma is angio with elective resection however the fact that she is tachy and hypotensive makes it seem like they want you to lap her

738
Q

Young woman, liver mass. Wants to get pregnant. Which requires resection?
A. Stellate scar
B .Hot sulfur colloid scan
C. High fat intensity on MRI
D. Peripheral enhancement with centripetal filling and washout on delay

A

C
Hepatic adenoma

A FNH
B FNH
D Hemangioma

739
Q

Women with dyspepsia and vague abdo symptoms. U/S shows two 5 mm gallbladder polyps. Mgmt.
A. Chole
B. Repeat ultrasound in 6 months

A

B

740
Q

Patient comes in with RUQ pain for 18hours, 38.6, US confirms acute chole. In the OR you find inflamed gallbladder with thickened area attached to duodenum suspicious for carcinoma. What should you do?
A. Antibiotics, stage and refer
B. Cholecystostomy tube
C. Cholecystectomy and partial duodenal resection
D. Lap chole with resection of your ports

A

A

741
Q

In the OR for a lap chole, you can’t get a plane between Hartman’s and CBD so you open. When open still can’t get a plane so you open the gallbladder and drain many stones. You find a large fistula between the GB and the CBD. What is the best management?
A. Partial cholecystectomy with closure of Hartman’s pouch
B. Primary closure of fistula opening
C. Leave a T tube through the fistula opening
D. Hepaticojejunostomy

A

A

742
Q

Scoping a patient with dyspepsia and you find a D1 ulcer eroding into biliary tract. What should you do? (poorly remembered, sorry)
A. ERCP and stent
B. Pantoloc IV
C. Hepaticojej

A

B

Shaw: only intervene if complications of hemorrhage or cholangitis.

743
Q

Woman with biliary pancreatitis with persistent pain (unclear if severe or not) 3 weeks out, CT shows 6cm collection in the lesser sac.
A. Percutaneous drain
B. Serial imaging
C. Endoscopic transgastric drainage

A

B

Too early to drain; acute fluid collection; 60-80% will improve with time

744
Q

Mild biliary pancreatitis being treated with conservative treatment, LFTs normal and no biliary tree dilation, what should you do?
A. Lap chole with IOC once pancreatitis resolves (same admission)
B. Lap chole with postop ERCP once pancreatitis resolves (same admission)
C. Immediate ERCP
D. Preop ERCP and then lap chole once pancreatitis resolves (same admission)

A

A

745
Q

Biliary pancreatitis treated conservatively, but 2 days later goes to ICU for clinical deterioration, on pressors, which of the following is the best indication for emergency ERCP?
A. Increasing Ranson’s criteria
B. Persistent increase in bilirubin

A

B

746
Q

Severe biliary pancreatitis, 72 hours of pain, now in the ICU, becomes febrile 38.9 and tachycardic (but no pressors), they give you a lot of labs, what we can remember is total bili was very mildly elevated (24?), normal direct bili, all LFTs MILDLY elevated, WBC 23. What should you do?
A. ERCP
B. CT
C. Antibiotics

A

B

747
Q

Endoscopic surveillance of pt with previous colectomy for FAP, you see two 1cm polyps in D2, well away from the ampulla, bx shows tubulovillous polyps with LGD, what should you do?
A. Whipple
B. Endoscopic removal
C. Transduodenal resection

A

B

748
Q

Old man in the OR doing a lap chole and at the end of the case he develops SC emphysema of the chest and neck. What should you do?
A. Extubate, give O2 and do CXR in PACU
B. Keep intubated and give 100% O2
C. Keep intubated until symptoms resolved
D. Bilateral chest tubes

A

A

749
Q
32F with recurrent pancreatitis persists despite cholecystectomy and cessation of all alcohol, now narcotic dependent for back pain, ERCP shows complete pancreatic divisum and chronic pancreatitis changes in the distal pancreas.
A. Open sphincterotomy
B. ERCP and minor papilla sphincterotomy
C. Distal pancreatectomy
D. ERCP and major papilla sphincterotomy
A

B

750
Q
In the OR for lap chole and incidentally the liver looks nodular and has perihepatic and omental varices.
A. Biopsy the liver
B. Cholecystostomy
C. Subtotal cholecystectomy
D. Total cholecystectomy
A

C

751
Q

Intraop for 1.5 pancreatic tail insulinoma, when you finish enucleation there is a signicant amount of clear fluid coming out. Best management?
A. Pack omentum in the defect and leave a drain
B. Distal panc
C. Pancreaticojejunostomy
D. Postop octreotide

A

B

752
Q

Previous resection of small bowel carcinoid symptomatic from carcinoid syndrome with multiple bilateral liver mets.
A. Liver debulking
B. Octreotide

A

B

753
Q

What increases your risk of ERCP pancreatitis?
A.Duodenal diverticulum
B. Sphincter of Oddi spasm
C. Pancreas divisum

A

B

The risk factors for acute pancreatitis include sphincter of Oddi dysfunction, female sex, young age, prior history of post-ERCP pancreatitis, history of recurrent acute pancreatitis, difficult cannulation, pancreatic duct injection, pancreatic sphincterotomy, failed attempts at placing a pancreatic duct stent, precut sphincterotomy, and low endoscopy volume.

754
Q
Newly diagnosed alcoholic cirrhosis. What should you do?
A. AFP Q6 months
B. US Q6 months
C. LFTs Q6 months
D. EtOH counselling
A

D

Up to Date
American Association for Study of Liver Diseases (AASLD) recommendes surveillance in patients at risk for HCC (cirrhosis, chronic HBV) with US q6 months; no recommendation for AFP alone – poor sensitivity and specificity.

755
Q
Cirrhotic getting screening EGD has esophageal varices with no bleeding and no ulcerations. What is the next line of treatment?
A. Beta blocker
B. Banding
C. Scleroinjection
D. TIPS
A

A

If ulceration considered high-risk for bleeding and banding should be done. Banding superior to medical therapy for primary prophylaxis, though with procedural complications. Recommend primary prophylaxis with beta blockers unless doesn’t tolerate, in which case EVL is used.

756
Q
Young woman previously healthy with weight loss, CT shows diffuse HCC with largest lestion 15cm (not 1.5). What should you do?
A. Resect
B. Transplant
C. TACE
D. Ytrium beads
A

C

Based on Barcelona algorithm.

757
Q

Guy with hep C cirrhosis (child Pugh B) with 3 lesions (largest was 2.9cm) consistent with HCC.
A. Resect
B. Transplant
C. TACE

A

B

758
Q

Old man with 2cm side branch IPMN in the tail of the pancreas. What should you do?
A. Follow up with imaging
B. Distal pancreatectomy

A

A (but want to know more about the lesion like whether there is enhancing mural nodules or duct diln)

759
Q

Patient who is an alcoholic, liver mass. Dx with HCC. Best sensitivity.
A. U/S
B. AFP

A

A

760
Q

Patient with severe gallstone pancreatitis. Day 4. Labs listed with increased WBC , bilirubin., lipase. What values would warrant intervention.
A. WBC
B. Lipase
C. Bilirubin

A

C

761
Q

Young female, chronic pancreatitis despite lap chole and abstinence from ETOH. Found to have pancreatic divisum, and on imaging, changes in distal pancreas consistent with chronic pancreatitis. Has pain, narcotic dependent
A. Endoscopic sphincterotomy of minor papilla
B. Operative sphincterotomy of minor papilla
C. Distal panc
D. Celiac block

A

A

762
Q
Guy with ETOH pancreatitis, admitted for exacerbation, has left sided portal HTN, 3cm pseudocyst in mid pancreas.  Pain not helped with NSAIDs and found to have antral gastritis. +ve Gastric varices
A. Oral omeprazole
B. Drainage of pseudocyst
C. Splenectomy
D. ?
A

A

763
Q
Female Ab discomfort.  15cm thick wall septated cystic mass in liver.  No other pertinent info give.  What to do?
A. Perc drain
B. Laparoscopic unroof
C. Open unroff
D. Open enucleate
A

D
Cystadenoma
Some ppl would advocate for formal resection

764
Q
Post op 8 hrs from liver resection with history of MI.  Lower U/O, low CVP, tachy/ hypotensive, what is the most likely cause?
A. Third spacing
B. Bleeding
C. MI 
D.	?
A

B

765
Q

During lap chole, found to have Mirrizi with conversion to open and identified 2mm fistula from hartmann’s pouch to CBD. What to do
A. Cholecestectomy + repair fistula
B. Cholecystectomy + HJ
C. Sub-total cholecystectomy + closure of hartmann’s pouch
D. Cholecystectomy + repair fistula over a T tube

A

C

766
Q

Patient w/ RUQ pain underwent imaging surveillance showing fundic lesion Suspected GB cancer (thickening), What to do
A. R hepatectomy
B. Lap chole + port excision
C. Segmental liver-reseciton + en bloc chole + LN dissection
D. Cholecystectomy + LN dissection

A

C

767
Q

Absolute contraindication for liver resection for CRC Liver Mets
A. Para-aortic LN
B. Stage 3 primary

A

A

768
Q

Male underwent extended R hepatic resection for CRLM. What is the most finding consistent with post-op liver failure?
A. INR 2 POD1
B. Bili 64 POD 6
C. Glucose 3.2 POD4

A

B

50/50 rule: 50% INR, 50 bili @ 5 days.

769
Q
Patient with CRLM, 8 of them, 1-2cm, bilobar.  Management?
A. Bevacizumab type chemo
B. Staged resection
C. Chemo then restage
D. ?
A

C

NACT indicated in borderline resectable disease, short DFI (<12mths).

Bevacizumab is commonly given in metastatic CRC; it always requires a chemo backbone (usually FOLFOX or FOLFIRI), despite little evidence of benefit in those regimens; useful as conversion therapy (cetuxumab not effective for conversion). BCZMB contraindicated if recent arterial thromboembolic events.

In the absence of validated predictive factors for treatment benefit, the default position in the palliative management of mCRC (intentionally neglecting financial considerations) is to add bevacizumab to the chemotherapy backbone chosen for the individual patient. For those who are at high risk for serious toxicity, in particular arterial thromboembolic events, where benefit from the addition of bevacizumab has to be balanced against a perceived risk for potentially life-threatening adverse events, a non-bevacizumab containing first-line regimen such as FOLFOX or FOLFIRI with or without cetuximab or panitumumab (if RAS wild-type) may be preferred.

When added to cytotoxic FU-based chemotherapy for mCRC, bevacizumab routinely improves outcomes, regardless of the specific chemotherapy backbone that is chosen. However, in some trials, bevacizumab appears to provide greater incremental gain in PFS when it is added to “weaker” chemotherapy regimens such as FU/LV or bolus IFL than to more active regimens such as FOLFOX

A major issue is whether to withhold bevacizumab from elderly patients who have a history of an arterial thromboembolic event (ATE) within the past 6 to 12 months. Bevacizumab increases the risk of an ATE. In one meta-analysis, the risk appeared to be higher in those over the age of 65 and in those with a prior history of an ATE

Cetuximab + irinotecan has benefit; cetuxumab + FOLFOX does not seem to add benefit.

770
Q

What’s the difference between whipple vs whipple + extended lymphadenectomy?
A. Less local recurrence
B. No difference in overall survival
C. Extended lymphadenctomy has better outcome for node positive patients
D. Similar operative morbidity

A

B

771
Q
Patient screened for AAA, found to have adenomyomatosis of GB.
A. Open chole
B. Lap chole
C. Radical chole
D. Observe
A

D

772
Q
Patient post AAA, RUQ, fever, elev WBC, normal bili.  U/S showed thick GB wall, distended GB, no stones
A. Cholecystectomy
B. Cholecystostomy
C. IV abx
D. ?
A

B

Sabiston

Treatment of acalculous cholecystitis is similar to that for calculous cholecystitis, with cholecystectomy being therapeutic. Given the substantial inflammation and high risk of gallbladder gangrene, an open procedure is generally preferred. However, many of these patients are critically ill and would not tolerate the physiologic insult of a laparotomy, explaining why the mortality rate of cholecystectomy for acalculous cholecystitis is up to 40%. Accordingly, percutaneous drainage of the distended and inflamed gallbladder is carried out in patients unable to
tolerate a laparotomy. The cholecystostomy tube used to drain the gallbladder can be placed by ultrasound or CT guidance. Approximately 90% of patients will improve with percutaneous drainage and the tube can eventually be removed. If follow-upimaging continues to demonstrate no stones, interval cholecystectomy is generally unnecessary.

773
Q

Elderly lady with biliary pancreatitis, admitted for a while. Lots of comorbidities and not a good surgical candidate. Has some GS in GB. Management?
A. ERCP sphinctertomy
B. Cholecystostomy
C. Abx

A

A

774
Q
Elderly patient with obstructive jaundice that resolved.  CBD is 10mm and there are stones in GB.  How to rule out choledocholithiasis?
A. ERCP
B. MRCP
C. EUS
D. ?
A

B

MRCP is preferred for many patients because it is noninvasive. However, the sensitivity of MRCP may be lower for small stones (<6 mm, (image 3)) [35], and biliary sludge can be detected by EUS, but generally not by MRCP. As a result, EUS should be considered in patients in whom the suspicion for choledocholithiasis remains moderate to high despite a negative MRCP. (See ‘Intermediate-risk patients’ above.)

775
Q
ERCP for choledocholithiasis.  Stones were extracted and on completion cholangiogram, there’s a contrast leak in intrapancreatic portion of CBD.  Management?
A. PTC
B. Percutaneous drainage
C. ERCP stent
D. ?
A

C

776
Q
Pregnant patient 14 weeks gestation, RUQ pain, febrile, sounds like cholecystitis.  Management
A. Abx
B. Lap chole
C. Cholecystostomy
D. ?
A

B

777
Q
Young female with RUQ pain, BP stable but tachy.  Found to have ruptured hepatic adenoma.  Management?
A. Angioembolization
B. Laparotomy and pack
C. Hepatic resection
D. Enucleation
A

A

Elective hepatic resection once stable

778
Q
Patient with pancreatitis, CT shows boggy pancreas and bowel, and SMV thrombosis.  Treatment?
A. Surgical thrombectomy
B. Thrombolysis
C. IV anticoagulation
D. ?
A

C

779
Q
Finding of hepatic adenoma on imaging?
A. Central scar
B. Hot of RBC scan
C. Cold on sulfur colloid scan
D. Peripheral enhancement then isodense on CT
A

C

A and D are both FNH
B is hemangioma

780
Q
81F, lap chole and found to have hole at base of cystic duct which was repaired with suture.  Comes back couple months later with intrahepatic duct dilatation, jaundiced.  Either a MRCP or ERCP showed stricture at mid CBD.  Management?
A. ERCP balloon and stent 
B. PTC stent
C. HJ
D. ?
A

A

Would try ERCP and stent first given her age. HJ is best management

781
Q
Patient undergoing Lap chole, intraop cholangiogram was performed (I think for elev enzymes).  Found to have 3mm stone in CBD.  Management?
A. Post-op ERCP
B. Laparoscopic CBD exploration
C. T-tube
D. Open CBD exploration?
A

A

Observe, glucagon, try to flush out. Stones that small should pass. If larger than 5mm, options are LCBDE or ERCP, with equivalent success, choice depends on local expertise.

782
Q
Old guy, RUQ, febrile, elevated WBC, Tbili and direct bili elevated.  GB thickened on U/S with GB stones.  
A. ERCP
B. Lap chole
C. Abx
D. ?
A

A

783
Q
Patient with vague abdominal symptoms and found to have multiple hepatic cysts and renal cysts.  Management
A. Albendazole
B. Percutaneous drainage
C. Operative unroofing largest cysts
D. Observe
A

D

784
Q
Old guy w/ choledocholithiasis, fever but otherwise stable, unable to remove stone w/ ERCP.  What to do next
A. Transhepatic drain
B. Cholecystostomy tube
C. ABx and perc drain
D. ABx and wait 24hr w/ repeat ERCP
A

A

785
Q
74 yo M w/ hep C cirrhosis present w/ 10cm HCC on segment 3, child A.  What to do
A. TACE 
B. RFA 
C. Transplant 
D. Resection
A

D

786
Q

Post AAA w/ RUQ pain, US found to have GB wall thickening <1cm consistent of adenomyomatosis. What to do next
A. Observe
B. Cholecystectomy

A

C

Adenomyomatosis is not an indication for resection but sounds like patient is symptomatic from gb

787
Q

Patient undergoing lap chole. Develop fever, tachycardic, cyanosis. Gas showing hypercarbia and metabolic acidosis. What to do next
A. Release pneumoperitoneum
B. Inc FiO2, and give methylpredinsone
C. d/c inhalation agent and give dantrolene

A

C

Malignant hyperthermia: are due to a hypercatabolic state, which presents as a very high temperature, an increased heart rate and breathing rate, increased carbon dioxide production, increased oxygen consumption, acidosis, rigid muscles, and rhabdomyolysis

788
Q

Patient had CRC resected sometime ago, now developed bilobar multiple CRC liver mets >8 lesion 1-2cm each. What to do
A. Staged resection
B. FOLFOX then re-stage
C. Bevacizumab chemotherapy

A

B

Immediately following the approval of bevacizumab in the US in 2004, FOLFOX plus bevacizumab (table 7) emerged as the most commonly used combination regimen for palliative front-line therapy of mCRC despite the absence of phase II or phase III data confirming the superiority of this regimen over any other. BCZ never given alone, always with a chemo backbone. Marginal benefit when added to FOLFOX, more when added to 5-FU. Can be used as conversion therapy to achieve resectability.

789
Q

Patient with food intolerance and dyspepsia, found to have GB sludge and two 5mm GB polyps. F/U in 6 months showed similar findings.
A. Repeat U/S 6mns
B. Lap chole
C. Open chole

A

B

790
Q

Hilar cholangioCa, unresectable. How to palliate
A. ERCP
B. PTC
C. Palliative resection

A

B

Percutaneous palliation is more effective, and preferred. The problem with endoscopy is that you may attempt ERCP, push bacteria beyond the obstruction, and then not be able to drain that system, leaving the patient cholangitic. For distal, ERCP with metal stents would be preferred.

SCNA – Proximal Biliary Tumours

Percutaneous drainage is more effective than endoscopic approaches due to technical and mechanical limitations of the latter; initial percutaneous drainage can be followed by self-expanding metallic stent placement. Although 2 or 3 stents may be necessary to drain all ducts, partial decompression can offer symptom relief.

791
Q

Which is not true regarding cholesterol gallstones:
A. The Pima Indians have a high incidence
B. The incidence increases with age
C. There is a higher incidence in diabetics
D. There is a higher incidence in females
E. There is a higher incidence in cirrhotics

A

E

792
Q

Concerning gallstone formation:
A. Cholesterol is soluble is aqua media as bile
B. Lecithin is not important
C. Bile salts are not important
D. Cholesterol supersaturation is not important
E. Mucous secretion is not important

A

A

793
Q

What two conditions favor gallstone formation:
A. Increased bile cholesterol and stasis
B. Increased bile cholesterol and non functioning gallbladder
C. Increased bile salts and stasis
D. Increased bile cholesterol and chronic inflammation
E. Increased bile cholesterol and increased bile salts

A

A

794
Q
Which are not prerequisites for dissolving gallstones with urso:
A. Radiolucent stones
B. Functioning gallbladder
C. Stone less than 15mm in size
D. Primarily cholesterol gallstones
E. Recurrent attacks of biliary colic
A

E

Small, (<15mm), free floating, funcitonal GB, not pigment stones, minimally symptomatic. Efficacy is 30-50%, but will reform.

795
Q
How does urso works to dissolve gallstones: 
A. Decreased bile salt secretion 
B. Decreases cholesterol synthesis 
C. Decreases gallbladder stasis
D. Increases fluidity of bile
A

B

Bile acids work by reducing biliary cholesterol secretion, increasing biliary bile acid concentrations, and as a result, reducing the cholesterol saturation index. Studies using strict inclusion criteria have found dissolution rates of between 50 to 60 percent, and a meta-analysis found a 37 percent dissolution rate with UDCA, with the best results occurring in patients with small, buoyant stones.

796
Q

Indication for ESWL:
A. Non functioning gallbladder with 2 radiolucent stones
B. Functioning gallbladder with 2 radiolucent stones
C. Acute cholecystitis with multiple stones
D. Large stones in CBD with acute biliary pancreatitis

A

B

797
Q

Regarding gallstones ileus:
A. Rare form of SBO*
B. Cholecystectomy should be done at operation
C. Resect
D. Milk backwards and perform enterotomy (plus run the entire bowel for other stones)

A

A &D

798
Q

74 diabetic presents with acute cholecystitis. She has rheumatoid arthritis and is on steroids. What will be your treatment:
A. IV antibiotics
B. Conservative tx, cholecystectomy later
C. Early cholecystectomy
D. ERCP & papillotomy
E. Percutaneous cholecystostomy

A

C

799
Q
65 yo female present tot he ER with a 16h history of acute cholecystitis. 2 years previously he had mitral valve replacement and is on coumadin. Your management would be:
A. Percutaneous cholecystostomy
B. Conservative with IV Ab
C. Cholecystectomy
D. Cholecystectomy under local
A

C

800
Q

Apatient has a leak from a cystic duct stump after laparoscopic cholecystectomy. The most appropriate treatment is:
A. PTC
B. Endoscopic stent insertion
C Laparotomy and ligation cystic duct
D. Laparotomy and Roux-Y choledochojejunostomy
E. Percutaneous drainage

A

E to drain collection then B

801
Q

During an open cholecystectomy the ligature on the cystic artery slips , and blood pools in the area. What should be done to control this event:
A. Apply a clamp in the pool of blood at the site of bleeding
B. Place avitine
C. Ligate the right hepatic artery
D. Pinch the hepatoduodenal ligament and suction out the blood
E. Cauterize in the pool of blood

A

D

Lap Pringle

802
Q

Patient undergoing elective laparotomy for another reason is found to have gallstones. You should:
A. Remove the gallbladder
B. Not remove the gallbladder
C. Remove only if previous symptoms
D. Remove is patients is otherwise stable
E. Remove stones only

A

B

803
Q
Sclerosing cholangitis is associated with all except:
A. UC
B. Cronh’s
C. Scleroderma
D. Reidel thyroiditis
E. Retroperitoneal fibrosis
F. Amyloidosis
A

C

804
Q
A patient develops acalculous cholecystitis after trauma. You recommend:
A. Cholecystectomy
B. Cholecystectomy under local
C. ERCP
D. Percutaneous drainage under US
A

D

805
Q
A patient develops acalculous cholecystitis after trauma. You recommend:
A. Cholecystectomy
B. Cholecystectomy under local
C. ERCP
D. Percutaneous drainage under US
A

D

806
Q
14 yo female presents with RUQ pain and mass after a mild bought of cholangitis. She has a fusiform dilation of the CBD. What should be the treatment of choice:
A. Cyst jejunostomy
B. Excision and hepaticoenterostomy
C. Excision and hepaticoduodenostomy
D. Excision with primary duct repair
E. ERCP and sphincterotomy
A

B

807
Q
18 yo female with intermittent jaundice, RUQ pain and RUQ mass. She most likely has:
A. Cholecystitis
B. Hepatitis
C. Pancreatitis
D. Choledochal cyst
E. Pancreatic division
A

D

808
Q
CholangioCa is typically:
A. Mucinous adenoca
B. Sclerosing adenoca
C. Squamous cell ca
D. Basal cell ca
A

B

809
Q
68 yo male presents with mild RUQ pain. Increased ALP, no jaundice, mildly elevated bili. He had a laparotomy and his pancreas, duodenum, ampulla and CBD are normal. Gallstones are not present.  Liver bx reveals intrahepatic cholestasis. What is the most likely diagnosis:
A. Periampullary ca
B. Sclerosing cholangitis
C. Hepatic duct adenoca
D. Pancreatic ca
E. Gilberts disease
A

C?

810
Q
65 male presents with known gallstones, fever, chills, hypotension and confusion. His platelet count is 30,000. He has a previous distal gastrectomy with Roux-Y reconstruction. What is the best treatment:
A. IV Ab
B. Operation to decompress the CBD
C. PTC
D. ERCP
E. Liver bx
A

C

811
Q
50 yo alcoholic with jaundice, fever, no pain, US showing sludge in gallbladder and ducts normal. Next steps; 
A. Cholecystectomy and CBD exploration
B. Lap chole and chlangiogram
C. ERCP
D. Liver bx
A

D? to assess for hepatitis or cirrhosis

Prev year said C

812
Q

Stone in intrahepatic ducts post chole. All except:
A. May lead to cirrhosis
B. More common in Asia than in North America
C. May lead to sepsis
D. Treated with hepatic lobectomy if only in 1 lobe
E. ERCP and sphinterotomy

A

E

813
Q
65 yo male had an open chole with CBDE 4 days ago. A T-tube cholangiogram reveals 3 CBD stones, each one approx 0.5 cms along with mild obstruction. What should be done: radiology consult for removal of stones through T-tube
A. Lithothripsy
B. Back to laparotomy and ECBD
C. Repeat cholangiogram in 2 weeks
D. Biliary pancreatitis
A

C

Small stones should pass, especially as should have dilated sphincter in CBDE.

814
Q

Which is the best indication for ERCP with papillotomy:
A. Sclerosing cholangitis
B. Pancreatic pseudocyst
C. Stenosis of pancreatic duct
D. CBD stones with previous cholecystectomy
E. Biliary pancreatitis

A

D

815
Q
The best indication for endoscopic sphinterotomy in acute pancreatitis is:
A. Worsening pancreatitis
B. Jaundice
C. Pancreatic ascites
D. Pancreatic pseudocyst
A

Last year said A. I think B is a better answer

816
Q
70 yo female presents with fever, no pain, jaundice, elevated ALP, normal LE’s normal CBD, dilated intrahepatic ducts:
A. Ampullary ca
B. Pancreatic ca
C.. Klatskin ca
D. CBD stone
A

C

817
Q
Patient transferred to you wit bile leak 1 week after transduodenal sphinterotomy. Now febrile, malnourished. On TPN:
A. Long NG post duodenal leak
B. Catheter into duodenal whole
C. Adequate drainage
D. Operate and repair now
A

C

818
Q
Treatment of bile leak following lap chole:
A. Open repair
B. ERCP
C. Percutaneous drainage
D. Observe
A

C first to control the sepsis then B

819
Q
What is the treatment of intrahepatic bile duct stones:
A. Chole and ECBD*
B. Hepatic lobectomy
C. Lithotripsy
D. L:iver transplant
A

B

820
Q
What is the best method to assess the proximal biliary tract:
A. ERCP
B. IV cholangiogram
C. HIDA
D. PTC
E. Laparoscopic
A

D

821
Q
Female undergoes lap chole. 36h after she is febrile, WCC 12.5, bili 52, ALT and AS elevated. US shows fluid collection around the live, 4x6cms.  What next:
A. HIDA
B. PTC
C. ERCP
D. Percutaneous drainage
E. OR
A

D

822
Q
70 female in MVA and on ventilator for 10 days now develops fever and RUQ pain.  The best way to confirm the diagnosis of acalculus cholecystitis?
A. U/S
B. History and physical and lab tests
C. Isotope scintigraphy
D. CT scan
E. MRI
A

A

823
Q

Patient with ERCP sphincterotomy. Bleeding post procedure, repeat ERCP shows bleeding from sphincterotomy, unable to control endoscopically. What next
A. IV octreotide
B. Angio with embolization of the gastroduodenal artery.
C. OR, tie off gastroduodenal
D. OR, open sphincterotomy

A

B

824
Q
55 year old male with 2 wk history of painless jaundice. ALP 360, Bili 280, AST/ALT Normal. At laparotomy, diffusely enlarged liver, normal pancreas, normal CBD, normal gallbladder. Diagnosis?
A. Hepatic duct stone
B. Hepatic duct tumour
C. Ampullary tumour
D. Pancreatic tumour
E. CBD tumour
A

B

825
Q
POD # 2 lap chole pt develops pain RUQ and elevated bili, with 12.5 wbc.  U/S shows 6cm RUQ collection. Next step?
A. HIDA
B. ERCP
C. Laparotomy
D. U/S guided drain
A

D

826
Q
Which is a clear indication for prophylactic cholecystectomy?
A. Porcelain gall bladder
B. Sickle cell
C. Diabetes
D. 5mm GB polyps
E. Adenomyosis
A

A

827
Q
Bleeding post ER sphincterotomy; uncontrollable with endoscopic means.  Best treatment:
A. Angioembolize gastroduodenal
B. Whipple
C. Open sphinteroplasty
D. Open gastroduodenal ligation
E. Treat with blood products
A

A

828
Q

Which is true regarding gall bladder cancer:
A. Survival depends on stage and treatment depends on depth of invasion
B. Gall stones aid in ultrasound screening
C. High prob. of cancer if polyp less than 1 cm
D. Not commonly found incidentally

A

A

829
Q
ERCP and sphincterotomy difficult, dangerous or impossible in all except which one of the following:
A. BII
B. R Y gastroenterostomy
C. Duodenal diverticulae periampullary
D. Divisum
E. Gastric outlet obstruction
A

D

ERCP sphincterotomy is treatment for divisum

830
Q
When is an open chole absolutely indicated?
A. Gall bladder cancer
B. Cirrhosis
C. Grade IV left ventricle
D. Severe acute disease
A

A

831
Q

What ratio is decreased in lithogenic bile:
A. Lecithin and bile salts to cholesterol
B. Cholesterol to bile salts
C. Cholesterol to lecithin
D. Lechithin to bilirubin

A

A

832
Q

All of the following are true about intrahepatic gallstones except:
A. Cholecystectomy and CBD exploration is curative
B. More common in East Asia compared to North America
C. May lead to hepatic fibrosis
D. May require resection of affected lobe
E. May lead to sepsis

A

A

833
Q
All the following are associated with jaundice due to hemolysis EXCEPT:
A. Hyperbilirubinemia
B. Decreased haptoglobin 
C. Increased LDH
D. Increased ALP
E. Increased urine hemosiderin
A

D

834
Q
Cholangiocarcinoma most commonly presents as:
A. Sclerosing adenocarcinoma
B. Squamous cell carcinoma
C. Mucinous adenocarcinoma
D. Transitional cell carcinoma
E. Neuroendocrine tumour
A

A

835
Q

Annular pancreas associated with:
A. Duodenal atresia
B. Ampullary obstruction causing jaundice
C. Presents with failure to pass meconium
D. Always associated with duodenal ulcer when presented in adulthood
E. Is associated with poor prognosis in childhood

A

A

836
Q

Which best describes the anatomy of the pancreas divisum:
A. Duct of Santorini is the main drainage of the pancreas *
B. The ventral analoge fails to migrate and join the dorsal analoge *
C. The duct of Wirsung drains the head, uncinate process, and body of the pancreas
D. The accessory duct of Santorini does not drain into the duodenum
E. The accessory duct of Santorini drains into the duct of Wirsung

A

A

Santorini/Minor = dorsal = most of panc
Wirsung /Major= ventral = head of panc

837
Q

What is the best treatment for gallstone pancreatitis:
A. Immediate open cholecystectomy with ECBD
B. Immediate ERCP
C. Delayed cholecystectomy performed when symptoms resolved, but at this admission
D. Delayed cholecystectomy a few weeks after
E. Immediate lap chole and ERCP

A

C

838
Q
24 yo female is in hospital for pancreatitis. She is improving until day 8 when more abdo pain develops. A CT scan reveals a peripancreatic fluid collection. No fever and normal WCC. What should be done: 
A. Percutaneous drainage of collection
B. Laparotomy and debridement
C. Continue observation
D. ERCP
E. IV Ab
A

C

839
Q

Which is most true regarding surgery for chronic pancreatitis:
A. Reverse endocrine insufficiency prevents development of endocrine insufficiency
B. Primarily indicated for pseudocyst
C. Primarily indicated for pain
D. Primarily indicated for dilated ducts

A

C

840
Q

Which is true regarding pancreatic pseudocysts:
A. Most are in the head
B. Immediate ERCP is indicated when they are detected
C. Have an epithelial lining
D. Form after any peripancreatic fluid collection develops
E. Complication rate is high after 3 to 4 weeks

A

E

841
Q
The most common complication of a pancreatic pseudocyst:
A. Infection
B. Rupture
C. Bleeding
D. Gastric outlet obstruction
E. Pancreatic ascites
A

D

842
Q
Tumor with best prognosis:
A. Proximal 1/3 CBD
B. Distal 1/3 CBD
C. Head of the pancreas
D. Duodenum	
E. Ampulla of Vater
A

D

843
Q
Which of the following is not associated with elevated amylase:
A. Parotiditis
B. Cardiac failure
C. Perforated DU
D. Ectopic pregnancy
E. Renal failure
A

D

844
Q
What are the typical findings in a glucagonoa:
A. Diarrhea
B. Weight gain
C. Migrating erythema
D. Polycythemia
E. Bradycardia
A

C but also A

Necrolytic migratory erythema present in 80% of patients.

845
Q

All are true of insulinoma except.
A. Usually benign and solitary
B. If symptomatic they are easily palpable at operation.
C. Diagnosis - fasting hypoglycaemia, hyperinsulinemia, and elevated c-terminal.
D. If metastatic, debulking may improve hypoglycaemic symptoms.
E. May be treated by enucleation if tumour does not involve the duct.

A

B

846
Q

Pancreatic duct leak. Nonhealing after 3/52 conservative management. Rx
A. Somatostatin
B. Surgery
C. ERCP

A

C

847
Q
All except which of the following is true with regards to islet cell tumours of the pancreas:
A. More common in head
B. Better prognosis if adenocarcinoma
C. Better prognosis if nonfunctioning
D. Related to MEN
E. Can be cured with resection
A

B

848
Q
A stable trauma patient is found to have a transected pancreatic duct to the left of the SMV.  Treatment
A. Distal pancreatectomy
B. Distal pancreatectomy and splenectomy
C. R Y repair
D. Drain
E. Primary repair
A

B

849
Q
Somatostatinoma and glucagonoma both have:
A. DM
B. Erythema
C. Anemia
D. Ulcers
A

A

850
Q
IPMN pancreatic resection, pathology shows PANIN-2
A. Total panc
B. Resect to negative margins
C. Annual MRI
D. Chemo post op
A

C

851
Q
45 yo women presents with back pain and a 2 cm cyst discovered by imaging in the head of the pancreas.  Appropriate management: 
ACT guide percutaneous drainage
B. Repeat CT in 3 months
C. Whipple
D. Observe
E. Cystgastrostomy
A

Answer key says C. That seems a bit extreme if EUS was an option I’d probably pic that. Of these answers, B?

852
Q
Which diuretic is contraindicated in hepatic failure:
A. Lasix
B. Spironolactone
C. Dyazide
D. Amiloride
E. Hydrochlorotiazide
A

C

853
Q
A patient has hepatomegaly but 3 liver bx had been reported as normal. The most likely diagnosis is:
A. Amyloidosis
B. Metastasis
C. Sarcoidosis
D. Hemochromatosis
E. Cirrhosis
A

C

854
Q

Polycythemia rubra vera with hepatic enlargement and shifting dullness and malaise could be cause by:
A. Hepatic vein thrombosis
B. Portal vein thrombosis
C. Mesenteric vein thrombosis

A

A

Thrombosis — A history of venous or arterial thromboses is common in PV. These complications were noted in 7 and 16 percent, respectively, in the WHO-defined PV patients. Although the mechanisms involved in this hypercoagulable state are unclear, abnormalities in blood viscosity, platelets, and leukocytes have been implicated (Uptodate)

855
Q
Portal vein thrombosis causes all but:
A. Varices
B. Hemorrhoids
C. Ascites
D. Testicular atrophy
A

D

Testicles drain into systemic circulation

856
Q

Which of the following is characteristic of bile duct hamartoma?
A. Associated with malignant transformation
B. Can be a large intrahepatic mass
C. Congenital lesion which increases in size over time
D. Needs to be promptly resected
E. Small white raised nodule on surface often found incidentally at laparotomy

A

E

857
Q
Portal vein thrombosis is associated with all except:
A. Hypersplenism and thromobocytopenia
B. Elevated splenic pulp pressure
C. Esophagogastric varices
D. Hypersplenism and leukopenia
E. Elevated Hepatic Wedge pressure
A

E

858
Q
Most common early complication of liver trauma:
A. Sepsis
B. Bleeding
C. Liver failure
D. Resp failure
A

B

859
Q
Falciform ligament leads to:
A. No embryological importance
B. Divide liver into right and left
C. Separates medial and lateral segments of left lobe
D. Site of division of hepatic artery
A

C

860
Q

Which of the following is not true regarding liver anatomy:
A. The quadrate lobe is segment IV
B. The medial division of the left lobe is segment IV
C. Biliary triads travel to the centre of the segments
D. The hepatic vein travels to the centre of the segments
E. The gallbladder fossa defines the division between r and l lobes

A

C

Biliary triads – at corner, with artery and portal v
Hepatic vein at the centre

861
Q
Patient with pancreatic pseudocyst undergoes cystgastrostomy then presents 1 week later with fluid collection in same area (as seen on CT). WBC 25, Hb 90, hemodynamically stable. Treatment
A. Laparotomy
B. Observe
C. Percutaneous drainage
D. ERCP
A

A in answer key

UptoDate:

Three clinical features suggest the possibility of a pseudoaneurysm:
●Unexplained gastrointestinal bleeding
●Sudden expansion of a walled-off pancreatic fluid collection
●An unexplained drop in hematocrit

Management: angioembolization, then surgery(distal pancreatectomy and closure of proximal pancreatic stump)

Persistent collection after cystgastrostomy, you should be concerned about:

  1. Pseudoanyerysm (drop in Hb, HCT, need angioembolization or OR)
  2. Ductal disruption (need MRCP/ERCP for anatomic delineation)
    a. Type 1 - no disruption, endoscopic or percutaneous drain
    b. Type 2 - stricture, OR
    c. Type 3 - complete occlusion, OR: pancreaticojejunostomy, or distal pancreatectomy+closure of proximal stump
862
Q

50 something year old female, hx of UC, finding of bile duct stricture in CBD, at confluence, at right hepatic duct, do what?
A. Transplant
B. Ercp stent
C. Ptc, stent
D. Right lobectomy with bile duct excision and HJ

A

A

UTD
The enthusiasm for biliary surgery declined further after a retrospective study found liver transplantation to be superior to biliary surgical procedures [78]. The actuarial symptom-free survival rate in 23 patients treated by nontransplantation biliary surgery was 35 percent at 10 years; the actuarial survival rate from the onset of PSC (56 percent at 10 years) was identical to that expected from the prognostic model. In comparison, the actuarial patient survival rate five years after transplantation in 28 patients was greater than that expected from prognostic models (89 versus 31 percent).

Thus, surgical therapies other than transplantation should generally be avoided in patients with PSC. The only exception may be in patients with isolated focal extrahepatic strictures and early histologic stage disease

Liver transplantation is the treatment of choice for patients with advanced liver disease due to PSC, and patients should generally be referred for liver transplantation once their Model for End-stage Liver Disease (MELD) score is ≥15

863
Q

What is true about accessory nerve:
A. Exits foramen ovale
B. Deficit is pure motor if injured
C. Goes behind jugular

A

B

864
Q
Thyroglossal duct cyst.  Which is true:
A. Must excise foramen cecum
B. Must excise portion of thyroid cartilage
C. Arises from second branchial cleft 
D. 1% papillary cancer
E. 10% squamous cancer
A

D

UTD
The incidence of primary carcinoma of the thyroglossal duct is less than 1 percent in all age groups

Article: Article Navigation
Thyroid Cancer in Thyroglossal Duct Cysts Requires a Specific Approach due to Its Unpredictable Extension
Thyroglossal duct cyst cancer histotype was papillary in 23 of 26 patients (88.5%) and follicular-Hurthle in 3 of 26 cases (11.5%)

865
Q
Where does RLN enter larynx:
A. Cricothyroid muscle
B. Cricothyroid membrane
C. Cricothyroid articulation
D. Thyrohyoid membrane
A

C

The RLN on both sides pass deep to the lower border of the inferior constrictor muscle and enter the larynx posterior to the cricothyroid articulation.

866
Q

A punch biopsy of a 1 cm mid tongue lesion reveals SCC. Best treatment:
A. Intraoral glossectomy
B. Neoadjuvant radiotherapy with surgery for residual disease only
C. Major resection with lymph node dissection

A

A

867
Q

A posterior triangle node is biopsied. Three months later the patient returns with difficulty abducting shoulder. The original path was T cell lymphoma. Most likely diagnosis:
A. Brachial plexus involvement with tumour
B. Axillary nerve injury
C. Accessory nerve injury
D. Suprascapular nerve injury

A

C

868
Q
A 60 yo female with a longstanding goiter presents with stridor.  Best treatment:
A. Nasotracheal intubation
B. Bronchoscopic guided intubation
C. Cricothyroidotomy
D. Tracheostomy
A

B

869
Q
75 year old man with painless enlargement of submandibular gland to 3x the size of the contralateral gland. Painless. Most likely pathology.
A. Mucoepidermoid carcinoma
B. Pleiomorphic adenoma
C. Pleiomorphic carcinoma
D. Warthin's tumour
E. Adenoid cystic carcinoma
A

A

MEC>ACC

870
Q
What is the best location for a tracheostomy:
A. Below the cricoid cartilage
B. At the cricoid
C. As low as possible
D. At the second tracheal ring
E. At the fourth tracheal ring
A

D

871
Q
Nerves encountered during submandibular gland resection:
A. XI, XII
B. V, VII
C. VII, XI
D. V, VIII
A

B
Injury to the hypoglossal nerve (CN XII) and lingual nerve (CN V3) are much less common (<1% of patients) than injury to the marginal mandibular nerve (branch of facial nerve–CN VII)
(probably 5-10%).

872
Q
Nerves at risk in submandibular gland dissection
A. Lingual nerve
B. Facial nerve
C. Submandibular ganglion
D. uriculotemporal nerve
A

A and B

Injury to the hypoglossal nerve (CN XII) and lingual nerve (CN V3) are much less common (<1% of patients) than injury to the marginal mandibular nerve (branch of facial nerve–CN VII)
(probably 5-10%).

873
Q
Which structure is likely to be injured with biopsy of a lymph node superficial to the submandibular gland:
A. Hypoglosal nerve
B. A motor branch of CN VII 
C. A motor branch of CN V 
D. Lingual nerve
E. Vagus nerve
A

B Marginal mandibular

Lingula is deep to mylohyoid

874
Q
Course of phrenic nerve. It runs anterior to..
A. Scalene anterior
B. Scalene media
C. Subclavius
D. Omohyoid
A

A

875
Q
Radical neck dissection would sacrifice all of the following except:
A. Internal jugular
B. Sternocleidomastoid muscle
C. Supraclavicular nodes
D. Hypoglossal nerve
A

D

876
Q
After a lymph biopsy in a posterior triangle a patient complains of tingle in the fingers, a sore aching shoulder and inability to elevate his arm above his head. The most likely diagnosis:
A. Brachial plexus injury
B. Musculoskeletal injury
C. Spinal accessory nerve injury
D. Malingering
A

C

http://practicalneurology.com/2015/10/spinal-accessory-nerve-injury/

877
Q
Which structure is not likely to be injured with resection of a brachial cleft cyst:
A. Facial nerve
B. Hypoglossal nerve
C. Accessory nerve
D. Superior laryngeal nerve
E. Vagus nerve
A

D

Elevate the cyst off the deep structures i.e. Xn, XIn, XIIn, descendens hypoglossi, internal jugular vein and carotid vessels take care not to traumatise the ranine/lingualveins where the tract crosses the XIIn

878
Q
Which of the following will be visualized with resection of a branchial cyst:
A. Hypoglossal nerve
B. Trachea
C. Spinal accessory nerve
D. Larynx
E. Submandibular gland
A

A

879
Q
Lymph node drainage from mid third of tongue:
A. Submental 
B. Submandibular digastric
C. Superficial anterior cervical
D. Posterior cervical
A

B

Tip of tongue–submental
Anterior 2/3 Lateral–Ipsilateral submandibular
Anterior 2/3 Central–Bilat Submandibular
Posterior 1/3–Upper Deep Cervical

880
Q
Midline lesions of the neck include all except;
A. Thyroglossal duct cyst
B. Lingual thyroid
C. Dermoid tumor
D. Ranula
E. Granular cell myoblastoma
A

C

Ranula -type of mucocele found on the floor of the mouth. Ranulas present as a swelling of connective tissue consisting of collected mucin from a ruptured salivary gland caused by local trauma.

Granular cell tumors can affect all parts of the body; however, the head and neck areas are affected 45% to 65% of the time. Of the head and neck cases 70% of lesions are located intraorally (tongue, oral mucosa, hard palate). The next most common location that lesions are found in the head and neck area is the larynx (10%)

881
Q
45 yo male alcoholic, smoker for 40 years has a painful tongue. On exam, he has poor dentition and a 1.5cm hard, painful, indurated ulcer at the base of the tongue. What is the most likely dx:
A. Lymphoma
B. Epidermoid ca
C. Syphilitic ulcer
D. Blastomycosis
E. Actinomycosis
A

B

SCC

882
Q

SCC of the tongue may receive curative rads except:
A. Recurrent SCC with previous radiation
B. When there is mandibular involvement

A

Both seem like contraindications

Radiation to mandible can lead to osteoradionecrosis of the jaw

883
Q

Which of the following is true regarding adenoca of the oral cavity:
A. It is as common as epidermoid ca
B. It is effective treated with radiation
C. The prognosis is the same as for epidermoid ca
D. It raises from the minor salivary glands

A

D

884
Q

Which statement is true regarding head and neck ca:
A. Surgery is better than radiation to treat occult neck disease
B. The 5 year survival for stage I & II ca approach 50%
C. Risk factors include smoking, drinking and viral infection

A

C

885
Q
Freys syndrome, injury to which nerve:
A. Submandibular
B. Marginal mandibular branch of facial nerve
C. Post-auricular nerve
D. Auriculotemporal nerve
A

D

886
Q
Best indication of parotid malignancy:
A. Serous otitis media
B. Increased swelling of gland
C. Facial nerve involvement
D. Fixed gland
E, Large size
F. Hearing deficit
A

D

887
Q
Which is the most common parotid tumor:
A. Warthins tumor
B. Monomorphic adenoma
C. Pleomorphic adenoma
D. Lymphangioma
E. Oncocytoma
A

C

888
Q

A firm rubbery lesion in the parotid of a female that has been present for 3 years is most likely:
A. Pleomorphic adenoma
B. Warthins
C. Lymph node

A

A

889
Q
Warthins tumor all except:
A. Bilateral in 10%
B. More common in men
C. Mandates surgical excision
D. Found in minor salivary glands
A

D

890
Q
Warthins tumor all except:
A. Benign
B. Often soft and compressible
C. More frequent in men than in women
D. Frequently bilateral
E. Radiation is the best treatment
A

E

891
Q
The most common malignant parotid salivary gland in children is :
A. Mucoepidermoid
B. Acinic
C. Lymphoma
D. Oxyntic
E. Malignant mixed salivary
A

A

Image result for malignant parotid tumor children
Salivary gland neoplasms are rare in children. Most tumors (65%) are benign, with hemangiomas being the most common, followed by pleomorphic adenomas. In children, 35% of salivary gland neoplasms are malignant. Mucoepidermoid carcinoma is the most common salivary gland malignancy in children.

892
Q

Treatment of adenoid cystic ca of the parotid not invading facial nerve:
A. Total parotidectomy sacrifice nerve, neck dissection
B. Superficial parotidectomy only
C. Total parotidectomy, spare nerve, radiation

A

C

893
Q

Apatient with adenocystic ca of the parotid. You have done a parotidectomy ( no involvement of the facial nerve). Do you:
A. Spare the facial nerve
B. Take the facial nerve
C. Take the facial nerve and neck dissection
D. Take the facial nerve, neck dissection and rads
E. Spare facial nerve, neck dissection and rads

A

E

894
Q

Which is true of the spinal accessory nerve (XI):
A. Nothing important lies above the nerve in the posterior triangle
B. It is surrounded by lymph nodes for the entire length
C. Exits skull via the foramen magnum (jugular foramen)
D. Lies on anterior scalene muscle

A

A

C = jugular foramen
D = phrenic nerve
895
Q

A fifteen y/o girl has a cystic mass in the midline of the neck, 1 cm above the level of the thyroid cartilage. The clinical suspicion is that is it a thyroglossal duct cyst.

Before surgical excision, which of the following investigations should be performed:

A. An ultrasound of the thyroid gland
B.  CT of the neck
C. Thyroid function tests
D. Radioactive Thyroid Scan
E. FNA of the mass
A

A

896
Q

A 23 y/o female has undergone a right hemithyroidectomy for a nodule that proved to be a benign follicular lesion on the frozen section examination. Two days later the final histopath. report confirms that it is a Hurthle cell carcinoma 1 cm in diameter.

What would you recommend for this patient

A. Observation and follow up
B. Total Thyroidectomy
C. Subtotal Thyroidectomy
D. Radioactive Iodine
E. External Radiation
A

B in prev answers. A is reasonable.

897
Q

During a thyroidectomy, clamping the vascular pedicle of the upper lobe, rather than careful ligature of the individual vessels, is most likely to injure:

A. The superior laryngeal nerve
B. The internal laryngeal nerve
C. The recurrent laryngeal nerve
D. The cricopharyngeal nerve
E. The cricothyroid nerve
A

A

898
Q
After a thyroidectomy, which electrolyte imbalance can worsen transient hypoparathyroidism:
A. Hypomagnesemia
B. Hypernatremia
C. Hyperkalemia
D. Hypophosphatemia
E. Hypokalemia
A

A

899
Q
The common cause of primary hyperparathyroidism
A. A single parathyroid adenoma	
B. Multiple parathyroid adenomas
C. Parathyroid carcinoma
D. A clear cell parathyroid adenoma
E. Chief cell hyperplasia
A

A

900
Q

A 27F presents with a small lump in the left lobe of her thyroid. This is proven to be medullary carcinoma on needle biopsy. There are no other abnormalities of the neck or thyroid.
The correct operation is:
A. Complete excision of the nodule
B. Left hemithyoidectomy
C. Total thyroidectomy
D. Total thyroidectomy with central neck dissection

A

D

901
Q

A 27F presents with a small lump in the left lobe of her thyroid. This is proven to be medullary carcinoma on needle biopsy. There are no other abnormalities of the neck or thyroid.
The correct operation is:
A. Complete excision of the nodule
B. Left hemithyoidectomy
C. Total thyroidectomy
D. Total thyroidectomy with central neck dissection

A
D 
Total thyroidectomy
-Known distant mets
Extrathyroidal extension 
Tumor >4 cm 
Cervical LN mets
Poorly differentiated 
Prior radiation 
Bilateral nodularity
902
Q
Which of the following should be the FIRST step in management of a patient in hypercalcemic crisis
A. Diuresis induced by IV saline
B. Lasix
C. Cortisone
D. Calcitonin
E. Emergency parathyroidectomy
A

A

903
Q
Which of the following is NOT associated with pheochromocytoma
A. Hyperparathyroidism
B. HTN
C. Neurofibromatosis
D. Follicular Carcinoma of the thyroid
A

D

904
Q

Level 4 LN shows metastatic papillary thyroid carcinoma. Treatment?
A. Lobectomy + ?RT/Chemo or something else
B. Lobectomy L neck dissection
C. Total thryoid L neck dissection
D. Total thyroid + something else

A

C

905
Q
Kid w/ painful, swollen mass anterior SCM. Complex cystic on u/s w/ enlarged node. Management?
A. IV abx
B. I &amp; D
C. US guided aspiration
D. Excision
A

A

906
Q
Parathyroid exploration. Can’t find R inferior parathyroid. Where is it most likely to be?
A. Tracheo-esophageal groove
B. Thyrothymic ligament
C. Carotid sheath
D. Intrathyroidal
A

B

907
Q
Lady w/ thyroid nodule. What result on FNA would be most useful preop?
A.  Colloid
B. Hurthle cells
C. Follicular cells
D. Papillary
A

B in prev answer but I think D is right.

You can’t dx follicular or huerthle cell neoplasm on FNA. Need to look at architecture

908
Q

A 50 year old female with a multinodular goiter. A CT of the chest shows a mass in the anterior superior mediastinum with minimal tracheal deviation. The patient has a goiter with retrosternal extension. The best treatment is:
B. Radioactive iodine ablation therapy.
C. Total thyroidectomy through a cervical incision.
D. Sternotomy and total thyroidectomy.

A

C

909
Q

Patient has had large asymptomatic goitre for a long time. Has URTI then gets stridor. Management?
A. Bronchoscopic guided orotracheal intubation
B. Perc trach
C. Open trach
D. Cric

A

A

910
Q
Previously healthy 62 yo man non smoker, non drinker. Has level 3 palpable neck node. Best next mgmt?
A. FNA 
B. Core
C. Excisional bx
D. CT
A

A

911
Q
2 yo child with a small opening in the left neck intermittently drains a small amount of clear fluid.  The opening is at the anterior edge of the SCM at the junction of the inferior and middle third. Surgery to dissect out the sinus will end at:
A. The left tonsillar fossa 
B. The left piriform fossa 
C. External auditory canal
D. Cyst deep to platysma
A

A

First branchial cleft cysts

  • Type I cysts- external auditory canal. Inferior and posterior to the tragus , but may also be in the parotid.
  • Type II cysts-angle of the mandible and may involve the submandibular gland

Second branchial cleft cysts

  • 95%
  • Anterior border of the upper third of the SCM and adjacent –Anywhere along the course of a second branchial fistula: skin of the lateral neck, between the internal and external carotid arteries, and into the palatine tonsil

Third branchial cleft cyst

  • Rare
  • Same skin location as a second branchial fistula
  • Courses posterior to the carotid arteries and pierces the thyrohyoid membrane to enter the larynx, terminating on the lateral aspect of the pyriform sinus.
  • (eg, inside the larynx), but they are characteristically located deep to SCM

Fourth branchial cleft

  • Extremely rare.
  • From the lateral neck and parallels the course of the recurrent laryngeal nerve (around the aorta on the left and around the subclavian artery on the right), terminating in the apex of the pyriform sinus
  • Cysts arise in various locations, including the thyroid gland and mediastinum.
912
Q

FNA of thryoid nodule comes back 75% hurthle cells.
A. Lobectomy
B. Total thryoidectomy
C. I-131

A

A

913
Q

36 yo female undergoing r thyroid lobectomy for 3 cm Hurthle cell lesion. Frozen section confirms the pathology. What should you do now?
A. Right hemithyroidectomy
B. Total thyroidectomy
C. Total thryoidectomy with central nodes
D. Total thyroidectomy with central and lateral nodes on the right

A

A

914
Q

35 yo old female with 1.5cm right thyroid Hurthle cell neoplasm on FNA. You do a right hemithyroidectomy and frozen section at OR confirms the lesion. What will you do now?
A. Left hemithyroidectomy
B. Close and wait for final path
C. Subtotal thyroidectomy
D. Left hemithyroidectomy plus central neck dissection

A

B

915
Q
Red, tender mass on anterior neck at hyoid bone. U/S shows cyst with thickened wall and LN. Treatment:
A. Antibiotics
B. I&amp;D
C. Sistrunk
D. Excise
A

A then Sistrunk electively

916
Q
50F with enlarging 2cm mass at angle of mandible over past 2 years.  No facial paralysis, no pain.  FNA performed and was inconclusive.  Management?
A. Core Biopsy
B. Observe
C. Superficial parotidectomy
D. Total parotidectomy
A

C

917
Q
What is a complication of low tracheal placement of a tracheostomy?
A. Tracheomalacia
B. Subglottic stenosis
C. Tracheo-esophageal fistula
D. Tracheo-innominate fistula
A

D

918
Q
35yo healthy F sent to you due to a thyroid nodule. This nodule is hot in the nuclear scan and the thyroid function is normal. Best management:
A.  Hemithyroidectomy
B. Discharge – no follow-up needed
C.  Suppress thyroid
D.  FNA
A

D

919
Q

22 year-old woman referred to you for an elevated calcitonin. She has a family history of adrenal and thyroid tumours. Clinical neck exam is negative. Best next step:
A. Urine metanephrines
B. CT abdomen
C. Total thyroidectomy
D. Total thyroidectomy & modified radical neck dissection

A

A

920
Q
A 67 M has had a previous total thyroidectomy and radioactive iodine treatment for papillary thyroid cancer.  He is now two years later, and has a persistently elevated thyrogloblulin level and Iodine 131 scan is negative.  What is the best test to localize the residual disease?
A. CT scan of chest and neck
B. MRI
C. Sestamibi scan
D. PET scan of neck and chest
A

A based on NCCN

921
Q
IDDM with CRF post parathyroidectomy for secondary hyperparathyroidism.  Despite appropriate Vit D and Ca replacement patient is still hypocalcemic.  Why?
A. Hypoalbuminemia
B.  Hypokalemia
C. Hypomagnesemia
D.  Hypophosphatemia
A

C

922
Q
30F had hemithyroidectomy for a 3cm FA. Pathologist calls you later to say that there is a 6mm papillary cancer in the specimen. Most appropriate mgt?
A. Total thyroidectomy
B. Thyroid suppression
C. RAI ablation
D. Total body I scan and regular F/U
A

B

923
Q
FNA of parotid mass comes back as Warthin’s tumor.  What diagnostic test would confirm the diagnosis?
A. MRI
B. CT scan
C. Technetium nuclear scan
D. Ultrasound
A

C

924
Q

Where is the most common location of a thyroglossal duct cyst?
A. Superior pole of thyroid
B. Adjacent to the thyroid isthmus
C. Midline neck mass at the level of the hyoid bone
D. Midline neck mass at the level of the cricoid

A

D

925
Q
A patient is post-thyroidectomy on the ward.  He develops stridor, O2 sats are good, and he is able to talk.  The wound is opened and evacuated, but patient remains stridorous. What is the next BEST management?
A. Go to OR and explore
B. Intubate on ward
C. Cricothyroidotomy on the ward
D. Perc trach on the ward
A

A

926
Q
A patient has Grave’s disease is scheduled to undergo a cholecystectomy for symptomatic gallstones.  She is unable to take PTU b/c allergic reaction.  What is the BEST way to manage perioperative state? 
A. Beta blocker
B. Beta block with lugol’s solution
C. Lugol’s solution alone
D. I 131 treatment
A

C

927
Q
A 35 year old male is found to have an elevated ESR at 60 (N < 30), elevated T4, and 4% uptake of radioactive iodine (which is low). What the MOST likely diagnosis?
A. Subacute thyroiditis
B. Hashimotos
C. Riedel’s thyroiditis
D. Graves disease
A

A

Subacute lymphocytic thyroiditis (also known as silent sporadic thyroiditis or painless sporadic thyroiditis) is clinically and pathologically similar to postpartum thyroiditis but occurs in the absence of pregnancy. It appears to be autoimmune in origin; the thyroid contains a lymphocytic infiltrate partially resembling Hashimoto’s disease but without the fibrosis, Askanazy cells, and extensive lymphoid follicle formation. Between 5 and 20 percent of patients exhibit hyperthyroidism from release of pre-formed T4 and T3, which may be followed by hypothyroidism and then a return to normal in the majority of patients. RAIU low/absent

Riedel thyroiditis, or Riedel’s thyroiditis (RT), is a rare, chronic inflammatory disease of the thyroid gland characterized by a dense fibrosis that replaces normal thyroid parenchyma. The fibrotic process invades adjacent structures of the neck and extends beyond the thyroid capsule. RAIU N/low

Hashimoto’s disease (also known as chronic lymphocytic thyroiditis or chronic auto-immune thyroiditis) is an autoimmune condition characterized by the infiltration of the thyroid by lymphocytes and the formation of Askanazy (Hürthle) cells. It is the most common inflammatory disorder of the thyroid in the United States. Patients may have normal thyroid function, frank clinical primary hypothyroidism associated with low free T4 and high TSH concentrations, or subclinical hypothyroidism with normal free T4 and elevated TSH levels. Circulating levels of antithyroid peroxidase antibodies, usually in high titers, are a hallmark of Hashimoto’s disease. RAIU N/low

Graves’ disease, also known as toxic diffuse goiter, is an autoimmune disease that affects the thyroid.[1] It frequently results in and is the most common cause of hyperthyroidism.RAIU is high.

928
Q
A patient underwent exploration for hyperparathyroidism, but at OR, unable to find the parathyroid adenoma.  Postop imaging suggests the adenoma is high in the jugulodigastric region (technetium suggests this).  What is the BEST management for this patient?
A. Same excision, cervical thymectomy
B. Explore carotid sheath
C. Exporation at jugulodigastric
D. Angioembolize the lesion
A

B

929
Q
What is the BEST way to treat symptomatic hypocalcemia?
A. Calcium Chloride over 5 minutes
B. Calcium Chloride over 20 minutes
C. Calcium Gluconate over 5 minutes
D. Calcium Gluconate over 20 minutes
A

D

930
Q

Female patient with left zone 5 lymph node. Excision biopsy demonstrates metastatic papillary thyroid cancer. Imaging demonstrates 0.8cm L thyroid nodule. Best management?
A. L hemithyroidectomy
B. Total thyroidectomy + external beam radiation
C. Total thyroidectomy + L modified radical neck ds
D. Total thyroidectomy + L modified radical neck ds + I-131

A

C

931
Q

Patient has undergone total thyroidectomy for medullary thyroid cancer. Post-op, the calcitonin level is undetectable. What is the BEST adjuvant therapy?
A. Thyroid replacement to TSH suppression
B. Thyroid replacement to normalize TSH
C. Thyroid replacement to normalize TSH and EBRT
D. Thyroid replacement to normalize TSH and 131 I ablation

A

B

932
Q

Female patient with left zone 5 lymph node. Excision biopsy demonstrates metastatic papillary thyroid cancer. Imaging demonstrates 0.8cm L thyroid nodule. Best management?
A. L hemithyroidectomy
B. Total thyroidectomy + external beam radiation
C. Total thyroidectomy + L modified radical neck ds
D. Total thyroidectomy + L modified radical neck ds + I-131

A

C in old answers. I would pick D cause patient has cervical LN mets which is an indication for RAIU per NCCN

933
Q
50 yo male with newly found jugulodigastric lymph node.  He is a non-smoker.  The rest of the head and neck is unremarkable.  What is the next BEST management?
A. CT scan and FNA
B. Ultrasound guided biopsy
C. Excisional 
D. Follow up in 6 months
A

A

934
Q
Guy gets into a fight and sustains a deep laceration to the submandibular triangle.  Which nerve is the MOST likely to be injured?
A. Marginal mandibular branch
B .Lingual
C. Hypoglossal
D. Ansa cervicalis
A

A

935
Q

40 yo woman after undergoing a lobectomy for 4 cm follicular adenoma pathologist calls and says there is a 1 cm papillary cancer in the specimen
A. Completion thyroidectomy
B. Ablate with radioactive iodine
C. Suppress thyroid with thyroxine
D. Radioiodine scan and continued follow-up

A

C

936
Q
40 yo male with 3 years history of cutaneous and subcutaneous nodules on his left upper chest. Biopsy shows dermatofibrosarcoma protuberans. What is the best management
A. Wide local excision
B. Wide local excision and radiation
C. Wide local excision and SLNBx
D. Radiation and chemotherapy
A

A

937
Q
Left neck lymph node that on biopsy stains cytokeratin positive, vimentin negative, Leukocyte common antigen (LCA) negative, HMB-45 negative. What is the primary
A. Lymphoma
B. Sarcoma 
C. Adenocarcinoma 
D. Melanoma
A

C
Cytokeratin = adenoca
Vimentin = sarcoma
HMB45 = melanoma

938
Q
25 yo female presents with hot nodule in left thyroid. She has low TSH, high T4 and is hyperthyroid. Best approach?
A. Radioactive iodine ablation 
B. Suppress TSH
C. L hemithyroidectomy
D. Antithyroid meds
A

A

939
Q
30 yo lady that had a thyroidectomy and radioactive iodine ablation 1 year ago presents for follow up. She is asymptomatic, normal exam, thyroglobulin not detectable. What should you do?
A. Clinical F/U 1 year
B. Radioactive iodine scan
C. Bone scan, CT scan
D. Neck US
A

D? vs B?

NCCN states periodic neck U?S

Local practice, we typically do one at 1 yr for follow up

940
Q

What is the most common reason for post thyroidectomy hypocalcemia?
A. Permanent ischemia of parathyroids
B. Temporary ischemia of parathyroids
C. Respiratory alkalosis and hypocalcemia
D.

A

B

941
Q

What is the most common cause of elevated Ca post surgery for excess PTH
A. Ectopic parathyroid
B. Missed hyperplasia
C. Missed adenoma

A

C

942
Q

A known MEN IIB pt presents with a 2.5 cm adrenal lesion. Management
A. Bilateral adrenalectomy
B. Unilateral adrenalectomy
C. Radical adrenal-nephrectomy

A

B

943
Q
Pt has resection of desmoid tumor from posterior triangle.  Initially has positive margins, re-exision is carried out, but abuts the accessory nerve.  What is optimal post operative treatment?
A. Excision of nerve
B. Radiation
C. Sulindac
D. Tamoxifen
A

C

944
Q
Recurrent medullary thyroid cancer after previous total thyroidectomy.  Optimal treatment
A. Radioactive iodine 
B. External beam radiation 
C. Chemoradiation  
D. Excise
A

D

Surgery first. If unresectable, radiated per NCCN

945
Q
For which FNA would you obtain an intraoperative frozen section
A. Hurthle cell 
B. Papillary  
C. Follicular 
D. Colloid
A

D?

We never do it for A, B or C

946
Q

2.5 cm nodule, FNA demonstrates 75 % Hurthle cells, 50 yo patient. Surgical treatment.
A. Hemithyroid with intraoperative frozen
B .Total thyroidectomy
C. Hemithyroid with total after permanent section

A

C

947
Q
Which nerve is most likely mistaken for accessory nerve when you’re at the lateral border of SCM
A. Phrenic 
B. Deep cervical 
C. Superior laryngeal
D. Vagus
A

A

948
Q
What is the margin for BCC.
A. 1 cm macroscopic
B. 1cm microscopic
C. Microscopic clear 
D. Macroscopic clear
A

C

949
Q

All of the following are correct EXCEPT:
A. The catabolic effects of PTH are on cortical bone
B. Post surgery for hyperparathyroidism the increased bone density is seen mainly in cancellous bone
C. Post surgery for hyperparathyroidism there is a 90% decrease in the incidence of kidney stone formation
D. Plain bone x-rays are part of the standard preoperative workup of a patient with hyperparathyroidism.

A

A

950
Q

All of the following are associated with superior laryngeal nerve injury EXCEPT:
A. Occurs when the surgeon doesn’t identify the nerve before dividing the superior pole
B. Causes loss of pitch and depth of projection
C. Is easily diagnosed with indirect or direct fiberoptic laryngoscopy
D. Causes the cord to shorten and bow

A

A

951
Q

The 2002 consensus guidelines for asymptomatic hyperparathyroidism lists all except for one of the following as indications for surgery:
A. Calcium levels 0.25 mmol/L above the upper limit of normal
B. Creatinine clearance decreased by 30%
C. Documented cardiac arrythmias or ventricular hypertrophy
D. T - Score < 2.5 at hip, lumbar spine or wrist.

A

C

952
Q
37 yo woman with previous radiation to the neck and chest for lymphoma.  Present with a 2.5 cm follicular neoplasm of the right thryroid.  What is the BEST management for this woman at this time?
A. Right Hemithryroidectomy
B. Subtotal or Total thyroidectomy
C. Observation
D. Repeat FNA
A

B

Prev radiation is one of the indications for total/subtotal

953
Q
34 yo male who has undergone a hemithryoidectomy for a 2 cm mass.  When may you not proceed with more surgery?
A. Capsular invasion
B. Well differentiated papillary cancer
C. Previous neck XRT
D. Positive lymph nodes
A

B

954
Q

Patient with a subungual melanoma of the great toe. What will you resect?
A.Local resection to include distal phalynx
B. At the MTP joint
C. Just proximal to the MTP joint
D. The toe and Metatarsal

A

A

955
Q
Cystic lesion in the posterior triangle that is positive for papillary cancer
A. Repeat FNA with immunohistochemistry
B. Total or near total
C. Total with central LN dissection
D. Total thryoidectomy with MRLND
A

D

956
Q
A female patient presents with elevated calcium and PTH, consistent with hyperparathyroidism.  The diagnosis would be confirmed with:
A. Bone scan
B. CT scan of the neck
C. Sestamibi scan
D. Urine phosphate / calcium ratio
A

C

957
Q

Frozen section is LEAST useful in:
A. Assessing margins during esophageal resection for carcinoma
B. Palpable lymph nodes during hemithyroidectomy
C. Hemithyroidectomy for follicular neoplasm
D. Morpheaform basal cell carcinoma (BCC) of the skin

A

C

958
Q
What is the most common region where the Recurrent Laryngeal Nerve is injured?
A. Tracheosophageal Grove
B. Suspensory Ligament of the Thyroid
C. Inferior Thryoid Artery
D. Thyroid
A

B

959
Q
RLN in relation to Berry’s ligament
A. Anterior
B. Posterior
C. Lateral
D. Medial
E. Superficial
A

C

https://www.ncbi.nlm.nih.gov/pubmed/9790290
Suspensory ligament of Berry: its relationship to recurrent laryngeal nerve and anatomic examination of 24 autopsies.

960
Q
Steroids are effective in treating elevated Ca in which of the following
A. Primary hyperparathyroidism
B. Secondary hyperparathyroidism
C. Metastatic ca
D. Familial hypocalciuric hypercalcemia
E. Milk alkali syndrome
A

C

961
Q
Regarding thyroglossal duct cyst
A. Forms from 2nd branchial B. 
b.	10% risk of squamous cell ca
C. Up to 1% risk papillary ca
D. Need to excise thyroid cartilage
E. Need to excise foramen cecum
A

C

962
Q
Worse prognosis with papillary thyroid ca in all except
A. apsular invasion
B. Lymph node involvement
C. Age>60
D. Size>4 cm
E. Tall cell variant
A

D

963
Q
Lymphocytic thyroiditis is associated with all of the following except:
A. Hypothyroidism
B. Transient hyperthyroidism
C. Hashimoto’s thyroiditis
D. Lymphoma
E. Medullary thyroid cancer
A

C

964
Q
Which of the following is not a histologic feature of papillary thyroid cancer?
A. Orphan Annie eyes
B. Psammoma bodies
C. Small vessel invasion
D. Lymph node metastases
E. Multicentricity
A

C

965
Q
The most common malignant tumour of the parotid is:
A. Acinic cell ca
B. Adenoid cystic ca
C. Mucoepidermoid ca
D. Mixed malignant tumour
E. Squamous cell ca
A

C

966
Q

A 10 year old girl is brought to your with a diagnosis of medullary thyroid carcinoma. She has a family history of thyroid tumours. A urinary VMA was done and is negative. What is the appropriate treatment?
A. Total parathyroidectomy
B. Total thyroidectomy
C. Additional work-up including iodonuclear scan
D. Subtotal thyroidectomy

A

B

967
Q

A hypertensive man is found to have an elevated urinary catecholamines. A CT scan of the abdomen does not localize an adrenal mass. What is the next appropriate step?
A. Laparotomy with bilateral adrenal exploration
B. Diagnostic laparoscopy
C. Observe and repeat scan
D. Order an MIBG scan

A

D

968
Q

In a patient with severe hypercalcemia (Ca > 3.2), what is the most likely cause?
A. Parathyroid carcinoma
B. Large parathyroid adenoma
C. Secondary hyperparathyroidism

A

B

In parathyroid carcinoma, calcium >3.5 typically

969
Q
A female patient presents with heat intolerance and an enlarged thyroid.  She may also have
A. Muscle weakness and diarrhea
B. Muscle hypertrophy and diarrhea
C. Muscle weakness and constipation
D. Muscle hypertrophy and constipation
A

A

970
Q
A patient with hypercalcemia may exhibit all of the following EXCEPT
A. Renal stones
B. Nausea
C. Psychosis
D. Diarrhea
A

D

971
Q

The recurrent laryngeal nerve innervates all EXCEPT
A. Sensation to the larynx below the cords
B. True cords
C. Intrinsic muscles of the larynx
D. Cricothyroid muscle

A

DC

ricothyroid is SLN

972
Q
What is the best way to localize a parathyroid adenoma?
A. US
B. Four gland exploration
C. Sestamibi
D. MRI
A

B